You are on page 1of 554

SAN BEDA UNIVERSITY

College of Law
First Semester AY 2020-2021

CASE DIGESTS
Submitted in Partial Fulfillment
of the requirements in
Criminal Law I

Submitted by:
Class of 1st Year Section B
First Semester AY 2020-2021

Submitted to:
Atty. Lyan David M. Juanico
Criminal Law I Professor

January 20, 2021

1
1st Year Section B Class List
1. AGUILAR, Jose Maria L. – Subject Beadle
2. ALGURA, Nino N.
3. AROZA, Maria Minette R
4. BELANO, Renato Jr. P.
5. BILTZ, Aralind Louise A.
6. CACHERO, Luis III L.
7. CALZADO, Anne Valerie
8. CARPIO, Anna Clarissa C.
9. CHUA, Kristy Anne
10. CRUZ, Jakielyn Anne O.
11. DELA CRUZ, Ma. Luzelle P.
12. DIMAKUTA, Jasmine A.
13. FLORES, Precious Eureka D.
14. GARCIA, Lexanne O.
15. MANANGO, John De Divine B
16. MARCELINO, Ferilynn T.
17. MAYUGA, Eunice Allaine G.
18. PALALA, Amer B.
19. PANGAN, Gabrielle L.
20. PROVIDO, Gemy Hale A
21. ROMERO, Ma. Camille Concepcion M.
22. SAMONTE, Vanessa Antoinette
23. SAMSON, Jessa Viena D.
24. TERTE, Karen A.
25. TIDALGO, Aimee Diane A.
26. VALDEZ, Ariane Faye V.
27. VILLANUEVA, Sean Ruthie L.

2
MODULE 1
GENERAL PRINCIPLES OF
CRIMINAL LAW

3
CONTRIBUTOR AGUILAR, Jose Maria L.
MODULE GENERAL PRINCIPLES OF CRIMINAL LAW (Nature and
TOPIC definition)
CASE TITLE IENT v. TULLETT PREBON G.R.NO. 189158
PONENTE LEONARDO-DE CASTRO, J.: DATE: JAN. 11, 2017
DOCTRINE Rule of lenity as a principle. The rule applies when the court is faced
with two possible interpretations of a penal statute, one that is
prejudicial to the accused and another that is favorable to him. The
rule calls for the adoption of an interpretation which is more lenient
to the accused.
FACTS Petitioner lent is a British national and the Chief Financial Officer of
Tradition Asia Pacific Pte. Ltd. (Tradition Asia) in
Singapore. Petitioner Schulze is a Filipino/German who does
Application Support for Tradition Financial Services Ltd. in London
(Tradition London). The Tradition Group and Tullett are competitors
in the inter-dealer broking business.

Tullett was the first to establish a business presence in the


Philippines and had been engaged in the inter-dealer broking
business. At sometime, in line with Tradition Group's motive of
expansion and diversification in Asia, petitioners lent and Schulze
were tasked with the establishment of a Philippine subsidiary of
Tradition Asia to be known as Tradition Financial Services
Philippines, Inc. (Tradition Philippines)

Later on, Tullett, through one of its directors, filed a Complaint-


Affidavit with the against the officers of the Tradition Group for
violation of the Corporation Code. Impleaded as respondents in the
Complaint-Affidavit were petitioners lent and Schulze, Jaime Villalon
(Villalon), who was formerly President and Managing Director of
Tullett, Mercedes Chuidian (Chuidian), who was formerly a member
of Tullett's Board of Directors, and other John and Jane Does.
Villalon and Chuidian were charged with using their former positions
in Tullett to sabotage said company by orchestrating the mass
resignation of its entire brokering staff in order for them to join
Tradition Philippines. With respect to Villalon, Tullett claimed that
the former held several meetings between with members of Tullett's
Spot Desk and brokering staff to convince them to leave the
company. Villalon likewise supposedly intentionally failed to renew
the contracts of some of the brokers. A meeting was also allegedly
conducted where petitioners and a lawyer of Tradition Philippines
were present. At said meeting, the brokers of complainant Tullett
were purportedly induced to sign employment contracts with
Tradition Philippines and were allegedly instructed by Tradition
Philippines' lawyer as to how they should file their resignation
letters.

4
According to Tullett, respondents Villalon and Chuidian violated
Sections 31 and 34 of the Corporation Code which made them
criminally liable under Section 144. As for petitioners lent and
Schulze, Tullett asserted that they conspired with Villalon and
Chuidian in the latter's acts of disloyalty against the company.

In defense, Villalon argued that his resignation from Tullett was


done in the exercise of his fundamental rights to the pursuit of life
and the exercise of his profession; he can freely choose to avail of
a better life by seeking greener pastures; and his actions did not fall
under any of the prohibited acts under Sections 31 and 34 of the
Corporation Code. It is likewise his contention that Section 144 of
the Corporation Code applies only to violations of the Corporation
Code which do not provide for a penalty while Sections 31 and 34
already provide for the applicable penalties for violations of said
provisions - damages, accounting and restitution. Thus, he
concluded that there was no probable cause that he violated the
Corporation Code nor was the charge of conspiracy properly
substantiated. While Chudian claimed that she left Tullett simply to
seek greener pastures. She stressed that her resignation from
Tullett and subsequent transfer to Tradition Philippines did not fall
under any of the prohibited acts under Sections 31 and 34. Section
144 of the Corporation Code purportedly only applies to provisions
of said Code that do not provide for any penalty while Sections 31
and 34 already provide for the penalties for their violation.
ISSUE/S Whether or not the Section 144 of the Corporation Code applies to
Sections 31 and 34 of the same code, thus, making it a penal
offense.
RULING/S No. The Court concludes that there is textual ambiguity in Section
144; moreover, such ambiguity remains even after an examination
of its legislative history and the use of other aids to statutory
construction, necessitating the application of the rule of lenity in the
case at bar.
The Supreme Court stated that there is no provision in the
Corporation Code using similarly emphatic language that evinces a
categorical legislative intent to treat as a criminal offense each and
every violation of that law. Consequently, there is no compelling
reason for the Court to construe Section 144 as similarly employing
the term "penalized" or "penalty" solely in terms of criminal liability.
The Corporation Code was intended as a regulatory measure, not
primarily as a penal statute. Sections 31 to 34 in particular were
intended to impose exacting standards of fidelity on corporate
officers and directors but without unduly impeding them in the
discharge of their work with concerns of litigation. Considering the
object and policy of the Corporation Code to encourage the use of
the corporate entity as a vehicle for economic growth, we cannot

5
espouse a strict construction of Sections 31 and 34 as penal
offenses in relation to Section 144 in the absence of unambiguous
statutory language and legislative intent to that effect.
When Congress intends to criminalize certain acts it does so in
plain, categorical language, otherwise such a statute would be
susceptible to constitutional attack. The court stress that had the
Legislature intended to attach penal sanctions to Sections 31 and
34 of the Corporation Code it could have expressly stated such
intent in the same manner that it did for Section 74 of the same
Code.

6
CONTRIBUTOR ALGURA, Nino N.
MODULE General Principles of Criminal Law
TOPIC
CASE TITLE KHOSROW MINUCHER G.R.NO. 142396
v.
HON. COURT OF APPEALS and
ARTHUR SCALZO
PONENTE VITUG, J.: DATE: Feb. 11, 2003
DOCTRINE Only "diplomatic agents," under the terms of the Convention, are
vested with blanket diplomatic immunity from civil and criminal suits.
FACTS Sometime in May 1986, an Information for violation of Section 4 of
Republic Act No. 6425, otherwise also known as the "Dangerous
Drugs Act of 1972," was filed against petitioner Khosrow Minucher
and one Abbas Torabian with the Regional Trial Court, Branch 151,
of Pasig City. The criminal charge followed a "buy-bust operation"
conducted by the Philippine police narcotic agents in the house of
Minucher, an Iranian national, where a quantity of heroin, a
prohibited drug, was said to have been seized.

The criminal charge followed a "buy-bust operation" conducted by


the Philippine police narcotic agents in the house of Minucher, an
Iranian national, where a quantity of heroin, a prohibited drug, was
said to have been seized. The narcotic agents were accompanied
by private respondent Arthur Scalzo who would, in due time,
become one of the principal witnesses for the prosecution.

On 08 January 1988, Presiding Judge Eutropio Migrino rendered a


decision acquitting the two accused.

On 03 August 1988, Minucher filed Civil Case No. 88-45691 before


the Regional Trial Court (RTC), Branch 19, of Manila for damages
on account of what he claimed to have been trumped-up charges of
drug trafficking made by Arthur Scalzo. The Manila RTC detailed
what it had found to be the facts and circumstances surrounding the
case.

ISSUE/S Whether or not Arthur Scalzo is indeed entitled to diplomatic


immunity.
RULING/S A foreign agent, operating within a territory, can be cloaked with
immunity from suit but only as long as it can be established that he
is acting within the directives of the sending state. The consent of
the host state is an indispensable requirement of basic courtesy
between the two sovereigns. Guinto and Shauf both involve officers
and personnel of the United States, stationed within Philippine

7
territory, under the RP-US Military Bases Agreement. While
evidence is wanting to show any similar agreement between the
governments of the Philippines and of the United States (for the
latter to send its agents and to conduct surveillance and related
activities of suspected drug dealers in the Philippines), the consent
or imprimatur of the Philippine government to the activities of the
United States Drug Enforcement Agency, however, can be gleaned
from the facts heretofore elsewhere mentioned. The official
exchanges of communication between agencies of the government
of the two countries, certifications from officials of both the Philippine
Department of Foreign Affairs and the United States Embassy, as
well as the participation of members of the Philippine Narcotics
Command in the "buy-bust operation" conducted at the residence of
Minucher at the behest of Scalzo, may be inadequate to support the
"diplomatic status" of the latter but they give enough indication that
the Philippine government has given its imprimatur, if not consent,
to the activities within Philippine territory of agent Scalzo of the
United States Drug Enforcement Agency. The job description of
Scalzo has tasked him to conduct surveillance on suspected drug
suppliers and, after having ascertained the target, to inform local law
enforcers who would then be expected to make the arrest. In
conducting surveillance activities on Minucher, later acting as the
poseur-buyer during the buy-bust operation, and then becoming a
principal witness in the criminal case against Minucher, Scalzo
hardly can be said to have acted beyond the scope of his official
function or duties.
All told, this Court is constrained to rule that respondent Arthur
Scalzo, an agent of the United States Drug Enforcement Agency
allowed by the Philippine government to conduct activities in the
country to help contain the problem on the drug traffic, is entitled to
the defense of state immunity from suit.

8
CONTRIBUTOR AROZA, Maria Minette R.
MODULE Characteristics of Criminal Law - Generality
TOPIC
CASE TITLE LIANG v. PEOPLE G.R.NO. 125865
PONENTE YNARES-SANTIAGO, J: DATE: JAN. 28, 2000
DOCTRINE Immunity from suit does not apply when a public official has acted
beyond his authority.
FACTS Petitioner is an economist working with the Asian Development
Bank (ADB). For allegedly uttering defamatory words against fellow
ADB worker Joyce Cabal, he was charged before the Metropolitan
Trial Court (MeTC) of Mandaluyong City with two counts of grave
oral defamation. MeTC judge received an "office of protocol" from
the Department of Foreign Affairs (DFA) stating that petitioner is
covered by immunity from legal process under Section 45 of the
Agreement between the ADB and the Philippine Government
regarding the Headquarters of the ADB in the country. The MeTC
judge without notice to the prosecution dismissed the two criminal
cases. Petitioner filed a motion for reconsideration which was
opposed by the DFA. When the motion was denied, the prosecution
filed a petition for certiorari and mandamus with the Regional Trial
Court (RTC) of Pasig City which set aside the MeTC rulings and
ordered the latter court to enforce the warrant of arrest it earlier
issued. After the motion for reconsideration was denied, petitioner
elevated the case to the Supreme Court via a petition for review
arguing that he is covered by immunity under the Agreement.
ISSUE/S Whether or not the petitioner is immune from suit.
RULING/S The Supreme Court held that the Petitioner in this case is not
immune from suit. Slandering a person could not possibly be
covered by the immunity agreement because our laws do not allow
the commission of a crime, such as defamation, in the name of
official duty. The imputation of theft is ultra vires and cannot be part
of official functions. It is well-settled principle of law that a public
official may be liable in his personal private capacity for whatever
damage he may have caused by his act done with malice or in bad
faith or beyond the scope of his authority or jurisdiction. It appears
that even the government's chief legal counsel, the Solicitor
General, does not support the stand taken by petitioner and that of
the DFA. Under the Vienna Convention on Diplomatic Relations, a
diplomatic agent, assuming petitioner is such, enjoys immunity from
criminal jurisdiction of the receiving state except in the case of an
action relating to any professional or commercial activity exercised
by the diplomatic agent in the receiving state outside his official
functions. As already mentioned above, the commission of a crime
is not part of official duty.

9
CONTRIBUTOR Belano, Renato Jr. P.
MODULE General Principles of Criminal Law, Characteristics of Criminal
TOPIC Law
CASE TITLE Del Socorro v. Van Wilsem G.R. NO. 193707
PONENTE Peralta, J. DATE: Dec 10, 2014
DOCTRINE Territoriality Principle - Penal laws and those of public security and
safety shall be obligatory upon all who live and sojourn in Philippine
territory, subject to the principle of public international law and to
treaty stipulations.
FACTS Petitioner and respondent contracted marriage in Holland in
September of 1990. Four years later they were blessed with a son
named Roderigo Norjo Van Wilsem. Their bond ended in July of
1995 by virtue of a Divorce Decree issued by the appropriate Court
of Holland. Thereafter, petitioner and her son came home to the
Philippines.

According to petitioner, respondent made a promise to provide


monthly support to their son amounting to Php 17,500 more or less.
However, since the arrival of petitioner and her son in the
Philippines, respondent never gave support to their son.

Because of the foregoing circumstances petitioner filed an


information with the RTC-Cebu stating that her son Roderigo was
economically abused due to the complete lack of support which was
due to him. Subsequently the case was dismissed on the ground
that the facts charged in the information do not constitute an offense
with respect to the respondent who is an alien. Petitioner thereafter
filed her Motion for Reconsideration reiterating respondent’s
obligation to support their child under Article 195 of the Family Code,
thus failure to do so makes him liable under R.A. No. 9262.

In September of 2010, the lower court denied the petitioner’s Motion


for Reconsideration stating that the arguments presented are
basically a rehash of those advanced earlier in the memorandum of
the prosecution.
ISSUE/S Whether or not a foreign national can be held criminally liable
under R.A. No. 9262 for his unjustified failure to support his minor
child.
RULING/S Yes, based on the foregoing legal precepts, we find that respondent
may be made liable under Section 5(e) and (i) of R.A. No. 9262 for
unjustly refusing or failing to give support to petitioner’s son, to wit:

10
SECTION 5. Acts of Violence Against Women and Their Children. -
The crime of violence against women and their children is
committed through any of the following acts:

xxx
(2) Depriving or threatening to deprive the woman or her
children of financial support legally due her or her family, or
deliberately providing the woman's children insufficient financial
support;
xxx

(i) Causing mental or emotional anguish, public ridicule or


humiliation to the woman or her child, including, but not limited to,
repeated verbal and emotional abuse, and denial of financial
support or custody of minor children of access to the woman's
child/children.

Under the aforesaid special law, the deprivation or denial of financial


support to the child is considered an act of violence against women
and children.

In addition, considering that respondent is currently living in the


Philippines, we find strength in petitioner’s claim that the
Territoriality Principle in criminal law, in relation to Article 14 of the
New Civil Code, applies to the instant case, which provides that:
“Penal laws and those of public security and safety shall be
obligatory upon all who live and sojourn in Philippine territory,
subject to the principle of public international law and to
treaty stipulations.” On this score, it is indisputable that the alleged
continuing acts of respondent in refusing to support his child with
petitioner is committed here in the Philippines as all of the parties
herein are resident of the Province of Cebu City. As such, our courts
have territorial jurisdiction over the offense charged against
respondent. It is likewise irrefutable that jurisdiction over the
respondent was acquired upon his arrest.

11
CONTRIBUTOR BILTZ, Aralind Louise A.
MODULE Characteristics of Criminal Law
TOPIC
CASE TITLE GONZALES VS. ABAYA G.R.NO. 164007
PONENTE SANDOVAL-GUTIERREZ, J. DATE: AUGUST 10, 2006
DOCTRINE Members of the AFP and other persons subject to military law who
commit crimes or offenses penalized under the Revised Penal
Code (like coup d'etat), other special penal laws, or local
ordinances shall be tried by the proper civil court.
FACTS President Gloria Macapagal Arroyo received intelligence reports
that some members of the AFP had abandoned their designated
places of assignment with the aim of destabilizing the government.
More than 300 heavily armed members of the AFP entered the
premises of Oakwood Premier Luxury Apartments and planted
explosive devices around the building.

Led by Navy Lt. Antonio Trillanes IV, the troops announced their
grievances against the administration through broadcast media
such as the graft and corruption in the military, illegal sale of arms
and ammunition the enemies of the State, and bombings in Davao
City intended to acquire more military assistance from the US
government. They declared their withdrawal of support from the
President and demanded that she resign as well as her cabinet
members.

On the same day, President Arroyo issues Proclamation No. 427


declaring a state of rebellion. This was followed by General Order
No. 4 directing the AFP and PNO to take all necessary measure to
suppress the rebellion taking place.

Pursuant to Article 70 of the Articles of War, respondent General


Narciso Abaya, then AFP Chief of Staff, ordered the arrest and
detention of the soldiers involved in the Oakwood incident.

The DOH filed an Information for coup d’etat against the soldiers
with the RTC. While respondent Chief of Staff issues Letter Order
No. 625 creating a Pre-Trial Investigation Panel tasked to determine
the propriety of filing with military tribunal charges for violations of
Articles of War under Commonweal Act No. 408 against the same
military personnel.

148 out of the 321 accused filed an Omnibus Motion praying that
the said trial court assume jurisdiction over all the charges filed with
the military tribunal invoking RA No. 7055. The Pre-Trial
Investigation Panel recommended that the accused be charged

12
before a general court martial with violations of the Articles of War.
On its final Pre-Trial Investigation, the Panel recommended that
following the “Doctrine of Absorption”, those charged with coup
d’etat before the RTC should not be charged before the military
tribunal for violation of Articles of War.

The RTC issued an Order stating that “all charges before the court
martial against the accused are hereby declared not service-
connected, but rather absorbed in furtherance of the alleged crime
of coup d’etat. Petitioners thus maintain that since the RTC has
made a determination that the offense for violation of Article 96 of
the Articles of war is not service-connected, the military tribunal
cannot compel them to submit to its jurisdiction.
ISSUE/S Whether or not the petitioners are entitled to the Writ of Prohibition
RULING/S No. Prohibition is the remedy to prevent inferior courts,
corporations, boards, or persons from usurping or exercising a
jurisdiction or power with which they have not been vested by law.
Petitioners, being officers of the AFP, are subject to military law.
Section 1 or RA No. 7055 lays down the general rule that
members of the AFP and other persons subject to military law who
commit crimes or offenses penalized under the RPC, special penal
laws, or local ordinances shall be tried by the proper civil court. It
also provides an exception to the general rule. Where the civil
court, before arraignment, has determined the offense to be
service-connected, the offending soldier shall be tried by a court
martial.
The second paragraph of Article 96 of the Articles of War identifies
the “service-connected crimes or offenses” as “limited to those
defined in Articles 54 to 70, 72 to 92, and 95 to 97”. Violations of
these articles are triable by court martial and delineates the
jurisdiction of civil courts and court martial over crimes or offenses
committed by military personnel. This is necessary to preserve the
peculiar nature of military justice system over military personnel
charged with service-connected offenses.
The charge against petitioners concerns the alleged violation of
their solemn oath as officers to defend the Constitution and the
duly-constituted authorities. Such violation allegedly caused
dishonor and disrespect to the military profession. Section 1 of RA
7055 deprives civil courts of jurisdiction over service-connected
offenses including Article 96 of the Articles of War. Thus, the
doctrine of absorption of crimes is not applicable in this case.
RA No. 7055 did not divest the military courts of jurisdiction to try
cases involving violations of Articles 54 to 70, 72 to 92, and 95 to
97 of the Articles of War as these are service-connected crimes or
offenses.

13
CONTRIBUTOR CACHERO, Luis III L.
MODULE Territoriality principle of Criminal Law
TOPIC
CASE TITLE PEOPLE V. ROGER TULIN, G.R.NO. 111709
ET AL.
PONENTE MELO, J: DATE: August 30, 2001
DOCTRINE Territoriality Principle of Criminal Law. The provisions of the Revised
Penal Code shall be enforced not only within the Philippine
archipelago and its interior waters, but also outside of its jurisdiction
such as when the offender should commit an offense while onboard
a Philippine ship or airship. This principle also applies to special
penal laws, such as the Presidential Decree No. 532 or the Anti-
Piracy and Anti-Highway Robbery Law of 1974.
FACTS M/T Tabangao, a cargo vessel owned by the PNOC Shipping and
Transport Corporation was sailing off the coast of Mindoro near
Silonay Island. The vessel is manned by 21 crew members when it
was suddenly boarded, with the use of an aluminum ladder by seven
fully armed pirates led by Emilio Changco, older brother of accused-
appelant Cecilio Changco. The pirates detained the crew and took
complete control of the vessel. Loyola, one of the accused-
appellants, ordered the three crew members to paint over the name
“M/T Tabangao” using black paint on the front and rear portions of
the vessel, as well as the PNOC logo on the chimney of the vessel.
The veseel was then painted with the name “Galilee” with registry at
San Lorenzo, Honduras. The crew was then forced to sail to
Singapore.
Upon arrival in the vicinity of Singapore on March 9, 1991, the vessel
cruised around the area presumably to await for another vessel to
arrive which, however, failed. The pirates were then forced to return
to the Philippines, arriving at Calatagan, Batangas on March 14,
1991 where it remained at sea for a few days. On March 28, 1991,
the vessel again sailed to and anchored about 10-18 nautical miles
from Singapore’s shoreline where another vessel called “Navi Pride”
anchored beside it. The cargo of M/T Tabangao was then
transferred to the Navi Pride upon the orders of Emilio Changco with
the supervision of accused-appellant Cheong San Hiong.
The transfer of the cargo was completed on March 30, 1991 and
the vessel returned to Calatagan, Batangas on April 8, 1991. The
crew members were also released on the same day.
The accused-appellants were then charged with qualified piracy
(violation of P.D. 532)
Accused-appellant Cheong San Hiong contends that the alleged
acts done by him were outside the jurisdiction of Philippine courts
since the acts were done or executed outside of the Philippine
waters and territory.

14
ISSUE/S Whether or not Philippine courts have jurisdiction over accused-
appellant Cheong San Hiong for the alleged acts done outside
Philippine territory.
RULING/S YES. The Philippine courts have jurisdiction over the alleged acts
done by Cheong San Hiong, even though these acts were done
outside Philippine waters. The attack and seizure of “M/T
Tabangao” and its cargo were committed in Philippine waters,
specifically off the coast of Mindoro near Silonay Island. Although
P.D. 532 requires that the attack and seizure of the vessel and its
cargo be committed in Philippine waters, the disposition by the
pirates of the vessel and its cargo is still deemed part of the act of
piracy, hence, the same need not be committed in Philippine
waters.

15
CONTRIBUTOR CALZADO, Anne Valerie L.
MODULE Characteristics of Criminal Law, Territorial (Art. 2 RPC)
TOPIC
CASE TITLE PEOPLE v. LOL-LO G.R.NO. 17958
PONENTE MALCOLM, J. DATE: FEB 27 1922
DOCTRINE Piracy is a crime not against any particular State but against all
mankind. It may be punished in the competent tribunal of any
country where the offender may be found or into which he may be
carried. The jurisdiction of piracy unlike all other crimes has no
territorial limits.
FACTS Six vintas manned by 24 Moros intercepted a Dutch boat between
the islands of Buang and Bukid in the Dutch East Indies. The Dutch
boat was carrying eleven men, women and children. At first, the
Moros asked for food, but once on the Dutch boat, took for
themselves all the vessel’s cargo, attacked some of the men and
brutally violated 2 of the women by methods too horrible to be
described. All of the persons on the boat, with the exception of the
2 young women, were again placed on it and holes were made on
it, with the idea that it would submerge. Two of the Moro marauders
were Lol-lo, who also raped one of the women, and Saraw later
returned to their home in Tawi-Tawi, Sulu where they were arrested
and charged with piracy in the Court of First Instance (CFI) of Suhn.

The Moros interposed a demurrer, saying that the charge was not
within the jurisdiction of the CFI, nor of any court in the Philippines,
since facts did not constitute a public offense under Philippine laws.
ISSUE/S Whether or not the CFI in the Philippines have jurisdiction over the
case of Lol-lo and Saraw
RULING/S Yes. The CFI has jurisdiction over the case.
Pirates are in law hostes humani generis (enemy of mankind).
Piracy is a crime not against any particular state but against all
mankind. It may be punished in the competent tribunal of any
country where the offender may be found or into which he may be
carried. The jurisdiction of piracy unlike all other crimes has no
territorial limits. As it is against all so may it be punished by all. Nor
does it matter that the crime was committed within the jurisdictional
3-mile limit of a foreign state, "for those limits, though neutral to war,
are not neutral to crimes.”

16
CONTRIBUTOR CARPIO, Anna Clarissa C.
MODULE Module 1: General Principles of Criminal Law
TOPIC
CASE TITLE WHITE LIGHT CORP. v. CITY G.R.NO. 122846
OF MANILA
PONENTE Tinga, J. DATE: January 20,
2009
DOCTRINE The Ordinance prevents the lawful uses of a wash rate depriving
patrons of a product and the petitioners of lucrative business ties as
a police power measure. However, for the same to be a valid
exercise of police power, (1) the interest of the public in general, as
distinguished from those of a particular class, require an
interference with private rights; (2) the means must be reasonably
necessary for the accomplishment of the purpose; (3) it (the means)
should not be unduly oppressive of private rights; (4) it must be
evident that no other alternative for the accomplishment for the
purpose less intrusive of private rights can work; and more
importantly, (5) a reasonable relation must exist between the
purposes of the measure and the means employed for its
accomplishment. Absence of such requisites, the police measure
shall be struck down as an arbitrary intrusion into private rights.
FACTS In December 1992, Manila Mayor Alfredo Lim penned into law, City
Ordinance No. 7774 which, among others, prohibits short-time
admission and rates, and wash-up rate schemes in hotels, motels,
inns, lodging houses, pension houses, and similar establishes in the
city of Manila; and prescribes penalty for any person or corporation
who shall violate any of its provisions ranging from fines to
imprisonment or both. Thereafter, the Malate Tourist and
Development Corporation (MTDC), owner and operator of Victoria
Court, a motel in Malate, filed for a temporary restraining order
(TRO) with RTC Manila assailing the constitutionality and validity of
the Ordinance averring that PD No. 259 has authorized it to admit
customers on a short-time basis as well as to charge customers
wash up rates for 3 hours of stay. Petitioners White Light
Corporation (WLC), Titanium Corporation (TC) and Sta. Mesa
Tourist and Development Corporation (STDC), components of the
Anito Group of Companies then filed a motion to intervene on the
ground that the Ordinance directly affects their business interests
as operators of drive-in-hotels and motels in Manila which was
granted by the RTC, as well as the TRO which directed the
respondent City of Manila to cease and desist from enforcing the
Ordinance.

In its Answer, respondent City maintained that the Ordinance is a


legitimate exercise of police power. In March 1993, the Solicitor
General filed its Comment arguing that the Ordinance is

17
constitutional. At the pre-trial conference, petitioners WLC, TC, and
STDC agreed to submit the case without trial as it involved a purely
legal question. The RTC declared the Ordinance, NULL and VOID
on the ground that it “strikes at the personal liberty of the individual
guaranteed and jealously guarded by the Constitution,” rooted on
the constitutional provision encouraging private enterprises and the
incentive to needed investment, as well as the right to operate
economic enterprises. Furthermore, the illicit relationships that the
Ordinance sought to dissuade could still be consummated
elsewhere not only by simply paying the assailed 12-hour stay.

On appeal, respondent City asserted that the Ordinance is a valid


exercise of police power pursuant to Section 458(4)(iv) of the Local
Government Code which confers on cities, among other local
government units, the power to regulate the establishment,
operation and maintenance of cafes, restaurants, beerhouses,
hotels, motels, inns, pension houses, lodging houses and other
similar establishments including tourist guides and transports. Also,
the same is a valid exercise of the City’s power to enact all
ordinances it may deem necessary and proper for the sanitation and
safety, furtherance of prosperity, and the promotion of morality,
peace, good order, comfort, convenience and general welfare of the
city and its inhabitants, among others, pursuant to the Revised
Manila Charter (Article III, Section 18(kk)). Petitioners argued the
contrary, stating that it violates the right to privacy and the freedom
of movement; an invalid exercise of police power; and an
unreasonable and oppressive interference in their businesses. The
Court of Appeals reversed the decision of the RTC and affirmed the
constitutionality of the Ordinance on the following grounds: (1) it only
penalizes owners/operators of establishments that admit individuals
for short time stays, hence, not violative of the right to privacy or the
freedom of movement; (2) the requisites of the valid exercise of
police power have been satisfied, that is, having a lawful object
obtained through a lawful method, that is to cub immoral activities
(lawful object), and lawful method since establishments are still
allowed to operate; (3) the adverse effect on the establishments is
justified by the well-being of its constitutes in general; and (4) liberty
is regulated by law.
ISSUE/S Whether or not the assailed Ordinance is constitutional?
RULING/S The test of a valid ordinance is well established. For an ordinance
to be valid, it must not only be within the corporate powers of the
local government unit to enact and pass according to the
procedure prescribed by law, it must also conform to the
following SUBSTANTIVE REQUIREMENTS: (1) it must not
contravene the Constitution or any statue; (2) not be unfair or
oppressive; (3) not be partial or discriminatory; (3) not prohibit by

18
regulate trade; (4) be general and consistent with public policy; and
(5) not be unreasonable.

The Ordinance prohibits 2 specific and distinct business practices:


wash rate admissions; and renting out a room more than twice a
day. The apparent goal of the Ordinance is to minimize if not
eliminate the use of the covered establishments for illicit activities
(sex, prostitution, drug use, etc.) and these goals, by themselves,
are unimpeachable and certainly fall within the ambit of the police
power of the State which is conferred upon the respondent City
through the Local Government Code. However, the desirability of
these ends do not sanctify any and all the means for their
achievements which must align with the Constitution, and shall not
infringe any of the rights guaranteed by the Bill of Rights. In the
case, the primary question is one of due process – that is, whether
the government has sufficient justification for depriving a person of
life, liberty, or property.

Liberty as guaranteed by the Constitution is the “right to exist


and the right to be free from arbitrary restraint and servitude,”
not the mere freedom from physical restraint of the citizen but the
right of man to enjoy the facilities with which he has been
endowed by his Creator, subject only to such restraint as are
necessary for the common welfare. In the case, the rights of the
citizen to be free to use his faculties in all lawful ways; to live and
work where he will; to earn his livelihood by any lawful calling; and
to pursue and avocation are all deemed embraced in the concept of
liberty. In a Constitution for a free people, there can be no doubt,
the meaning of “liberty” must be broad indeed.

It cannot be denied that the primary animus behind the Ordinance


is the curtailment of sexual behavior and the respondent City
asserts that the subject establishments “have gained notoriety as
venue of prostitution, adultery, and fornications in Manila since they
provide the necessary atmosphere for clandestine entry, presence
and exit and thus become the ideal haven for prostitutes and thrill-
seekers. Whether or not this depiction is accurate, it is undeniable
that legitimate sexual behavior among willing married or consenting
single adults which is constitutionally protected will be curtailed as
well. Thus, the concept of liberty compels respect for the individual
whose claim to privacy and interference demands respect.

Indeed, the invasion of the right to privacy as a constitutional right


should be justified by a compelling state interest. The recognition of
the right of privacy must be dealt independently of its identification.
Recognition in itself is fully deserving of constitutional protection and

19
governmental powers should stop short of certain intrusions into the
personal life of the citizen. Thus, the other legitimate activities which
the Ordinance would proscribe or impair cannot be discounted as
there are very legitimate uses for a wash rate or renting the room
out for more than twice a day such as the use of families who are
momentarily out of their homes, or transit passengers who use the
same to wash up and rest. Thus, the use of motel or hotels for a few
hours with purposes other than having sex or using illegal drugs is
legitimate although it is only for a span of a few hours.

The Ordinance prevents the lawful uses of a wash rate depriving


patrons of a product and the petitioners of lucrative business ties as
a police power measure. However, for the same to be a valid
exercise of police power, (1) the interest of the public in general, as
distinguished from those of a particular class, require an
interference with private rights; (2) the means must be reasonably
necessary for the accomplishment of the purpose; (3) it (the means)
should not be unduly oppressive of private rights; (4) it must be
evident that no other alternative for the accomplishment for the
purpose less intrusive of private rights can work; and more
importantly, (5) a reasonable relation must exist between the
purposes of the measure and the means employed for its
accomplishment. Absence of such requisites, the police measure
shall be struck down as an arbitrary intrusion into private rights.
Thus, the exercise of police power is subject to judicial review when
life, liberty or property is affected; but is not an in any way meant to
take it away from the vastness of State’s police power its exercise
enjoys presumption of validity.

A plain reading of the Ordinance’s Section 3 shows that it makes no


classification of places of lodging, thus deems them all susceptible
to illicit patronage and subject them without exception to the
unjustified prohibition. The behavior which the Ordinance seeks to
curtail is in fact already prohibited and could not in fact be
diminished simply by applying existing laws. Less intrusive
measures such as curbing the proliferation of prostitutes and drug
dealers through active police work would be more effective and
would have minimal intrusion on the petitioners’ businesses and
other legitimate merchants. It is also apparent that the Ordinance
can easily be circumvented by merely paying the whole day rate
without any hindrance to those engaged in illicit activities as the drug
dealers and prostitutes can in fact collect “wash rates” from their
clients by charging the same to them.

While individual rights may be adversely affected as fairly required


by the legitimate demands of public interest or public welfare, the

20
State is a leviathan that must be restrained from needlessly
intruding into the lives of its citizens. As well-intentioned the
Ordinance may be, it is in effect an arbitrary and whimsical intrusion
into the rights or the establishment and their patrons. It needlessly
restrains operation of the petitioners’ businesses as well as
restricting the rights of their patrons without sufficient justification as
it rashly equates wash rates and renting out a room more than twice
a day with immorality without accommodating innocuous intentions.

The promotion of public welfare and a sense of morality among


citizens deserves the full enforcement of the judiciary PROVIDED
that such measures do not trample rights the Court is sworn to
protect. The notion that the promotion of public morality is a function
of the State ever since and the advancement of moral relativism as
a school of philosophy does not de-legitimize the role of morality in
law. The American maxim “you cannot legislate morality” is
ultimately legitimate as a matter of law. It means efforts to legislate
morality will fail if they are widely at variance with public attitudes
about right and wrong. Our penal laws, for one, are founded on age-
old moral tradition, and as long as they are widely accepted
distinctions between right and wrong, they will remain so oriented.
Our democracy is distinguished for its recognition that the individual
liberty to make the choices is innate and protected by the State.

21
CONTRIBUTOR CHUA, Kristy Anne
MODULE GENERAL PRINCIPLES OF CRIMINAL LAW (Due process and
TOPIC equal protection: Art. 3, Sec. 1, 1987 Constitution)
CASE TITLE GARCIA V. DRILON G.R.NO. 179267
PONENTE PERLAS-BERNABE, J.: DATE: JUNE 25, 2013
DOCTRINE The essence of due process is to be found in the reasonable
opportunity to be heard and submit any evidence one may have in
support of one's defense. "To be heard" does not only mean verbal
arguments in court; one may be heard also through pleadings.
Equal protection simply requires that all persons or things similarly
situated should be treated alike, both as to rights conferred and
responsibilities imposed.
FACTS For the years 2000-2003, 90% of all forms of abuse and violence
consisted of female violence and more than 90% were committed
by their husbands or live-in partners. On March 8, 2004, Congress
enacted R.A. No. 9262, entitled “An Act Defining Violence Against
Women and Their Children, Providing for Protective Measures for
Victims, Prescribing Penalties Therefore, and for Other Purposes.”
which took effect on March 27, 2004.

On March 23, 2006, Jaype-Garcia filed on behalf of her minor


children and for herself, a petition before the Regional Trial Court of
Bacolod City requesting for the grant of Temporary Protection Order
against petitioner Garcia under R.A. 9262. In 2002, the couple
celebrated their marriage with that they have three children, namely:
Jo-Ann, a natural child of Garcia but adopted by Jaype-Garcia;
Jessie, their common child; and lastly Joseph, also their common
child. Jaype-Garcia is a dutiful and faithful wife while Garcia is a
dominant, controlling and demands absolute obedience from his
wife and children. Prayers were forbidden and the wife was isolated
from her friends. When she took up law and worked in a law office
part time, she was trivialized for her ambitions and persuaded her
to stay at home. He was often jealous of other men eyeing his wife
and even threatened the wife that he would get any men eyeing her
killed.

In 2004, Garcia had an affair with a godmother of one of their sons


which he admitted to Jaype-Garcia. Garcia’s infidelity created
numerous fights which caused Jaype-Garcia physical and emotional
wounds. In one of their fights, Garcia grabbed Jaype-Garcia on both
arms and shook her with force that eventually caused bruises and
hematoma. At another fight, Garcia hit Jaype-Garcia’s lip causing it
to bleed. He also hit Jo-Ann on the chest and slapped her multiple
times. All of the children were aware of Jaype-Garcia’s sufferings.
Jaype-Garcia even attempted suicide by cutting her wrist. Garcia
left the family home for good and warned Jaype-Garcia that if there

22
is a legal battle with him, she will receive nothing. On March 24,
2006, RTC issued a TPO effective for thirty days. A petition for
prohibition with prayer for injunction and temporary restraining order
while challenging the constitutionality of R.A. No. 9262 and validity
of the TPO was filed before the CA, the appellate court amended
the TPO and other orders, however also dismissing the petition on
January 24, 2007, for failure of Garcia to raise the constitutional
issue and the annulment of the protection order by the trial court
equates to a collateral attack on the law.
ISSUE/S Whether or not the court erred in not declaring R.A. 9262, an act in
violation of the equal protection clause and due process clauses of
the constitution and which causes undue delegation of judicial
power to barangay officials.
RULING/S No, according to the Supreme Court, the lower court did not commit
an error regarding their judgment on R.A. 9262. First and foremost,
R.A. 9262 does not violate the equal protection clause by favoring
women over men as victims of violence and abuse to whom the
State extends it protection since this clause simply means that every
individual or thing similarly situated should be treated alike, both as
to rights conferred and responsibilities imposed. This clause does
not guarantee equality in the application of the laws upon every
citizen of the state. Next, regarding the due process clause, it should
be taken into account that on April 26, 2006, the petitioner filed a
petition in opposition to the Urgent Ex-Parte Motion for Renewal of
the TPO which was granted two days earlier. The trial court dated
September 26, 2006, provided him five days within which provide
cause why the TPO should not be renewed or extended but the
petitioner chose not to file the required comment arguing that it
would result into an “exercise in futility,” conveniently forgetting that
the TPO is effective for 30 days each time. Failure to provide does
not equate to denial of due process of law. Lastly, there is no undue
delegation of judicial power to the barangay officials because the
BPO issued by the Punong Barangay or by any available Barangay
Kagawad when Punong Barangay is unavailable merely orders the
perpetrator to desist from causing physical harm and threatening to
cause physical harm to the woman or her child. With this, the
function is purely executive in nature in accordance with the Local
Government Code to “enforce all laws and ordinances,” and to
“maintain public order in the barangay.”

23
CONTRIBUTOR CRUZ, Jakielyn Anne O.
MODULE MODULE 1: Constitutional Limitations on Criminal Law – Freedom
TOPIC of Expression (Art. 3, Sec. 4, 1987 Constitution)
CASE TITLE US v. BUSTOS G.R.NO. L-12592
PONENTE MALCOLM, J: DATE: MAR. 8, 1918
DOCTRINE The interests of society and the maintenance of good government
demand a full discussion of public affairs. Complete liberty to
comment on the conduct of public men is necessary for free speech.
The guarantees of free speech and a free press include the right to
criticize judicial conduct.
The doctrine of privileged communications rests upon public policy,
"which looks to the free and unfettered administration of justice,
through, as an incidental result, it may, in some instances, afford an
immunity to the evil-disposed and malignant slanderer."
FACTS In 1915, numerous citizens of the Province of Pampanga
assembled, the prepared and signed a petition to the Executive
Secretary, and and five individuals signed affidavits, charging
Roman Punsalan, justice of the peace of Macabebe and Masantol,
Pampanga, with malfeasance in office and asking for his removal.
The complainant stated in their petition three different instances that
the justice of peace would demanded something in return from the
interested parties when they filed a complaint or wanted to ascertain
the results of the trial, and if the demand would not be complied they
would not get the results that they wanted. The justice of the peace
was notified and denied the charges. The justice of peace filed a
motion of new trial, and the judge of first instance ordered a
suppression of the charges against Punsalan and acquitted him of
the same.
Hence, a criminal action was filed against the petitioners,
for voluntarily, illegally, and criminality and with malicious intent to
prejudice and defame Mr. Roman Punsalan Serrano and published
a writing which was false, scandalous, malicious, defamatory, and
libelous against the justice of the peace Mr. Roman Punsalan
Serrano.
The Honorable Percy M. Moir found all the defendants, with the
exception of Felix Fernandez, Juan S. Alfonso, Restituto Garcia,
and Manuel Mallari, guilty and sentenced each of them to pay a fine
of P10 and one thirty-second part of the costs, or to suffer subsidiary
imprisonment in case of insolvency.

24
ISSUE/S Whether or not the defendants and appellants are guilty of a libel
of Roman Punsalan, justice of the peace of Macabebe and
Masantol, Province of Pampanga?
RULING/S NO. We find the defendants and appellants entitled to the protection
of the rules concerning qualified privilege, growing out of
constitutional guaranties in our bill of rights.
The guarantees of a free speech and a free press include the right
to criticize judicial conduct. The administration of the law is a matter
of vital public concern. Whether the law is wisely or badly enforced
is, therefore, a t subject for proper comment. If the people cannot
criticize a justice of the peace or a judge the same as any other
public officer, public opinion will be effectively muzzled.
The right to assemble and petition is the necessary consequence of
republican institutions and the complement of the right of free
speech. Assembly means a right on the part of citizens to meet
peaceably for consultation in respect to public affairs. Petition
means that any person or group of persons can apply, without fear
of penalty, to the appropriate branch or oce of the government for a
redress of grievances. The persons assembling and petitioning
must, of course, assume responsibility for the charges made.
Public policy, the welfare of society, and the orderly administration
of government have demanded protection for public opinion. The
inevitable and incontestable result has been the development and
adoption of the doctrine of privilege.
A pertinent illustration of the application of qualified privilege is a
complaint made in good faith and without malice in regard to the
character or conduct of a public official when addressed to an officer
or a board having some interest or duty in the matter. Even when
the statements are found to be false, if there is probable cause for
belief in their truthfulness and the charge is made in good faith, the
mantle of privilege may still cover the mistake of the individual. But
the statements must be made under an honest sense of duty; a self-
seeking motive is destructive. Personal injury is not necessary. All
persons have an interest in the pure and efficient administration of
justice and of public affairs. The duty under which a party is
privileged is sucient if it is social or moral in its nature and this
person in good faith believe he is acting in pursuance thereof
although in fact he is mistaken. The privilege is not defeated by the
mere fact that the communication is made in intemperate terms. A
further element of the law of privilege concerns the person to whom
the complaint should be made. The rule is that if a party applies to
the wrong person through some natural and honest mistake as to
the respective functions of various officials such unintentional error
will not take the case out of the privilege.

25
CONTRIBUTOR DELA CRUZ, Ma. Luzelle P.
MODULE Freedom of Speech
TOPIC
CASE TITLE Guingguing v. People G.R.NO. 128959
PONENTE TINGA, J.: DATE: SEPT. 30, 2005
DOCTRINE No law shall be passed abridging the freedom of speech, of
expression, or of the press, or the right of the people peaceably to
assemble and petition the government for redress of grievances.
FACTS A criminal complaint for libel filed by Cirse "Choy" Torralba against
Lim and Guingguing. Torralba was a broadcast journalist who
handled two programs for radio stations DYLA and DYFX.
On 13 October 1991, Lim caused the publication of records of
criminal cases filed against Torralba as well as photographs of the
latter being arrested. These were published by means of a one-page
advertisement paid for by Lim in the Sunday Post, a weekly
publication edited and published by petitioner. The Sunday Post
was circulated in the province of Bohol, as well as in the Visayas
and Mindanao.
Lim, in his defense, claimed that complainant was allegedly making
scurrilous attacks against him and his family over the airwaves. Lim
then paid advertisements via newspaper to answer the attacks, as
a measure of self-defense. Lim also argued that Torralba, as a
media man and member of the fourth estate, occupied a position
almost similar to a public functionary and should not be onion-
skinned and be able to absorb the thrust of public scrutiny.
Lower court ruled in favor of Torralba.

Guingguing contends inter alia that as editor-publisher of the


Sunday Post and as a member of the fourth estate, the lower courts’
finding of guilt against him constitutes an infringement of his
constitutional right to freedom of speech and of the press.
ISSUE/S WON the publication was libelous.
WON Guingguing was deprived of his constitutional right to freedom
of speech and of the press.
RULING/S In convicting the defendants, the lower courts paid particular heed
to Article 354 of the Revised Penal Code, which provides that "every
defamatory imputation is presumed to be malicious, even if it be
true, if no good intention and justifiable motive for making it is
shown…". SC ruled that if applied to public figures complaining of
criminal libel, must be construed in light of the constitutional
guarantee of free expression, and this Court’s precedents upholding
the standard of actual malice with the necessary implication that a
statement regarding a public figure if true is not libelous. The
provision itself allows for such leeway, accepting as a defense "good
intention and justifiable motive." The exercise of free expression,

26
and its concordant assurance of commentary on public affairs and
public figures, certainly qualify as "justifiable motive," if not "good
intention."
The publication of the subject advertisement by petitioner and Lim
cannot be deemed by this Court to have been done with actual
malice. Aside from the fact that the information contained in said
publication was true, the intention to let the public know the
character of their radio commentator can at best be subsumed
under the mantle of having been done with good motives and for
justifiable ends. The advertisement in question falls squarely within
the bounds of constitutionally protected expression under Section 4,
Article III, and thus, acquittal is mandated.

27
CONTRIBUTOR DIMAKUTA, Jasmine A.
MODULE FREEDOM OF RELIGION
TOPIC
CASE TITLE ALEJANDRO ESTRADA vs. A.M. NO. P-02-1651
SOLEDAD S. ESCRITOR
PONENTE REYES, J. DATE: JUNE 22,
2006
DOCTRINE The free exercise of religion is one of the fundamental rights in our
Constitution and is the most inalienable and sacred of human rights
(Jefferson). The State’s interest in enforcing its prohibition cannot
be merely abstract or symbolic in order to be sufficiently compelling
to outweigh a free exercise claim. In the case at bar, the State has
not evinced any concrete interest in enforcing the concubinage or
bigamy charges against respondent or her partner. Thus the State’s
interest only amounts to the symbolic preservation of an unenforced
prohibition. Furthermore, a distinction between public and secular
morality and religious morality should be kept in mind. The
jurisdiction of the Court extends only to public and secular morality.

FACTS Administrative case against Soledad S. Escritor, a court interpreter


in RTC Las Piñas City. Complainant Alejandro Estrada requested
for an investigation of respondent Soledad Escritor for
living with a man not her husband, and having borne a child within
this live-in arrangement. Respondent Escritor was already a widow
when she entered the judiciary in 1999, her husband having died in
1998.

Respondent Escritor's testified that: She had been living with


Luciano Quilapio, Jr. without the benefit of marriage for more than
20 years while her husband was still alive but living with another
woman. After 10 years of living together, she executed a
"Declaration of Pledging Faithfulness" which allows members of
Jehovah's Witness who have been abandoned by their spouses to
enter into marital relations.

The Declaration makes the resulting union moral and binding within
the congregation. At the time the declarations are executed, the
congregation requires that: (1) the couple cannot secure the civil
authorities’ approval of the marital relationship because of legal
impediments, and (2) couples should be baptized and in good
standing.

The Jehovah’s congregation believes that once all legal


impediments for the couple are lifted, the validity of the declarations
ceases, and the couple should legalize their union. However,

28
although Escritor was widowed in 1998, thereby lifting the legal
impediment to marry on her part, her mate was still not capacitated
to remarry. Thus, their declarations remain valid.

ISSUE/S Should Escritor be penalized for such conjugal arrangement?

RULING/S No. Escritor for she is exercising her right to freedom of religion. The
free exercise of religion is one of the fundamental rights in our
Constitution and is the most inalienable and sacred of human rights
(Jefferson). The State’s interest in enforcing its prohibition cannot
be merely abstract or symbolic in order to be sufficiently compelling
to outweigh a free exercise claim. In the case at bar, the State has
not evinced any concrete interest in enforcing the concubinage or
bigamy charges against respondent or her partner. Thus the State’s
interest only amounts to the symbolic preservation of an unenforced
prohibition. Furthermore, a distinction between public and secular
morality and religious morality should be kept in mind. The
jurisdiction of the Court extends only to public and secular morality.
The PH Constitution adheres to the benevolent neutrality approach
which gives room for accommodation of religious exercises as
required by the Free Exercise Clause. This benevolent neutrality
could allow for accommodation of morality based on religion,
provided it does not offend compelling state interests. Assuming
arguendo that the OSG has proven a compelling state interest, it
has to further demonstrate that the state has used the least intrusive
means possible so that the free exercise is not infringed any more
than necessary to achieve the legitimate goal of the state. Thus the
conjugal arrangement cannot be penalized for it constitutes an
exemption to the law based on her right to freedom of religion.

29
CONTRIBUTOR FLORES, Precious Eureka D.
MODULE No excessive fines, nor cruel, degrading or inhuman punishment
TOPIC
CASE TITLE PEOPLE v. ECHAGARAY G.R.NO. 117472
PONENTE PER CURIAM DATE: FEBRUARY 7, 1997
DOCTRINE Excessive fines shall not be imposed, nor cruel, degrading or
unhuman punishment inflicted. Neither shall death penalty be
imposed, unless, for compelling reasons involving heinous crimes,
the Congress hereafter provides for it. Any death penalty already
imposed shall be reduced to reclusion perpetua.
FACTS On June 25, 1996, the court rendered a decision affirming the
conviction of the accused for the crime of rape. The crime having
been committed during the time when the Republic Act No. 7659,
otherwise known as the “Death Penalty Law” was in effect wherein
the accused was sentenced to be punished of the supreme penalty
of death.
ISSUE/S Whether or not RA No. 7659 is constitutional.
RULING/S Section 19(1) of the 1987 Constitution states:

SECTION 19. (1) Excessive fines shall not be imposed,


nor cruel, degrading or unhuman punishment inflicted.
Neither shall death penalty be imposed, unless, for
compelling reasons involving heinous crimes, the
Congress hereafter provides for it. Any death penalty
already imposed shall be reduced to reclusion
perpetua.

Xxx

The above-mentioned Section of the 1987 Constitution plainly vests


in Congress the power to re-impose the death penalty “for
compelling reasons involving heinous crimes”. This power is no
subsumed in the plenary legislative power of the Congress, for it is subject
to a clear showing of “compelling reasons involving heinous crimes.”

The constitutional exercise of this limited power to re-impose the


death penalty entails (1) that Congress define or describe what is
meant by heinous crimes; (2) that Congress specify and penalize by
death, only crimes that qualify as heinous in accordance with the
definition or description set in the death penalty bill and/or designate
crimes punishable by reclusion perpetua to death in which latter
case, death can only be imposed upon the attendance of
circumstances duly proven in court that characterize the crime to be
heinous in accordance with the definition or description set in the
death penalty bill; and (3) that Congress, in enacting this death

30
penalty bill be singularly motivated by "compelling reasons involving
heinous crimes."

Heinous crime is an act or series of acts which, by the flagrantly


violent manner in which the same was committed or by the reason
of its inherent viciousness, shows a patent disregard and mockery
of the law, public peace and order, or public morals. It is an offense
whose essential and inherent viciousness and atrocity are
repugnant and outrageous to a civilized society and hence, shock
the moral self of a people.

Under RA No. 7659, it was discussed that the death penalty shall
also be imposed if the crime of rape is committed with any of the
following attendant circumstances:

1. When the victim is under 18 years of age and the offender is


a parent, ascendant, step-parent, guardian, relative by
consanguinity or affinity within the third civil degree, or the
common-law spouse of the parent or the victim.

Xxx

"Rape is the forcible violation of the sexual intimacy of another


person. It does injury to justice and charity. Rape deeply wounds the
respect, freedom, and physical and moral integrity to which every
person has a right. It causes grave damage that can mark the victim
for life. It is always an intrinsically evil act xxx an outrage upon
decency and dignity that hurts not only the victim but the society
itself."

We are not unaware that for all the legal posturings we have so
essayed here, at the heart of the issue of capital punishment is the
wistful, sentimental life-and-death question to which all of us,
without thinking, would answer, "life, of course, over death". But
dealing with the fundamental question of death provides a context
for struggling with even more basic questions, for to grapple with the
meaning of death is, in an indirect way, to ask the meaning of life.
Otherwise put, to ask what the rights are of the dying is to ask what
the rights are of the living.

"Capital punishment ought not to be abolished solely because it is


substantially repulsive, if infinitely less repulsive than the acts which
invoke it. Yet the mounting zeal for its abolition seems to arise from
a sentimentalized hyperfastidiousness that seeks to expunge from
the society all that appears harsh and suppressive. If we are to

31
preserve the humane society we will have to retain sufficient
strength of character and will to do the unpleasant in order that
tranquility and civility may rule comprehensively. It seems very likely
that capital punishment is xxx necessary, if limited factor in that
maintenance of social tranquility and ought to be retained on this
ground. To do otherwise is to indulge in the luxury of permitting a
sense of false delicacy to reign over the necessity of social
survival."

32
CASE TITLE Corpuz v. People G.R NO. G.R. No. 180016
PONENTE PERALTA, J DATE: 29 April 2014
CONTRIBUTOR GARCIA, LEXANNE O.

DOCTRINE The primordial duty of the Court is merely to apply the law in such
a way that it shall not usurp legislative powers by judicial
legislation. For acts born out of a case which is not punishable by
law and the court finds it proper to repress, the remedy is to render
the proper decision and thereafter, report to the Chief Executive,
through the Department of Justice (DOJ), the reasons why
the same act should be the subject of penal legislation.
FACT/S
Danilo and the petitioner met at the Admiral Royale Casino in
Olongapo. Petitioner approached Danilo at the same casino
and offered to sell the said pieces of jewelry on commission basis.
Danilo agreed and turned over pieces of jewelry with an aggregate
value of P98,000.00. They both agreed that petitioner shall remit
the proceeds of the sale, and/or, if unsold, to return the same
items, within 60 days. However, the period expired without
petitioner remitting the proceeds of the sale or returning the pieces
of jewelry.
MAIN ISSUE/S
Can the Court revise the penalties on crimes against property on
the ground that they are now cruel and unusual punishment?
RULING ON
MAIN ISSUE/S The Court found the petitioner guilty.
There was misappropriation when petitioner failed to remit
the proceeds of those pieces of jewelry sold, or if no sale took
place, failed to return the same pieces of jewelry within or after
the agreed period despite demand from the private
complainant, to the prejudice of the latter.

However, the Court recognized the perceived injustice brought


about by the range of penalties
that the courts continue to impose on crimes against
property committed today, based on the amount of damage
measured by the value of money eighty years ago in 1932.

But, this Court cannot modify the said range of penalties because
that would constitute judicial legislation. The primordial duty of the
Court is merely to apply the law in such a way that it
shall not usurp legislative powers by judicial legislation. For acts
born out of a case which is not punishable by law and the court find
it proper to repress, the remedy is to render the proper decision

33
and thereafter, report to the Chief Executive, through the
Department of Justice (DOJ), the reasons why the same act
should be the subject of penal legislation.

34
CONTRIBUTOR Manango, John De Divine B.
MODULE Module I.IV.E. Constiutional Limitations of Criminal Law – Non-
TOPIC imprisonment for debt or non-payment of poll tax (Art 3 Sec 19,
1987 Constitution)
CASE TITLE Lozano v. Martinez G.R.NO. G.R. No. L-63419
PONENTE J. Yap DATE: 18 December 1986
DOCTRINE Statute is construed in favor of its constitutionality
FACTS Petitioners assail the validity of BP 22, also known as the Bouncing
Check Law. BP 22 punishes a person “who makes or draws and
issues any check on account for value, knowing at the time of issue
that he does not have sufficient funds in or credit with the drawee
bank..”. It is aimed at putting a stop to the practice of issuing checks
that are worthless which causes injury to the public interest.
Contentions on the law are that: 1) it offends constitutional provision
forbidding imprisonment for debt; 2) it impairs freedom of contract;
3) it contravenes the equal protection clause; 4) it unduly delegates
legislative and executive powers; and 5) its enactment is flawed
because the Interim Batasan violated the prohibition on
amendments in the Third Reading
ISSUE/S Whether or not BP 22 is a valid law
RULING/S The offense punished by BP 22 is the act of making and issuing a
worthless check, not the non-payment of an obligation which the
law punishes. The effects of issuance of a worthless check
transcends the private interests of the parties directly involved in
the transaction and touches the interests of the community at large
since putting valueless commercial papers in circulation can
pollute the channels of trade and commerce, injure the banking
system and eventually hurt the welfare of society and the public
interest. Hence, the enactment of BP 22 is a valid exercise of
police power and is not in conflict with the constitutional inhibition
against imprisonment for debt.
There is no valid ground to sustain the contention the BP 22
impairs freedom of contract since contracts which contravene
public policy are not lawful. The statute does not deny the equal
protection clause since it only penalizes the drawer of the check
and not the payee. Additionally, BP 22 does not constitute an
undue delegation of legislative powers. Contrary to the contention,
the power to define the offense and to prescribe the penalty are
not delegated to the payee. On the last contention, the Interim
Batasan investigated the matter and reported that the clause in
question was an authorized amendment of the bill. With all the
foregoing reasons, the constitutionality of BP 22 is upheld.

35
CONTRIBUTOR MARCELINO, Ferilynn T.
MODULE General Principles of Criminal Law; Constitutional Limitations on
TOPIC Criminal Law – Bill of Attainder
CASE TITLE PEOPLE V. FERRER G.R.NO. 32613-14
PONENTE CASTRO, J: DATE: 27 December 1972
DOCTRINE A bill of attainder is a legislative act which inflicts punishment without
trial. Its essence is the substitution of a legislative for a judicial
determination of guilt. The constitutional ban against bills of
attainder serves to implement the principle of separation of powers
by confining legislatures to rule-making and thereby forestalling
legislative usurpation of the judicial function.
FACTS On 05 March 1970, a criminal complaint for violation of Sec. 4 of the
Anti-Subversion Act was filed against the respondent Feliciano Co
in the Court of First Instance of Tarlac. Respondent was an officer
and/or ranking leader of the Communists Party of the Philippines
(CPP), an outlawed ang illegal organization aimed to overthrow the
government of the Philippines by illegal means for the purpose of
establishing in the Philippines a totalitarian regime. He is an
instructor in the Mao Tse Tung University, the training school of
recruits of the NPA (military arm of CPP).

On 10 March, Judge Jose C. de Guzman conducted a preliminary


investigation, and finding a prima facie case against Co, directed the
government prosecutors to file the corresponding information. Co
moved to quash the information on the ground that the Anti-
Subversion Acts is a bill of attainder.

On 25 May 1970, another criminal complaint was filed with the same
court, charging respondent Nilo Tayag and five others with
subversion for knowingly, willfully and by overt acts organized,
joined and/or remained as officers and/or ranking leaders of the
Kabataang Makabayan (KM), a subversive organization as defined
in RA 1700 (Anti-Subversion Law). On July 21, 1970, Tayag moved
to quash on the grounds that (1) RA 1700 is a bill of attainder; (2) it
is vague; (3) it embraces one subject not expressed in the title
thereof; and (4) it denies him equal protection of the laws.

The trial court is of opinion that (1) Congress usurped the “powers
of the judge” ; (2) assumed “judicial magistracy by pronouncing the
guilt of CCP without any of forms or safeguards of judicial trial” and
(3) created a presumption of organizational guilt which the accused
can never hope to overthrow.

The Government appealed.


ISSUE/S Whether or not the Anti-Subversion Law is a bill of attainder

36
RULING/S The court holds the validity of the Anti- Subversion Act.

Article III, Section 1 (11) of the Constitution states that “No bill of
attainder or ex post facto law shall be enacted.” A bill of attainder is
a legislative act which inflicts punishment without trial. Its essence
is the substitution of a legislative for a judicial determination of guilt.
History in perspective, bills of attainder were employed to suppress
unpopular causes and political minorities and it is against this evil
that the constitutional prohibition is directed. The singling out of a
definite class, the imposition of a burden on it and a legislative intent,
suffice to stigmatize a statute as a bill of attainder.

However, the Act when viewed in its actual operation, it will be seen
that it does not specify the CCP or the members thereof for the
purpose of punishment. In fact, the Act applies not only to the CCP
but also “any other organizations having the same purpose and their
successors.” Its focus is not on individuals but on conduct.

Membership to this organizations, to be unlawful, it must be shown


that membership was acquired with the intent to further the goals of
the organization by overt acts. This is the element of membership
with knowledge that is punishable. This is the required proof of a
member’s direct participation. Why is membership punished.
Membership renders aid and encouragement to the organization.
Membership makes himself party to its unlawful acts.

The court set basic guidelines to be observed in the prosecution


under RA1700. In addition to proving circumstances/evidences of
subversion, the following elements must also be established:

1. Subversive Organizations besides the CPP, it must be proven


that the organization purpose is to overthrow the present
Government of the Philippines and establish a domination of a
FOREIGN POWER. Membership is willfully and knowingly done
by overt acts.

2. In case of CPP, the continued pursuance of its subversive


purpose. Membership is willfully and knowingly done by overt acts.

The court did not make any judgment on the crimes of the accused
under the Act. The Supreme Court set aside the resolution of the
TRIAL COURT.

37
CONTRIBUTOR MAYUGA, Eunice Allaine G.
MODULE Ex Post Facto Laws
TOPIC
CASE TITLE US v. DIAZ CONDE G.R.NO. L-18208
PONENTE JOHNSON, J: DATE: 14 FEB 1922
DOCTRINE Ex post facto laws are prohibited in this jurisdiction unless they are
favorable to the accused. Every law that makes an action, done
before the passage of the law and which was innocent when done,
criminal, and punishes such action, is an ex post facto law.
FACTS On 30 December 1915, Bartolome Oliveros and Engracia Lianco
executed a contract with Vicente Diaz Conde and Apolinaria R. De
Conde, the defendants, evidencing that the former had borrowed
the sum of P300 to the defendants. Under the terms of the contract,
Bartolome Oliveros and Engracia Lianco obligated themselves to
pay the defendants an interest at the rate of 5% per month payable
within the first ten days of every month, the first payment to be made
on the 10 day of January 1916. On 06 May 1916, a complaint was
th

presented charging the defendants with a violation of Usury Law.


The defendants contend that the contract in which the usurious
interest was collected was executed before the Usury Law took
effect which was on 01 May 1916, or four months and a half after
the contract was executed. Hence, there is no Usury Law in force at
the time the contract was made. Moreover, defendants claimed that
the law could have no retroactive effect and that the Usury Law
impairs the obligation of a contract. Thereby for all the said reasons,
the judgment imposed by the lower court should be revoked and the
complaint should be dismissed. However, the lower court stated that
although there was no law in force in the Philippine Islands
punishing usury, but the fact that the defendants had collected a
usurious rate of interest after the adoption of the Usury Law, the
defendants were guilty of violating said law. Hence, this petition.

ISSUE/S Whether or not the defendants are guilty of violating the Usury Law.
RULING/S No. Ex post facto laws, unless they are favorable to the accused,
are prohibited in this jurisdiction. Every law that makes an action,
done before the passage of law, and which was innocent when
done, criminal and punishes such action, is an ex post facto law.
The laws in force in the Philippine Islands prior to any legislation by
the American sovereignty prohibited the legislature from giving to
any penal law a retroactive effect unless such law was favorable to
the accused. Moreover, laws adopted after the execution of a
contract, changing or altering the rate of interest, cannot be made
to apply to such contract without violating the provision of the
constitution which prohibits the adoption of a law impairing the
obligation of the contract.

38
Laws in force at the time the contract was made must govern its
interpretation and application. Laws must be construed
prospectively and not retrospectively. If a contract is legal at its
inception, it cannot be rendered illegal by any subsequent
legislation. Hence, in this case, the acts complained of were legal at
the time of their occurrence and cannot be made criminal by any
subsequent or ex post facto legislation.

39
CONTRIBUTOR PALALA, Amer B.
MODULE General Principles Of Criminal Liability
TOPIC
CASE TITLE People v. Abilong G.R.NO. L-1960
PONENTE MONTEMAYOR, J. DATE: November 26, 1948
DOCTRINE The Revised Penal Code was originally approved and enacted in
Spanish. In case of doubt, the Spanish text governs.
FACTS Florentino Abilong was charged in the Court of First Instance of
Manila with evasion of service of sentence. The said accused,
being then a convict sentenced and ordered to serve two years, four
months and one day of destierro during which he should not enter
any place within the radius of 100 kilometers from the City of Manila,
by virtue of final judgment rendered by the municipal court on April
5, 1946, in criminal case No. B-4795 for attempted robbery, did then
and there willfully, unlawfully and feloniously evade the service of
said sentence by going beyond the limits made against him and
commit vagrancy.

Upon arraignment he pleaded guilty and was sentenced to two


years, four months and one day of prision correccional, with the
accessory penalties of the law and to pay the costs. He is appealing
from that decision with the following assignment of error:

Counsel for the appellant contends that a person like the accused
evading a sentence of destierro is not criminally liable under the
provisions of the Revised Penal Code, particularly article 157 of the
said Code for the reason that said article 157 refers only to persons
who are imprisoned in a penal institution and completely deprived
of their liberty. He bases his contention on the word "imprisonment"
used in the English text of said article which in part reads as follows:

Evasion of service of sentence. — The penalty of prision


correccional in its medium and maximum periods shall be imposed
upon any convict who shall evade service of his sentence by
escaping during the term of his imprisonment by reason of final
judgment.

ISSUE/S Whether or not Abilong is liable for evasion of service of sentence.


RULING/S Yes, appellant Abilong is guilty of evasion of service of sentence
under article 157 of the Revised Penal Code (Spanish text), in that
during the period of his sentence of destierro by virtue of final
judgment wherein he was prohibited from entering the City of
Manila, he entered the said city.

40
It is the Spanish text that is controlling in case of doubt for
the Revised Penal Code was originally approved and enacted in
Spanish, the Spanish text governs.
It is clear that the word "imprisonment" used in the English text is a
wrong or erroneous translation of the phrase "sufriendo privacion de
libertad" used in the Spanish text.
Destierro is a deprivation of liberty, though partial, in the sense that
as in the present case, the appellant by his sentence
of destierro was deprived of the liberty to enter the City of Manila.

41
CONTRIBUTOR PANGAN, Gabrielle L.
MODULE Retroactive application when favorable to the accused
TOPIC
CASE TITLE HERNAN V. SANDIGANBAYAN G.R.NO. 217874
PONENTE PERALTA, J: DATE: DEC. 5, 2017
DOCTRINE For as long as it is favorable to the accused, recent legislation
shall find application regardless of whether its effectivity comes
after the time when the judgment of conviction is rendered and
even if service of sentence has already begun. The accused, in
these applicable instances, shall be entitled to the benefits of the
new law warranting him to serve a lesser sentence, or to his
release, if he has already begun serving his previous sentence,
and said service already accomplishes the term of the modified
sentence.
FACTS Petitioner Ophelia Hernan was a Supervising Fiscal Clerk by virtue
of which she was designated as cashier, disbursement and
collection officer. As such, petitioner received cash and other
collections from customers and clients for the payment of
telegraphic transfers, toll fees, and special message fees. The
collections she received were deposited at the bank account of the
DOTC at the Land Bank of the Philippines (LBP), Baguio City
Branch.

Due to a conducted cash examination of the accounts handled by


petitioner, an auditor of Commission on Audit (COA) discovered
deposit slips that did not bear the stamp of receipt by the LBP. Upon
further investigation, the deposit slip bearing the amount of P11,300
has not been remitted with the LBP. COA filed a complaint of
malversation of public funds against petitioner with the Office of the
Ombudsman which, after due investigation, recommended her
indictment for the loss of P11,300.00. Accordingly, petitioner was
charged before the RTC of Baguio City in an Information.

After trial, the RTC found petitioner guilty beyond reasonable doubt
of the crime charged in the Information. Upon appeal of the
petitioner, Sandiganbayan affirmed the decision of the RTC and
sentenced her to, after applying the Indeterminate Sentence Law, 6
years and 1 day of prision mayor as minimum, to 11 years, 6
months, and 21 days of prision mayor as maximum, together with
the accessory penalties under Article 42 of the Revised Penal Code,
and that interest of only 6% shall be imposed on the amount of
P11,300.00 to be restored by the accused.

Petitioner led the instant petition for certiorari. Petitioner claims,


among others, that during trial, she could not obtain the necessary
evidence for her defense due to the fact that the odds were against

42
her. Thus, the Sandiganbayan's denial of petitioner's motion to
reopen the case is capricious, despotic, and whimsical since the
admission of her additional evidence will prevent a miscarriage.
Because of this, she asks the Court to relax the strict application of
the rules and consider remanding the case to the lower court for
further reception of evidence.
ISSUE/S Whether or not the case shall be reopened and remanded to the
lower court for further reception of evidence
RULING/S The petition lacks merit. Nevertheless, the Court finds that it is still
necessary to reopen the instant case and recall the Entry of
Judgment of the Sandiganbayan, not for further reception of
evidence, however, as petitioner prays for, but in order to modify
the penalty imposed by said court.
The general rule is that a judgment that has acquired finality
becomes immutable and unalterable, and may no longer be
modified in any respect even if the modification is meant to correct
erroneous conclusions of fact or law and whether it will be made
by the court that rendered it or by the highest court of the land.
When, however, circumstances transpire after the finality of the
decision rendering its execution unjust and inequitable, the Court
may sit en banc and give due regard to such exceptional
circumstance warranting the relaxation of the doctrine of
immutability.
To the Court, the recent passage of Republic Act (R.A.) No. 10951
entitled An Act Adjusting the Amount or the Value of Property and
Damage on which a Penalty is Based and the Fines Imposed
Under the Revised Penal Code Amending for the Purpose Act No.
3815 Otherwise Known as the “Revised Penal Code” as Amended,
which accordingly reduced the penalty applicable to the crime
charged herein is an example of such exceptional circumstance.
Pursuant to the provision for malversation of public funds, there is
a novel situation wherein the judgment convicting the accused,
petitioner herein, has already become final and executory and yet
the penalty imposed thereon has been reduced by virtue of the
passage of said law. Because of this, not only must petitioner's
sentence be modified respecting the settled rule on the retroactive
effectivity of laws, the sentencing being favorable to the accused,
she may even apply for probation, as long as she does not
possess any ground for disqualification in view of recent legislation
on probation.
Recent legislation shall find application in cases where the
imposable penalties of the affected crimes such as theft, qualified
theft, estafa, robbery with force upon things, malicious mischief,
malversation, and such other crimes, the penalty of which is
dependent upon the value of the object in consideration thereof,
have been reduced, as in the case at hand, taking into

43
consideration the presence of existing circumstances attending its
commission.
For as long as it is favorable to the accused, said recent legislation
shall find application regardless of whether its effectivity comes
after the time when the judgment of conviction is rendered and
even if service of sentence has already begun. The accused, in
these applicable instances, shall be entitled to the benefits of the
new law warranting him to serve a lesser sentence, or to his
release, if he has already begun serving his previous sentence,
and said service already accomplishes the term of the modified
sentence. In the latter case, moreover, the Court, in the interest of
justice and expediency, further directs the appropriate filing of an
action before the Court that seeks the reopening of the case rather
than an original petition filed for a similar purpose.

44
CONTRIBUTOR PROVIDO, Gemy Hale A.
MODULE Prescribed but underserved penalties
TOPIC
CASE TITLE PEOPLE v. FORMIGONES G.R.NO. L-3246
PONENTE MONTEMAYOR, J: DATE: NOV. 29, 1950
DOCTRINE Under Article 5 of the Revised Penal Code, one can serve lighter
penalties if brought to the attention of the Chief Executive through
his executive clemency or with his discretion
FACTS Abelardo Formigones, together with his family, lives in the house of
his half-brother, Zacarias Formigones, for around a month. He was
a harvester in that place. Filled with jealousy towards his wife and
his brother, he stabbed his wife at the back using his bolo. It resulted
to the death of his wife as the blade penetrated in her right lung.
When the wife was already dead, he carried her inside the house
and together they laid in the living room, with him beside his wife.
He was charged with the crime of parricide, however, he pleaded
not guilty and his counsel argued that he was not in the right mind.
There were two testimonies from the two guards of the jail where he
is serving his sentence that he is not of sound mind and sometimes
act like an insane person would act. Based from his actions, the
counsel pleaded that he is an imbecile and because of that, he is to
be excempt from criminal liability. However, Dr. Francisco Gomez
reported that Formigones is not an imbecile person, but rather a
feebleminded person.
ISSUE/S Whether or not Formigones can use the defense of
feeblemindedness to be exempt from criminal liability
RULING/S No, feeblemindedness is not an exempting circumstance. For one
to be one to be exempted from criminal liability, there must be a
complete deprivation of intelligence or reason which would make
that person not responsible for his acts, such as an imbecile or
insane person. Being feebleminded does not take away one’s ablity
to discern right from wrong, hence, Formigones is held criminally
liable for the prescribed penalties of the crime he has committed.
Since Formigones suffers from feeblemindedness, the mitigating
circumstance of ¨suffering some physical defect which thus restricts
his means of action, defense, or communication with his fellow
beings,¨ or such illness ¨as would diminish the exercise of his will
power¨, either paragraph 8 or 9 of Article 13 of the Revised Penal
Code is to be appreciated in this case. In addition is paragraph 6 of
Article 13 of the Revised Penal Code where one acted with ¨an
impulse so powerful as naturally to have produced passion or
obfuscation.¨
For the penalty to be imposed in the crime, Paragraph 2, Rule 3 of
Article 63 of the Revised Penal Code is to be applied, which
provides that when there are mitigating circumstances and no

45
aggravating circumstance present in the case, the lesser penalty
shall be applied. The case should be brought to the Chief Executive
who has discretion in reducing the penalty of reclusion perpetua to
death or apply executive clemency, whichever he sees fit.

46
CONTRIBUTOR ROMERO, Ma. Camille Concepcion M
MODULE Suppletory application of RPC to Special Penal Laws
TOPIC
CASE TITLE Ladonga v. People G.R.NO. 141066
PONENTE AUSTRIA-MARTINEZ, J. DATE: FEB. 17, 2005
DOCTRINE Lex specialis derogant generali is a principle where special legal
provisions prevail over general ones.
FACTS In 1989 spouses Adronico and petitioner Ladonga were Alfredo
Oculam regular customers in his pawnshop in Tagbilaran City,
Bohol, the spouses availed three consecutive loans guaranteed by
UCPB post dated checks issued by Adronico worth (1) P9,075.55;
(2) P12,730.00; and (3) P8,496.55. Oculam stated that the three
checks bounced for the reason of "CLOSED ACCOUNT"; when the
Ladonga spouses failed to redeem the check, despite repeated
demands, he filed a criminal complaint against them.

Spouses Ladonga claim to have instructed Oculam to not encash


the said checks when they mature as the checks were issued only
to guarantee the obligations and that the petitioner is not a signatory
of the checks and had no participation in the issuance.

Adronico managed to appeal for a probation while the petitioner


averred that she did not conspire with her husband as the principle
of conspiracy is inapplicable to B.P. Blg. 22 which is a special law.
ISSUE/S Whether or not the petitioner could be held liable for violating B.P.
Blg. 22 as conspirator to her husband in the issuance of the three
bouncing checks.
RULING/S No, Ladonga could be held liable for violating B.P. Blg. 22 as a
conspirator.
The petitioner insisted that she cannot be held criminally liable for
violation of B.P. Blg. 22. She contends that the Court of Appeals
gravely erred in applying the principle of conspiracy, as defined
under the RPC, to violations of B.P. Blg. 22. She posits that the
application of the principle of conspiracy would include situations not
intended by the lawmakers, such as penalizing the petitioner, who
had no participation in the drawing or issuance of checks. The Office
of the Solicitor General disagrees with petitioner and echoes the
declaration of the Court of Appeals that some provisions of the RPC,
especially with the addition of the second sentence in Article 10, are
applicable to special laws.
ART. 10. of the RPC provides that, “Offenses not subject to the
provisions of this Code. – Offenses which are or in the future may
be punishable under special laws are not subject to the provisions
of this Code. This Code shall be supplementary to such laws, unless
the latter should specially provide the contrary.”

47
The first clause of the article should be understood to mean only
that the special penal laws are controlling with regard to offenses
therein specifically punished. Said clause only restates the
elemental rule of statutory construction that special legal provisions
prevail over general ones. Lex specialis derogant generali. In fact,
the clause can be considered as a superfluity, and could have been
eliminated altogether. The second clause of the article contains the
soul or main idea and purpose of the article that the "code shall be
supplementary" to special laws, unless the latter should specifically
provide the contrary.
B.P. Blg. 22 does not expressly B.P. Blg. 22 does not expressly
provide prohibition on the applicability in a suppletory character of
the provisions of the RPC. Thus, the general provisions of the RPC
which by their nature, are necessarily applicable, may be applied
suppletorily.
Article 8 of the RPC provides that “a conspiracy exists when two or
more persons come to an agreement concerning the commission of
a felony and decide to commit it.” In the present case the
prosecution failed to prove that Ladonga performed an overt act on
the issuance of the three checks. The overt act or acts may consist
of active participation in the actual commission of the crime itself or
may consist of moral assistance to his co-conspirators by moving
them to execute or implement the criminal plan.

48
CONTRIBUTOR SAMONTE, Vanessa Antoinette
MODULE SUPPLETORY APPLICATION OF REVISED PENAL CODE TO
TOPIC SPECIAL PENAL LAWS
CASE TITLE PEOPLE v. MARTIN SIMON A.M. NO. G.R. NO. 93028
PONENTE REGALADO, J: DATE: JULY 29,1994
DOCTRINE The Revised Penal Code shall have supplementary application to
the special laws whenever the latter use the nomenclature of
penalties in the Code, indicating the intent of Congress to make the
Code apply suppletorily to such special laws.

FACTS Accused appellant Simon was arrested after selling tea bags of
dried marijuana leaves to a Narcotic Command ( NARCOM) poseur-
buyer, during a buy-bust operation. The 4 tea bags confiscated from
appellant were submitted to the Crime Laboratory Service for
examination and were found positive for and had a total weight of
3.8 grams of marijuana. Appellant was convicted for violation of
Section 4, Article II of Republic Act No.6425,or the Dangerous
Drugs Act of 1972 and was sentenced to suffer the penalty of
reclusion perpetua. While appellant was still serving his sentence,
Republic Act No. 6425 was amended by Republic Act No. 7659
which adapted the scale of penalties under the Revised Penal Code.
Accused-appellant prayed for a lesser punishment under the
Revised Penal Code pursuant to Republic Act No. 7659.

ISSUE/S Whether or not the Revised Penal Code can be applied suppletory
to a special law like the Dangerous Drugs Act.

RULING/S YES. Republic Act No. 6425, as amended by Republic Act No. 7659,
unqualifiedly adapted the penalties under the Revised Penal Code
in their technical terms hence, with their technical signification and
effects.
Although the offense is defined in and ostensibly punished under a
special law, the penalty therefore is actually taken from the Revised
Penal Code in its technical nomenclature and, necessarily, with its
duration, correlation and legal effects under the system of penalties
native to said Code.
Thus, the Revised Penal Code shall have supplementary
application to a special law whenever the latter uses the
nomenclature of penalties in the Code, indicating the intent of
Congress to make the Code apply suppletorily to such special law.

49
MODULE 2
GENERAL PRINCIPLES OF
CRIMINAL LIABILITY

50
CONTRIBUTOR SAMSON, Jessa Viena D.
MODULE Elements of criminal liability
TOPIC
CASE TITLE PEOPLE v. SYLVESTRE AND G.R.NO. 35748
ATIENZA
PONENTE VILLA-REAL, J.: DATE: DEC. 14,
1931
DOCTRINE Mere passive presence at the scene of another's crime, mere
silence, and failure to give the alarm without evidence of agreement
or conspiracy do not constitute the cooperation required by Article
14 of the Penal Code for complicity in the commission of the crime
witnessed passively, or with regard to which one has kept silent.
FACTS This is an appeal on the judgment of the Court of First Instance (CFI)
convicting the defendants-appellants Martin Atienza and Romana
Silvestre upon the information of the crime of arson.
Romana Silvestre, wife of Domingo Joaquin by her second
marriage, cohabited with co-defendant Martin Atienza. In view
thereof, Domingo filed a sworn complaint for adultery. After being
arrested and released on bail, the defendants begged to speak to
the complainant, and urge him to withdraw the complaint. The two
accused bound themselves to discontinue cohabitation and
promised not to live again in the barrio of Masocol. The said
promise was signed by Atienza. Consequently, Domingo Joaquin
filed a motion for the dismissal of his complaint which afterwards
was dismissed.
Romana happened to meet her son by her former marriage, Nicolas
de la Cruz, in the barrio of Santo Niño, and under pretext of asking
him for some nipa leaves, followed him home to the village of
Masocol, and remained there. Co-defendant Martin, who continued
to cohabit with Romana, followed her and lived in the home of
Nicolas. Before the incident, Nicolas, his wife Antonia, and the
appellants were gathered after supper. Subsequently, Martin told
the [spouses] to take their furniture out of the house for he was going
to set fire to it. [He] said that it was the only way he could be
revenged upon the people of Masocol who, according to him, had
instigated the charge of adultery against him and Romana. The
former was armed with a pistol. Due to this, spouses went to ask
for help, but were too late. Unfortunately, the fire took about 48
houses. Witnesses saw Martin and Romana leaving the house on
fire.
The CFI convicted Martin and Romana of arson. The former was
convicted as principal by direct participation, whereas the latter was
convicted as accomplice. The court-appointed counsel for the
accused-appellant prays for the affirmance of the CFI’s decision
with regard to Martin, however assigned errors with reference to
Romana.

51
ISSUE/S Whether or not Romana can be convicted as accomplice
RULING/S Article 14 of the Penal Code, corollary with Article 13 of the same
Code, defines an accomplice to be one who does not take a direct
part in the commission of the act, who does not force or induce other
to commit it, nor cooperates in the commission of the act by another
act without which it would not have been accomplished, yet
cooperates in the execution of the act by previous or simultaneous
actions.
The complicity, which is penalized, requires a certain degree of
cooperation, whether moral, through advice, encouragement, or
agreement, or material, through external acts. As regards
Romana’s case, there is no evidence of moral or material
cooperation, and none of an agreement to commit the crime in
question. Her mere presence and silence while they are
simultaneous acts, do not constitute cooperation, for it does not
appear that they encouraged or nerved Martin to commit the crime
of arson; and as for her failure to give the alarm, that being a
subsequent act it does not make her liable as an accomplice.
Hence, mere passive presence at the scene of another's crime,
mere silence, and failure to give the alarm without evidence of
agreement or conspiracy, do not constitute the cooperation required
by Article 14 of the Penal Code for complicity in the commission of
the crime witnessed passively, or with regard to which one has kept
silent.
Wherefore, the decision is affirmed with reference to Martin Atienza,
and reversed with reference to Romana Silvestre, who is acquitted.

52
CONTRIBUTOR TERTE, Karen A.
MODULE Determination of Criminal Responsibility; Accessory
TOPIC
CASE TITLE PEOPLE v. TALINGDAN G.R.NO. L-32126
PONENTE REYES, J: DATE: JULY 6, 1978
DOCTRINE Under Article 19 paragraph 3 of the Revised Penal code, the act of
concealing or assisting in the escape of the principal in the crime
makes a person liable as an accessory. These acts, which
constitutes to their participation, are subsequent to the commission
of the offense. Although an accessory has no direct participation in
the commission of the crime, they have knowledge of the
commission of the crime and have either: profited from the effects;
concealed the body of the crime; or assisted in the escape of the
principals.
FACTS On Saturday, June 24, 1967, Bernardo was shot in his own house.
Nemesio Talingdan, Magellan Tobias, Augusta Berras, Pedro Bides
and Teresa Domogma, the supposed wife of the victim, are the
accused-appellants of this case. Bernardo and Teresa were living
together prior to his death. However, there was no proof of their
marriage and their relationship has been strained for Teresa had
deserted her family home a couple of times and each time Bernardo
took time out to look for her. Accused Talingdan, a policeman, came
armed to the vicinity of Bernardo's house and warned Bernardo that
someday he would kill him.

Corazon, child of the deceased, testified that on Friday, the day


before the crime, she saw her mother meeting with the other
accused and heard one of them say "Could he elude a bullet." When
Teresa noticed the presence of her daughter, she shoved her away
saying "You tell your father that we will kill him". On Saturday,
Corazon was in the kitchen cooking for supper when she saw her
mother with her companions armed with long guns. informed
Bernardo, who was then working on a plow, about the presence of
persons downstairs, but Bernardo paid no attention.
Bernardo proceeded to the kitchen and sat himself on the floor near
the door where he was fired upon from below the stairs of the
batalan. The four accused climbed the stairs of the batalan and upon
seeing that Bernardo was still alive, Talingdan and Tobias fired at
him again. Corazon saw her mother come out of her room and when
she informed Teresa that she recognized the killers, her mother
threatened to kill the latter if she revealed the matter to anyone.
ISSUE/S Whether or not Teresa Domogma is an accessory to the murder of
Bernardo Bagabag.
RULING/S Yes, Teresa Domogma is an accessory to the murder of Bernado.

53
There exists no proof that Teresa Domogma has a direct hand in
the murder of Bernardo nor has she helped in the planning and
preparation of the crime. Although her direct participation in the
conspiracy is not proven beyond reasonable doubt, there is in the
record morally convincing proof that she is at the very least an
accessory to the offense committed. Teresa’s actions of ordering
her daughter to not reveal what she knew; claiming to the peace
officers that she has no suspects in mind; and her passive attitude
towards the conspiracy whilst active in cooperating with the co-
appellants, constitutes "concealing or assisting in the escape of
the principal in the crime."
Teresa Domogma, being an accessory to the crime, is sentenced
to suffer the indeterminate penalty of five years of prision
correccional as minimum to eight years of prision mayor as
maximum.

54
CONTRIBUTOR TIDALGO, Aimee Diane A.
MODULE Deliberate Intent (Dolo), Defined
TOPIC
CASE TITLE MANUEL V. PEOPLE G.R.NO. 165842
PONENTE CALLEJO, SR., J. DATE: NOV. 29, 2005
DOCTRINE For one to be criminally liable for a felony by dolo, there must be a
confluence of both an evil act and an evil intent—actus non facit
reum, nisi mens sit rea
FACTS Eduardo Manuel, 39 years old, was first married to Rubylus Gaña,
who was charged with Estafa, in 1975. Gaña got imprisoned and
was never seen again by Manuel after his last visit. Then, sometime
in January 1996, Manuel met Tina Ganalera, a 21-year old Guest
Relations Officer, in Dagupan City. Because of Manuel’s love for
Ganalera, he proposed marriage on several occasions, assuring her
that he was single. Manuel even brought his parents to Baguio City
to meet Tina’s parents, and was assured by them that their son was
still single.

After three years of their married life, Manuel started making himself
scare and went to their house only twice or thrice a year. Also,
Ganalera was jobless, and whenever she asked money from
Eduardo, he would slap her. Sometime in January 2001, Eduardo
took all his clothes, left, and did not return. Worse, he stopped giving
financial support. Later on, Tina became curious and made inquiries
from the National Statistics Office (NSO) in Manila where she
discovered that Eduardo had been previously married. She secured
an NSO-certified copy of the marriage contract. She was so
embarrassed and humiliated when she learned that Eduardo was in
fact already married when they exchanged their own vows.

The trial court ruled that the elements of bigamy under Article 349
of the Revised Penal Code. Hence, Eduardo appealed the decision
to the CA alleging that he was not criminally liable for bigamy
because when he married the private complainant, he did so in good
faith and without any malicious intent. He maintained that at the time
that he married the private complainant, he was of the honest belief
that his first marriage no longer subsisted. He insisted that
conformably to Article 3 of the Revised Penal Code, there must be
malice for one to be criminally liable for a felony.
ISSUE/S Whether or not Eduardo committed bigamy under Article 349 of the
Revised Penal Code
RULING/S YES, Manuel is criminally liable for bigamy. Art. 349 of RPC states
that “The penalty of prision mayor shall be imposed upon any person
who shall contract a second or subsequent marriage before the
former marriage has been legally dissolved, or before the absent

55
spouse has been declared presumptively dead by means of a
judgment rendered in the proper proceedings.”

The reason why bigamy is considered a felony is to preserve and


ensure the juridical tie of marriage established by law. The phrase
“or before the absent spouse had been declared presumptively
dead by means of a judgment rendered in the proper proceedings”
was incorporated in the Revised Penal Code because the drafters
of the law were of the impression that “in consonance with the civil
law which provides for the presumption of death after an absence of
a number of years, the judicial declaration of presumed death like
annulment of marriage should be a justification for bigamy.”

Although Manuel believed in good faith that the 20-year absence of


his first wife had been dissolved, this did not exculpate him from
being liable. Prosecution was able to prove all the elements of
bigamy. The prosecution proved that the petitioner was married to
Gaña in 1975, and such marriage was not judicially declared a
nullity; hence, the marriage is presumed to subsist. The prosecution
also proved that the petitioner married the private complainant in
1996, long after the effectivity of the Family Code.

The petitioner is presumed to have acted with malice or evil intent


when he married the private complainant. As a general rule, mistake
of fact or good faith of the accused is a valid defense in a
prosecution for a felony by dolo; such defense negates malice or
criminal intent. However, ignorance of the law is not an excuse
because everyone is presumed to know the law. Ignorantia legis
neminem excusat.

It was the burden of the petitioner to prove his defense that when he
married the private complainant in 1996, he was of the well-
grounded belief that his first wife was already dead, as he had not
heard from her for more than 20 years since 1975. He should have
adduced in evidence a decision of a competent court declaring the
presumptive death of his first wife as required by Article 349 of the
Revised Penal Code, in relation to Article 41 of the Family Code.
Such judicial declaration also constitutes proof that the petitioner
acted in good faith, and would negate criminal intent on his part
when he married the private complainant and, as a consequence,
he could not be held guilty of bigamy in such case. The petitioner,
however, failed to discharge his burden.

The above Article of the Family Code now clearly provides that for
the purpose of the present spouse contracting a second marriage,
he or she must file a summary proceeding as provided in the Code

56
for the declaration of the presumptive death of the absentee, without
prejudice to the latter’s reappearance. This provision is intended to
protect the present spouse from a criminal prosecution for bigamy
under Art. 349 of the Revised Penal Code because with the judicial
declaration that the missing spouses presumptively dead, the good
faith of the present spouse in contracting a second marriage is
already established.

57
CONTRIBUTOR Valdez, Ariane Faye V.
MODULE Module 2- Mental element (Mens rea)-General and Specific Intent
TOPIC
CASE TITLE People v. Puno G.R.NO. 97471
PONENTE REGALADO, J. DATE: 17 Feb. 1993
DOCTRINE Proverbial rule of ancient respectability: “That for kidnapping to
exist, there must be indubitable proof that the actual intent of the
malefactors was to deprive the offended party of her liberty.”
FACTS Maria Socorro Mutuc-Sarmiento (victim) owns a bakeshop in
Quezon City. At around 5:00pm of Jan. 13, 1988, the driver of her
husband, Isabelo Puno (accused) arrived at the bakeshop. The
latter told Mrs. Socorro that her own driver Fred had to go to
Pampanga on an emergency, so he will temporary take his place,
as the husband of the victim is in Davao on account of local election
there.

As the victim’s time to go home in Pasig came, she got into the
Mercedes Benz of her husband with Isabelo Puno on the wheel.
After the car turned right in Araneta Avenue, it stopped. A young
man, who was introduced to be the nephew of Isabelo Puno,
Enrique Amurao (accused), boarded the car beside the
driver. Once inside, Enrique clambered on top of the back side of
the front seat and went onto where Ma. Socorro was seated at the
rear. He pokes a gun at her and announced that she wanted to get
money from her. The victim gave her bag containing P7, 000.00
however, the two accused told her that they wanted P100, 000.00
more. As the car sped off north towards the North superhighway.
There Isabelo, asked Ma. Socorro to issue a check for P100.000.00
and the latter complied. Mrs. Socorro drafted three checks in
denomination of two P30, 000.00 and one for P40, 000.00. After
which, Isabelo turned the car around towards Metro Manila. Later,
he changed his mind and turned the car again towards Pampanga.

After which, according to the victim, she jumped out of the car and
fell down on the ground and was injured. She, then crossed to the
other side of the superhighway and was able to flag down a fish
vendor's van. On reaching Balintawak, Ma. Socorro reported the
matter to CAPCOM.

Both accused were, day after, arrested. Enrique was arrested trying
to encash the victim’s P40.000.00 check at PCI Bank in Makati.

On September 26, 1990, the trial which ultimately resulted in a


judgment finding them guilty of robbery with extortion committed on
a highway, punishable under P.D. No. 532. Hence this petition.

58
ISSUE/S 1. Whether or not accused-appellants committed the felony of
kidnapping for ransom under Article 267 of the Revised Penal
Code;
2. Whether or not the said robbery is in violation of Presidential
Decree No. 532 (Anti-Piracy and Anti-Highway Robbery Law
of 1974);
3. Whether or not accused appellants committed the offense of
simple robbery punished by Paragraph 5, Article 294 of the
Revised Penal Code.
RULING/S 1. No. It is an accepted tenet in criminal law that in the
determination of the crime for which the accused should be
held liable in those instances where his acts partake of the
nature of variant offenses, his motive and specific intent in
perpetrating the acts complained of are invaluable aids in
arriving at a correct appreciation and accurate conclusion
thereon. In the case at bar, there is no showing whatsoever
that appellants had any motive, nurtured prior to or at the time
they committed the wrongful acts against complainant, other
than the extortion of money from her under the compulsion
of threats or intimidation. This much is admitted by both
appellants, without any other esoteric qualification or dubious
justification.
With respect to the specific intent of appellants vis-à-vis the
charge that they had kidnapped the victim, we can rely on the
proverbial rule of ancient respectability that for this crime to
exist, there must be indubitable proof that the actual
intent of the malefactors was to deprive the offended
party of her liberty, and not where such restraint of her
freedom of action was merely an incident in the commission
of another offense primarily intended by the offenders. That
appellants in this case had no intention whatsoever to kidnap
or deprive the complainant of her personal liberty is clearly
demonstrated in the veritably confessional testimony of
appellant Puno.
Neither can we consider the amounts given to appellants as
equivalent to or in the nature of ransom.
Ransom, in municipal criminal law, “is the money,
price or consideration paid or demanded for
redemption of a captured person or persons, a
payment that releases from captivity”.
It can hardly be assumed that when complainant readily gave
the cash and checks demanded from her at gunpoint, what
she gave under the circumstances of this case can be
equated with or was in the concept of ransom in the law of

59
kidnapping. Therefore, accused-appellants did not commit
the felony of kidnapping for ransom under Article 267 of the
Revised Penal Code
2. No. Accordingly, while the SC hold that the crime committed
is robbery under Art 293 of the RPC, it rejected the theory that the
same constitutes the highway robbery contemplated in and
punished by Presidential Decree No. 532. The said decree punishes
as highway robbery or brigandage only acts of robbery perpetrated
by outlaws indiscriminately against any person or persons on
Philippine highways as defined therein, and not acts of robbery
committed against only a predetermined or particular victim, is
evident from the preambular clauses thereof, to wit:
“WHEREAS, reports from law-enforcement agencies
reveal that lawless elements are still committing acts
of depredation upon the persons and properties of
innocent and defenseless in habitants who travel
from one place to another, thereby disturbing the
peace, order and tranquility of the nation and stunting
the economic and social progress of the people;
"WHEREAS, such acts of depredations constitute
x x x highway robbery / brigandage which are
among the highest forms of lawlessness
condemned by the penal statutes of all countries:”
Erroneous advertence is nevertheless made by the court
below to the fact that the crime of robbery committed by
appellants should be covered by the said amendatory decree
just because it was committed on a highway. Aside from what
has already been stressed regarding the absence of the
requisite elements which thereby necessarily puts the
offense charged outside the purview and intendment of that
presidential issuance, it would be absurd to adopt a literal
interpretation that any unlawful taking of property committed
on our highways would be covered thereby. It is an
elementary rule of statutory construction that the spirit or
intent of the law should not be subordinated to the letter
thereof.
Therefore, that the coincidental fact that the robbery in the
present case was committed inside a car which, in the natural
course of things, was casually operating on a highway, is not
within the situation envisaged by Section 2(e) of the decree
in its definition of terms. Besides, that particular provision
precisely defines "highway robbery/ brigandage" and, as
we have amply demonstrated, the single act of robbery

60
conceived and committed by appellants in this case does not
constitute highway robbery or brigandage.
3. Yes. It is sufficient that the elements of unlawful taking, with
intent to gain, of personal property through intimidation of the owner
or possessor thereof shall be, as it has been, proved in the case at
bar.
From the facts given, it can be remembered that the bag of
the victim containing cash and the drafting of checks had
been unlawfully taken and made to her from the intimidation
made by the accused with their intent to gain. Intent to gain
(animus lucrandi) is presumed to be alleged in an
information where it is charged that there was unlawful taking
(apoderamiento) and appropriation by the offender of the
things subject of the robbery.
Therefore, the offense committed by appellants is simple
robbery defined in Article 293 and punished under Paragraph
5 of Article 294 of the RPC.

61
CONTRIBUTOR VILLANUEVA, Sean Ruthie L.
MODULE GENERAL PRINCIPLES OF CRIMINAL LIABILITY
TOPIC
CASE TITLE PEOPLE OF THE PHILIPPINES, plaintiff- G.R 142773
appellee, vs. MARLON DELIM, LEON DELIM, NO.
MANUEL DELIM alias "BONG" (At Large),
ROBERT DELIM (At Large), and RONALD
DELIM alias "BONG", accused-appellants.
PONENTE CALLEJO, SR., J DATE: January
28, 2003
DOCTRINE Intent is used to describe a state of mind which exists where
circumstances indicate that an offender actively desired certain
criminal consequences or objectively desired a specific result to
follow his act or failure to act. It is the particular purpose or specific
intention in doing the prohibited act. Intent is not synonymous with
motive. Motive generally is referred to as the reason which prompts
the accused to engage in a particular criminal activity. Motive is not
an essential element of a crime and hence the prosecution need not
prove the same. As a general rule, proof of motive for the
commission of the offense charged does not show guilt and
absence of proof of such motive does not establish the innocence
of accused for the crime charged such as murder.

FACTS On January 23, 1999, at around 6:30 in the evening, Modesto, Rita
and Randy were preparing to have their supper in their home.
Joining them were Modesto and Rita's two young grandchildren,
aged 5 and 7 years old. They were about to eat their dinner when
Marlon, Robert and Ronald suddenly barged into the house and
closed the door. Each of the three intruders was armed with a short
handgun. Marlon poked his gun at Modesto while Robert and
Ronald simultaneously grabbed and hog-tied the victim. A piece of
cloth was placed in the mouth of Modesto. Marlon, Robert and
Ronald herded Modesto out of the house on their way towards the
direction of Paldit, Sison, Pangasinan. Rita and Randy were warned
by the intruders not to leave the house. Leon and Manuel, who were
also armed with short handguns, stayed put by the door to the house
of Modesto and ordered Rita and Randy to stay where they were.
Leon and Manuel left the house of Modesto only at around 7:00 a.m.
the following day, January 24, 1999.
At around 3:00 in the afternoon of January 27, 1999, Randy, in the
company of his relatives, Nida Pucal, Pepito Pucal, Bernard Osias
and Daniel Delim, returned to the housing project in Paldit, Sison,
Pangasinan and this time they found Modesto under thick bushes in
a grassy area. He was already dead. The cadaver was bloated and
in the state of decomposition. It exuded a bad odor. The cadaver

62
was autopsied and the cause of death was a gunshot wound in the
head.

ISSUE/S Whether the crime charged in this case is murder or kidnapping.

RULING/S The crime charged in this case is Murder under Article 248 of the
Revised Penal Code and not Kidnapping under Article 268 thereof.

It bears stressing that in determining what crime is charged in an


information, the material inculpatory facts recited therein describing
the crime charged in relation to the penal law violated are
controlling. Where the specific intent of the malefactor is
determinative of the crime charged such specific intent must be
alleged in the information and proved by the prosecution

If the primary and ultimate purpose of the accused is to kill the


victim, the incidental deprivation of the victim's liberty does not
constitute the felony of kidnapping but is merely a preparatory act to
the killing, and hence, is merged into, or absorbed by, the killing of
the victim. The crime committed would either be homicide or murder.

What is primordial then is the specific intent of the malefactors as


disclosed in the information or criminal complaint that is
determinative of what crime the accused is charged with — that of
murder or kidnapping.

In this case, it is evident on the face of the Information that the


specific intent of the malefactors in barging into the house of
Modesto was to kill him and that he was seized precisely to kill him
with the attendant modifying circumstances. The act of the
malefactors of abducting Modesto was merely incidental to their
primary purpose of killing him. Moreover, there is no specific
allegation in the information that the primary intent of the
malefactors was to deprive Modesto of his freedom or liberty and
that killing him was merely incidental to kidnapping.

63
CONTRIBUTOR AGUILAR, Jose Maria L.
MODULE GENERAL PRINCIPLES OF CRIMINAL LIABILITY
TOPIC (Malum prohibitum as exception to the requirement of mens rea)
CASE TITLE MAGNO v. CA G.R.NO. 96132
PONENTE PARAS, J: DATE: JUNE 26, 1992
DOCTRINE Utilitarian theory, the "protective theory" in criminal law, "affirms that
the primary function of punishment is the protective (sic) of
society against actual and potential wrongdoers."
“Criminal law is founded upon that moral disapprobation of actions
which are immoral, which are detrimental to those conditions upon
which depend the existence and progress of human society.
FACTS Petitioner was in the process of putting up a car repair shop but did
not have complete equipment that could make his venture workable.
Thus, petitioner, representing Ultra Sources International
Corporation, approached Corazon Teng, (private complainant) Vice
President of Mancor Industries (hereinafter referred to as Mancor)
for his needed car repair service equipment of which Mancor was a
distributor. The arrangement went through on condition that
petitioner must put up a warranty deposit equivalent to thirty per
centum (30%) of the total value of the pieces of equipment to be
purchased.

As part of the arrangement, petitioner and LS Finance entered into


a leasing agreement whereby LS Finance would lease the garage
equipments and petitioner would pay the corresponding rent with
the option to buy the same. To replace the first check issued,
petitioner issued another set of six (6) postdated checks. Two (2)
checks dated July 29, 1983 were deposited and cleared while the
four (4) others, which were the subject of the four counts of the
aforestated charges subject of the petition, were held momentarily
by Corazon Teng, on the request of Magno as they were not
covered with sufficient funds. It was then on this occasion that
petitioner became aware that Corazon Teng was the one who
advanced the warranty deposit.

Petitioner with his wife went to see Corazon Teng and promised to
pay the latter but the payment never came and when the four (4)
checks were deposited they were returned for the reason "account
closed.". Consequently, the accused-petitioner was convicted for
violations of BP Blg. 22.
ISSUE/S Whether or not the petitioner violated BP Blg. 22 which is a special
statutory law, violations which are mala prohibita.
RULING/S No. The Supreme Court stated that the transaction did not ripen into
a purchase but remained a lease with rentals being paid for the
loaned equipment, which were pulled out by the Lessor (Mancor)

64
when the petitioner failed to continue paying possibly due to
economic constraints or business failure, then it is lawful and just
that the warranty deposit should not be charged against the
petitioner. The court got intrigued that respondent did not want the
petitioner to know that it was she who "accommodated" petitioner's
request. Thus, it unfolds the kind of transaction that is shrouded with
mystery and doubtful legality. It is in simple language, a scheme
whereby respondent as the supplier of the equipment in the name
of her corporation, Mancor, would be able to "sell or lease" its goods
as in this case, and at the same time, privately financing those who
desperately need petty accommodations as this one. This modus
operandi has in so many instances victimized unsuspecting
businessmen, who likewise need protection from the law, by availing
of the deceptively called "warranty deposit" not realizing that they
also fall prey to leasing equipment under the guise of a lease-
purchase agreement when it is a scheme designed to skim off busin
ess clients.
This maneuvering has serious implications especially with respect
to the threat of the penal sanction of the law in issue, as in this case.
And, with a willing court system to apply the full harshness of the
special law in question, using the "mala prohibitia" doctrine, the
noble objective of the law is tainted with materialism and
opportunism in the highest degree. For all intents and purposes, the
law was devised to safeguard the interest of the banking system and
the legitimate public checking account user. It did not intend to
shelter or favor nor encourage users of the system to enrich
themselves through manipulations and circumvention of the noble
purpose and objective of the law.

65
CONTRIBUTOR ALGURA, Nino N.
MODULE General Principles of Criminal Liability
TOPIC
CASE TITLE ARSENIA B. GARCIA G.R.NO. 157171
vs.
HON. COURT OF APPEALS and the
PEOPLE OF THE PHILIPPINES
PONENTE QUISUMBING, J.: DATE: March 4,
2006
DOCTRINE Mala in se felonies are defined and penalized in the Revised Penal
Code. When the acts complained of are inherently immoral, they are
deemed mala in se, even if they are punished by a special law. On
the other hand, in crimes that are mala prohibita, the criminal acts
are not inherently immoral but become punishable only because the
law says they are forbidden
FACTS This petition seeks the review of the judgment of the Court of
Appeals in CA-G.R. CR No. 24547 that affirmed the conviction of
petitioner by the Regional Trial Court of Alaminos City, Pangasinan,
Branch 54, for violation of Section 27(b) of Republic Act No. 6646
The RTC convicted the herein petitioner for the said crime and
acquitted all other accused for insufficiency of evidence. Petitioner
appealed before the Court of Appeals, which affirmed with
modification the RTC Decision. The Court of Appeals likewise
denied the Motion for Reconsideration. Hence, this appeal before
the appellate court.
ISSUE/S Whether or not, violation of Section 27(b) of Rep. Act No. 6646,
classified under mala in se or mala prohibita? Could good faith and
lack of criminal intent be valid defenses?
RULING/S Petitioner contends that (1) the Court of Appeals’ judgment is
erroneous, based on speculations, surmises and conjectures,
instead of substantial evidence; and (2) there was no motive on her
part to reduce the votes of private complainant.
Respondent on the other hand contends that good faith is not a
defense in the violation of an election law, which falls under the class
of mala prohibita.
The main issue is a violation of Section 27(b) of Rep. Act No. 6646,
classified under mala in se or mala prohibita? Could good faith and
lack of criminal intent be valid defenses?
Generally, mala in se felonies are defined and penalized in the
Revised Penal Code. When the acts complained of are inherently
immoral, they are deemed mala in se, even if they are punished by
a special law. Accordingly, criminal intent must be clearly
established with the other elements of the crime; otherwise, no
crime is committed. On the other hand, in crimes that are mala

66
prohibita, the criminal acts are not inherently immoral but become
punishable only because the law says they are forbidden. With
these crimes, the sole issue is whether the law has been violated.
Criminal intent is not necessary where the acts are prohibited for
reasons of public policy.

67
CONTRIBUTOR AROZA, Maria Minette R.
MODULE General Principles of Criminal Liability – Imprudence or Lack of
TOPIC Skill
CASE TITLE PEOPLE v. PUGAY G.R.NO. L-74324
PONENTE MEDIALDEA, J: DATE: NOV. 17, 1988
DOCTRINE
FACTS The deceased Miranda, a 25-year-old retardate, and the accused
Pugay were friends. In an evening sometime in May, while a town
fiesta fair was being held, Eduardo Gabion was sitting in the ferris
wheel and reading a comic book with his friend Henry. Later, the
accused Pugay and Samson with several companions arrived
appearing to be drunk as they were all happy and noisy. When the
group saw the deceased walking nearby, they started making fun of
him. They made the deceased dance by tickling him with a piece of
wood. Pugay suddenly took a can of gasoline from under the engine
of the ferris wheel and poured its contents on the body of the
deceased. Gabion told Pugay not to do so while latter was already
in the process of pouring the gasoline. Then, the accused Samson
set Miranda on fire making a human torch out of him. Miranda was
brought to a hospital, however, he died due to the incident.
ISSUE/S What are the respective criminal responsibility of the accused-
appellants?
RULING/S In ruling that Pugay is guilty of homicide through reckless
imprudence, according to the Supreme Court, Pugay having taken
the can from under the engine of the ferris wheel and holding it
before pouring its contents on the body of the deceased, he knew
that the can contained gasoline. The stinging smell of this flammable
liquid could not have escaped his notice even before pouring the
same. It was clear that he failed to exercise all the diligence
necessary to avoid every undesirable consequence arising from any
act that may be committed by his companions who at the time were
making fun of the deceased.
With respect to the accused Samson, the Supreme Court ruled that
he is guilty of the crime of homicide with one mitigating circumstance
of no intention to commit so grave a wrong than that committed, as
the qualifying circumstance of treachery was not appreciated
because for such to exist, the attack must be deliberate and the
culprit employed means, methods, or forms in the execution thereof
which tend directly and specially to insure its execution, without risk
to himself arising from any defense which the offended party might
make. In this case, there was no proof in the record that Samson
had some reason to kill the deceased before the incident. On the
contrary, there is adequate evidence showing that his act was
merely a part of their fun-making that evening. However, there is no
doubt that the accused Samson knew very well that the liquid

68
poured on the body of the deceased was gasoline and a flammable
substance for he would not have committed the act of setting the
latter on fire if it were otherwise. Giving him the benefit of doubt, it
can be conceded that as part of their fun-making he merely intended
to set the deceased's clothes on fire. His act, does not relieve him
of criminal responsibility. Burning the clothes of the victim would
cause at the very least some kind of physical injuries on his person,
a felony defined in the Revised Penal Code. If his act resulted into
a graver offense, as what took place in the instant case, he must be
held responsible therefor. Article 4 of the aforesaid code provides,
inter alia that criminal liability shall be incurred by any person
committing a felony (delito) although the wrongful act done be
different from that which he intended.

69
CONTRIBUTOR Belano, Renato Jr. P.
MODULE General Principles of Criminal Liability, Constructive Intent
TOPIC
CASE TITLE Ivler v. San Pedro G.R. NO. 172716
PONENTE Carpio, J. DATE: Nov 17. 2010
DOCTRINE Reckless imprudence under Article 365 is a single quasi-offense by
itself and not merely a means to commit other crimes such that
conviction or acquittal of such quasi-offense bars subsequent
prosecution for the same quasi-offense, regardless of its various
resulting acts.
FACTS Petitioner was charged before the Metropolitan Court of Pasig City
(MeTC) with two separate offenses of Reckless Imprudence
Resulting in Slight Physical Injuries (Criminal Case 82367) for the
injuries sustained by respondent Evangeline L. Ponce and Reckless
Imprudence Resulting in Homicide and Damage to Property
(Criminal Case 82366) for the death of Ponce’s husband and
damage don to the spouses’ vehicle.

In September of 2004, petitioner pleaded guilty to the charge of


Reckless Imprudence Resulting in Slight Physical Injuries and was
meted the penalty of public censure. Invoking this conviction,
petitioner moved to quash the Information in the second criminal
case for placing him in jeopardy of second punishment for the same
offense of reckless imprudence.

The MeTC refused quashal, finding no identity of offenses in the two


cases.

Petitioner elevated the matter to the Regional Trial Court of Pasig in


a petition for certiorari. Meanwhile, petitioner sought from the MeTC
suspension of the proceedings in the second criminal case including
the arraignment on May 17, 2005 invoking his petition for certiorari
as a prejudicial question. MeTC however, did not act on petitioner’s
motion and proceeded with his arraignment and because of his
absence cancelled his bail and ordered his arrest.

Seven days later, the MeTC issued a resolution denying petitioner’s


motion to suspend proceedings and postponing his arraignment
until after his arrest. Petitioner sought reconsideration but as of the
filing of this petition, the motion remained unresolved.
ISSUE/S Whether petitioner’s constitutional right under the Double Jeopardy
Clause bars further proceedings in Reckless Imprudence Resulting
in Homicide and Damage to Property for the death of respondent
Ponce’s husband and damage to the spouses’ vehicle.

70
RULING/S Yes. The two charges against petitioner, arising from the same
facts, were prosecuted under the same provision of the Revised
Penal Code, as amended, namely, Article 365 defining and
penalizing quasi-offenses.

The doctrine that reckless imprudence under Article 365 is a single


quasi-offense by itself and not merely a means to commit other
crimes such that conviction or acquittal of such quasi-offense bars
subsequent prosecution for the same quasi-offense,
regardless of its various resulting acts, undergirded this Court's
unbroken chain of jurisprudence on double jeopardy as applied to
Article 365

These cases uniformly barred the second prosecutions as


constitutionally impermissible under the Double Jeopardy Clause.

Our ruling today secures for the accused facing an Article 365
charge a stronger and simpler protection of their constitutional right
under the Double Jeopardy Clause. True, they are thereby denied
the beneficent effect of the favorable sentencing formula under
Article 48, but any disadvantage thus caused is more than
compensated by the certainty of non-prosecution for quasi-crime
effects qualifying as "light offenses" (or, as here, for the more
serious consequence prosecuted belatedly). If it is so minded,
Congress can re-craft Article 365 by extending to quasi-crimes the
sentencing formula of Article 48 so that only the most severe penalty
shall be imposed under a single prosecution of all resulting acts,
whether penalized as grave, less grave or light offenses. This will
still keep intact the distinct concept of quasi-offenses. Meanwhile,
the lenient schedule of penalties under Article 365, befitting crimes
occupying a lower rung of culpability, should cushion the effect of
this ruling.

71
CONTRIBUTOR BILTZ, Aralind Louise A.
MODULE Aberratio Ictus
TOPIC
CASE TITLE PEOPLE VS. JULIO G.R.NO. L-1477
GUILLEN
PONENTE PER CURIAM DATE: JANUARY 18, 1950
DOCTRINE Criminal liability is incurred by any person committing felony
(delito) although the wrongful act done be different from that which
he intended. In criminal negligence, the injury caused to another
should be unintentional, it being simply the incident of another act
performed without malice. A deliberate intent to do an unlawful act
is essentially inconsistent with the idea of reckless imprudence.
FACTS Julio Guillen became voted for the defeated candidate in the 1946
presidential elections. He became disappointed with the president-
elect Roxas for his alleged failure to fulfill the promises made by him
during his campaign.

On the evening of March 10, 1947, during a popular meeting held


by the Liberal Party at Plaza Miranda, Guillen decided to carry out
his plan to assassinate the president. He was carrying two hand
grenades and buried one in a plant pot located close to the platform.
He then hurled the other grenade at the President when the latter
just closed his speech and about to leave the platform.

General Castaneda, who saw the grenade, kicked it away from the
platform towards an open space where he thought the grenade was
likely to do the least harm and covered the President with his body.
The grenade fell to the ground and exploded in the middle of a group
of persons who were standing close to the platform. This resulted to
a serious injury on Simeon Varela who died the following day and
mortal wound on four other people. Guillen was arrested by the
Police two hours after the incident.
ISSUE/S Whether or not the accused is guilty of murder for the death of
Simeon Varela
RULING/S In throwing hand grenade at the President with the intention of
killing him, the appellant acted with malice. He is therefore liable
for all the consequences of his wrongful act; for in accordance with
article 4 of the Revised Penal Code, criminal liability is incurred by
any person committing felony (delito) although the wrongful act
done be different from that which he intended.

72
CONTRIBUTOR CACHERO, Luis III L.
MODULE Abberatio Ictus or Mistake in the blow
TOPIC
CASE TITLE PEOPLE V. ROLLY ADRIANO Y G.R.NO. 205228
SANTOS, ET. AL.
PONENTE PEREZ, J. DATE: July 15, 2015
DOCTRINE Aberratio Ictus. Criminal liability shall be incurred by any person
committing a felony (delito) although the wrongful act done be
different from that which he intended.
FACTS Two police officers, namely PO1 Matthew Garabiles, and PO2
Alejandro Santos were on their way to Camp Olivas, Pampanga
riding a motorcycle along Olongapo-Gapan National Road. A
speeding blue Toyota Corolla heading towards the same direction
overtook them and the car in front of them, a maroon Honda CR-V.
When the Corolla reached alongside the CR-V, the passenger on
the front seat of the Corolla shot the CR-V and cause the CRV to
swerve and fall in the canal of the road embankment. Four (4)
armed men alighted the Corolla and started shooting at the driver
of the CR-V, who later was identified as Danilo Cabiedes. During
the shooting, a bystander named Ofelia Bulanan, who was
standing near the road embankment, was hit by a stray bullet. Both
Cabiedes and Bulanan died from fatal gunshot wounds. The
Accused Rolly Adriano was then charged with two (2) counts of
Murder. The RTC then convicted Adriano for the charges filed
against him.

ISSUE/S Whether or not Rolly Adriano should be held liable for the death of
the bystander Bulanan, even though he had no intention of killing
her.
RULING/S YES. Logically, Bulanan’s death was random and unintentional
and the method used to kill her, and she was killed by a stray
bullet, was, by no means, deliberate. Nonetheless, Adriano is
guilty of the death of Bulanan under Article 4 of the Revised Penal
Code, pursuant to the doctrine of aberratio ictus, which imposes
criminal liability for the acts committed in violation of law and for all
the natural and logical consequences resulting therefrom. The
Supreme Court affirmed the decision of the lower court.

73
CONTRIBUTOR CALZADO, Anne Valerie L.
MODULE Elements of Criminal Liability, Mental Element, Transferred Intent,
TOPIC Error in personae (Art. 4, par. 1, RPC)
CASE TITLE PEOPLE V. SABALONES G.R.NO. 123485
PONENTE PANGANIBAN, J. DATE: AUG 31, 1998
DOCTRINE Where the case involves the killing of persons other than the
intended victims, the same is better characterized as error in
personae or mistake in the identity of the victims, rather than
aberratio ictus which means mistake in the blow, characterized by
aiming at one but hitting the other due to imprecision in the blow.

Mistake in the identity of the victim carries the same gravity as when
the accused zeroes in on his intended victim.
FACTS Accused-appellants, Rolusape Sabalones, Artemio Timoteo
Beronga, Teodulo Alegarbes and Eufemio Cabanero, were
convicted of two counts of murder and three counts of frustrated
murder. The conviction arose from a shooting incident on June 1,
1985, at around 11:45pm, in Talisay Cebu, which resulted in the
killing of two persons - Glenn Tiempo, Alfredo Nardo – and the
wounding of three others - Rey Bolo, Rogelio Presores and Nelson
Tiempo.

The victims were riding two vehicles, a jeep and a car, when the
appellants allegedly ambushed them as they were approaching the
house of Stephen Lim.

Previously, the rival of Sabalones, Nabing Velez, got into an


altercation with his father in a cockpit, and was killed in a separate
incident. The RTC ruled that the attack by the group of Sabalones
was the effect of their assumption that the people of Nabing Velez
will retaliate. Unfortunately, the victims of the incident had nothing
to do with the case.
ISSUE/S Whether or not there was aberratio ictus in the present case as
appellants allege
RULING/S NO. The trial court only relied on the concept of aberration ictus to
explain why the appellants staged the ambush, not to prove that
appellants did in fact commit the crimes. Assuming that the trial
court did err in explaining the motive of the appellants, this does not
detract from its findings, that the guilt of the appellants was proven
beyond reasonable doubt.

The conclusion of the courts that the appellants killed the wrong
persons was based on the extrajudicial statement of Appellant
Boronga and a testimony of Jennifer Binghoy. The appellants
believed that they were suspected of killing a Nabing Velez, and that

74
they expected his group to retaliate against them. Hence, upon
arrival of the victims’ vehicles, which they mistook to be carrying the
retaliating men of Nabing Velez, they opened fire. Nonetheless, the
fact that they were mistaken does not diminish their culpability. The
court held that mistake in the identity of the victim carries the same
gravity as when the accused zeroes in on his intended victim.

The case, observed by the OSG, is better characterized as error in


personae or mistake in the identity of the victim, rather than
aberratio ictus which means mistake in the blow, characterized by
aiming at one but hitting the other due to imprecision in the blow.

75
CONTRIBUTOR CARPIO, Anna Clarissa C.
MODULE Module 2: General Principles of Criminal Liability
TOPIC
CASE TITLE PEOPLE v. ALBUQUERQUE G.R.NO. L-38773
PONENTE AVANCEÑA, C.J. DATE: December 19,
1933
DOCTRINE Article 49 of RPC which refers to cases where the crime is
committed is different from that intended is only a reproduction of
Article 64 of the old Code and is only applicable in cases where the
crime befalls a different person (abberatio ictus).
FACTS Defendant-appellant Gines Alburqueque is a 55-year-old widower,
a father of 9 children, and has since been suffering from partial
paralysis, walking dragging one leg and has lost control of the
movement of his right arm. As such, he has been unable to work
since the stroke of his paralysis. Two of his daughters, one of which
named Maria, are married while the other is a nun. With the
exception of one of the married daughters and the nun, all of the
daughters, including defendant Gines, live with Maria who supports
them financially. Among the daughters living with Maria, one named
Pilar became acquainted and had intimate relations with deceased-
victim Manuel Osma at the end of 1928 who frequently visited their
house and it was also then when defendant Gines come to know
Osma. Soon after, the relationship between Pilar and Osma bore
them a child which defendant Gines was unaware of. As such,
patriarch Gines had to be deceived with the information that Pilar
had gone to her godfather’s house in Singalong when in fact, she
had been taken to the hospital for delivery. Gines only learned the
truth when Pilar returned home with her child.

Naturally, Gines was deeply affect by the deception and appeared


sad and worried not only because of the dishonor it brought upon
his family but also because the child meant an added burden to
Maria upon whom they all depended for support. Thereafter, the
attitude of Giles swings from writing letters, at times he was hostile
and threatening; and at times, entreating the deceased Osma to
marry Pilar or at least support her and the child financially. Although
Osma agreed to provide financial support by way of monthly
allowance, he never complied with the same.

One day, he appeared in Osma’s office in such a mood and asked


for the manager to speak with the former. Both of them went
downstairs and the events thereafter was unknown to anyone but
the undisputed fact is that on that occasion, defendant Gines
inflicted a wound at the base of Osma’s neck which caused the
latter’s death. The court replied on defendant’s testimony that he
asked the deceased to marry his daughter but the latter refused and

76
then he whipped out his penknife. The deceased tried to seized him
by the neck whereupon defendant Gines stabbed him on the face
but due to his lack of movement of his right arm, the weapon landed
on the base of Osma’s neck.

The defendant claims self-defense but the RTC convicted him of the
crime of homicide. Further, the RTC found that Gines did not intend
to cause so grave an injury as Osma’s death. Thus, was sentenced
to suffer the penalty of prision mayor (8 years and 1 day) and to
indemnify Osma’s heirs.
ISSUE/S Whether or not defendant Gines properly invoked praeter
intentionem?
RULING/S YES. The Supreme Court sustained the ruling of the trial court that
defendant Giles did not intend to cause so grave an injury as to the
death of Osma as supported by the evidence.

In his testimony, defendant Giles emphatically affirmed that he only


wanted to inflict a wound that would leave a permanent scar on
Osma’s face or one that would hospitalize him for a week or two but
never intended to kill him because such would frustrate his plan of
compelling him to marry or at least financially support his daughter.
Giles’ intention was stated in some of his letters to the deceased by
way a threat to induce the latter to accept his proposal that would
benefit Pilar. The act of the defendant in stabbing Osma which
resulted in his death was solely because of the fact that Giles did
not have control of his right arm because of his paralysis; thus, the
blow, although intended for the face, landed at the base of the neck.

The Court appreciated the mitigating circumstances of lack of


intention to cause so grave an injury (praeter intentionem), voluntary
surrender to authorities, and acted under the influence of passion
and obfuscation; but discarded the claim for self-defense as Giles
provoked and commenced the aggression by whipping out and
brandishing his penknife.

However, the contention of the defense that Article 49 of RPC which


refers to cases where the crime is committed is different from that
intended should be applied to the case at bar is untenable. Article
49 is only a reproduction of Article 64 of the old Code and is only
applicable in cases where the crime befalls a different person
(abberatio ictus).

With the presense of 3 mitigating circumstances without any


aggravating circumstance, the penalty next lower in degree (from
reclusion temporal to prision mayor) should be imposed.

77
CONTRIBUTOR CHUA, Kristy Anne
MODULE GENERAL PRINCIPLES OF CRIMINAL LIABILITY
TOPIC (Causation)
CASE TITLE BATACLAN V. MEDINA G.R.NO. L-10126
PONENTE MONTEMAYOR, J.: DATE: OCT. 22, 1957
DOCTRINE The proximate legal cause is acting first and producing the injury,
either immediately or by setting other events in motion, all
constituting a natural and continuous chain of events, each having
a close causal connection with its immediate predecessor, the final
event in the chain immediately affecting the injury as a natural and
probable result of the cause which first acted, under such
circumstances that the person responsible for the first event should,
as an ordinary prudent and intelligent person, have reasonable
ground to expect at the moment of his act or default that an injury to
some person might probably result therefrom.
FACTS On September 13, 1952 bus no. 30 of the Medina Transportation
owned by Mariano Medina shortly after midnight, left the town of
Amadeo, Cavite, on its way to Pasay City, driven by Conrado, the
chauffeur. Including the driver, Bataclan(the deceased) and the
conductor, there were eighteen passengers in total. During the ride,
the bus was in Imus, Cavite when one of the front tires burst and the
vehicle began to zig-zag until it fell into a canal or ditch on the right
side of the road and turned turtle. The driver, conductor, and some
passengers were able to escape the overturned bus except for
Bataclan and the other three passengers. The four passengers
stuck in the bus called for help, ten men arrived after half an hour,
one of them had a lighted torch. The gasoline began leaking from
the gasoline tank on the side of the chassis when the bus was
overturned, spreading over, under and around the bus, the lighted
torch bought by one of the men set the bus by accident on fire.
Those ten men received no prior warning from the driver or
conductor that the gasoline was leaking, with the fire that engulfed
the bus, it resulted in the death of Bataclan and the three other
passengers trapped in the bus.
ISSUE/S Whether or not the proximate cause of the death of Juan Bataclan
and the other three passengers was burning by reason of the lighted
torch which ignited the gasoline from the leaking overturned bus.
RULING/S No, in this case, the proximate cause held by the Supreme Court
was the overturning of the bus because the bus turned completely
on its back not just on its side, the leaking of the gasoline was
unexpected. Since it was dark, the lighted torch used around 2:30
in the early morning was necessary since in rural areas, lanterns
and flashlights were unavailable. The rescuers are not to blame and
they were innocently approaching the bus in order to provide aid
and rescue to the trapped passengers, therefore the man bringing

78
a lighted torch was a natural sequence of the overturned bus, the
trapped passengers and the call for help. The negligence of the
carrier was attributed by the burning of the bus, through the driver
and conductor. Negligence on the part of the agents of the carrier is
explained in the codal provisions of Article 1733, 1759 and 1763. An
injured passenger even overheard Mariano Medina speaking to one
of his bus inspectors to change the tires of the bus since it is already
old and that he has been telling the driver to change the tires but the
driver did not follow instructions.

79
CONTRIBUTOR CRUZ, Jakielyn Anne O.
MODULE MODULE 2: Elements of criminal liability – Causation
TOPIC
CASE TITLE PEOPLE v. ILIGAN G.R.NO. 75369
PONENTE FERNAN, C.J: DATE: NOV. 26, 1990
DOCTRINE The essential requisites of Article 4 are: (a) that an intentional felony
has been committed, and (b) that the wrong done to the aggrieved
party be the direct, natural and logical consequence of the felony
committed by the offender.
Proximate legal cause is defined as "that acting first and producing
the injury, either immediately or by setting other events in motion,
all constituting a natural and continuous chain of events, each
having a close causal connection with its immediate predecessor,
the final event in the chain immediately effecting the injury as a
natural and probable result of the cause which first acted, under
such circumstances that the person responsible for the first event
should, as an ordinarily prudent and intelligent person, have
reasonable ground to expect at the moment of his act or default that
an injury to some person might probably result therefrom.
FACTS The victim Esmeraldo Quiñones, Jr. and his companions, Zaldy
Asis and Felix Lukban were on their way home after attending the
barrio fiesta dance when they met the group of the accused
Fernando Iligan, his nephew, Edmundo Asis, and Juan Macandog
in front of the ricemill. Edmundo Asis pushed them aside thereby
prompting Zaldy Asis to box him. Felix Lukban quickly told the group
of the accused that they had no desire to fight but Fernando Iligan
drew his bolo and hacked Zaldy Asis but missed. The group of
Quñones, Jr. ran as they were being chased by the group of the
accused, they only stopped when they were no longer being
chased.
The group of Quñones, Jr. were on their way to his house to change
to their work clothes when the group of the accused appeared from
nowhere and hacked Quiñones, Jr. with a bolo hitting him in his
forehead which caused him to fall down. Quiñones, Jr. companions
fled as they were terrified of what happened. The companions
returned to the crime scene and saw Quiñones, Jr. dead with his
head busted. The body of Quiñones was examined, and the
autopsy report shows that he died of "shock and massive cerebral
hemorrhages due to a vehicular accident."
The accused denied the accusations and claimed that they were in
their respective houses during the incident. Fernando Iligan claimed
that he left his house at midnight to fetch some visitors from the

80
dance hall and invited his drunk nephew Edmundo Asis to
accompany him to the dance hall. However, they were unable to
reach their destination because Edmundo was boxed by somebody,
hence he decided to bring home his nephew.
The RTC ruled that Iligan's group conspired to kill anyone or all
members of the group of the victim to vindicate the boxing on the
face of Edmundo Asis.
ISSUE/S Whether or not Fernando Iligan is criminally liable for the death of
Esmeraldo Quiñones, Jr. given that it was due to vehicular accident
and not the hacking incident?
RULING/S YES. We are convinced beyond peradventure that indeed, after
Quiñones, Jr. had fallen from the bolohacking perpetrated by Iligan,
he was run over by a vehicle. This 􀀾nding, however, does not in
any way exonerate Iligan from liability for the death of Quiñones, Jr.
Under Article 4 of the Revised Penal Code, criminal liability shall be
incurred "by any person committing a felony (delito) although the
wrongful act done be different from that which he intended." The
essential requisites of Article 4 are: (a) that an intentional felony has
been committed, and (b) that the wrong done to the aggrieved party
be the direct, natural and logical consequence of the felony
committed by the offender. We hold that these requisites are
present in this case.
The intentional felony committed was the hacking of the head of
Quiñones, Jr. by Iligan. What is material is that by the instrument
used in hacking Quiñones, Jr. and the location of the wound, the
assault was meant not only to immobilize the victim but to do away
with him as it was directed at a vital and delicate part of the body:
the head.
Under these circumstances, we hold that while Iligan's hacking of
Quiñones, Jr.'s head might not have been the direct cause, it was
the proximate cause of the latter's death. In other words, the
sequence of events from Iligan's assault on him to the time
Quiñones, Jr. was run over by a vehicle is, considering the very
short span of time between them, one unbroken chain of events.
Having triggered such events, Iligan cannot escape liability.

81
CONTRIBUTOR DELA CRUZ, Ma. Luzelle P.
MODULE Mens rea Causation
TOPIC
CASE TITLE URBANO v. PEOPLE OF THE G.R.NO. 72964
PHILIPPINES
PONENTE GUTIERREZ, JR., J.: DATE: JAN. 07,
1988
DOCTRINE Mens rea Causation:
Criminal liability shall be incurred: (1) By any person committing a
felony (delito) although the wrongful act done be different from that
which he intended ..."
an accused is criminally responsible for acts committed by him in
violation of law and for all the natural and logical consequences
resulting therefrom
FACTS Filomeno Urbano went to his ricefield located at about 100 meters
from the tobacco seedbed of Marcelo Javier. He found his palay
flooded with water coming from the irrigation canal nearby which
had overflowed. Urbano went to the elevated portion of the canal to
see what happened and there he saw Marcelo Javier and Emilio
Erfe cutting grass. He asked them who was responsible for the
opening of the irrigation canal and Javier admitted that he was the
one. Urbano then got angry and demanded that Javier pay for his
soaked palay. A quarrel between them ensued. Urbano unsheathed
his bolo (about 2 feet long, including the handle, by 2 inches wide)
and hacked Javier hitting him on the right palm of his hand, which
was used in parrying the bolo hack. Javier who was then unarmed
ran away from Urbano but was overtaken by Urbano who hacked
him again hitting Javier on the left leg with the back portion of said
bolo, causing a swelling on said leg. When Urbano tried to hack and
inflict further injury, his daughter embraced and prevented him from
hacking Javier.

Antonio Erfe, Emilio Erfe, and Felipe Erfe brought Javier to Dr.
Guillermo Padilla, rural health physician of San Fabian, who did not
attend to Javier but instead suggested that they go to Dr. Mario
Meneses because Padilla had no available medicine.

After Javier was treated by Dr. Meneses, he and his companions


returned to Dr. Guillermo Padilla who conducted a medico-legal
examination.
At about 1:30 a.m. on November 14, 1980, Javier was rushed to the
Nazareth General Hospital in a very serious condition. When
admitted to the hospital, Javier had lockjaw and was having
convulsions. Dr. Edmundo Exconde who personally attended to
Javier found that the latter's serious condition was caused by

82
tetanus toxin. He noticed the presence of a healing wound in
Javier's palm which could have been infected by tetanus.

On November 15, 1980 at exactly 4:18 p.m., Javier died in the


hospital.

In an information dated April 10, 1981, Filomeno Urbano was


charged with the crime of homicide before the then Circuit Criminal
Court of Dagupan City, Third Judicial District.

Upon arraignment, Urbano pleaded "not guilty." After trial, the trial
court found Urbano guilty as charged.
ISSUE/S WON there was an efficient intervening cause from the time Javier
was wounded until his death which would exculpate Urbano from
any liability for Javier's death.(WON Urbano is criminally liable for
death of Javier.)
RULING/S The incubation period of tetanus, i.e., the time between injury and
the appearance of unmistakable symptoms, ranges from 2 to 56
days. However, over 80 percent of patients become symptomatic
within 14 days. A short incubation period indicates severe disease,
and when symptoms occur within 2 or 3 days of injury the mortality
rate approaches 100 percent.
Mild tetanus is characterized by an incubation period of at least 14
days and an onset time of more than 6 days. Trismus is usually
present, but dysphagia is absent and generalized spasms are brief
and mild.

If, therefore, the wound of Javier inflicted by the appellant was


already infected by tetanus germs at the time, it is more medically
probable that Javier should have been infected with only a mild
cause of tetanus because the symptoms of tetanus appeared on the
22nd day after the hacking incident or more than 14 days after the
infliction of the wound. Therefore, the onset time should have been
more than six days. Javier, however, died on the second day from
the onset time. The more credible conclusion is that at the time
Javier's wound was inflicted by the appellant, the severe form of
tetanus that killed him was not yet present. Consequently, Javier's
wound could have been infected with tetanus after the hacking
incident. Considering the circumstance surrounding Javier's death,
his wound could have been infected by tetanus 2 or 3 or a few but
not 20 to 22 days before he died.

The rule is that the death of the victim must be the direct, natural,
and logical consequence of the wounds inflicted upon him by the
accused. (People v. Cardenas, supra) And since we are dealing with
a criminal conviction, the proof that the accused caused the victim's

83
death must convince a rational mind beyond reasonable doubt. The
medical findings, however, lead us to a distinct possibility that the
infection of the wound by tetanus was an efficient intervening cause
later or between the time Javier was wounded to the time of his
death. The infection was, therefore, distinct and foreign to the crime.
(People v. Rellin, 77 Phil. 1038).
ACQUITTED

84
CONTRIBUTOR DIMAKUTA, Jasmine A.
MODULE IMPOSSIBLE CRIMES
TOPIC
CASE TITLE SULPICIO INTOD V. HONORABLE G.R. 103119
COURT OF APPEALS and PEOPLE NO.
OF THE PHILIPPINES
PONENTE CAMPOS, JR. DATE: OCTOBER 21,
1992
DOCTRINE Legal impossibility occurs where the intended acts, even if
completed, would not amount to a crime. Thus: Legal impossibility
would apply to those circumstances where (1) the motive, desire
and expectation is to perform an act in violation of the law; (2) there
is intention to perform the physical act; (3) there is a performance of
the intended physical act; and (4) the consequence resulting from
the intended act does not amount to a crime. The impossibility of
killing a person already dead falls in this category.

FACTS In the morning of February 4, 1979, Sulpicio Intod, Jorge Pangasian,


Santos Tubio and Avelino Daligdig went to Salvador Mandaya's
house in Katugasan, Lopez Jaena, Misamis Occidental and asked
him to go with them to the house of Bernardina Palangpangan.
Thereafter, Mandaya and Intod, Pangasian, Tubio and Daligdig had
a meeting with Aniceto Dumalagan. He told Mandaya that he
wanted Palangpangan to be killed because of a land dispute
between them and that Mandaya should accompany the 4 men,
otherwise, he would also be killed.

At about 10:00 pm of the same day, Petitioner, Mandaya,


Pangasian, Tubio and Daligdig, all armed with firearms, arrived at
Palangpangan's house in Katugasan, Lopez Jaena, Misamis
Occidental. At the instance of his companions, Mandaya pointed the
location of Palangpangan's bedroom. Thereafter, Petitioner,
Pangasian, Tubio and Daligdig fired at said room. It turned out,
however, that Palangpangan was in another City and her home was
then occupied by her son-in-law and his family. No one was in the
room when the accused fired the shots. No one was hit by the gun
fire. Petitioner and his companions were positively identified by
witnesses. One witness testified that before the 5 men left the
premises, they shouted: "We will kill you (the witness) and especially
Bernardina Palangpangan and we will come back if you were not
injured".

Petitioner contends that, Palangpangan's absence from her room


on the night he and his companions riddled it with bullets made the
crime inherently impossible. On the other hand, Respondent People
of the Philippines argues that the crime was not impossible. Instead,

85
the facts were sufficient to constitute an attempt and to convict Intod
for attempted murder. Respondent alleged that there was intent.
Further, in its Comment to the Petition, respondent pointed out that:
The crime of murder was not consummated, not because of the
inherent impossibility of its accomplishment (Art. 4(2), Revised
Penal Code), but due to a cause or accident other than petitioner's
and his accused's own spontaneous desistance. Palangpangan did
not sleep at her house at that time. Had it not been for this fact, the
crime is possible, not impossible.

ISSUE/S Whether or not the RTC and the CA erred in their decision for
convicting the herein petitioner of attempted murder

RULING/S Yes, the RTC and the CA erred in their decision for convicting the
herein petitioner of attempted murder. The rationale of Article 4(2)
is to punish such criminal tendencies. Under this article, the act
performed by the offender cannot produce an offense against
person or property because: (1) the commission of the offense is
inherently impossible of accomplishment: or (2) the means
employed is either (a) inadequate or (b) ineffectual.

That the offense cannot be produced because the commission of


the offense is inherently impossible of accomplishment is the focus
of this petition. To be impossible under this clause, the act intended
by the offender must be by its nature one impossible of
accomplishment. There must be either impossibility of
accomplishing the intended act in order to qualify the act an
impossible crime.

Legal impossibility occurs where the intended acts, even if


completed, would not amount to a crime. Thus: Legal impossibility
would apply to those circumstances where (1) the motive, desire
and expectation is to perform an act in violation of the law; (2) there
is intention to perform the physical act; (3) there is a performance of
the intended physical act; and (4) the consequence resulting from
the intended act does not amount to a crime. The impossibility of
killing a person already dead falls in this category.

On the other hand, factual impossibility occurs when extraneous


circumstances unknown to the actor or beyond his control prevent
the consummation of the intended crime. One example is the man
who puts his hand in the coat pocket of another with the intention to
steal the latter's wallet and finds the pocket empty. The case at bar
belongs to this category. Petitioner shoots the place where he
thought his victim would be, although in reality, the victim was not
present in said place and thus, the petitioner failed to accomplish
his end.

86
In the Philippines, the Revised Penal Code, in Article 4(2), expressly
provided for impossible crimes and made them punishable.
Whereas, in the United States, the Code of Crimes and Criminal
Procedure is silent regarding this matter. What it provided for were
attempts of the crimes enumerated in the said Code. Furthermore,
in said jurisdiction, the impossibility of committing the offense is
merely a defense to an attempt charge. In this regard,
commentators and the cases generally divide the impossibility
defense into two categories: legal versus factual impossibility. In
U.S. vs. Wilson, the Court held that: Factual impossibility of the
commission of the crime is not a defense. If the crime could have
been committed had the circumstances been as the defendant
believed them to be, it is no defense that in reality the crime was
impossible of commission.

In our jurisdiction, impossible crimes are recognized. The


impossibility of accomplishing the criminal intent is not merely a
defense, but an act penalized by itself. Furthermore, the phrase
"inherent impossibility" that is found in Article 4(2) of the Revised
Penal Code makes no distinction between factual or physical
impossibility and legal impossibility. Ubi lex non distinguit nec nos
distinguere debemos. The factual situation in the case at bar
present a physical impossibility which rendered the intended crime
impossible of accomplishment. And under Article 4, paragraph 2 of
the Revised Penal Code, such is sufficient to make the act an
impossible crime.

To uphold the contention of respondent that the offense was


Attempted Murder because the absence of Palangpangan was a
supervening cause independent of the actor's will, will render
useless the provision in Article 4, which makes a person criminally
liable for an act "which would be an offense against persons or
property, were it not for the inherent impossibility of its
accomplishment . . ." In that case all circumstances which prevented
the consummation of the offense will be treated as an accident
independent of the actor's will which is an element of attempted and
frustrated felonies.

87
CONTRIBUTOR FLORES, Precious Eureka D.
MODULE General Principles of Criminal Liability: Impossible Crimes
TOPIC
CASE TITLE PEOPLE v. SALADINO G.R.NO. L-3634
PONENTE Not mentioned DATE: MAY 30, 1951
DOCTRINE Concealment is not only wrong but also unlawful, a person is not
exempt from criminal liability, even though he acted in obedience to
a command from his superior, because such command was illegal
and in conflict with the law and justice. Therefore it cannot be
alleged that obedience was due, or that it exempts the defendant
from criminal liability.
FACTS On June 23, 1948, Corporal Bartolo Saladino and Private Anastacia
Alejo of the Philippine Constabulary were resting in the house of
Celso Abucay in Paoay, Ilocos Norte, together with policemen
Melchor Quevedo, Wilfredo Osman and George Plan. They had
gone on patrol duty to the barrio for the purpose of apprehending
those who on a previous night had fired upon the dwelling. About
midnight they were suddenly awakened by cries of help. They went
down and was approached by Felix Pasion who reported he had
been robbed, one of the robbers being Luis Bernabe.

The next morning, Saladino and lejo, accompanied by the


policemen proceeded to the house of Luis Bernabe . Having found
the suspect, they brought him, for questioning, to the residence of
Felix Pasion. Saladino led Bernabe up to the house for further
investigation and was followed by Alejo and the other policemen.
However, Bernabe denied the charge. To extract a confession,
Saladino repeatedly boxed and kicked the victim in different parts of
the body but still Bernabe continued to deny his guilt. Saladino got
a piece of wood, two (2) inch thick and one (1) yard long, and
clubbed the victim several times on the chest, abdomen and the
back.

Saladino called Alejo to take his turn. Alejo whipped Bernabe four
(4) times with the branch of a tree, and retired to the kitchen.
Saladino once again questioned the victim but Bernabe won’t admit
his guilt. Saladino repeated the severe beating and tying Bernabe’s
wrists together with a rubber strap, tied the strap to a beam in the
ceiling which result to Bernabe hanging in the air. Saladino,
cudgeled Bernabe with the wooden club, on the sides, armpits,
stomach, hips and back. Policeman Plan interceded and told
Saladino to stop and just bring the victim to the headquarters for
further interrogation, however he was ignored and
Saladino resumed the maltreatment, predicting that Bernabe would
confess. Saladino untied the victim and made him sit on a chair. Not
long after, Saladino kicked the chair and pounced on Bernabe and

88
booted him several times until he became motionless. When they
confirmed that he is dead, Saladino ordered two (2) civilians to carry
the victim down and told Alejo, to shoot him and they will say that
he ran away. Alejo complied and shot Bernabe four (4) times.
ISSUE/S Whether or not the accused shall incur criminal liability?
RULING/S It is plain that Bernabe died as a consequence of a violent mauling
by Saladino, the latter must be declared guilty of assassination.
Anastacia Alejo does not conspire with him, and is not liable either
as principal or as an accomplice of the murder. But he is guilty as
an accessory after the fact or having performed acts tending to
conceal Saladino’s crime by making it appear that Bernabe had run
away. Applying the jurisprudence in US v. Cuison, which states:

“But we do find criminal liability in the acts performed


by Corporal Cuison, even though he obeyed orders
from his Lieutenant, Poggi; such liability consists in his
having intervened subsequently to the commission of
the crime, by furnishing the means to make it appear
that the deceased was armed and that it was
necessary to kill him on account of his resistance to
the constabulary man, who, to lend color to such
pretended resistance, discharged their firearms into
the air, under the direction of Cuison, at the place
there where the corpse was lying; and also consists in
his having tried to find a dagger to place beside the
deceased. Such acts must be characterized as
concealment, and since they are not only wrong but
also unlawful, the defendant is not exempt from
liability, even though he acted in obedience to a
command from his superior, because such command
was illegal and in conflict with law and justice.
Therefore it can not be alleged that obedience was
due, or that it exempts the defendant from criminal
liability.”

Therefore, Saladino is guilty of a assassination while Alejo is guilty


of being an accessory to the crime.

89
CASE TITLE Jacinto v. People G.R NO. G.R. No.
162540
PONENTE PERALTA, J. DATE: 13 July 2009
CONTRIBUTOR GARCIA, LEXANNE O.
DOCTRINE
To be impossible under this clause, the act intended by the must
be by its nature one impossible of accomplishment. There must
be either legal impossibility, or physical impossibility of
accomplishing the intended act in order to qualify the act as an
impossible crime. Legal impossibility occurs where the intended
acts, even if completed, would not amount to a crime.

FACT/S
Baby Aquino handed petitioner a Banco De Oro (BDO) Check in
the amount of P10,000.00. The check was payment for Baby
Aquino’s purchases from Mega Foam Int’l. and petitioner was then
the collector of Mega Foam.

However, the check was deposited in the Land Bank account of


Generoso Capitle, the husband of Jacqueline Capitle - the latter is
the sister of petitioner and the former pricing, merchandising and
inventory clerk of Mega Foam. Such check however was
dishonored by the bank.

Company records showed that petitioner never remitted the


subject check to Mega Foam. However, Baby Aquino contended
that she had already paid Mega Foam P10,000.00 cash in August
1997 as replacement for the dishonored check.

MAIN ISSUE/S
Was the crime of theft produced? Was the petitioner guilty of theft
or an impossible crime?

RULING ON
MAIN ISSUE/S The petitioner is guilty of an impossible crime. The personal
property subject of the theft must have some value, as the
intention of the accused is to gain from the thing stolen.

On the other hand, to be an impossible crime, the act intended by


the offender must be by its nature one impossible of
accomplishment. There must be either legal impossibility, or
physical impossibility of accomplishing the intended act in order to
qualify the act as an impossible crime. Legal impossibility occurs
where the intended acts, even if completed, would not amount to
a crime.

90
In this case, petitioner unlawfully took the postdated check
belonging to Mega Foam, but the same was apparently without
value, as it was subsequently dishonored.

This case is closely akin to the factual impossibility given in Intod.


Petitioner performed all the acts to consummate the crime of
qualified theft, which is a crime against property. Petitioner’s evil
intent cannot be denied as the mere act of unlawfully taking the
check meant for Mega Foam showed her intent to gain or be
unjustly enriched. Were it not for the fact that the check bounced
she would have received the face value thereof, which was not
rightfully hers.

Therefore, it was only due to the extraneous circumstance of the


check being unfunded, a fact unknown to petitioner at the time
that prevented the crime from being produced. The thing
unlawfully taken by petitioner turned out to be absolutely
worthless, because the check was eventually dishonored, and
Mega Foam had received the cash to replace the value of said
dishonored check. Thus, petitioner is found guilty of an
impossible crime.

91
CONTRIBUTOR ALGURA, Nino N.
MODULE General Principles of Criminal Liability
TOPIC
CASE TITLE THE UNITED STATES G.R.NO. L-5272
v.
AH CHONG
PONENTE CARSON, J.: DATE: March 19, 1910
DOCTRINE Since evil intent is in general an inseparable element in every crime,
any such mistake of fact as shows the act committed to have
proceeded from no sort of evil in the mind necessarily relieves the
actor from criminal liability, provided always there is no fault or
negligence on his part.
FACTS The defendant, Ah Chong, was employed as a cook at "Officers'
quarters, No. 27," Fort Mc Kinley, Rizal Province, and at the same
place Pascual Gualberto, deceased, was employed as a house boy
or muchacho. "Officers' quarters No. 27" as a detached house
situates some 40 meters from the nearest building, and in August,
19087, was occupied solely as an officers' mess or club. No one
slept in the house except the two servants, who jointly occupied a
small room toward the rear of the building, the door of which opened
upon a narrow porch running along the side of the building, by which
communication was had with the other part of the house room.

On the night of August 14, 1908, at about 10 o'clock, the defendant,


who had received for the night, was suddenly awakened by some
trying to force open the door of the room. He sat up in bed and called
out twice, "Who is there?" He heard no answer and was convinced
by the noise at the door that it was being pushed open by someone
bent upon forcing his way into the room.

The deceased and the accused, who roomed together and who
appear to have on friendly and amicable terms prior to the fatal
incident, had an understanding that when either returned at night,
he should knock at the door and acquiant his companion with his
identity. Pascual had left the house early in the evening and gone
for a walk with his friends, Celestino Quiambao and Mariano Ibañez,
servants employed at officers' quarters No. 28, the nearest house to
the mess hall. The three returned from their walk at about 10 o'clock,
and Celestino and Mariano stopped at their room at No. 28, Pascual
going on to his room at No. 27. A few moments after the party
separated, Celestino and Mariano heard cries for assistance and
upon returning to No. 27 found Pascual sitting on the back steps
fatally wounded in the stomach, whereupon one of them ran back to
No. 28 and called Liuetenants Jacobs and Healy, who immediately
went to the aid of the wounded man.

92
ISSUE/S Whether or not in this jurisdiction one can be held criminally
responsible who, by reason of a mistake as to the facts, does an act
for which he would be exempt from criminal liability if the facts were
as he... supposed them to be, but which would constitute the crime
of homicide or assassination if the actor had known the true state
of the facts at the time when he committed the act.
RULING/S In such circumstances there is no criminal liability, the alleged
ignorance or mistake of fact was not due to negligence or bad faith.
In broader terms, ignorance or mistake of fact, if such ignorance or
mistake of fact is sufficient to negative a particular intent which
under the law is a necessary ingredient of the offense charged...
cancels the presumption of intent," and works an acquittal
Article 1 of the Penal Code is as follows:
Any person voluntarily committing a crime or misdemeanor shall
incur criminal liability, even though the wrongful act committed be
different from that which he had intended to commit.
a voluntary act is a free, intelligent, and intentional act, and roundly
asserts that without intention (intention to do wrong or...
criminal intention) there can be no crime;
Since evil intent is in general an inseparable element in every crime,
any such mistake of fact as shows the act committed to have
proceeded from no sort of evil in the mind necessarily relieves the
actor from criminal liability, provided always there is no fault or
negligence on his part;
A careful examination of the facts as disclosed in the case at bar
convinces us that the defendant Chinaman struck the fatal blow
alleged in the information in the firm belief that the intruder who
forced open the door of his sleeping room was a thief, from whose...
assault he was in imminent peril, both of his life and of his property
and of the property committed to his charge; that in view of all the
circumstances, as they must have presented themselves to the
defendant at the time, he acted in... good faith, without malice, or
criminal intent, in the belief that he was doing no more than
exercising his legitimate right of self-defense; that had the facts
been as he believed them to be he would have been wholly exempt
from criminal liability on account of his ... act; and that he cannot be
said to have been guilty of negligence or recklessness or even
carelessness in falling into his mistake as to the facts, or in the
means adopted by him to defend himself from the imminent danger
which he believed threatened his person and his property and the
property under his charge.

93
CONTRIBUTOR AROZA, Maria Minette R.
MODULE General Principles of Criminal Liability – Mistake of Fact
TOPIC
CASE TITLE PEOPLE v. OANIS G.R.NO. 47722
PONENTE MORAN, J DATE: JUL 27, 1943
DOCTRINE In criminal negligence, the injury caused to another should be
unintentional, it being simply the incident of another act performed
without malice.
FACTS The Provincial Inspector at Cabanatuan, Nueva Ecija, received a
telegram: "Information received escaped convict Anselmo Balagtas
with bailarina Irene in Cabanatuan. Get him dead or alive." The
Defendants were instructed to arrest Balagtas and, if overpowered,
to follow the instruction in the telegram. When their group arrived at
supposedly Irene's house, Oanis approached one Brigida Mallare,
who was then stripping banana stalks, and asked her where Irene's
room was. Brigida indicated the place and upon further inquiry also
said that Irene was sleeping with her paramour. Defendants Oanis
and Galanta then went to the room of Irene, and seeing a man
sleeping with his back towards the door where they were,
simultaneously or successively fired at him with their .32 and .45
caliber revolvers. Awakened by the gunshots, Irene saw her
paramour already wounded, and looking at the door where the shots
came, she saw the defendants still firing at him, it was later on found
that the person shot was not the notorious criminal Anselmo
Balagtas but one Serapio Tecson, Irene's paramour. The lower
court found the defendants guilty of homicide through reckless
imprudence taking into consideration the fact that Oanis and
Galanta acted in innocent mistake of fact in the honest performance
of their official duties, both of them believing that Tecson was
Balagtas.
ISSUE/S Whether or not the defendants may be held responsible for the
death of Tecson.
RULING/S The Supreme Court held that the defendants are criminally liable for
murder, not homicide through reckless imprudence with qualifying
circumstance of alevosia. Although an officer in making a lawful
arrest is justified in using such force as is reasonably necessary to
secure and detain the offender, he is never justified in using
unnecessary force or in resorting to dangerous means when the
arrest could be effected otherwise. It may be true that Anselmo
Balagtas was a notorious criminal, a life-termer, but these facts
alone constitute no justification for killing him when in effecting his
arrest, he offers no resistance or in fact no resistance can be
offered, as when he is asleep. The crime committed by them is not
merely criminal negligence, the killing being intentional and not
accidental. In criminal negligence, the injury caused to another

94
should be unintentional, it being simply the incident of another act
performed without malice. As the deceased was killed while asleep,
the crime committed is murder with the qualifying circumstance of
alevosia. There is, however, a mitigating circumstance of weight
consisting in the incomplete justifying circumstance defined in article
11, No. 5, of the Revised Penal Code: a person incurs no criminal
liability when he acts in the fulfillment of a duty or in the lawful
exercise of a right or office. There are two requisites in order that
the circumstance may be taken as a justifying one, in the case at
bar, only the first requisite is present, that the offender acted in the
performance of a duty or in the lawful exercise of a right.

95
CONTRIBUTOR Belano, Renato Jr. P.
MODULE General Principles of Criminal Liability
TOPIC
CASE TITLE Padilla v. Dizon A.C. NO. 3086
PONENTE Per Curiam DATE: Feb 23,1988
DOCTRINE Proof of malice or deliberate intent (mens rea) is not essential in
offenses punished by special laws, which are mala prohibita.
FACTS This is an administrative complaint, dated August 6, 1987, filed by
the then Commissioner of Customs, Alexander Padilla, against
respondent Baltazar R. Dizon, RTC Judge, Branch 115, Pasay City,
for rendering a manifestly erroneous decision due, at the very least,
to gross incompetence and gross ignorance of the law, in Criminal
Case No. 86-10126-P, entitled "People of the Philippines vs. Lo Chi
Fai", acquitting said accused of the offense charged, i.e., smuggling
of foreign currency out of the country.

The respondent rendered a decision of acquitting Lo Chi Fai, who


was caught by a customs guard and two PAFSECOM officers on
July 9, 1986, while on board Flight PR 300 of the Philippine Air Lines
bound for Hongkong while attempting to smuggle foreign currency
and foreign exchange instruments out of the country amounting to
USD 355,349.57, in various currency and denominations.

The accused claimed that he was a businessman from Hongkong


engaged in the trade of garments. That his reason for coming to the
Philippines was to invest in business inside the country and also
play in the casino. He also testified that his business associates
started putting their money into a common fund, hence every time
anyone of them came to the Philippines, they would declare the
money they were bringing in, and all declarations were handed to
and kept by him. He also testified that his reason for going back to
Hongkong and bringing with him all the money intended for investing
in the Philippines was because of fear of his group that the
revolution taking place in Manila might become widespread.

The respondent judge decided to acquit the accused and allowed


the release of USD 3,000.00 based on the presented evidence
which, according to him, showed that the accused had no willful
intention to violate the law.
ISSUE/S Whether or not the respondent judge is guilty of gross incompetence
or gross ignorance of the law in deciding that criminal intent should
be established in order to penalize the accused for violating Section
6 of Central Bank Circular No. 960
RULING/S Yes. The respondent judge has shown gross incompetence or
gross ignorance of the law in holding that to convict the accused

96
for violation of Central Bank Circular No. 960, the prosecution must
establish that the accused had the criminal intent to violate the law.
The respondent ought to know that proof of malice or
deliberate intent (mens rea) is not essential in offenses
punished by special laws, which are mala prohibita.

It did not matter to the respondent that the foreign currency and
foreign currency instruments found in the possession of the accused
when he was apprehended at the airport ---- 380 pieces in all---- and
the amounts of such foreign exchange did not correspond to the
foreign currency declarations presented by the accused at the trial.
It did not matter to the respondent that the accused by his own story
admitted, in effect, that he was a "carrier" of foreign currency for
other people. The respondent closed his eyes to the fact that the
very substantial amounts of foreign exchange found in the
possession of the accused at the time of his apprehension consisted
of personal checks of other people, as well as cash in various
currency denominations (12 kinds of currency in all), which clearly
belied the claim of the accused that they were part of the funds
which he and his supposed associates had brought in and kept in
the Philippines for the purpose of investing in some business
ventures. The respondent ignored the fact that most of the CB
Currency declarations presented by the defense at the trial were
declarations belonging to other people which could not be utilized
by the accused to justify his having the foreign exchange in his
possession.

These and other circumstances which make the story concocted by


the accused so palpably unbelievable as to render the findings of
the respondent judge obviously contrived to favor the acquittal of the
accused, thereby clearly negating his claim that he rendered the
decision "in good faith."

In invoking the provisions of CB Circular No. 960 to justify the


release of USD 3,000.00 to the accused, the respondent judge
again displayed gross incompetence and gross ignorance of the
law. There is nothing in the said CB Circular which could be taken
as authority for the trial court to release the said amount of U.S.
Currency to the accused.

97
MODULE 3
STAGES OF EXECUTION

98
CONTRIBUTOR Manango, John De Divine B.
MODULE Module 3: Stages of Execution
TOPIC
CASE TITLE US v. Eduave, G.R.NO. G.R. No. 12155
PONENTE J. Moreland DATE: 2 February 1917
DOCTRINE Subjective phase of a crime, difference between attempted and
frustrated felony
FACTS Defendant Protasio Eduave is the querido of the victim’s mother.
Eduave attacked the victim from behind using a bolo creating a gash
8 1/2 inches long and 2 inches deep because the latter accused
defendant of having commiting rape against said victim. Upon
thinking that he has already killed the victim, he threw the body into
the bushes and left.
ISSUE/S What is the crime committed by Eduave?
RULING/S Accused is guilty of frustrated murder. The fact that Eduave
attacked the victim from behind, in a vital portion of the body,
shows treachery qualifying it as murder. The crime was not
consummated because the elements of the crime’s execution and
accomplishment were not complete as the victim did not die.
Neither was the crime an attempted one because the accused’s
actions has already passed the subjective phase, that is, there was
no external force preventing defendant from performing all the acts
of execution necessary to commit the felony. Consequently, the
victim did not die because an external element has prevented such
death after Eduave has performed all the necessary acts of
execution that would have caused the death of the victim.

99
CONTRIBUTOR MARCELINO, Ferilynn T.
MODULE Stages of Execution – Consummated Crimes
TOPIC
CASE TITLE RIVERA v. PEOPLE G.R.NO. 166326
PONENTE CALLEJO, SR., J: DATE: 25 January 2006
DOCTRINE An essential element of murder and homicide, whether in their
consummated, frustrated or attempted stage, is intent of the
offenders to kill the victim immediately before or simultaneously with
the infliction of injuries. Intent to kill is a specific intent which the
prosecution must prove by direct or circumstantial evidence, while
general criminal intent is presumed from the commission of a felony
by dolo.
FACTS As the victim, Ruben Rodil, went to a nearby store to buy food,
accused Edgardo Rivera mocked him for being jobless and
dependent on his wife for support. Ruben resented the rebuke and
thereafter, a heated exchange of words ensued. In the evening of
the following day, when Ruben and his three-year-old daughter went
to the store to buy food, Edgardo, together with his brother
Esmeraldo Rivera and Ismael Rivera, emerged from their house and
ganged up on him. Esmeraldo and Ismael mauled Ruben with fist
blows. And as he fell to the ground, Edgardo hit him three times with
a hollow block on the parietal area. Esmeraldo, Ismael and Edgardo
fled to their house only when the policemen arrived. Ruben
sustained injuries and was brought to the hospital. The doctor
declared that the wounds were slight and superficial, though the
victim could have been killed had the police not promptly intervened.

The trial court found the accused guilty of the crime of frustrated
murder. An appeal was made by the accused, but the Court of
Appeals affirmed the trial court’s decision with modification,
changing the crime to attempted murder and imposed an
indeterminate penalty of 2 years of prision correccional as minimum
to 6 years and 1 day of prision mayor as maximum.
ISSUE/S 1) Whether or not there was intent to kill.
2) Whether or not the Court of Appeals was correct in modifying the
crime from frustrated to attempted murder.
RULING/S 1) YES. The Court declared that evidence to prove intent to kill in
crimes against persons may consist, inter alia, in the means used
by the malefactors, the nature, location and number of wounds
sustained by the victim, the conduct of the malefactors before, at
the time, or immediately after the killing of the victim, the
circumstances under which the crime was committed and the
motives of the accused. In the present case, Esmeraldo and Ismael
pummeled the victim with fist blows, while Edgardo hit him three
times with a hollow block. Even though the wounds sustained by the

100
victim were merely superficial and could not have produced his
death, intent to kill was presumed.

2) YES. Article 6 of the Revised Penal Code provides that there is


an attempt when the offender commences the commission of a
felony directly by overt acts, and does not perform all the acts of
execution which should produce the felony by reason of some cause
or accident other than his own spontaneous desistance. Although
the wounds sustained by the victim were merely superficial and
could not have produced his death, it does not negate criminal
liability of the accused for attempted murder. The intent to kill was
already presumed based on the overt acts of the accused. In fact,
victim could have been killed had the police not promptly intervened.

101
CONTRIBUTOR MAYUGA, Eunice Allaine G.
MODULE Stages of Execution: Attempted Stage, Frustrated Stage, and
TOPIC Consummated Stage
CASE TITLE BALEROS v. PEOPLE G.R.NO. 138033
PONENTE GARCIA, J: DATE: 22 FEB 2006
DOCTRINE Overt or external act has been defined as some physical activity or
deed, indicating the intention to commit a particular crime, more
than a mere planning or preparation, which if carried out to its
complete termination following its natural course, without being
frustrated by external obstacles nor by the voluntary desistance of
the perpetrator, will logically and necessarily ripen into a concrete
offense.

FACTS One evening, Malou went to her bedroom and retired at 10:30PM.
Outside, right in front of her bedroom door is her maid, Marvilou,
who is sleeping on a folding bed. Early morning of the following day,
Malou was awakened by the smell of a chemical on a piece of cloth
that is pressed on her face. She struggled, fighting the attacker until
her right hand got free and she was able to grab hold of the
attacker’s sex organ which she then squeezed. The man let her go
Malou rushed to the bedroom door and woke up Marvilou. Malou
told Ferolin over the intercom “May pumasok sa kwarto ko
pinagtangkaan ako”. Malou didn’t know who attacked her at first but
eventually, she learned that it was Renato D. Baleros, Jr. alias
“Chito” who was courting her but she rejected.

The trial court rendered a decision convicting the petitioner of the


crime of attempted rape. Aggrieved, the petitioner went to CA but
CA affirmed the decision of the lower court. Petitioner moved for
reconsideration but his motion was denied hence this petition.
ISSUE/S Whether or not the CA erred in affirming the ruling of the RTC finding
the petitioner guilty beyond reasonable doubt of the crime of
attempted rape.
RULING/S Yes. Article 6 of the RPC, in relation to Article 335 of the RPC, a
felony is attempted when the offender commences the commission
of the act directly by overt acts and does not perform all the acts of
execution which should produce the felony by reason of some cause
or accident other than his own spontaneous desistance.

The court ruled that it would be too strained to construe the


petitioner's act of pressing a chemical-soaked cloth in the mouth of
Malou which would induce her to sleep as an overt act that will
logically and necessarily ripen into rape. As it were, petitioner did
not commence at all the performance of any act indicative of an
intent or attempt to rape Malou. It cannot be overemphasized that
petitioner was fully clothed and that there was no attempt on his part

102
to undress Malou, let alone touch her private part. For what reason
petitioner wanted the complainant unconscious, if that was really his
immediate intention, is anybody’s guess.

In Perez vs. Court of Appeals, the Court held that:


“xxx. In the crime of rape, penetration is an essential act of execution
to produce the felony. Thus, for there to be an attempted rape, the
accused must have commenced the act of penetrating his sexual
organ to the vagina of the victim but for some cause or accident
other than his own spontaneous desistance, the penetration,
however, slight, is not completed. Petitioner’s act of lying on top of
the complainant, embracing and kissing her, mashing her breasts,
inserting his hand inside her panty, and touching her sexual organ,
while admittedly obscene and detestable acts, do not constitute
attempted rape absent any showing that petitioner actually
commenced to force his penis into the complainant’s sexual organ.
xxx”
Therefore, the acts of Renato Baleros do not determine attempted
rape but rather constitute unjust vexation punishable as light
coercion as provided under the second paragraph of Article 287 of
the RPC.

103
CONTRIBUTOR PALALA, Amer B.
MODULE Stages Of Execution
TOPIC
CASE TITLE Valenzuela v. People G.R.NO. 160188
PONENTE TINGA, J. DATE: June 21, 2007
DOCTRINE Article 308 of the Revised Penal Code, theft cannot have a
frustrated stage. Theft can only be attempted (no unlawful taking)
or consummated (there is unlawful taking).
FACTS Aristotel Valenzuela and Jovy Calderon were charged with the crime
of theft. On May 19, 1997, Valenzuela and Calderon were seen
outside of Super Sale Club in SM by Lorenzo Lago, a security guard
of SM. Lago saw Valenzuela hauling a push cart full of cases of Tide
detergent. Valenzuela unloaded these cases in an open parking
space, where Calderon was waiting. He returned inside the
supermarket, and after several minutes, emerged with more cartons
of Tide Ultramatic and again unloaded these boxes to the same area
in the open parking space. He then hailed a taxi and loaded the
cases of Tide inside the taxi. Lago stopped the taxi and asked
Valenzuela and Calderon for the receipt. The two reacted by fleeing
on foot, but Lago fired a warning shot to alert his fellow security
guards of the incident. They were eventually apprehended. The
stolen items seized from the duo were four cases of Tide Ultramatic,
one case of Ultra 25 grams, and three additional cases of detergent,
the goods with total value of ₱12,090.00.

The two were first brought to the SM security office before they were
transferred on the same day to the Baler Station II of the Philippine
National Police, Quezon City, for investigation. Eventually, the RTC
convicted them of the crime of consummated theft. It was affirmed
by the CA. However, Valenzuela (only) argued that he should only
be convicted of frustrated theft since at the time he was
apprehended, he was never placed in a position to freely dispose of
the articles stolen. Valenzuela cites two decisions rendered many
years ago by the Court of Appeals: People v. Diño and People v.
Flores. Both decisions elicit the interest of this Court, as they
modified trial court convictions from consummated to frustrated
theft.
ISSUE/S Whether or not Valenzuela should be guilty of consummated theft.
RULING/S Yes, Valenzuela should be guilty of consummated theft. In this case,
the Supreme Court discussed the three stages of crime.

Article 6 defines those three stages, namely the


consummated, frustrated and attempted felonies.

104
• Consummated - when all the elements necessary for
its execution and accomplishment are present.
• Frustrated - when the offender performs all the acts of
execution which would produce the felony as a
consequence but which, nevertheless, do not produce
it by reason of causes independent of the will of the
perpetrator.
• Attempted - when the offender commences the
commission of a felony directly by overt acts, and does
not perform all the acts of execution which should
produce the felony by reason of some cause or
accident other than his own spontaneous desistance.
Each felony under the Revised Penal Code has a:
• Subjective phase - portion of the acts constituting the
crime included between the act which begins the
commission of the crime and the last act performed by the
offender which, with prior acts, should result in the
consummated crime

NOTE: If the offender never passes the subjective phase of


the offense, the crime is merely attempted

• Objective phase - After that point of subjective phase


has been breached

NOTE: subjective phase is completely passed in case of


frustrated crimes

The Court says that the critical distinction between frustrated and
consummated is whether the felony itself was actually produces by
the acts of the offender. In this case, the crime committed is crime
of theft, and the accused is arguing that he should only be convicted
of frustrated crime since he was not given an opportunity to return
the stolen articles.

The Court said that "unlawful taking" is most material in this respect.
Unlawful taking, which is the deprivation of one’s personal property,
is the element which produces the felony in its consummated stage.
At the same time, without unlawful taking as an act of execution, the
offense could only be attempted theft, if at all.

The Court concluded that under Art 308 of the RPC, there can be
no frustrated stage in theft. It is either attempted or consummated.

105
Hence, the Court ruled beyond reasonable doubt that the taking by
the petitioner was completed in this case. With intent to gain, he
acquired physical possession of the stolen cases of detergent for a
considerable period of time that he was able to drop these off at a
spot in the parking lot, and long enough to load these onto a taxicab.

106
CONTRIBUTOR PANGAN, Gabrielle L.
MODULE Stages of execution in relation to specific felonies
TOPIC
CASE TITLE PEOPLE V. DIO G.R.NO. L-36461
PONENTE ABAD SANTOS, J: DATE: JUNE 29, 1984
DOCTRINE Where the accused is unsuccessful in their criminal venture of
divesting the victim of his property so as to constitute the
consummated crime of robbery and when the killing of the victim
who was not divested of his property which may be considered as
merely incidental to and an offshoot of the plan to carry out the
robbery, the crime would properly come under the provision of Art.
297 of the Revised Penal Code.
FACTS Crispulo Alega, a civil engineer by profession working at the Sugar
Construction Company, went to the Southeastern College, Pasay
City to fetch his girlfriend, Remedios Maniti, a third year high school
student thereat. They proceeded to the Pasay City Public Market.
As they were going up the stairs leading to the Teresa and Sons
Restaurant, Remedios witnessed the victim being strangled by
Danilo Tobias while the appellant was holding his two hands.

The appellant and his companion tried to divest Crispulo of his


'Seiko' wrist watch, but Crispulo resisted their attempt and fought
the robbers. At this juncture, the man who was twisting the neck of
Crispulo stabbed the latter on the left side of his chest. Crispulo ran
down the stairs followed by Remedios who shouted for help. When
he reached the front of the Pasay Commercial Bank he fell down
and expired. At the time of his death, the 'Seiko' watch was still
strapped to his wrist.
The trial court found the appellant is guilty of crime of Robbery with
Homicide as defined under Article 294 of the Revised Penal Code
and sentenced him to suffer the penalty of death.

The appellant claims that he should not have been convicted of the
special complex crime of robbery with homicide because the
robbery was not consummated. He states that there was only an
attempted robbery.
ISSUE/S Whether or not the robbery is consummated
RULING/S The judgement of the trial court is modified. The appellant is found
guilty beyond reasonable doubt of the special complex crime of
attempted robbery with homicide.
The evidence adduced show that the appellant and his companion
were unsuccessful in their criminal venture of divesting the victim
of his wrist watch so as to constitute the consummated crime of

107
robbery. Indeed, as adverted to earlier, when the victim expired,
the 'Seiko' watch was still securely strapped to his wrist.
The killing of Crispulo Alega may be considered as merely
incidental to and an offshoot of the plan to carry out the robbery,
which however was not consummated because of the resistance
offered by the deceased. Consequently, this case would properly
come under the provision of Art. 297 of the Revised Penal Code
which states that — 'When by reason or on occasion of an
attempted or frustrated robbery a homicide is committed, the
person guilty of such offenses shall be punished by reclusion
temporal in its maximum period to reclusion perpetua, unless the
homicide committed shall deserve a higher penalty under the
provisions of this Code.

108
CONTRIBUTOR PROVIDO, Gemy Hale A.
MODULE Robbery, when consummated
TOPIC
CASE TITLE PEOPLE v. SALVILLA G.R.NO. 86163
PONENTE MELENCIO-HERRERA, J: DATE: APRIL 26, 1990
DOCTRINE For robbery to be consummated, there should be an unlawful taking
of a personal property belonging to another person.
FACTS On April 12, 1986 in Iloilo CIty, Bienvenido Salvilla and his co-
accused robbed Severino Choco, Mary Choco, Mimia Choco and
Rodita Hablero, armed with guns and grenade. With an intent to
gain, they unlawfully took the cash amounting to P20000.00, two
men’s wrist watches, one lady’s wrist watch and assorted pieces of
jewelry which are valued all together in the amount of P50000.00.
One of the victims, Mary Choco, suffered serious physical injuries.
In the commencement of the crime, the Phlippine National Police
surrounded the premises of the lumber yard where the crime is
taking place. The police officers negotiated with the offenders, but
they have refused to surrender. This resulted to the firing of the
police officers which caused injuries to the victims as well as the
offenders. Bienvenido Salvilla appealed to the court that the crime
of robbery was still on the attempted and was not yet consummated
in which the lower court made a mistake on the crime that was
charged.
ISSUE/S Whether or not the crime of robbery was consummated in the case
RULING/S Yes, the crime of robbery was consummated in the case the
offenders were able to gain possesion of the paper bag which
contains the victim’s personal proerty and even though they were
not able to dispose of the property when they were interrupted by
the police.
The defense contends that "The complete crime of larceny
(theft/robbery) as distinguished from an attempt requires asportation
or carrying away, in addition to the taking, In other words, the crime
of robbery/theft has three consecutive stages:
1. the giving;
2. the taking; and
3. the carrying away or asportation. Without asportation the
crime committed is only attempted" (Memorandum for
Appellant Salvilla, Records, p. 317).
However, it is already consummated at the time the offenders were
able to get a hold of another person’s personal property, even if they
were not able to get it out or dispose of it, as unlawful taking is
already present at that time.
Wherefore, the offenders were charged with “Robbery with Serious
Physcal Injuries and Serious Illegal Detention” and the penalty

109
imposed is reclusion perpetua. They were charged with a Complex
Crime under Article 48 of the Revised Penal Code.

110
CONTRIBUTOR ROMERO, Ma. Camille Concepcion M
MODULE Stages of Execution : Illegal Tresspass
TOPIC
CASE TITLE People v. Lamahang G.R.NO. L-43530
PONENTE Recto, J. DATE: August 3, 1935
DOCTRINE The attempt to commit an offense that the Penal Code punishes is
that which has a logical relation to a particular concrete offense.
That the beginning of the execution of offense by overt acts of the
perpetrator leading directly to its realization and consummation .
FACTS On early dawn of March 2, 1935 while a police officers was
patrolling in Fuentes St. Of the City of Iloilo they caught the accused
making an opening of the wall of a store of cheap goods with an iron
bar. At the time of the commission the owner Tan Yu was sleeping
inside with another Chinaman. The accused succeeded in breaking
one board and in unfastening another from the wall, when the
policeman showed up, who instantly arrested him and placed him
under custody.
ISSUE/S Whether or not the accused was erroneously declared guilty of
attempted robbery.
RULING/S No, the facts of the case does not constitute attempted robbery but
attempted trespass to dwelling.
What is clear in the case is the intention of the accused to enter the
store by means of violence to commit an offense that did not develop
beyond the beginning of the execution due to the arrival of the
policemen, but it is not enough to impose penal sanction.
Against the accused must be taken into consideration the
aggravating circumstances of nighttime and former convictions,
inasmuch as the record shows that several final judgments for
robbery and theft have been rendered against him and in his favor,
the mitigating circumstance of lack of instruction. The breaking of
the wall should not be taken into consideration as an aggravating
circumstance inasmuch as this is the very fact which in this case
constitutes the offense of attempted trespass to dwelling.
Wherefore, the sentence appealed from is revoked and the accused
is hereby held guilty of attempted trespass to dwelling, committed
by means of force, with the aforesaid aggravating and mitigating
circumstances and sentenced to three months and one day of
arresto mayor, with the accessory penalties thereof and to pay the
costs.

111
CONTRIBUTOR SAMONTE, Vanessa Antoinette
MODULE STAGES OF EXECUTION
TOPIC
CASE TITLE PEOPLE v. BASILIO A.M. NO. G.R. No. 33463
BORINAGA
PONENTE MALCOM, J.: DATE: DECEMBER 18, 1930

DOCTRINE The essential condition of a frustrated crime, that the author perform
all the acts of execution, attended the attack.

FACTS Harry Mooney contracted with Juan Lawaan for the construction of
a fish corral. Defendant-appellant Basilio Borinaga was associated
with Lawaan. When about two-thirds of the fish corral was already
finished, Lawaan and his men went to Mooney to collect from him
the whole amount fixed by the contract. Mooney refused, which
made Lawaan warn the former that if he did not pay, something
would happen to him. On that evening, Mooney was in the store of
his neighbor Perpetua Najarra, sitting with his back towards a
window when Borinaga from the window struck Mooney with a knife
but did not hit Mooney, instead lodged in the back of the chair where
Mooney was seated. Borinaga left but returned after ten minutes
with a knife in hand but was frightened away by Mooney and
Perpetua. The Court of First Instance convicted Borinaga with the
crime of Frustrated murder.

ISSUE/S Whether or not Borinaga committed the crime of Frustrated murder.

RULING/S Yes. Borinaga performed all the acts of execution. Nothing


remained to be done to accomplish the work of the assailant
completely. The cause resulting to the failure of the attack arose by
reason of forces independent of the will of the perpetrator. The
assailant voluntarily desisted from further acts. What is known is the
subjective phase of the criminal act was passed.

112
CONTRIBUTOR SAMSON, Jessa Viena D.
MODULE Stages of execution in relation to physical injuries, homicide, and
TOPIC murder
CASE TITLE PEOPLE v. KALALO G.R.NO. 39303-05
PONENTE DIAZ, J.: DATE: MAR. 17, 1934
DOCTRINE Murder, the circumstance of "abuse of superior strength", if proven
to have been presented, raises homicide to the category of
murder. Moreover, performing everything necessary to commit the
crime that one has determined to commit, but failed by reason of
causes independent of his will, such act constitutes attempted
homicide.
FACTS Herein appellants Felipe Kalalo, Marcelo Kalalo, Juan Kalalo, and
Gregorio Ramos, were tried in the Court of First Instance (CFI)
jointly with Alejandro Garcia, Fausta Abrenica, and Alipia Abrenica
in criminal cases, the first two for murder, and the last for frustrated
murder.
Appellant Marcelo and Isabela Holgado, the latter being the sister
of the deceased Arcadio Holgado and a cousin of the other
deceased Marcelino Panaligan, had a litigation over a parcel of
land. Hence, Marcelo filed separate complaints on separate dates,
however, both were dismissed.
Isabela and her brother Arcadio decided to order the aforesaid land
plowed, and employed several laborers for that purpose. These
men, together with Arcadio, went to the said land. However,
Marcelo, who had been informed thereof, proceeded to the place
accompanied by his brothers Felipe and Juan, his brother-in-law
Gregorio and by Alejandro, who were later followed by Fausta and
Alipia, mother and aunt, respectively, of the first three.
Relative thereto, the first five were all armed with bolos. Upon their
arrival at the said land, they ordered those who were plowing it by
request of Isabela and Arcadio to stop which they subsequently
obeyed due to the threatening attitude of those who gave them said
order. Shortly, Isabela, Maria, and Hilarion arrived at the place with
food for the laborers. Before the men resumed their work, they were
given their food and not long after they had finished eating,
Marcelino likewise arrived. Having been informed of the cause of
the suspension of the work, Marcelino ordered said Arcadio and the
other laborers to again hitch their respective carabaos to continue
the work already began. At this juncture, the appellant Marcelo
approached Arcadio, while the appellants Felipe, Juan, and
Gregorio Ramos, in turn, approached Marcelino. At a remark from
Fausta, mother of the Kalalos, about as follows, "what is detaining
you?" they all simultaneously struck with their bolos, the appellant
Marcelo slashing Arcadio, while the appellants Felipe, Juan, and
Gregorio Ramos slashed Marcelino inflicting 14 wounds. Arcadio

113
and Marcelino died instantly from the wounds received by them in
the presence of Isabela and Maria, not to mention the accused.
Thereafter, appellant Marcelo took from its holster on the belt of
Marcelino's body, the revolver which the deceased carried, and fired
four shots at Hilarion who was then fleeing from the scene in order
to save his own life.
Thus, the appellants attempted to prove that the fight, which
resulted in the death of the two deceased, was provoked by
Marcelino who fired a shot at Marcelo upon seeing the latter's
determination to prevent Arcadio and his men from plowing the land
in question.
ISSUE/S Whether or not the appellants are guilty of attempted homicide
RULING/S Under Article 248 of the Revised Penal Code, which defines murder,
the circumstance of "abuse of superior strength", if proven to have
been presented, raises homicide to the category of murder;
however, the Court is of the opinion that said circumstance may not
properly be taken into consideration in the two cases at bar, either
as a qualifying or as a generic circumstance, if it is borne in mind
that the deceased were also armed, one of them with a bolo, and
the other with a revolver. The risk was even for the contending
parties and their strength was almost balanced because there is no
doubt but that, under circumstances similar to those of the present
case, a revolver is as effective as, if not more than three bolos.
As regards the evidence, it shows that Marcelo fired four successive
shots at Hilarion while the latter was fleeing to save his own life. The
fact that the said appellant, not having contended himself with firing
only once, fired said successive shots at Hilarion, added to the
circumstance that immediately before doing so, he and his co-
appellants had already killed Arcadio and Marcelino, shows that he
was then bent on killing said Hilarion. He thus performed everything
necessary on his pat to commit the crime that he determined to
commit however failed by reason of causes independent of his will,
either because of his poor aim or because his intended victim
succeeded in dodging the shots, none of which found its
mark. Hence, the acts committed by the said appellant Marcelo
constitute attempted homicide with no modifying circumstance to be
taken into consideration, for none has been established.

114
CONTRIBUTOR TERTE, Karen A.
MODULE Stages of Execution; Attempted stage
TOPIC
CASE TITLE PEOPLE v. TRINIDAD G.R.NO. 79123-25
PONENTE MELENCIO-HERRERA, J. DATE: JANUARY 9, 1989
DOCTRINE There is an attempt when the accused had commenced the
commission of the felony directly by overt acts but was unable to
perform all the acts of execution which would have produced it by
reason of causes other than his spontaneous desistance,
FACTS Lolito Soriano, a fish dealer, together with his helpers, Ricardo Tan
and Marcial Laroa, were driving on their way to Davao City to sell
fish. Accused Emeliano Trinidad, a member of the Integrated
National Police, was in uniform and had two firearms when he asked
them for a ride to Agusan del norte.

Ricardo Tan was driving when he was asked to slow down by


Trinidad as they were treading on dangerous territory. Suddenly,
Tan heard two gunshots and when he looked back, Soriano was
shot by Trinidad while Laroa was slumped dead. Tan got off the
moving vehicle and hid in the bushes before alighting a passenger
jeepney. Unfortunately, Trinidad was in the same jeepney and saw
Tan sitting in the front. Tan immediately got off, ran around the
jeepney and when boarded it again once it started driving away. He
was shot in the leg but was able to jump and transfer to the second
jeepney. However, the passengers in the latter jeep did not want to
get involved in the affray and told him to get out. Wounded, Tan
crawled until a member of the Philippine Constabulary chanced
upon him and helped him board a bus for Butuan City.

Trinidad, in his defense, contended that he was in Cagayan de Oro


City on the date of the incident. He narrated that he was at his
sister's house and he saw Sgt. Caalim, Mrs. Andoy, one Paelmo, in
addition to his sister. Sgt. Caalim corroborated having seen Trinidad
then. He alleges that Tan was an unreliable witness hence, the trial
court erred in giving full credit to him. However, Trinidad’s defense
of alibi is inherently weak and cannot prevail over the
straightforward and detailed descriptive narration of Tan.
ISSUE/S Whether or not Emeliano Trinidad is guilty of attempted murder
and not Frustrated Murder.
RULING/S Yes, Trinidad can only be convicted of Attempted Murder.
There is an attempt when the accused had commenced the
commission of the felony directly by overt acts but was unable to
perform all the acts of execution which would have produced it by
reason of causes other than his spontaneous desistance

115
Clear in the facts that Trinidad had fired shots to Tan even if the
jeep to which Tan was clinging to was already in motion. In
addition, had it not been for the spare tire that shielded the other
parts of Tan’s body, the shot could have been more fatal or
severe. However, since the wound on his thigh was not fatal and
the doctrinal rule is that where the wound inflicted on the victim is
not sufficient to cause his death, the crime is only Attempted
Murder, the accused not having performed all the acts of execution
that would have brought about death.
Despite having two counts of Murder, the penalty for Trinidad was
modified from capital punishment to reclusion temporal in its
maximum period to reclusion Perpetua as the former was
abolished in the 1987 Constitution. For the crime of Attempted
Murder, Trinidad was sentenced the penalty of prision
correccional, in its minimum, to prision mayor, in its maximum
period.

116
CONTRIBUTOR TIDALGO, Aimee Diane A.
MODULE Stages of Execution
TOPIC
CASE TITLE MARTINEZ V. COURT OF G.R.NO. 168827
APPEALS
PONENTE CALLEJO, SR., J. DATE: APRIL 13, 2007
DOCTRINE There is treachery when the offender commits any of the crimes
against the person, employing means, methods or forms in the
execution thereof which tend directly and specially to insure its
execution without risk to himself arising from the defense which the
offended party might take
FACTS Dean Dongui-is was a teacher at the Tubao National High School,
La Union. Petitioner Benjamin Martinez was the husband of Dean’s
co-teacher, Lilibeth Martinez. Petitioner eked out a living as a
tricycle driver.

On October 28, 1998, Dean and his wife Freda filed a complaint for
damages against the spouses Martinez alleging that Martinez had
been peddling false reports that Dean and Elvisa had illicit relations;
he even told Freda that Elvisa was Dean’s mistress. This led to a
quarrel between Dean and Freda, and the latter was hospitalized for
her heart ailment. Dean requested Lilibeth to stop her husband from
spreading lies, and she replied that Elvisa had been her husband’s
mistress.

Later that afternoon, Dean went to the Tubao Credit Cooperative


(TCC) office to pick up the dividend certificate of his wife who was a
member of the cooperative. He left the building and walked to his
car which was parked in front. As he did, he read the dividend
certificate of his wife. Dean was about a step away from an L-300
van which was parked in front of the building when petitioner, armed
with a bolo, suddenly emerged from behind the vehicle and stabbed
him on the left breast. Dean instantly moved backward and saw his
assailant. Dean fled to the bank office and was able to gain entry
into the bank. Petitioner ran after him and upon cornering him, tried
to stab him again. Dean was able to parry the blow with his right
hand, and the bolo hit him on the right elbow. Dean fell to the floor
and tried to stand up, but petitioner stabbed him anew on his left
breast. Dean managed to run to the counter which was partitioned
by a glass. Unable to get inside the counter, petitioner shouted at
Dean: “Agparentomeng ka tatta ta talaga nga patayen ka tatta nga
aldawen (You kneel down because I will really kill you now this
day).”

Meantime, SPO1 Henry Sulatre, who was about 100 meters aways
from the scene, was informed about the fight and ruseg to the place.

117
On March 10, 1999, SPO1 Sulatre filed a criminal complaint for
frustrated murder against petitioner. The court opted not to act on
the crime pending the arrest report and SPO1 Sulatre’s submission
of Dean’s sworn statement.

The IRH issued a medical certificate on February 28, 1999, stating


that Dean’s wounds would need medical attendance of more than
30 days. Barangay Captain Oller and SPO1 Sulatre executed an
affidavit on petitioner’s arrest. Dean had his affidavit sworn before
the Public Prosecutor on March 30, 1999.
ISSUE/S Whether or not CA’s affirmation of Martinez to be convicted for
frustrated homicide without any proof beyond reasonable doubt is
correct
RULING/S YES. Although Martinez argues that he should only be convicted of
less serious physical injuriesbecause the single wound suffered by
the victim was not life threatening and that the latter was transferred
to undergo operation in another hospital only because the medical
staff where he was first rushed bungled their job. He makes much
of the fact that Dr. Darius R. Pariñas who issued the Medical
Certificate never testified for the prosecution.

If one inflicts physical injuries on another but the latter survives, the
crime committed is either consummated physical injuries, if the
offender had no intention to kill the victim or frustrated or attempted
homicide or frustrated murder or attempted murder if the offender
intends to kill the victim. Intent to kill may be proved by evidence of
the following: (a) motive; (b) the nature or number of weapons used
in the commission of the crime; (c) the nature and number of wounds
inflicted on the victim; (d) the manner the crime was committed; and
(e) words uttered by the offender at the time the injuries are inflicted
by him on the victim. Petitioner insists that he had no intent to kill
Dean. However, the physical evidence belies petitioner’s pose.

To begin with, as between petitioner and the victim, the former had
more hatred to harbor arising from the fact that the victim filed a
lawsuit against him and his wife. Petitioner thus had more motive to
do harm than the victim. By his own account, he and Dean had a
history of personal animosity.

Finding that the prosecution failed to prove the qualifying


circumstances of treachery, the trial court convicted petitioner of
frustrated homicide. Petitioner is guilty of frustrated murder under
Article 248 in relation to Article 6, first paragraph of the Revised
Penal Code which provides “A felony is consummated when all the
elements necessary for its execution and accomplishment are
present; and it is frustrated when the offender performs all the acts

118
of execution which would produce the felony as a consequence but
which, nevertheless, do not produce it by reason of causes
independent of the will of the perpetrator.”

The essential elements of a frustrated felony are as follows:


1. The offender performs all the acts of execution;
2. All the acts performed would produce the felony as a
consequence;
3. But the felony is not produced;
4. By reason of causes independent of the will of the perpetrator.

119
CONTRIBUTOR Valdez, Ariane Faye V.
MODULE Module 3- Physical Injuries, Homicide, and Murder (Art. 248, 249,
TOPIC and 263, RPC)
CASE TITLE Mondragon v. People G.R.NO. L-17666
PONENTE ZALDIVAR, J. DATE: 30 June 1966
DOCTRINE The intent to kill being an essential element of the offense of
frustrated or attempted homicide, said element must be proved by
clear and convincing evidence and with the same degree of
certainty as is required of the other elements of the crime. The
inference of intent to kill should not be drawn in the absence of
circumstances sufficient to prove such intent beyond reasonable
doubt.
FACTS At about 5:00 in the afternoon of July 11, 1954, while complainant
Serapion Nacionales was opening the dike of his rice field situated
in Antandan, Miagao, Iloilo, to drain the water therein and prepare
the ground for planting the next day, he heard a shout from afar
telling him not to open the dike, Nacionales continued opening the
dike, and the same voice shouted again,

‘Don’t you dare open the dike.’ When he looked up, he saw Isidoro
Mondragon coming towards him. Nacionales informed appellant
that he was opening the dike because he would plant the next
morning.

Without much ado, Mondragon tried to hit the complainant who


dodged the blow. Thereupon, appellant drew his bolo and struck
complainant on different parts of his body. Complainant backed out,
unsheathed his own bolo, and hacked appellant on the head and
forearm and between the middle and ring fingers in order to defend
himself. The appellant retreated, and the complainant did not pursue
him but went home instead.

The petitioner, Isidoro Mondragon, was prosecuted in the Court of


First Instance of Iloilo of the crime of frustrated homicide. After trial
the Court of First Instance of Iloilo found him guilty of the crime of
attempted homicide. Mondragon appealed to the Court of Appeals,
and the latter court affirmed the decision of the Court of First
Instance of Iloilo in all its parts, with costs. This case is now before
us on a petition for certiorari to review the decision of the Court of
Appeals.
ISSUE/S Whether or not the Court of Appeals erred in finding him guilty of
the crime of attempted homicide.
RULING/S Yes. The facts as found by the Court of Appeals, do not establish
the intent to kill on the part of the petitioner. The determinants of
intent to kill include the following:

120
1. The means used by the malefactors;
2. The nature, location and number of wounds sustained by
the victim;
3. The conduct of the accused before, during and
immediately after the commission of the crime;
4. The circumstances under which the crime was
committed and the motives of the accused.
In the case at bar, we apply the determinants of intent to kill such as
the following:
1. The petitioner started the assault on the offended party by
just giving him fist blows;
2. The wounds inflicted on the offended party were of slight
nature, indicating no, homicidal urge on the part of the
petitioner;
3. The petitioner retreated and went away when the offended
party started hitting him with a bolo;
4. Thereby indicating that if the petitioner had intended to kill the
offended party he would have held his ground and kept on
hitting the offended party with his bolo to kill him. Moreover,
the petitioner had no motive to kill the offended party.
The inference of intent to kill should not be drawn in the absence of
circumstances sufficient to prove such intent beyond reasonable
doubt. The element of intent to kill not having been duly established,
and considering that the injuries suffered by the offended party were
not necessarily fatal and could be healed in less than 30 days.
Therefore, the offense that was committed by the petitioner is only
that of less serious physical injuries.

121
CONTRIBUTOR VILLANUEVA, Sean Ruthie L.
MODULE STAGES OF EXECUTION (Physical Injuries, Homicide, Murder)
TOPIC
CASE TITLE PEOPLE v. SY PIO G.R.NO. L-5848
PONENTE LABRADOR, J.: DATE: APRIL 30, 1954
DOCTRINE The fact that there was escape, which was seen, must have
produced in the mind that the individual was not hit in the vital part
of the body. In other words, all the acts of execution necessary to
kill the victim were not accomplished. Under these circumstances, it
can not be said that the subjective phase of the acts of execution
has been completed when there is no pursuit.
FACTS In the early morning of September 3, 1949, Sy Pio went to the store
at 511 Misericordia, Sta. Cruz, Manila. Upon entering the premise,
he started firing his .45 caliber pistol, the first shot hit Jose Sy, Tan
Siong Kap witnessed the said shooting incident but he was also shot
on the shoulder after Jose Sy which caused him to run into a room
behind the store and hide. He kept hearing gunshots until Sy Pio ran
away. Sy Pio shot Tan Siong Kap, Ong Pian and Jose Sy. Tan Siong
Kap was brought to Chinese General Hospital to get his wound
treated. He stayed in the hospital from September 3 to September
12, 1949 but was asked by the physician to return to the hospital for
further treatment. He did so five times for a period of more than ten
days and spent P300 for hospital and doctor’s fees.

On September 5, Sy Pio was in custody of the Constabulary in


Tarlac, Police officer Lomotan went to Tarlac and had a
conversation with him. Sy Pio admitted to shooting the three
individuals mentioned. The pistol used was also presented as
Exhibit C and a magazine as Exhibit C-1, Sy Pio was then under
Lomotan’s custody, where his statement was taken down in writing.

Sy Pio stated that some months before the shooting incident, he


was employed by Ong Pian in his restaurant. Prior to September 3,
Sy Pio’s wife, Vicenta’s father was sick. Both of the couple asked
money from their respective employer, Sy Pio received P1 while his
wife received P20 from her employer. Sy Pio was dismissed from
his work and he became a peddler. Ong Pian presented a list of Sy
Pio’s borrowings and had it deducted in his wife’s salary. He did not
recognize these sums as his indebtedness thus the reason why he
resented Ong Pian’s conduct. As to Tan Siong Kiap, Sy Pio had
been able to realize the sum of P70 from the sales of medicine that
he peddled. The following morning, he realized that the money
placed in his room disappeared. Tan Siong Kap and Jose Sy told
him that he must have given it to his wife but after the incident, he
heard the two and other Chinamen stating that he prolly lost the
money through gambling. With that, Sy Pio, resented the two.

122
ISSUE/S Whether or not Sy Pio should be charged of the crime frustrated
murder.
RULING/S No, Sy Pio should be found guilty beyond reasonable doubt of the
crime of attempted murder since he did not perform all the acts of
execution, actual and subjective. According to the cases of U.S. vs
Eduave, People vs. Dagman and People vs. Borinaga, the Court
held that it was not necessary that the accused actually commit all
the acts of execution necessary to produce the death of his victim
but that it was sufficient that they believed that they were able to
commit all the acts.
In the case at bar, Sy Pio fired at his victim which hit the latter on
the right shoulder but was able to escape into a room. The fact that
Tan Siong Kap was able to escape, which the accused must have
seen, must have produced in the mind of the accused that he was
not able to hit the victim in the vital part of the body. Sy Pio knew
that he had not actually performed all the acts of execution
necessary to kill him completely, also Sy Pio did not continue in the
pursuit, as a matter of fact, he ran away. When there is doubt , it
should be resolved in favor of the defendant-appellant. He was
sentenced to 4 years, 2 months and 1 day of prision correccional to
109 years of prision mayor.

123
CONTRIBUTOR BILTZ, Aralind Louise A.
MODULE Stages of Execution
TOPIC
CASE TITLE PEOPLE vs. ORITA G.R.NO. 88724
PONENTE MEDIALDEA, J DATE: APRIL 3, 1990
DOCTRINE In the crime of rape, from the moment the offender has carnal
knowledge of his victim he actually attains his purpose and, from
that moment also all the essential elements of the offense have
been accomplished. Nothing more is left to be done by the
offender, because he has performed the last act necessary to
produce the crime. Thus, the felony is consummated.
FACTS Complainant Cristina Abayan, 19 years old, just arrived at her
boarding house when somebody held her and poked a knife to her
neck. He ordered her to go upstairs an look for a room. Upon
entering the room, he order complainant to take off her clothes. He
then mounted on her but could not fully penetrate her so he
commanded her to mount him, allowing only a small part of his penis
to penetrate into her vagina.

Complainant dashed out of the room naked and darted further to the
municipal building. She was found by the policemen naked while
sitting on the stairs crying. She was then brought to the hospital and
was physically examined.
ISSUE/S Whether or not the trial court erred in declaring that the crime of
frustrated rape was committed by the accused
RULING/S YES. In concluding that there is no conclusive evidence of
penetration of the genital organ of the victim, the trial court relied
on the testimony of Dr. Zamora when he "categorically declared
that the findings in the vulva does not give a concrete disclosure of
penetration. As a matter of fact, he tossed back to the offended
party the answer as to whether or not there actually was
penetration."
For the consummation of rape, perfect penetration is not essential.
Any penetration of the female organ by the male organ is
sufficient. Entry of the labia or lips of the female organ, without
rupture of the hymen or laceration of the vagina is sufficient to
warrant conviction. Necessarily, rape is attempted if there is no
penetration of the female organ (People v. Tayaba, 62 Phil. 559
People v. Rabadan et al., 53 Phil. 694; United States v. Garcia: 9
Phil. 434) because not all acts of execution was performed. The
offender merely commenced the commission of a felony directly by
overt acts.

124
CONTRIBUTOR CACHERO, Luis III L.
MODULE Stages of Execution
TOPIC
CASE TITLE PEOPLE V PRIMO G.R.NO. 129433
CAMPUHAN
PONENTE BELLOSILLO, J. DATE: March 30, 2000
DOCTRINE Under Art 6, in relation to Art. 335 of the Revised Penal Code, rape
is attempted when the offender commences the commission of rape
directly by overt acts, and does not perform all the acts of execution
which should produce the crime of rape by reason of some cause of
accident other than his own spontaneous desistance.
FACTS On April 25, 1996, at around 4 o’clock in the afternoon, Ma. Corazon
P. Pamintuan, mother of four (4)-year old Crysthel Pamintuan, went
down from the second floor of their house to prepare Milo chocolate
drinks for her two (2) children. As Corazon was preparing the drinks,
she heard one of her daughters cry “Ayoko, ayoko!” prompting
Corazon to rush upstairs. Thereupon, she saw Primo Campuhan
inside her children’s room kneeling before Crysthel whose pajamas
or “jogging pants” and panty were already removed, while his short
pants were down to his knees. According to Corazon, Primo was
forcing his penis into Crysthel’s vagina.
Accused-appellant then tried to escape but Corazon then ran out
and shouted for help thus prompting her brother, a cousin, and an
uncle to chase the accused. Seconds later, Primo was then
apprehended by those who answered Corazon’s call for help.
Accused-appellant was then charged, then eventually found guilty,
or statutory rape and sentenced by the court a quo to the extreme
penalty of death.
Hence, this case before the Court on automatic review.

ISSUE/S Whether or not all the stages of execution were present to


consummate the felony of rape.
RULING/S The acts of execution were only on the ATTEMPTED stage. The
prosecution was not able to prove beyond reasonable doubt that the
felony was consummated. Even though Corazon witnessed the
accused’s shorts and her daughter’s undergarments already
removed, this did not consummate the crime of rape. In People v
Orita, it was held that rape was consummated from the moment the
offender had carnal knowledge of the victim, even though then that
perfect penetration need not to happen. Even the slightest
penetration would already consummate the crime of rape. In the
case at Bar, it was not proven that the accused-appellant’s penis
had the slightest touch in the vagina of the victim.

125
The Supreme Court modified the decision from rape to attempted
rape and is sentenced two degrees lower than the imposable
penalty of death for the offense charged.

126
CONTRIBUTOR CALZADO, Anne Valerie L.
MODULE Stages of execution in relation to specific felonies, Arson (Art. 320,
TOPIC RPC)
CASE TITLE US v. VALDES G.R.NO. 14128
PONENTE TORRES, J. DATE: DEC 10, 1918
DOCTRINE When the accused performs all the steps conducive to the burning
of a property, but, notwithstanding these acts, he did not
accomplish the criminal act which he had intended to consummate
by reason of causes independent of his will, it only qualifies as
frustrated arson.
FACTS A neighbor saw smoke issuing from the lower floor of the house
owned by Mrs. Lewis. The owner, upon being informed of the
smoke, ordered the servant Banal to look for the source. It was
found to be coming between a post of the house and a partition of
the entresol, where a piece of a jute sack and a rag soaked with
kerosene oil were burning. At that moment the defendant Valdes
was in the entresol, engaged in his work of cleaning. He was later
on arrested and made a statement that he had set the fire to the
said rag and piece of sack under the house due to the inducement
of other accused, Hugo Labarro and that he had started several
other fires which had occurred in said house on previous days.
ISSUE/S Whether of not the accused committed the crime of arson in its
consummated stage
RULING/S No. The fact of setting fire to a jute sack and a rag, soaked with
kerosene oil and placed beside an upright of the house and a
partition of the entresol of the building, thus endangering the burning
of the latter, constitutes the crime of frustrated arson of an inhabited
house, on an occasion when some of its inmates were inside of it.
The crime is classified only as frustrated arson, inasmuch as the
defendant performed all the acts conceive to the burning of said
house, but nevertheless., owing to causes independent of his will,
the criminal act which he intended was not produced. The offense
committed cannot be classified as consummated arson by the
burning of said inhabited house, for the reason that no part of the
building had yet commenced to burn, although, as the piece of sack
and the rag, soaked in kerosene oil, had been placed near partition
of the entresol, the partition might have started to burn, had the fire
not been put out on time.

127
MODULE 4
CONSPIRACY AND
PROPOSALS TO COMMIT A
FELONY (ART. 8, RPC) AND
FELONIES AS TO SEVERITY
(ART. 7 AND 9, RPC)

128
CONTRIBUTOR AGUILAR, Jose Maria L.
MODULE CONSPIRACY AND PROPOSALS TO COMMIT A FELONY AND
TOPIC FELONIES AS TO SEVERITY
CASE TITLE DUNGO v. PEOPLE G.R.NO. 209464
PONENTE MENDOZA, J: DATE: JULY 1, 2015
DOCTRINE Generally, mere presence at the scene of the crime does not in itself
amount to conspiracy. Exceptionally, under R.A. No. 8049, the
participation of the offenders in the criminal conspiracy can be
proven by the prima facie evidence due to their presence during the
hazing, unless they prevented the commission of the acts therein.
FACTS At around 3:20 o'clock in the morning, the victim Marlon Villanueva
was brought to the emergency room of Dr. JP Rizal Hospital. The
Dr. who was then the attending physician at the emergency room,
observed that Villanueva was motionless, not breathing and had no
heartbeat and was pronounced dead. The Dr. noticed a big
contusion hematoma on the left side of the victim's face and several
injuries on his arms and legs.

The said Dr. disclosed that two men brought the victim to the
hospital. The two told him that they found the victim lying motionless
on the ground at a store and brought him to the hospital. When he
asked them where they came from, one of them answered that they
came from Los Baños, Laguna, en route to San Pablo City. The Dr.
stated that due to the nature, extent and location of the injuries, he
opined that the victim was a victim of hazing.

Susan Ignacio was the owner of the sari-sari store located in front
of Villa Novaliches Resort. That around 8:30 to 9:00 o'clock in the
evening, she was tending her store when she saw a jeepney with
more than twenty persons arrive at the resort. Ignacio identified
Dungo as the person seated beside the driver of the jeepney.
Ignacio saw about fifteen persons gather on top of the terrace of the
resort who looked like they were praying, and then the lights of the
resort were turned off. Later on, she learned from the policemen
visiting the resort that the deceased person was the victim.

Magat, a tricycle driver testified that a man approached him and told
him that someone inside the resort needed a ride. Afterwards, he
saw three men in their carrying another man, who looked very weak,
like a vegetable, towards his tricycle. The security guards on duty at
JP Rizal Hospital, stated that two men brought the lifeless body of a
person and that they called the police station so that an investigation
could be conducted. During his testimony, he identified Sibal and
Dupgo as the two persons.

129
The officers of the Student Disciplinary Tribunal of the UP Los
Baños, testified that an administrative disciplinary against the APO
Fraternity regarding the death of the victim. The accused in the case
at bar were members of the said fraternity.

The RTC found Dungo and Sibal guilty of the crime of violating
Section 4 of the Anti-Hazing Law and sentenced them to suffer the
penalty of reclusion perpetua. It was been explained that even if
there was no evidence that Dungo and Sibal participated to bodily
assault and harm the victim, it was irrefutable that they brought
Villanueva to the resort for their final initiation rites. Clearly, they did
not merely induce Villanueva to attend the final initiation rites, but
they also brought him to Villa Novaliches Resort.
ISSUE/S Whether or not the accused can be convicted of the crime of hazing.
RULING/S Yes. The Supreme Court ruled that the accused can be convicted in
violation of the Anti-Hazing Law.
A conspiracy exists when two or more persons come to an
agreement concerning the commission of a felony and decide to
commit it. To determine conspiracy, there must be a common
design to commit a felony. The overt act or acts of the accused may
consist of active participation in the actual commission of the crime
itself or may consist of moral assistance to his co-conspirators by
moving them to execute or implement the criminal plan.
In conspiracy, it need not be shown that the parties actually came
together and agreed in express terms to enter into and pursue a
common design. The assent of the minds may be and, from the
secrecy of the crime, usually inferred from proof of facts and
circumstances which, taken together, indicate that they are parts of
some complete whole. Responsibility of a conspirator is not
confined to the accomplishment of a particular purpose of
conspiracy but extends to collateral acts and offenses incident to
and growing out of the purpose intended
Verily, the disputable presumption under R.A. No. 8049 can be
related to the conspiracy in the crime of hazing. The common design
of offenders is to haze the victim. Some of the overt acts that could
be committed by the offenders would be to (1) plan the hazing
activity as a requirement of the victim's initiation to the fraternity; (2)
induce the victim to attend the hazing; and (3) actually participate in
the infliction of physical injuries.
In this case, there was prima facie evidence of the petitioners'
participation in the hazing because of their presence in the venue.
As correctly held by the RTC, the presence of Dungo and Sibal
during the hazing at Villa Novaliches Resort was established by the
testimony of Ignacio.
Because of the uncontroverted prima facie evidence against the
petitioners, it was shown that they performed an overt act in the

130
furtherance of the criminal design of hazing. Not only did they induce
the victim to attend the hazing activity, the petitioners also actually
participated in it based on the prima facie evidence. These acts are
sufficient to establish their roles in the conspiracy of hazing.

131
CONTRIBUTOR ALGURA, Nino N.
MODULE Conspiracy and Proposals to Commit a Felony (Art. 8, RPC) and
TOPIC Felonies as to Severity (Art. 7 and 9, RPC)
CASE TITLE People v. Aguilos G.R.NO. 121828
PONENTE CALLEJO, SR., J.: DATE: June 27, 2003
DOCTRINE Art. 4. Criminal liability. – Criminal liability shall be incurred:
• By any person committing a felony (delito) although the wrongful act
done be different from that which he intended.

Art. 18. Accomplices. – Accomplices are the persons who, not


being included in Article 17, cooperate in the execution of the
offense by previous or simultaneous acts.
FACTS That on or about the 5th day of February, 1988 in the Municipality
of Mandaluyong, Metro Manila, Philippines, a place within the
jurisdiction of this Honorable Court, the above-named accused,
conspiring and confederating together with one Ronnie Diamante
who is still at-large and no fixed address and mutually helping and
aiding with one another, armed with double-bladed knives and a
bolo and with intent to kill, treachery and taking advantage of
superior strength, did then and there willfully, unlawfully and
feloniously attack, assault hack and stab one Joselito Capa y
Rulloda, as a result of which the latter sustained hack and stab
wounds on the different parts of his body, which directly caused his
death.
Of the three accused, Odilon Lagliba was the first to be arrested and
tried, and subsequently convicted of murder. The decision of the trial
court became final and executory. Accused Edmar Aguilos remains
at large while accused Ronnie Diamante reportedly died a month
after the incident. Meanwhile, herein appellant Rene Gayot Pilola
was arrested. He was arraigned on March 9, 1994, assisted by
counsel, and pleaded not guilty to the charge. Thereafter, trial of the
case ensued
ISSUE/S Whether or not Pilola is guilty of murder?
RULING/S YES. Rene Gayot Pilola GUILTY beyond reasonable doubt of the
crime of murder is AFFIRMED WITH MODIFICATION

The identity of the person who hit the victim with a hollow block is of
de minimis importance. Elisa’s testimony is corroborated by the
autopsy report of Dr. Bienvenido Muñoz. No showing of any
improper motive on the part of a witness to testify falsely against the
accused or to falsely implicate the latter in the commission of the
crime. The trial court gave credence and full probative weight to
Elisa’s testimony.

132
There is conspiracy when two or more persons agree to commit a
felony and decide to commit it. Conspiracy as a mode of incurring
criminal liability must be proved separately from and with the same
quantum of proof as the crime itself. Conspiracy need not be proven
by direct evidence. After all, secrecy and concealment are essential
features of a successful conspiracy. It may be inferred from the
conduct of the accused before, during and after the commission of
the crime, showing that they had acted with a common purpose and
design. Conspiracy may be implied if it is proved that two or more
persons aimed by their acts towards the accomplishment of the
same unlawful object, each doing a part so that their combined acts,
though apparently independent of each other, were, in fact,
connected and cooperative, indicating a closeness of personal
association and a concurrence of sentiment. There may be
conspiracy even if an offender does not know the identities of the
other offenders, and even though he is not aware of all the details
of the plan of operation or was not in on the scheme from the
beginning. One need only to knowingly contribute his efforts in
furtherance of it. One who joins a criminal conspiracy in effect
adopts as his own the criminal designs of his co-conspirators. If
conspiracy is established, all the conspirators are liable as co-
principals regardless of the manner and extent of their participation
since in contemplation of law, the act of one would be the act of all.
Each of the conspirators is the agent of all the others.

The mere presence of an accused at the situs of the crime will not
suffice. There must be intentional participation in the transaction
with a view to the furtherance of the common design and purpose.

Even if two or more offenders do not conspire to commit homicide


or murder, they may be held criminally liable as principals by direct
participation if they perform overt acts which mediately or
immediately cause or accelerate the death of the victim. Art.
4. Criminal liability. – Criminal liability shall be incurred:
o 1. By any person committing a felony (delito) although the wrongful act
done be different from that which he intended.

Art. 18. Accomplices. – Accomplices are the persons who, not


being included in Article 17, cooperate in the execution of the
offense by previous or simultaneous acts.

o To hold a person liable as an accomplice, two elements must concur:


1. the community of criminal design; that is, knowing the criminal design
of the principal by direct participation, he concurs with the latter in his
purpose;

133
2. the performance of previous or simultaneous acts that are not
indispensable to the commission of the crime

Accomplices come to know about the criminal resolution of the


principal by direct participation after the principal has reached the
decision to commit the felony and only then does the accomplice
agree to cooperate in its execution. Accomplices do not decide
whether the crime should be committed; they merely assent to the
plan of the principal by direct participation and cooperate in its
accomplishment.

However, where one cooperates in the commission of the crime by


performing overt acts which by themselves are acts of execution, he
is a principal by direct participation, and not merely an accomplice.

Odilon all by himself initially decided to stab the victim. However,


while Odilon was stabbing the victim, the appellant and Ronnie
agreed to join. All the overt acts of Odilon, Ronnie and the Pilola
before, during, and after the stabbing incident indubitably show that
they conspired to kill the victim. Since the victim is not yet dead, the
crime is not yet consummated so Pilola is a principal by direct
participation.
Alibi is a weak, if not the weakest of defenses in a criminal
prosecution, because it is easy to concoct but hard to disprove. To
serve as basis for acquittal, it must be established by clear and
convincing evidence. For it to prosper, the accused must prove not
only that he was absent from the scene of the crime at the time of
its commission, but also that it was physically impossible for him to
have been present then.

Pilola knew that he was charged for the stabbing but instead of
surrendering to the police authorities, he evaded arrest and this
flight is evidence of guilt.

There is treachery when the offender commits any of the crimes


against persons, employing means, methods or forms in the
execution thereof which tend directly and specially to insured its
execution, without risk to himself arising from the defense which the
offended party might make. The essence of treachery is the swift
and unexpected attack on the unarmed victim without the slightest
provocation on his part - attack on the unarmed victim was
sudden. The aggravating circumstance of abuse of superior
strength is absorbed by treachery.

134
CONTRIBUTOR AROZA, Maria Minette R.
MODULE Conspiracy
TOPIC
CASE TITLE PEOPLE v. LARRAÑAGA G.R.NO. 138874-75
PONENTE PER CURIAM DATE: FEB. 3, 2004
DOCTRINE In conspiracy, direct proof of a previous agreement to commit a
crime is not necessary. It may be shown by the conduct of the
accused before, during, and after the commission of the crime.
The responsibility of a conspirator is not confined to the
accomplishment of a particular purpose of conspiracy but extends
to collateral acts and offenses incident to and growing out of the
purpose intended.
FACTS On the night of July 16, 1997, victims Marijoy and Jacqueline Chiong
failed to come home on the expected time. After two days of their
disappearance, a young woman was found dead at the foot of a cliff.
Her pants were torn, her t-shirt was raised up to her breast and her
bra was pulled down. Her face and neck were covered with masking
tape and attached to her left wrist was a handcuff. The woman was
identified by Chiong Family as Marijoy. One of the accused,
Davidson Rusia, surfaced and admitted before the police having
participated in the abduction of the sisters. He identified the other
appellants as co-perpetrators in the crime. As the state witness,
Rusia testified in the court that in the evening of July 16, 1997, he
met Rowen and Josman and told him to ride with them in a white
car. Following them were Larrañaga, James Anthony, and James
Andrew who were riding in a red car. Josman stopped the white car
in front of the waiting shed where the sisters Marijoy and Jacqueline
were standing and forced them to ride the car. Rusia taped their
mouths while Rowen handcuffed them jointly. After stopping by a
safe house in Guadalupe, they headed to the South Bus Terminal
where they met Alberto and Ariel, driver and conductor of the van,
respectively. They traveled towards South of Cebu City, leaving the
red car at the South Bus Terminal. After parking their cars near a
precipice, they drank and had a pot session. Later, they started
raping Marijoy inside the vehicle and thereafter raped Jacqueline.
Josman instructed Rowen and Ariel to bring Marijoy to the cliff and
push her into the ravine. Rusia’s statements were corroborated by
other witnesses.
ISSUE/S Whether or not there was conspiracy.
RULING/S From the evidence of the prosecution, there is no doubt that all the
appellants conspired in the commission of the crimes charged. Their
concerted actions point to their joint purpose and community of
intent. Well settled is the rule that in conspiracy, direct proof of a
previous agreement to commit a crime is not necessary. It may be
deduced from the mode and manner by which the offense was

135
perpetrated, or inferred from the acts of the accused themselves
when such point to a joint design and community of interest.
Otherwise stated, it may be shown by the conduct of the accused
before, during, and after the commission of the crime. Appellants'
actions showed that they have the same objective to kidnap and
detain the Chiong sisters. Rowen and Josman grabbed Marijoy and
Jacqueline from the vicinity of Ayala Center. Larrañaga, James
Andrew and James Anthony who were riding a red car served as
back-up of Rowen and Josman. Together in a convoy, they
proceeded to Fuente Osmeña to hire a van, and thereafter, to the
safehouse of the "Jozman Aznar Group " in Guadalupe, Cebu where
they initially molested Marijoy and Jacqueline. They headed to the
South Bus Terminal where they hired the white van driven by
Alberto, with Ariel as the conductor. Except for James Andrew who
drove the white car, all appellants boarded the white van where they
held Marijoy and Jacqueline captive. In the van, James Anthony
taped their mouths and Rowen handcuffed them together. They
drank and had a pot session at Tan-awan. They encircled
Jacqueline and ordered her to dance, pushing her and ripping her
clothes in the process. Meanwhile, Larrañaga raped Marijoy,
followed by Rowen, James Anthony, Alberto, and Ariel. On other
hand, Josman and James Andrew raped Jacqueline. Upon
Josman's order, Rowen and Ariel led Marijoy to the cliff and pushed
her. After leaving Tan-awan, they taunted Jacqueline to run for her
life. And when Rusia got off from the van near Ayala Center, the
appellants jointly headed back to Cebu City. Clearly, the argument
of Rowen, Ariel and Alberto that they were not part of the
"conspiracy" as they were merely present during the perpetration of
the crimes charged but not participants therein, is bereft of merit. To
hold an accused guilty as co-principal by reason of conspiracy, he
must be shown to have performed an overt act in pursuance or
furtherance of the complicity. There must be intentional participation
in the transaction with a view to the furtherance of the common
design and purpose. Responsibility of a conspirator is not confined
to the accomplishment of a particular purpose of conspiracy but
extends to collateral acts and offenses incident to and growing out
of the purpose intended. As shown by the evidence for the
prosecution, Rowen, Ariel and Alberto were not merely present at
the scene of the crime. Indeed, all appellants, except James
Anthony who was 16 years old when the crimes charged were
committed, share the same degree of responsibility for their criminal
acts.

136
CONTRIBUTOR Belano, Renato Jr. P.
MODULE Conspiracy and Proposals to Commit a Felony and Felonies as to
TOPIC Severity
CASE TITLE People v. Garchitorena G.R. NO. 175605
PONENTE Leonardo-De Castro, J. DATE: Aug 28, 2009
DOCTRINE Direct proof is not essential, for conspiracy may be inferred from the
acts of the accused prior to, during or subsequent to the incident.
Such acts must point to a joint purpose, concert of action or
community of interest. All conspirators are liable as co-principals
regardless of the intent and the character of their participation,
because the act of one is the act of all.
FACTS Accused Arnold Garchitorena, Joey Pamplona and Jessie Garcia
were charged with the stabbing and killing of Mauro Biay.

Dulce Borero, the victim’s sister testified that on the evening of


September 22, 1995, she and her brother Mauro were selling “balut”
when she was called by the accused Jessie Garcia. When Mauro
approached Jessie, the latter twister his hand behind his back while
later being stabbed by his companions Arnold and Joey repeatedly
with a shiny bladed instrument, until Mauro slumped facedown on
the ground. Arnold then instructed his two companions to run away.

The accused denied the charge against them. Specifically, accused


Joey denied his participation in the stabbing of Mauro, accused
Jessie interposed the defense of alibi, while accused Arnold
interposed the defense of insanity.
ISSUE/S Whether or not conspiracy was present in the commission of the
felony.
RULING/S Yes. Conspiracy exists when two or more persons come to an
agreement concerning the commission of a felony and decide to
commit it. Direct proof is not essential, for conspiracy may be
inferred from the acts of the accused prior to, during or subsequent
to the incident. Such acts must point to a joint purpose, concert of
action or community of interest.

Conspiracy was shown because the three accused were together in


performing the concerted acts in pursuit of their common objective.
Garcia grabbed the victim's hands and twisted his arms, in turn,
Pamplona, together with Garchitorena, strangled him and straddled
him on the ground, then stabbed him. The victim was trying to free
himself from them, but they were too strong. All means through
which the victim could escape were blocked by them until he fell to
the ground and expired. The three accused's prior act of waiting for
the victim outside affirms the existence of conspiracy, for it speaks
of a common design and purpose.

137
Where there is conspiracy, as here, evidence as to who among the
accused rendered the fatal blow is not necessary. All conspirators
are liable as co-principals regardless of the intent and the character
of their participation, because the act of one is the act of all.

138
CONTRIBUTOR BILTZ, Aralind Louise A.
MODULE Conspiracy and Proposals to Commit a Felony
TOPIC
CASE TITLE PEOPLE VS. CARANDANG G.R.NO. 175926
PONENTE LEONARDO-DE CASTRO, J. DATE: JULY 6, 2011
DOCTRINE Conspiracy exists when two or more persons come to an
agreement concerning the commission of a felony and decide to
commit it. Evidence need not establish the actual agreement
among the conspirators showing a preconceived plan or motive for
the commission of the crime. Proof of concerted action before,
during and after the crime, which demonstrates their unity of
design and objective, is sufficient. When conspiracy is established,
the act of one is the act of all regardless of the degree of
participation of each
FACTS The sister of accused Milan requested for assistance from the drug
enforcement unit of the La Loma Police Station 1 regarding a drug
deal that would allegedly take place in her house. When the team
reached the place, the door to Milan’s room was open, enabling the
police officers to see Carandang, Milan and Chua inside. When the
group introduced themselves as police officers, Milan immediately
shut the door.

The police officers pushed the door open, gunshots were heard,
hitting SPO2 Alonzo and PO2 Red who were instantly killed. SPO2
Motecalvo who was right behind SPO2 Red was still aiming his
firearm at the assailants when Carandang shot and hit him.

The RTC found all the accused guilty of two counts of murder and
one frustrated murder.

The CA affirmed the ruling of the Trial Court.


ISSUE/S Whether or not conspiracy exists among the appellants
RULING/S In the case at bar, the conclusion that Milan and Chua conspired
with Carandang was established by their acts (1) before
Carandang shot the victims (Milan’s closing the door when the
police officers introduced themselves, allowing Carandang to wait
in ambush), and (2) after the shooting (Chua’s directive to Milan to
attack SPO1 Montecalvo and Milan’s following such instruction).
Contrary to the suppositions of appellants, these facts are not
meant to prove that Chua is a principal by inducement, or that
Milan’s act of attacking SPO1 Montecalvo was what made him a
principal by direct participation. Instead, these facts are convincing
circumstantial evidence of the unity of purpose in the minds of the
three. As co-conspirators, all three are considered principals by
direct participation.

139
CONTRIBUTOR CACHERO, Luis III L.
MODULE Conspiracy
TOPIC
CASE TITLE PEOPLE V. MARCELINO G.R.NO. 201860
DADAO, ET. AL.
PONENTE LEONARDO-DE CASTRO, J. DATE: January 22,
2014
DOCTRINE A conspiracy exists when two or more persons come to an
agreement concerning the commission of a felony and decide to
commit it.
FACTS Accused-appellants Marcelino Dadao, Antonio Sulindao, Eddie
Malogsi (Deceased), and Alfemio Malogsi were charged with the
crime of Murder for the death of Pionio Yacapin. Ronie Dacion, the
prosecution’s first witness, testified that on July 11, 1993 at about
7:30 in the evening, he saw accused Dadao, Sulindao, Antonio and
Eddie Malogsi helping each other and with the use of firearms and
bolos, shot to death the victim, Pionio Yacapin in their house at
Barangay Saculot, Talakag, Bukidnon. This was corroborated by his
brother Edgar, the second witness. Another witness, SPO2 Nestor
Aznar, testified that he was the one who prepared the sketch of the
hut where the incident happened and further testified that the four
accused were in the custody of the government and in the following
morning of the incident, he was at the scene of the crime and found
in the yard of the hut eight (8) garand empty shells caliber 30.

ISSUE/S Whether or not the accused-appellants are equally liable for the
death of the victim.
RULING/S Yes they are all equally liable. There is conspiracy when two or more
persons come to an agreement concerning the commission of a
felony and then decide to commit it. It does not matter who inflicted
the mortal wound, as each of the actors incurs the same criminal
liability, because the act of one is the act of all.

140
CONTRIBUTOR CALZADO, Anne Valerie L.
MODULE Conspiracy and proposals to commit a felony (Art. 8, RPC)
TOPIC
CASE TITLE PEOPLE v. OCTA G.R.NO. 195196
PONENTE SERENO, CJ. DATE: JULY 13, 2015
DOCTRINE To hold an accused guilty as a coprincipal by reason of conspiracy,
he must be shown to have performed an overt act in pursuance or
furtherance of the complicity
FACTS On September 25, 2003, Johnny Corpuz and Mike Adrian Batuigas,
onboard a Honda Civic, were blocked by a Mitsubishi box type
Lancer. The four armed occupants of the Lancer car alighted. One
of the armed men fired his pistol at the left window of the civic car,
thus compelling Johnny to open the locked door of the car. Inside
the car, Johnny was handcuffed, blindfolded and boxed. After being
taken against their will by four armed occupants of a Mistubishi
Lancer, the kidnappers communicated with Johnny’s wife, Ana
Marie Corpuz, informing her that they have in their custody her
husband, Johnny, and her brother, Mike Adrian. The kidnappers
started demanding the amount of P20 million for the release of her
husband and her brother but the amount was considerably reduced
to the amount of P538,000.00 which was accepted by the
kidnappers.

On September 30, 2003, the kidnappers set up the manner on how


the ransom money would be delivered. She was instructed to deliver
the wrapped bundled ransom money to the man wearing red cap.
When she saw the man with red cap at the designated place, she
was asked for the money. At first, she did not give the money
because she wanted to be sure that she was giving the money to
the right man. Using her own cellphone, she called up the man who
had been instructing her and asked him to confirm if the man in front
of her is the right man to give the ransom money to. The man in the
phone and the man in the red cap talked for a while in another
dialect which Ana Marie did not understand. When she asked the
man to give back her cellphone back, he refused and, instead
instructed her to give the money to him. She described the man
wearing red cap to be good looking, lightly built, in his early 20s,
around 5’4" in height and with dimples, which she later identified in
court as the accused-appellant.

On October 1, 2003, Johnny and Mike Adrian were released after


the payment of the ransom money.
ISSUE/S Whether or not the court gravely erred in finding accused-appellant
to be a conspirator to the crime charged.

141
RULING/S No. Accused-appellant is liable as a principal for being a
coconspirator in the crime of Kidnapping for ransom.
Conspiracy exists when two or more persons come to an agreement
concerning the commission of a felony and decide to commit it.
Where all the accused acted in concert at the time of the
commission of the offense, and it is shown by such acts that they
had the same purpose or common design and were united in its
execution, conspiracy is sufficiently established. It must be shown
that all participants performed specific acts with such closeness and
coordination as to indicate a common purpose or design to commit
the felony.

Evidently, to hold an accused guilty as a coprincipal by reason of


conspiracy, he must be shown to have performed an overt act in
pursuance or furtherance of the complicity.

Accused-appellant was the one who received the ransom money,


the commonality of purpose of the acts of accused-appellant
together with the other accused can no longer be denied. Such acts
have the common design or purpose to commit the felony of
kidnapping for ransom.

Moreover, at the time accused-appellant received the ransom


money, the crime of kidnapping was still continuing, since both
victims were still being illegally detained by the kidnappers. While
his receipt of the ransom money was not a material element of the
crime, it was nevertheless part of the grand plan and was in fact the
main reason for kidnapping the victims.

142
CONTRIBUTOR CARPIO, Anna Clarissa C.
MODULE Module 4: Conspiracy and Proposals to commit felony; and
TOPIC felonies as to severity
CASE TITLE PEOPLE v. FELICIANO G.R.NO. 196735
PONENTE Leonen, J. DATE: May 5, 2014
DOCTRINE Conspiracy once proven, has the effect of attaching liability to all the
accused, regardless of their degree of participation, thus, once an
express or implied conspiracy is proved, all of the conspirators are
liable as co-principals regardless of the extent and character of their
respective active participation in the commission of the crime or
crimes perpetrated in furtherance of the conspiracy because in
contemplation of law the act of one is the act of all. The moment it
is established that the malefactors conspired and confederated in
the commission of the felony proved, collective liability of the
accused conspirators attaches by reason of the conspiracy, and the
court shall not speculate nor even investigate as to the actual
degree of participation of each of the perpetrators present at the
scene of the crime.
FACTS At around 12:30 p.m. to 1:00 p.m. in December 1994, 7 members of
the Sigma Rho fraternity were eating lunch at the Beach House
Canteen near the main library of U.P. Diliman, when they were
attacked by several masked men carrying baseball bats and lead
pipes and some of them were hospitalized due to the injuries
sustained while Dennis Venturina died. Venturina was hit on the
head and clubbed on different parts of the body, inflicting upon him
serious and mortal injuries which were the direct and immediate
cause of his death. Thereafter, an Information for murder was filed
against 12 members of the Scintilla Juris fraternity including the
accused-appellants Danilo Feliciano Jr., Julius Victor Medalla,
Christopher Soliva, Warren Zingapan and Robert Michael Alvir at
the RTC-Quezon City. Separate Informations were also filed against
the 12 for attempted murder of Cesar Mangrobang, Jr., Cristobal
Gaston, Jr., and Leandro Lanchica; and the frustrated murder of
Mervin Natalicio and Mel Fortes.

The prosecution maintains that all the victims were members of the
Sigma Rho Fraternity and were having lunch when suddenly,
deceased Venturina shouted “Brads, brods!” Thereafter, 2 members
saw that about 10 men who were armed with baseball bats and lead
pipes and their heads covered with either handkerchiefs or shirts,
were charging toward them and suddenly attacked them; and during
which, Leandro Lanchica, recognized one of the attackers as Alvir,
one of the accused-appellants, because his mask fell off. Lanchica
tried to parry the blows and suffered scratches and contusions but
was able to run to the nearby building. Before Lanchica reached the
building, he looked back and saw accused Zingapan and Medalla

143
holding lead pipes and standing where the commotion was, without
their masks on. Lanchica was familiar with them because he often
saw them in the campus and Zingapan used to be his friend. The
attacked lasted about 30 to 45 minutes.

Mervin Natalicio testified that there were about 15 to 20 men, most


of them wearing masks who attacked them. He ran but the attackers
came after him and beat him up until he fell down. During the attack,
which lasted about 30 seconds, he was able to recognize 2 of the
attackers, Zingapan and Soliva, as both were not wearing masks.
He got up after and saw another group of 4 to 5 men coming toward
him, who also beat him up. Afterwhich, several bystanders brought
him to the infirmary where he stayed for a week for treatment.

Cesar Mangrobang, Jr. stated that he recognized accused Gilbert


Magpantay and Carlo Fajardo whose masks fell off while he was
parrying the blows. He successfully ran to the library but returned to
the scene to help and there he saw Venturina lying on the ground,
being beaten by accused Feliciano Jr. with a lead pipe while
accused Narag was aiming to hit at the same. When the 2 saw
Cesar, they charged towards him but ran away when someone
shouted that policemen were coming. Victim Amel Fortes also saw
Feliciano beating Veturina and also saw Zingapan and George
Morano in the scene. Venturina was brought to the U.P. Infirmary
but was later transferred to St. Lukes Hospital where he died 2 days
after which, as per autopsy was due to “traumatic head injuries”
caused by a hard blunt object.

The defense however avers that the identity of the attackers was not
established. A member of the U.P. Police who responded to the
scene asked Natalicio who attacked him but the latter only replied
that they were wearing masks. The canteen utility worker also
testified that the attackers wore masks hence they are
unrecognizable but he admitted not having seen the attack but saw
a man sprawled on the ground. Likewise, another worker assigned
in the library testified that the men who attacked wore masked.
Students who were sorority members also testified for the defense
stating that the attackers wore masks so they were unidentifiable
and also claimed that none of the masks fell off but one of them
admitted that she was approached by Scintilla Juris to make a
statement. Another corroborated the statement.

Accused Feliciano’s mother meanwhile testified that Feliciano could


not have been at the scene because her son went to Pampanga that
day. Likewise, accused Alvir stated that he was sick since days
before the incident and was absent from school and work on that

144
day. He further stated that it was impossible for Lachica to have
identified him as he wore glasses and he was not enrolled in U.P. at
that time as he was working. Medalla, Soliva and Zingapan each
averred that they were not in U.P. during the incident and all their
statements were corroborated by other testimonies. Medalla also
stated he was injured and was working on a school project during
the rumble, Soliva was out eating lunch with friends, while Zingapan
was buying a gift at a mall.

The trial court found Alvir, Feliciano, Jr., Soliva, Medalla and
Zingapan guilty of murder and attempted murder and was
sentenced, among other penalties with reclusion perpetua. Other
accused were acquitted. On appeal, the Court of Appeals modified
the liabilities as guilty of the attempted murder of Natalicio and
Fortes but not against Mangrobang, Lachica, and Gaston which was
only deemed as slight physical injuries, on the ground that to the
latter, they “were no longer chased by the attackers” thus, the
accused voluntarily desisted from pursuing them and absent of
intent to kill. Thus this case.
ISSUE/S 1. Whether or not the accused-appellant’s constitutional rights
were violated when the information against them contained
the aggravating circumstance of the use of masks despite the
prosecution witnesses to prove that the masks fell off?
2. Whether or not there is conspiracy in the case at bar?
RULING/S 1. NO, accused-appellant’s constitutional rights were NOT
violated. The Information is sufficient when the accused is
fully apprised of the charged against him to enable him to
prepare his defense.

The defense’s argument that the prosecution should not have


included in the Information the phrase “wearing masks and/or other
forms of disguise” since they presented testimonial evidence that
not all accused were wearing masks or that the same fell off is
untenable. A complaint or information is sufficient if it states the (1)
name of the accused; the (2) designation of the offense given by the
statue; the (3) acts or omissions complained of as constituting the
offense; the (4) name of the offended party; the (5) approximate date
of the commission of the offense; and the (6) place where the
offense was committed. The test of sufficiency of Information is
whether it enables a person of common understanding to know the
charge against him, and the court to render judgement properly with
the purpose of allowing the accused to fully prepare for his defense,
precluding surprises during trial.

In the case, the inclusion of the assailed phrase does not violate the
accused’s constitutional rights. The introduction of the prosecution’s

145
testimonial evidence that tends to prove that the accused were
masked but the masks fell off does NOT prevent them from including
disguise as an aggravating circumstance. What is important in
alleging disguise as an aggravating circumstance is that there was
concealment of identity by the accuse. The inclusion of disguise
in the information was, therefore, enough to sufficiently apprise the
accused that in the commission of the offense they were being
charged with, they tried to conceal their identity.

Likewise, the introduction of evidence which shows that some of the


accused were not wearing masks did not violate their right to be
informed of their offenses. The information charges conspiracy
among the accused. Conspiracy presupposes that “the act of
one is the act of all,” meaning, all the accused had been one in
their plan to conceal their identity even if there was evidence
late on to prove that some of them might have not done so. All
that is needed for the information to be sufficient is that the elements
of the crime (murder, frustrated and attempted murder) have been
alleged and that there are sufficient details as to the time, place, and
persons involved in the offense.

2. YES. The trial court ruled the presence of conspiracy among


the accused-appellants and the CA sustained this finding.
Therefore, the modification of the liabilities from attempted murder
to slight physical injuries is erroneous.

Conspiracy once proven, has the effect of attaching liability to


all the accused, regardless of their degree of participation, thus,
once an express or implied conspiracy is proved, all of the
conspirators are liable as co-principals regardless of the extent
and character of their respective active participation in the
commission of the crime or crimes perpetrated in furtherance
of the conspiracy because in contemplation of law the act of
one is the act of all. Although it is axiomatic that no one is liable for
acts other than his own, when two or more persons agree or
conspire to commit a crime, each is responsible for all the acts of
the others, done is furtherance of the agreement or conspiracy.

The imposition of collective liability upon the conspirators is because


it is impossible to graduate the separate liability of each without
taking into consideration the close and inseparable relation of each
of them with the criminal act, for the commission of which they all
acted by common agreement. Therefore, the crime must be in the
view of the solidarity of the act and intent which existed between
them.

146
The moment it is established that the malefactors conspired
and confederated in the commission of the felony proved,
collective liability of the accused conspirators attaches by
reason of the conspiracy, and the court shall not speculate nor
even investigate as to the actual degree of participation of each
of the perpetrators present at the scene of the crime.

In the case, the liabilities of the accused-appellants arose from a


single incident wherein they were armed with baseball bats and lead
pipes, all in agreement to do the highest amount of damage possible
to the victims. The intent to kill was already present the moment of
attack and that intent was shared by all of the accused-appellants
alike when the presence of conspiracy was proven. It is, therefore,
immaterial to distinguish between the seriousness of the injuries
suffered by the victims to determine the respective liabilities of their
attackers. What is relevant only as to whether the death occurs as
a result of that intent to kill and whether there are qualifying,
aggravating or mitigating circumstances that can be appreciated.

147
CONTRIBUTOR CHUA, Kristy Anne
MODULE CONSPIRACY AND PROPOSALS TO COMMIT A FELONY AND
TOPIC FELONIES AS TO SEVERITY
CASE TITLE PEOPLE v. MORILLA G.R.NO. 189833
PONENTE PEREZ, J.: DATE: FEB. 5, 2014
DOCTRINE A conspiracy exists when two or more persons come to an
agreement concerning the commission of a felony and decide to
commit it. To determine conspiracy, there must be a common
design to commit a felony.
FACTS On October 13, 2001, in Barangay Kiloloran, Municipality of Real,
Province of Quezon, Philippines, the following individuals; Morilla,
Mayor Mitra, Yang and Dequilla belonging in an syndicate crime
group were transporting shabu or methamphetamine hydrochloride
with an approximate weight of five hundred three point sixty eight
kilos without authority by means of two motor vehicles, namely a
Starex van with a commemorative plate to read “Mayor” and a
municipal ambulance of Panukulan, Quezon Province. The
Regional Trial Court found Mayor Mitra and Morilla guilty while
Dequilla and Yang were acquitted due to prosecution’s failure to
present sufficient evidence to convict those individuals. The search
at a checkpoint conducted by police officers on the vehicles driven
by Mayor Mitra and Morilla was ruled valid by the court because the
police officers had prior knowledge of the suspected vehicles used
to transport dangerous drugs.

Mayor Mitra’s defense was dismissed stating that he was without


knowledge of the contents of the sacks and that he was merely
transporting the contents for Ben Tan, the buyer of his fishing boat.
The defense of Morilla was also dismissed, Morilla stated that he
lacked knowledge of the illegality of the contents, insisting that he
thought those were mere wooden tiles and electronic spare parts.
Dequilla and Yang were passengers of the ambulance. Mitra and
Morilla were caught in flagrante delicto of transporting dangerous
drugs in separate vehicles. It was found that the Starex van passed
the checkpoint while the ambulance was stopped by police officers,
even with a tinted window, the police officers noticed multiple sacks
inside. The officers requested to inspect the sacks but Morilla tried
explaining that he was with Mayor Mitra which was rejected by the
police officers, during the inspection, it was proven that the sacks
contained shabu. Thus, causing the police officers to overtake
Mayor Mitra’s van and inspect his van, finding the same contents.
ISSUE/S 1. ) Whether or not Morilla can be convicted for conspiracy to
commit the offense alleged.

148
2. ) Whether or not his culpability as alleged in the information
was proven by the prosecution.
RULING/S 1. ) Yes, the Supreme Court affirms the finding of conspiracy. As
stated in conspiracy, it does not need to show that the parties
physically came together and agreed expressly to enter into and
pursue a common design. The factual circumstances lead to a
conclusion that Morilla conspired with Mayor Mitra to transport
illegal drugs. Both vehicles were loaded with several sacks of
shabu while on convoy from Quezon to Manila. Morilla was not
able to pass the checkpoint successfully unlike Mayor Mitra,
Morilla was asked to open the sacks in the ambulance but in an
attempt to be free, he told the police officers that he was with
Mayor Mitra which was rejected by the police officers.
2.) Yes, Mayor Mitra and Morilla were caught in flagrante delicto in
the act of transporting dangerous drugs. The act of transporting
shabu is considered malum prohibitum which is a punishable
offense under a special law. Transportation of dangerous drugs
does not require, the mere fact of such does not require the proof of
criminal intent, motive or knowledge.

149
CONTRIBUTOR CRUZ, Jakielyn Anne O.
MODULE MODULE 4: Conspiracy and Proposals to Commit a Felony (Art.
TOPIC 8, RPC) and Felonies as to Severity (Art. 7 And 9, RPC)
CASE TITLE PEOPLE v. BOKINGCO G.R.NO. 187536
PONENTE PEREZ, J: DATE: AUG 10, 2011
DOCTRINE Conspiracy exists when two or more persons come to an agreement
to commit an unlawful act. It may be inferred from the conduct of
the accused before, during, and after the commission of the crime.
Conspiracy may be deduced from the mode and manner in which
the offense was perpetrated or inferred from the acts of the accused
evincing a joint or common purpose and design, concerted action,
and community of interest. Unity of purpose and unity in the
execution of the unlawful objective are essential to establish the
existence of conspiracy.
As a rule, conspiracy must be established with the same quantum
of proof as the crime itself and must be shown as clearly as the
commission of the crime.
FACTS The victim Noli Pasion and his wife, Elsa owned a pawnshop which
formed part of his house and maintained two rows of apartment
units at the back of his house. The first row was finished and one of
the units was leased to Pasion’s brother-in-law Dante Vitalico, while
the second row was still under construction.
On February 29, 2000, while Vitalico was doing his laundry, he
heard a commotion in one of the apartments and decided to check
it. He saw from the screen door the accused Michael Bokingco
hitting something on the floor. When the accused Bokingco saw
him, he was attacked by Bokingco using a hammer. Vitalico was hit
by Bokingco and was only able to escape when he bit the neck of
Bokingco. Vitalico was informed by his wife that Pasion was found
dead in the kitchen of Apartment No. 3. Both Pasion and Vitalico
were brought to the hospital but Pasion eventually died.
During the commotion, Elsa was in the master’s bedroom when she
heard the banging sound and her husband’s moans. She
immediately got off the bed and went down but was blocked by
Reynante Col. Col ran towards Elsa, sprayed tear gas and poked
her with a sharp object in the chin, asking her to open the vault in
the pawnshop. Elsa offered money instead since she does not know
the vault combination, but Col dragged her towards the back door
by holding her neck and pulling her backward. Before they reach
the door, Bokingco called Col saying, “tara, patay na siya.” Col
immediately let Elsa go and ran away with Bokingco.

150
ISSUE/S Whether or not Reynante Col is guilty of murder as co-conspirator
of Michael Bokingco?
RULING/S NO. Conspiracy exists when two or more persons come to an
agreement to commit an unlawful act. It may be inferred from the
conduct of the accused before, during, and after the commission of
the crime. Conspiracy may be deduced from the mode and manner
in which the offense was perpetrated or inferred from the acts of the
accused evincing a joint or common purpose and design, concerted
action, and community of interest. Unity of purpose and unity in the
execution of the unlawful objective are essential to establish the
existence of conspiracy. As a rule, conspiracy must be established
with the same quantum of proof as the crime itself and must be
shown as clearly as the commission of the crime.
Nobody witnessed the commencement of the attack. Col was not
seen at the apartment where Pasion was being attacked by
Bokingco. Based on these acts alone, it cannot be logically inferred
that Col conspired with Bokingco in killing Pasion. At the most, Col's
actuations can be equated to attempted robbery, which was actually
the initial information filed against appellants before it was
amended, on motion of the prosecution, for murder.
Elsa testified that she heard Bokingco call out to Col that Pasion
had been killed and that they had to leave the place. This does not
prove that they acted in concert towards the consummation of the
crime. It only proves, at best, that there were two crimes committed
simultaneously and they were united in their efforts to escape from
the crimes they separately committed.
Their acts did not reveal a unity of purpose that is to kill Pasion.
Bokingco had already killed Pasion even before he sought Col.
Their moves were not coordinated because while Bokingco was
killing Pasion because of his pent-up anger, Col was attempting to
rob the pawnshop.
As much as Bokingco's extrajudicial confession is inadmissible
against him, it is likewise inadmissible against Col, specifically
where he implicated the latter as a cohort. Under Section 28, Rule
130 of the Rules of Court, the rights of a party cannot be prejudiced
by an act, declaration, or omission of another. In order that the
admission of a conspirator may be received against his or her co-
conspirators, it is necessary that first, the conspiracy be first proved
by evidence other than the admission itself; second, the admission
relates to the common object; and third, it has been made while the
declarant was engaged in carrying out the conspiracy. As we have
previously discussed, we did not find any sufficient evidence to
establish the existence of conspiracy. Therefore, the extrajudicial
confession has no probative value and is inadmissible in evidence
against Col.

151
CONTRIBUTOR DELA CRUZ, Ma. Luzelle P.
MODULE Wheel Conspiracy
TOPIC
CASE TITLE FERNAN, JR. v. PEOPLE OF THE G.R.NO. 145927
PHILIPPINES
PONENTE VELASCO, JR., J.: DATE: AUG. 24,
2007
DOCTRINE Conspiracy may be implied if it is proved that two or more persons
aimed their acts towards the accomplishment of the same unlawful
object, each doing a part so that their combined acts, though
apparently independent of each other, were in fact, connected and
cooperative, indicating a closeness of personal association and a
concurrence of sentiment.
FACTS Sometime in February, 1977, accused Rolando Mangubat (Chief
Accountant), Delia Preagido (Accountant III), Jose Sayson (Budget
Examiner), and Edgardo Cruz (Clerk II), all of MPH Region VII, met
at the Town and Country Restaurant in Cebu City and hatched an
ingenious plan to siphon off large sums of money from government
coffers. Mangubat had found a way to withdraw government money
through the use of fake LAAs, vouchers and other documents and
to conceal traces thereof with the connivance of other government
officials and employees. In fine, the fraudulent scheme involved the
splitting of LAAs and RSEs so that the amount covered by each
general voucher is less than P50,000.00 to do away with the
approval of the Regional Auditor; the charging of disbursements to
unliquidated obligations due the previous year to provide the
supposed source of funds; and the manipulation of the books of
account by negation or adjustment, i.e., the cancellation of checks
through journal vouchers to conceal disbursements in excess of the
cash disbursement ceiling (CDC), so as not to reflect such
disbursements in the trial balances submitted to the Regional Office.

The nuclei of this massive conspiracy, namely: Rolando Mangubat,


Jose Sayson, and Edgardo Cruz, all of MPH Region VII, were found
guilty in all 119 counts and were accordingly sentenced by the SB.
The other conniver, Delia Preagido, after being found guilty in some
of the cases, became a state witness in the remainder. On the basis
of her testimony and pertinent documents, Informations were filed,
convictions were obtained, and criminal penalties were imposed on
the rest of the accused.

The four formed the nucleus of the nefarious conspiracy. Other


government employees, tempted by the prospect of earning big
money, allowed their names to be used and signed spurious
documents.

152
Fernan, Jr. and Torrevilas (Petitioners) were both Civil Engineers of
the MPH assigned to the Cebu First Highway Engineering District.
Petitioner Fernan, Jr. was included among the accused in Criminal
Case Nos. 2879, 2880, 2881, 2885, 2914, and 2918 allegedly for
having signed six (6) tally sheets or statements of deliveries of
materials, used as bases for the preparation of the corresponding
number of general vouchers. Fund releases were made to the
suppliers, contractors, and payees based on these general
vouchers.
ISSUE/S WON petitioners' guilt has not been established beyond reasonable
doubt and, hence, exculpation is in order.

WON Petitioners are indeed co-conspirators as ruled by Hon.


Sandiganbayan.
RULING/S Reasonable doubt is present when: after the entire comparison and
consideration of all the evidences, leaves the minds of the [judges]
in that condition that they cannot say they feel an abiding conviction,
to a moral certainty, of the truth of the charge; a certainty that
convinces and directs the understanding, and satisfies the reason
and judgment of those who are bound to act conscientiously upon
it.
Petitioners readily admitted that they either signed the tally sheets
and/or delivery receipts, reports of inspection, requests for supplies
and materials, and other related documents which became part of
the supporting documents that led to the issuance of general
vouchers and eventually the disbursement of public funds.
Conspiracy may be implied if it is proved that two or more persons
aimed their acts towards the accomplishment of the same unlawful
object, each doing a part so that their combined acts, though
apparently independent of each other, were in fact, connected and
cooperative, indicating a closeness of personal association and a
concurrence of sentiment.
To hold an accused guilty as a co-principal by reason of conspiracy,
he must be shown to have performed an overt act in pursuance or
furtherance of the complicity. There must be intentional participation
in the transaction with a view to the furtherance of the common
design and purpose.

We find that the conspiracy in the instant cases resembles the


"wheel" conspiracy. The 36 disparate persons who constituted the
massive conspiracy to defraud the government were controlled by
a single hub, namely: Rolando Mangubat (Chief Accountant), Delia
Preagido (Accountant III), Jose Sayson (Budget Examiner), and
Edgardo Cruz (Clerk II), who controlled the separate "spokes" of the
conspiracy. Petitioners were among the many spokes of the wheel.

153
The feeble defense of petitioners that they were not aware of the
ingenuous plan of the group of accused Mangubat and the
indispensable acts to defraud the government does not merit any
consideration. The State is not tasked to adduce direct proof of the
agreement by petitioners with the other accused, for such
requirement, in many cases, would border on near impossibility.
The State needs to adduce proof only when the accused committed
acts that constitute a vital connection to the chain of conspiracy or
in furtherance of the objective of the conspiracy. In the case at
bench, the signing of the fake tally sheets and/or delivery receipts,
reports of inspection, and requests for supplies and materials by
petitioners on separate occasions is vital to the success of the
Mangubat Group in siphoning off government funds. Without such
fabricated documents, the general vouchers covering the supply of
materials cannot be properly accomplished and submitted to the
disbursing officer for the preparation of checks.
When each and everyone of the accused in the instant cases
performed their assigned tasks and roles with martinet-like precision
and accuracy, by individually performing essential overt acts, so
much so that the common objective is attained, which is to secure
the illegal release of public funds under the guise of fake or
simulated public documents, then each and everyone of said
accused are equally liable as co-principals under the well-
established and universally-accepted principle that, once a
conspiracy is directly or impliedly proven, the act of one is the act
of all and such liability exists notwithstanding no-participation
in every detail in the execution of the offense.each and everyone
of the accused in the instant cases performed their assigned tasks
and roles with martinet-like precision and accuracy, by individually
performing essential overt acts, so much so that the common
objective is attained, which is to secure the illegal release of public
funds under the guise of fake or simulated public documents, then
each and everyone of said accused are equally liable as co-
principals under the well-established and universally-accepted
principle that, once a conspiracy is directly or impliedly proven, the
act of one is the act of all and such liability exists notwithstanding
no-participation in every detail in the execution of the offense.
DENY the petition and AFFIRM the Decision of the SB in the
consolidated criminal cases subject of this petition.

154
CONTRIBUTOR DIMAKUTA, Jasmine
MODULE CONSPIRACY
TOPIC
CASE TITLE GLORIA MACAPAGAL ARROYO V. G.R. NO. 220598
PEOPLE OF THE PHILIPPINES
PONENTE BERSAMIN, J. DATE: JULY 19,
2016
DOCTRINE Conspiracy exists when two or more persons come to an agreement
concerning the commission of a felony, and decide to commit it. In
this jurisdiction, conspiracy is either a crime in itself or a mere means
to commit a crime.


The community of design to commit an offense must be a conscious


one. Conspiracy transcends mere companionship, and mere
presence at the scene of the crime does not in itself amount to
conspiracy. Even knowledge of, or acquiescence in, or agreement
to cooperate is not enough to constitute one a party to a conspiracy,
absent any active participation in the commission of the crime with
a view to the furtherance of the common design and purpose.
Hence, conspiracy must be established, not by conjecture, but by
positive and conclusive evidence.

FACTS The State presented Atty. Aleta Tolentino as its main witness
against all the accused. Presently, she is a Member of the Board of
Directors of the PCSO. The Board appointed her as Chairman of an
Audit Committee. The audit review proceeded when she reviewed
the COA Annual Reports of the PCSO for 2006 2007 2008 and
2009, and the annual financial statements contained therein for the
years 2005 to 2009. The reports were given to them by the COA.
These are transmitted to the PCSO annually after the subject year
of audit.
One of her major findings was that the former management of the
PCSO was commingling the charity fund, the prize fund and the
operating fund. By commingling she means that the funds were
maintained in only one main account. This violates Section 6 of
Republic Act 1169 (PCSO Charter) and generally accepted
accounting principles. The Audit Committee also found out that
there was excessive disbursement of the Confidential and
Intelligence Fund (CIF). There were also excessive disbursements
for advertising expenses. The internal audit department was also
merged with the budget and accounting department, which is a
violation of internal audit rules.
There was excessive disbursement of the CIF because the PCSO
was given only P10 million in 2002, i.e. P5 million for the Office of

155
the Chairman and P5 million for the Office of the General Manager.
Such allocation was based on the letters of then Chairman Lopez
and then General Manager Golpeo, asking for P5 million intelligence
fund each. Both were dated February 21, 2000, and sent to then
President Estrada, who approved them. This allocation should have
been the basis for the original allocation of the CIF in the PCSO, but
there were several subsequent requests made by the General
Manager during the time of, and which were approved by, former
President Arroyo.
The allocation in excess of P10 million was in violation of the PCSO
Charter. PCSO did not have a budget for this. They were working
on a deficit from 2004 to 2009. The charter allows only 15% of the
revenue as operating fund, which was already exceeded. The
financial statements indicate that they were operating on a deficit in
the years 2006 to 2009. It is within the power of the General
Manager to ask for additional funds from the President, but there
should be a budget for it. The CIF should come from the operating
fund, such that, when there is no more operating fund, the other
funds cannot be used. The funds were maintained in a commingled
main account and PCSO did not have a registry of budget utilization.
The excess was not taken from the operating fund, but from the
prize fund and the charity fund.
In 2005, the deficit was P916 million; in 2006, P1,000,078,683.23.
One of the causes of the deficit for 2006 was the CIF expense of
P215 million, which was in excess of the approved allocation of P10
million. The net cash provided by operating expenses in 2006 is
negative, which means that there were more expenses than what
was received.
Atty. Tolentino received a copy of the PCSO corporate operating
budget (COB) for the year 2008 in 2010 because she was already
a member of its Board of Directors. The 2008 approved COB has a
comparative analysis of the actual budget for 2007. It is stated there
that the budget for CTF in 2007 is only P25,480,550. But the
financial statements reflect P77 million. The budget was prepared
and signed by then PCSO General Manager Rosario Uriarte. It had
accompanying Board Resolution No. 305, Series of 2008, which
was approved by then Chairperson Valencia, and board members
Valdes, Morato, Domingo, and attested to by Board Secretary Atty.
Ronald T. Reyes.

In the 2008 COA report, it was noted that there was still no deposit
to the prize and charity funds, adverted in the 2007 COA report.
There was already a recommendation by the COA to separate the
deposits or funds in 2007. But the COA noted that this was not

156
followed. The financial statements show the Confidential and the
Extra-Ordinary Miscellaneous Expenses account is P38,293,137,
which is more than the P10 million that was approved. The
Confidential and Extra-Ordinary Miscellaneous Expenses is the
account being used for confidential and intelligence expenses. The
amount in the financial statements is over the budgeted amount of
P28 million. Further, the real disbursement is more than that, based
on a summary of expenditures she had asked the treasurer to
prepare.
After Atty. Tolentino concluded the audit review, she reported her
findings to the Board of Directors in one of their executive meetings.
The Board instructed her to go in-depth in the investigation of the
disbursements of CIF. The Audit Committee also asked Aguas why
there were disbursements in excess of P10 million. He explained
that there were board resolutions confirming additional CIF which
were approved by former President Arroyo. The board secretary,
Atty. Ed Araullo, gave them the records of those resolutions. In the
records that Araullo submitted to Atty. Tolentino, it appears that
Uriarte would ask for additional CIF, by letter and President Arroyo
approves it by affixing her signature on that same letter-request.
There were seven letters or memoranda to then President Arroyo,
with the subject "Request for Intelligence Fund."
She then asked their Treasurer, Mercy Hinayon, to give her a
summary of all the disbursements from CIF from 2007 to 2010. The
total of all the amounts in the summaries for three years is
P365,997,915. Atty. Tolentino considered the following laws: R.A.
1169, as amended (PCSO Charter); P.D. 1445 (COA Code); LOI
1282; COA Circular 92-385, as amended by Circular 2003-002,
which provides the procedure for approval of disbursements and
liquidation of confidential intelligence funds. She made a
handwritten flowchart of the allocations/disbursements/liquidation
and audit of the CIF, based on LOI 1282 and the COA Circulars.
The first step is the provision or allotment of a budget because no
CIF fund can be disbursed without the allocation. This is provided in
the second whereas clause of Circular 92-385. For GOCCs,
applying Circular 2003-002, there must be allocation or budget for
the CIF and it should be specifically in the corporate operating
budget or would be taken from savings authorized by special
provisions.

This was not followed in the PCSO CIF disbursement in 2008. The
disbursement for that year was P86,555,060. The CIF budget for
that year was only P28 million, and there were no savings because
they were on deficit. This was also not followed for the year 2009.

157
The CIF disbursement for that year was P139,420,875. But the CIF
budget was only P60 million, and there was also no savings, as they
were in deficit. For the year 2010, the total disbursement, as of June
2010, was P141,021,980. The budget was only P60 million.
The requirements in the disbursement of the CIF are the budget and
the approval of the President. If the budget is correct, the President
will approve the disbursement or release of the CIF. In this case, the
President approved the release of the fund without a budget and
savings. Also, the President approved the same in violation of LOI
1282, because there were no detailed specific project proposals and
specifications accompanying the request for additional CIF. The
requests for the year 2008, 2009 and 2010 were uniform and just
enumerated the purposes, not projects. They did not contain what
was required in the LOI.
The third step is the designation of the disbursing officer. In this
case, the Board of Directors designated Uriarte as Special
Disbursing Officer (SDO) for the portion of the CIF that she
withdrew. For the portion withdrawn by Valencia, there was no
special disbursing officer designated on record. The designation of
Uriarte was in violation of internal control which is the responsibility
of the department head, as required by Section 3 of Circular 2003-
002. When she went through copies of the checks and
disbursement vouchers submitted to her, she found out that Uriarte
was both the SDO and the authorized officer to sign the vouchers
and checks. She was also the payee of the checks. All the checks
withdrawn by Uriarte were paid to her and she was also the
signatory of the checks. Aside from Uriarte, Valencia also disbursed
funds in the CIF. For the funds withdrawn by Valencia, he was also
the authorized officer to sign the vouchers and checks. He was also
the payee of the checks.
Uriarte and Valencia signed the vouchers to certify to the necessity
and legality of the vouchers; they also signed to approve the same,
signify they are "okay" for payment and claim the amount certified
and approved as payee. Gloria P. Araullo signed as releasing
officer, giving the checks to the claimants. Accused Aguas signed
the vouchers to certify that there are adequate funds and budgetary
allotment, that the expenditures were properly certified and
supported by documents, and that the previous cash advances were
liquidated and accounted for. This certification means that the cash
advance voucher can be released. This is because the COA rule on
cash advance is that before any subsequent cash advance is
released, the previous cash advance must be liquidated first. This
certification allowed the requesting party and payee to get the cash
advance from the voucher. Without this certification, Uriarte and

158
Valencia could not have been able to get the cash advance.
Otherwise, it was a violation of P.D. 1445 (Government Auditing
Code).

Valencia designated himself as SDO upon the recommendation of


COA Auditor Plaras. There was no board resolution for this
designation. There was just a certification dated February 2, 2009.
This certification was signed by Valencia himself and designates
himself as the SDO since he is personally taking care of the funds
which are to be handled with utmost confidentiality. The witness is
familiar with Valencia's signature because it appears on PCSO
official documents. Under COA rules, the Board of Directors has
authority to designate the SDO. The chairman could not do this by
himself.

COA Circulars 92-385 and 2003-002 indicate that to disburse CIF,


one must be a special disbursing officer or SDO. All disbursing
officers of the government must have fidelity bonds. The bond is to
protect the government from and answer for misappropriation that
the disbursing officer may do. The bond amount required is the
same as the amount that may be disbursed by the officer. It is based
on total accountability and not determined by the head of the agency
as a matter of discretion. The head determines the accountability
which will be the basis of the bond amount. The Charter states that
the head of the agency is the Board of Directors, headed by the
Chairman. But now, under the Governance of Government
Corporation law, it is the general manager.
Plaras should have disallowed or suspended the cash advances
because there was no fidelity bond and the disbursing officer was
not authorized. There was no bond put up for Valencia. The records
show that the bond for Uriarte was only for the amount of P1.5
million. This is shown in a letter dated August 23, 2010, to COA
Chairman Villar through Plaras from Aguas, with an attachment from
the Bureau of Treasury, dated March 2, 2009. It appears there that
the bond for Uriarte for the CIF covering the period February 2009
to February 2010 was only P1.5 million. Once an SDO is
designated, the specimen signature must be submitted to COA,
together with the fidelity bond and the signatories for the cash
advances.
The approval of the President pertains to the release of the budget,
not its allocation. Atty. Tolentino thinks the action of the Board was
done because there was no budget. The Board's confirmation was
needed because it was in excess of the budget that was approved.
They were trying to give a color of legality to them approval of the
CIF in excess of the approved corporate operating budget. The

159
Board approval was required for the amount to be released, which
amount was approved in excess of the allotted budget for the year.
The President cannot approve an additional amount, unless there is
an appropriation or a provision saying a particular savings will be
used for the CIF. The approvals here were all in excess of the
approved budget.
Cash advances can be given on a per project basis for CIF. For one
to get a cash advance, one must state what the project is as to that
cash advance. No subsequent cash advance should be given, until
previous cash advances have been liquidated and accounted for. If
it is a continuing project, monthly liquidation reports must be given.
All of the liquidation papers should go to the COA Chair, given on a
monthly basis.

In this case, the vouchers themselves are couched generally and


just say cash advance from CIF of the Chairman or from the GM's
office in accordance with her duties. There is no particular project
indicated for the cash advance. Also, the requirement that prior
advances be liquidated first for subsequent advances to be given
was not followed. The witness prepared a summary of the cash
advances withdrawn by the two disbursing officers covering the
years 2008, 2009 and 2010. The basis for this summary is the
record submitted to them by Aguas, which were supposedly
submitted to COA. It shows that there were subsequent cash
advances, even if a prior advance has not yet been liquidated.
Valencia submitted liquidation reports to Villar, which consists of a
letter, certification and schedule of cash advances, and liquidation
reports. One is dated July 24, 2008 (Exh. "G") and another is dated
February 13, 2009 (Exh. "H").
When Atty. Tolentino secured Exhibit "G5", together with the
attached documents, she did not find any supporting documents
despite the statement in Exhibit "G5" that the supporting details of
the expenses that were incurred from the fund can be made
available, if required. Aguas, the person who processed the cash
advances said he did not have the details or suppmiing details of
documents of the expenditures.
Normally, when liquidating CIF, the certification of the head of the
agency is necessary. If there were vouchers or receipts involved,
then all these should be attached to the liquidation report. There
should also be an accomplishment report which should be done on
a monthly basis. All of these should be enclosed in a sealed
envelope and sent to the Chairman of the COA, although the agency
concerned must retain a photocopy of the documents. The report
should have a cover/transmittal letter itemizing the documents, as

160
well as liquidation vouchers and other supporting papers. If the
liquidation voucher and the supporting papers are in order, then the
COA Chairman or his representative shall issue a credit
memorandum. Supporting papers consist of receipts and sales
invoices. The head of the agency would have to certify that those
were all actually incurred and are legal. In this case, there were no
supporting documents submitted with respect to Valencia's cash
advances in 2008. Only the certifications by the SDO were
submitted. These certifications stated that he has the documents in
his custody and they can be made available, if and when necessary.
Atty. Tolentino was shown the four liquidation reports attached to
the transmittal letter and was asked whether they were properly and
legally accomplished. She replied that they were couched in general
terms and the voucher for which the cash advance was liquidated is
not indicated and only the voucher number is specified. She adds
that the form of the liquidation is correct, but the details are not there
and neither are the supporting papers.
When Plaras wrote Valencia on December 15, 2008, Aguas wrote
back on behalf of Valencia, who had designated himself as SDO.
However, their designations, or in what capacity they signed the
voucher are not stated. Among the attachments is also a
memorandum dated April 2, 2008, containing the signature of
Arroyo, indicating her approval to the utilization of funds. These two
memoranda bear the reasons for the cash advances, couched in
general terms. The reasons were donated medicines that were sold
and authorized expenditures on endowment fund. The reasons
stated in the memoranda are practically the same. Uriarte did not
submit any accomplishment reports regarding the intelligence fund.
Aguas submitted an accomplishment report, but the
accomplishments were not indicated in definite fashion or with
specificity.
The witness narrated, based on her Summary of Liquidation
Reports in 2009, that the total cash advance made by Uriarte was
P132,760,096. Therefore, the total cash advances by these two
officials, Uriarte and Valencia, were P138,420,875, but all of these
were never liquidated in 2009. Uriarte and Valencia only submitted
a liquidation voucher and a report to COA on April 12, 2010. For the
January 22, 2009 disbursements, the date of the liquidation voucher
was June 30, 2009, but it was submitted to COA on April 12, 2010.
Witness identified the transmittal letter for P28 million by Uriarte,
dated October 19, 2009, which was received by the COA only on
April 12, 2010, with an accompanying certification from Uriarte as to

161
some of the documents from which the witness's Summary of
Liquidation was based.
The cash advances made by Uriarte and Valencia violated par. I,
Sec. 4 and Sec. 84 of P.D. I445 and par. 2, III, COA Circular No. 92-
385. Since these cash advances were in excess of the
appropriation, in effect, they were disbursed without any
appropriation. These cash advances were also made without any
specific project, in violation of par. 2 of COA Circular No. 92-385. In
this case, the cash advances were not for a specific project. The
vouchers only indicate the source of the fund. The vouchers did not
specify specific projects. The total cash advances for the years
2008, 2009 and 2010 to accused Uriarte and Valencia is more than
P366,000,000. Valencia cash advanced PI 3.3 million. The rest was
made by Uriarte.
The memoranda to President Arroyo stated only the problems
encountered by the PCSO. These problems, as stated in each
memorandum, included donated medicines sometimes ending up in
store for sale, unofficial use of ambulances, rise of expenditures of
endowment fund, lotto sweepstakes scams, fixers for programs of
the PCSO, and other fraudulent schemes. No projects were
mentioned.
As regards the credit notice, the same was not validly issued by the
COA. The credit notice is a settlement or an action made by the
COA Auditors and is given once the Chairman, in the case of CIF
Fund, finds that the liquidation report and all the supporting papers
are in order. In this case, the supporting papers and the liquidation
report were not in order, hence, the credit notice should not have
been issued. Further, the credit notice has to follow a specific form.
The COA Chairman or his representative can: 1) settle the cash
advance when everything is in order; 2) suspend the settlement if
there are deficiencies and then ask for submission of the
deficiencies; or 3) out rightly disallow it in case said cash advances
are illegal, irregular or unconscionable, extravagant or excessive.
Instead of following this form, the COA issued a document dated
January 10, 2011, which stated that there is an irregular use of the
price fund and the charity fund for CIF Fund. The document bears
an annotation which says, "wait for transmittal, draft" among others.
The document was not signed by Plaras, who was the Head of the
Confidential and Intelligence Fund Unit under COA Chairman Villar.
Instead of the earlier finding of irregularity, suddenly, the COA
issued a credit notice as regards the total of P140,000,000. The
credit notice also did not specify that the transaction had been
audited, indicating that no audit was made. A letter dated May 11,

162
2009 from the COA and signed by Plaras, states that the credit
notice is hereby issued. Thus, it is equivalent to the credit notice,
although it did not come in the required form. It merely stated that
the credit notice is issued for P29,700,000, without specifying for
which vouchers and for which project the credit notice was being
given. It merely says "First Semester of 2008". In other words, it is
a "global" credit notice that she issued and it did not state that she
made an audit. Another letter, dated July 14, 2010 and signed by
Plaras, supposedly covers all the cash advances in 2009, but only
up to the amount of P116,386,800. It also did not state that an audit
was made.
The certification dated July 24, 2008 by Valencia was not in
conformity with the certification required by COA. The required form
should specify the project for which the certification was being
issued, and file code of the specific project. The certification dated
July 24, 2008, however, just specified that it was to certify that the
P2 million from the 2008 CIF Fund was incurred by the undersigned,
in the exercise of his functions as PCSO Chairman for the various
projects, projects and activities related to the operation of the office,
and there was no specific project or program or file code of the
intelligence fund, as required by COA. Furthermore, the certification
also did not contain the last paragraph as required by COA. No
details or supporting documents were reviewed by the witness, and
though she personally asked Aguas, the latter said that he did not
have the supporting papers, and they were not in the official files of
the PCSO. Two people should have custody of the papers, namely,
The Chairman of COA and the PCSO or its Special Disbursing
Officer.
A certification dated February 13, 2009, stating that P2,857,000 was
incurred by Valencia in the exercise of his function as PCSO
Chairman, related to the operations of his office without the specific
intelligence project. No details of this certification were submitted by
Aguas. Another certification dated July 24, 2008 was presented, and
it also did not specify the intelligence and confidential project, and it
did not contain any certification that the amount was disbursed
legally or that no benefits was given to any person. Similarly, the
fourth paragraph of the same document states that Uriarte certified
that details and supporting papers of the cash advance that she
made of P27,700,000 are "kept in their confidential" (sic). The same
were not in the PCSO official records.
The certification dated October 19, 2009 for the amount of
P2,498,300, was submitted to the witness by Aguas. It also did not
conform to the COA requirements. Again, no supporting documents
were found and none were given by Aguas. Similarly, a certification

163
dated February 8, 2010 for the amount of P2,394,654 was
presented, and it also does not conform with the COA circular, as it
only stated that the amount was spent or incurred by Valencia for
projects covering the period of July 1 to December 31, 2009 to
exercise his function as PCSO Chairman, thus no particular
intelligence fund or project was stated. As in the other certifications,
it appeared that these were not in the possession of PCSO. Another
certification dated October 19, 2009 submitted by Uriarte was
examined by the witness in the course of her audit, and found that
it also did not conform to the requirements, as it only stated that the
P25 million and P10 million intelligence and confidential fund dated
January 29, 2009 and April 27, 2009 were used in the exercise of
her function as PCSO Vice Chairman and General Manager.

All the documents were furnished by Aguas during the course of the
audit of the financial transactions of PCSO. Despite the statement
certifying that there were documents for the audit, no documents
were provided and the same were not in the official files of PCSO.
There are two kinds of audit on disbursements of government funds:
pre-audit and post-audit. Both are defined in COA Circular 2009-
002. Pre-audit is the examination of documents supporting the
transaction, before these are paid for and recorded. The auditor
determines whether: (1) the proposed expenditure was in
compliance with the appropriate law, specific statutory authority or
regulations; (2) sufficient funds are available to enable payment of
the claim; (3) the proposed expenditure is not illegal, irregular,
extravagant, unconscionable or unnecessary, and (4) the
transaction is approved by the proper authority and duly supported
by authentic underlying evidence. On the other hand, the post-audit
requirement is the process where the COA or the auditor will have
to do exactly what was done in the pre-audit, and in addition, the
auditor must supplement what she did by tracing the transaction
under audit to the books of accounts, and that the transaction is all
recorded in the books of accounts. The auditor, in post-audit, also
makes the final determination of whether the transaction was not
illegal, irregular, extravagant, excessive, unconscionable or
unnecessary.
In this case, no audit was conducted. In a letter dated May 11, 2009
signed by Plaras, it was stated that a credit advice was given.
However, the letter did not conform to the requirements or form of a
credit notice. Such form was in COA Circular 2003-002, and should
specify the liquidation report number, the amount, check numbers,
and the action taken by the auditor. The auditor should also include
a certification that these have been audited. In this instance, no
certification that the transaction was audited was given by Plaras.

164
Other similar letters did not conform with the COA Circular. All
transactions of the government must be subject to audit in
accordance with the provisions of the Constitution. Nevertheless,
the requirements for audit are the same.
The effect of the issuance of the credit notice by the COA was that
the agency will take it up in the books and credit the cash advance.
This is the seventh step in the flowchart. Once there is a cash
advance, the liability of the officers who obtained the cash advance
would be recorded in the books. The credit notice, when received,
would indicate that the account was settled. The agency will credit
the receivable or the cash advance, and remove from the books as
a liability of the person liable for the cash advance. The effect of this
was that the financial liabilities of Uriarte and Valencia were
removed from the books, but they could still be subject to criminal
liability based on Sec. 10 of COA Circular 91-368 (Government
Accounting and Auditing Manuals, Vol. 1, implementing P.O. 1445),
which states: "The settlement of an account whether or not on
appeal has been made within the statutory period is no bar to
criminal prosecution against persons liable." From the 2008 COA
Annual Audited Financial Statements of PCSO, it was seen that the
procedure was not followed because the liability of the officers was
already credited even before the credit notice was received. In the
financial statements, it was stated that the amount due from officers
and employees, which should include the cash advances obtained
by Uriarte and Valencia, were not included because the amount
stated therein was P35 million, while the total vouchers of Uriarte
and Valencia was P86 million.
She found out that the recorded CIF fund expense, as recorded in
the corporate operating budget as actually disbursed, was only
P21,102,000. As such, she confronted her accountants and asked
them "Saan tinago itong amount na to?" The personnel in the
accounting office said that the balance of the P86 million or the
additional P21 million was not recorded in the operating fund budget
because they used the prize fund and charity fund as instructed by
Aguas. Journal Entry Voucher No. 8121443 dated December 31,
2008, signed by Elmer Camba, Aguas (Head of the Accounting
Department), and Hutch Balleras (one of the staff in the Accounting
Department), showed that this procedure was done. The total
amount of the receivables from Uriarte and Valencia that was
removed was P106,386,800 and P90,428,780 respectively which
came from the prize fund and charity fund.
The witness reported the discrepancy because there were violations
of R.A. 1169, Sec. 6, which provides for the different funds of PCSO
namely: prize fund (55% of the net receipts), charity fund (30% of

165
the net receipts), and operating fund (15%). The proceeds of the
lotto and sweepstakes ticket sales provide the money for these
different funds, removing first the printing cost and the net proceeds
(98%) is divided among the three funds mentioned. The prize fund
is the fund set aside to be used to pay the prizes for the winnings in
the lotto or sweepstakes draws, whether they are jackpot or
consolation prizes. Incentives to the lotto operators or horse owners
are also drawn from this fund, as all of the expenses connected to
the winnings of the draw. On the other hand, the charity fund is
reserved for charity programs approved by the board of PCSO, and
constitutes hospital and medical assistance to individuals, or to help
facilities and other charities of national character. Operating
expenses are charged to the expenses to operate, personnel
services, and MOOE. One kind of fund cannot be used for another
kind, as they become a trust fund which should only be used for the
purpose for which it was authorized, not even with the approval of
the board.
The amounts obtained from the charity fund and prize fund for 2008
was P63,750,000, and in 2009 P90,428,780. The Board of Directors
was given a copy of the COA Audit Reports for years 2008 and
2009. The Board of Directors for both years was composed of:
Chairman Valencia, and Board Members Morato, Roquero, Taruc
and Valdez. Uriarte was the Vice Chairman of the Board of
Directors. The witness did not know whether the Board checked the
COA reports, but there was no action on their part, and neither did
they question the correctness of the statements. They also had the
Audit Committee (which was composed of members of the board)
at that time, and one of the duties of the Audit Committee was to
verify the balances.

The witness identified the documents referring to the confirmation


by the Board of Directors of PCSO of the CIF. Board Resolution No.
217, approved on February 18, 2009, confirms the CIF approved by
the President. It did not state which CIF they were approving. They
also assigned Uriarte as the Special Disbursing Officer of the CIF,
but it did say for what year. The signatories to the same Board
Resolution were Valencia, Taruc, Valdes, Uriarte, Roquero and
Morato. The same were the witness's findings for Board Resolution
No. 2356 S. 2009, approved on December 9, 2009. As for Board
Resolution No. 29, S. 2010, approved on January 6, 2010, the Board
confirmed the fund approved by the President for 2010, though the
approval of the President was only received on August 13, 2010 as
shown in the Memorandum dated January 4. In effect, the Board
was aware of the requests, and because they ratified the cash
advances, they agreed to the act of obtaining the same.

166
Apart from the President violating LOI 1282, the witness also
observed that the President directly dealt with the PCSO, although
the President, by Executive Order No. 383 dated November 14,
2004, and Executive Order No. 455 dated August 22, 2005,
transferred the direct control and supervision of the PCSO to the
Department of Social Welfare and Development (DSWD), and later
to the Department of Health (DOH). A project should first be
approved by the Supervising and Controlling Secretary of the
Secretary of Health; that the President had transferred her direct
control and supervision, and lost the same. The witness said her
basis was administrative procedure. In this regard, President Aquino
now has transferred the control and supervision of the PCSO back
to the Office of the President through Executive Order No. 14, S.
2010, dated November 19, 2010.
Uriarte should not have gone directly to the President to ask for the
latter's approval for allocation. Nonetheless, the release of the CIF
must still be approved by the President.
After the Prosecution rested its case, GMA, Aguas, Valencia,
Morato, Taruc V, Roquero and Villar separately filed their demurrers
to evidence asserting that the Prosecution did not establish a case
for plunder against them.


On April 6, 2015, the Sandiganbayan granted the demurrers to


evidence of Morato, Roquero, Taruc and Villar, and dismissed the
charge against them. It held that said accused who were members
of the PCSO Board of Directors were not shown to have diverted
any PCSO funds to themselves, or to have raided the public treasury
by conveying and transferring into their possession and control any
money or funds from PCSO account; that as to Villar, there had
been no clear showing that his designation of Plaras had been
tainted with any criminal design; and that the fact that Plaras had
signed "by authority" of Villar as the COA Chairman could not
criminally bind him in the absence of any showing of conspiracy.

ISSUE/S Whether or not the State sufficiently established the existence of


conspiracy among GMA, Aguas, and Uriarte

RULING/S The Prosecution did not properly allege and prove the existence of
conspiracy among GMA, Aguas and Uriarte. Conspiracy exists
when two or more persons come to an agreement concerning the
commission of a felony, and decide to commit it. In this jurisdiction,
conspiracy is either a crime in itself or a mere means to commit a
crime.

As a rule, conspiracy is not a crime unless the law considers it a
crime, and prescribes a penalty for it. The exception is exemplified

167
in Article 115 (conspiracy and proposal to commit treason), Article
136 (conspiracy and proposal to commit coup d'etat, rebellion or
insurrection) and Article 141 (conspiracy to commit sedition) of the
Revised Penal Code. When conspiracy is a means to commit a
crime, it is indispensable that the agreement to commit the crime
among all the conspirators, or their community of criminal design
must be alleged and competently shown.

We also stress that the community of design to commit an offense
must be a conscious one. Conspiracy transcends mere
companionship, and mere presence at the scene of the crime does
not in itself amount to conspiracy. Even knowledge of, or
acquiescence in, or agreement to cooperate is not enough to
constitute one a party to a conspiracy, absent any active
participation in the commission of the crime with a view to the
furtherance of the common design and purpose. Hence, conspiracy
must be established, not by conjecture, but by positive and
conclusive evidence.

In terms of proving its existence, conspiracy takes two forms. The
first is the express form, which requires proof of an actual agreement
among all the co-conspirators to commit the crime. However,
conspiracies are not always shown to have been expressly agreed
upon. Thus, we have the second form, the implied conspiracy. An
implied conspiracy exists when two or more persons are shown to
have aimed by their acts towards the accomplishment of the same
unlawful object, each doing a part so that their combined acts,
though apparently independent, were in fact connected and
cooperative, indicating closeness of personal association and a
concurrence of sentiment. Implied conspiracy is proved through the
mode and manner of the commission of the offense, or from the acts
of the accused before, during and after the commission of the crime
indubitably pointing to a joint purpose, a concert of action and a
community of interest.

In her case, GMA points out that all that the State showed was her
having affixed her unqualified "OK" on the requests for the additional
CIFs by Uriarte. She argues that such act was not even an overt act
of plunder because it had no immediate and necessary relation to
plunder by virtue of her approval not being per se illegal or irregular.
However, the Sandiganbayan, in denying the Motions for
Reconsideration of GMA and Aguas vis-a-vis the denial of the
demurrers, observed that:

xxxx accused Arroyo insists that there was no proof of the fact of
amassing the ill-gotten wealth, and that the "overt act" of approving
the disbursement is not the "overt act" contemplated by Jaw. She
further stresses that there was no proof of conspiracy between
accused Arroyo and her co-accused and that the Prosecution was

168
unable to prove their case against accused Arroyo. What accused
Arroyo forgets is that although she did not actually commit any
"overt act" of illegally amassing CIF funds, her act of approving not
only the additional CIF funds but also their releases, aided and
abetted accused Uriarte's successful raids on the public treasury.
Accused Arroyo is therefore rightly charged as a co-conspirator of
Uriarte who accumulated the CIF funds. Moreover, the performance
of an overt act is not indispensable when a conspirator is the
mastermind.

It is in this regard that the Sandigabayan gravely abused its
discretion amounting to lack or excess of its jurisdiction. To start
with, its conclusion that GMA had been the mastermind of plunder
was plainly conjectural and outrightly unfounded considering that
the information did not aver at all that she had been the mastermind;
hence, the Sandigabayan thereby acted capriciously and arbitrarily.
In the second place, the treatment by the Sandiganbayan of her
handwritten unqualified "OK" as an overt act of plunder was
absolutely unwarranted considering that such act was a common
legal and valid practice of signifying approval of a fund release by
the President. Indeed, pursuant to People v. Lizada, supra, an act
or conduct becomes an overt act of a crime only when it evinces a
causal relation to the intended crime because the act or conduct will
not be an overt act of the crime if it does not have an immediate and
necessary relation to the offense.

In Estrada v. Sandiganbayan, the Court recognized two nuances of
appreciating conspiracy as a means to commit a crime, the wheel
conspiracy and the chain conspiracy. The wheel conspiracy occurs
when there is a single person or group (the hub) dealing individually
with two or more other persons or groups (the spokes). The spoke
typically interacts with the hub rather than with another spoke. In the
event that the spoke shares a common purpose to succeed, there
is a single conspiracy. However, in the instances when each spoke
is unconcerned with the success of the other spokes, there are
multiple conspiracies.

An illustration of wheel conspiracy wherein there is only one
conspiracy involved was the conspiracy alleged in the information
for plunder filed against former President Estrada and his co-
conspirators. Former President Estrada was the hub while the
spokes were all the other accused individuals. The rim that enclosed
the spokes was the common goal in the overall conspiracy, i.e., the
amassing, accumulation and acquisition of ill-gotten wealth. The
chain conspiracy recognized in Estrada v. Sandiganbayan exists
when there is successive communication and cooperation in much
the same way as with legitimate business operations between

169
manufacturer and wholesaler, then wholesaler and retailer, and then
retailer and consumer.

Once the State proved the conspiracy as a means to commit a
crime, each co-conspirator is as criminally liable as the others, for
the act of one is the act of all. A co-conspirator does not have to
participate in every detail of the execution; neither does he have to
know the exact part performed by the co-conspirator in the
execution of the criminal act. Otherwise, the criminal liability of each
accused is individual and independent.

A review of the records of the case compels us to reject the
Sandiganbayan's declaration in light of the information filed against
the petitioners, and the foregoing exposition on the nature, forms
and extent of conspiracy. On the contrary, the Prosecution did not
sufficiently allege the existence of a conspiracy among GMA, Aguas
and Uriarte.

A perusal of the information suggests that what the Prosecution
sought to show was an implied conspiracy to commit plunder among
all of the accused on the basis of their collective actions prior to,
during and after the implied agreement. It is notable that the
Prosecution did not allege that the conspiracy among all of the
accused was by express agreement, or was a wheel conspiracy or
a chain conspiracy. This was another fatal flaw of the Prosecution.

The law on plunder requires that a particular public officer must be
identified as the one who amassed, acquired or accumulated ill-
gotten wealth because it plainly states that plunder is committed by
any public officer who, by himself or in connivance with members of
his family, relatives by affinity or consanguinity, business
associates, subordinates or other persons, amasses, accumulates
or acquires ill-gotten wealth in the aggregate amount or total value
of at least P50,000,000.00 through a combination or series of overt
criminal acts as described in Section l(d) hereof. Surely, the law
requires in the criminal charge for plunder against several
individuals that there must be a main plunderer and her co-
conspirators, who may be members of her family, relatives by
affinity or consanguinity, business associates, subordinates or other
persons. In other words, the allegation of the wheel conspiracy or
express conspiracy in the information was appropriate because the
main plunderer would then be identified in either manner. Of course,
implied conspiracy could also identify the main plunderer, but that
fact must be properly alleged and duly proven by the Prosecution.

This interpretation is supported by Estrada v. Sandiganbayan,
where the Court explained the nature of the conspiracy charge and
the necessity for the main plunderer for whose benefit the
amassment, accumulation and acquisition was made, thus:


170
There is no denying the fact that the "plunder of an entire nation
resulting in material damage to the national economy" is made up
of a complex and manifold network of crimes. In the crime of
plunder, therefore, different parties may be united by a common
purpose. In the case at bar, the different accused and their different
criminal acts have a commonality - to help the former President
amass, accumulate or acquire ill-gotten wealth. Sub-paragraphs (a)
to (d) in the Amended Information alleged the different participation
of each accused in the conspiracy. The gravamen of the conspiracy
charge, therefore, is not that each accused agreed to receive
protection money from illegal gambling, that each misappropriated
a portion of the tobacco excise tax, that each accused ordered the
GSIS and SSS to purchase shares of Belle Corporation and receive
commissions from such sale, nor that each unjustly enriched himself
from commissions, gifts and kickbacks; rather, it is that each of
them, by their individual acts, agreed to participate, directly or
indirectly, in the amassing, accumulation and acquisition of ill-gotten
wealth of and/or for former President Estrada.

Here, considering that 10 persons have been accused of amassing,
accumulating and/or acquiring ill-gotten wealth aggregating
P365,997,915.00, it would be improbable that the crime charged
was plunder if none of them was alleged to be the main plunderer.
As such, each of the 10 accused would account for the aliquot
amount of only P36,599,791.50, or exactly 1/10 of the alleged
aggregate ill-gotten wealth, which is far below the threshold value of
ill-gotten wealth required for plunder.

We are not unmindful of the holding in Estrada v. Sandiganabayan
to the effect that an information alleging conspiracy is sufficient if the
information alleges conspiracy either: (1) with the use of the word
conspire, or its derivatives or synonyms, such as confederate,
connive, collude, etc; or (2) by allegations of the basic facts
constituting the conspiracy in a manner that a person of common
understanding would know what is being conveyed, and with such
precision as would enable the accused to competently enter a plea
to a subsequent indictment based on the same facts. We are not
talking about the sufficiency of the information as to the allegation
of conspiracy, however, but rather the identification of the main
plunderer sought to be prosecuted under R.A. No. 7080 as an
element of the crime of plunder. Such identification of the main
plunderer was not only necessary because the law required such
identification, but also because it was essential in safeguarding the
rights of all of the accused to be properly informed of the charges
they were being made answerable for. The main purpose of
requiring the various elements of the crime charged to be set out in
the information is to enable all the accused to suitably prepare their

171
defense because they are presumed to have no independent
knowledge of the facts that constituted the offense charged.

Nevertheless, the Prosecution insists that GMA, Uriarte and Aguas
committed acts showing the existence of an implied conspiracy
among themselves, thereby making all of them the main plunderers.
On this score, the Prosecution points out that the sole overt act of
GMA to become a part of the conspiracy was her approval via the
marginal note of "OK" of all the requests made by Uriarte for the use
of additional intelligence fund.

The insistence of the Prosecution is unwarranted. GMA's approval
of Uriarte's requests for additional CIFs did not make her part of any
design to raid the public treasury as the means to amass,
accumulate and acquire ill-gotten wealth. Absent the specific
allegation in the information to that effect, and competent proof
thereon, GMA' s approval of Uriarte' s requests, even if unqualified,
could not make her part of any criminal conspiracy to commit
plunder or any other crime considering that her approval was not by
any means irregular or illegal.

We opine and declare, however, that Aguas' certifications and
signatures on the disbursement vouchers were insufficient bases to
conclude that he was into any conspiracy to commit plunder or any
other crime. Without GMA's participation, he could not release any
money because there was then no budget available for the
additional CIFs. Whatever irregularities he might have committed
did not amount to plunder, or to any implied conspiracy to commit
plunder.

Under the circumstances, the Sandiganbayan's finding on the
existence of the conspiracy to commit plunder was unsustainable. It
then becomes unavoidable for the Court to rule that because the
Prosecution failed to properly allege the elements of the crime, as
well as to prove that any implied conspiracy to commit plunder or
any other crime existed among GMA, Aguas and Uriarte there was
no conspiracy to commit plunder among them. As a result, GMA and
Aguas could be criminally responsible only for their own respective
actions, if any.

In view of the foregoing, the Court inevitably concludes that the


Sandiganbayan completely ignored the failure of the information to
sufficiently charge conspiracy to commit plunder against the
petitioners; and ignored the lack of evidence establishing the corpus
delicti of amassing, accumulation and acquisition of ill-gotten wealth
in the total amount of at least P50,000,000 through any or all of the
predicate crimes. The Sandiganbayan thereby acted capriciously,
thus gravely abusing its discretion amounting to lack or excess of
jurisdiction. WHEREFORE, the Court GRANTS the petitions for

172
certiorari; ANNULS and SETS ASIDE the resolutions issued in
Criminal Case No. SB-12-CRM-0174 by the Sandiganbayan on
April 6, 2015 and September 10, 2015; GRANTS the petitioners'
respective demurrers to evidence; DISMISSES Criminal Case No.
SB-12-CRM-0174 as to the petitioners GLORIA MACAPAGAL-
ARROYO and BENIGNO AGUAS for insufficiency of evidence;
ORDERS the immediate release from detention of said petitioners;
and MAKES no pronouncements on costs of suit.


CONTRIBUTOR FLORES, Precious Eureka D.


MODULE Conspiracy and Proposals to Commit Felony and Felonies as to
TOPIC Severity
CASE TITLE GO-TAN v. TAN G.R.NO. 168852
PONENTE AUSTRIA-MARTINEZ, DATE: SEPTEMBER 30, 2008
J.
DOCTRINE Legal principles developed from the Penal Code may be applied in
a supplementary capacity to crimes punished under special laws, in
which the special law is silent on a particular matter.
FACTS On April 18, 1999, Sharica Mari L. Go-Tan and Steven L. Tan were
married and out of this union, two female children were born, Kyra
Danielle and Kristen Denise. On January 12, 2005, Go-Tan filed a
petition with Prayer for the Issuance of a Temporary Protective
Order against her husband, Steven, and her parents-in-law
Spouses Perfecto C. Tan and Juanita L. Tan, herein respondents.
Go-Tan alleged that Steven, in conspiracy with respondents, were
causing verbal, psychological and economic abuses upon her in
violation of Republic Act No. 9262 (RA 9262), otherwise known as
the “Anti-Violence Against Women and Their Children Act of 2004”.

However, the respondents filed a motion contending that the RTC


lacked jurisdiction over their persons since, as parents-in-law of the
petitioner, they were not covered by RA 9262.
ISSUE/S Whether or not the respondent spouses may be included in the
petition for the issuance of a protective order, in accordance with
RA 9262?
RULING/S Section 3 of RA 9262 defines violence against women and their
children as any act or a series of acts committed by any person
against a woman who is his wife, former wife, or against a woman
with whom the person has or had a sexual or dating relationship, or
with whom he has a common child, or against her child whether
legitimate or illegitimate, within or without the family abode, which
result in or is likely to result in physical, sexual, psychological harm
or suffering, or economic abuse including threats of such acts,
battery, assault, coercion, harassment or arbitrary deprivation of
liberty.

173
While the provision provides that the offender be related or
connected to the victim by marriage, former marriage or a sexual or
dating relationship, it does not preclude the application of the
principle of conspiracy under the RPC.

As stated in Section 47 of RA 9262 expressly provides for the


suppletory application of RPC, thus:

SEC. 47. Suppletory Application. – For purposes of


this Act, the Revised Penal Code and other applicable
laws, shall have suppletory application.

Parenthetically, Article 10 of the RPC provides:

ART. 10 Offenses not subject to the provisions of this


code. - Offenses which are or in the future may be
punishable under special laws are not subject to the
provisions of this Code. This Code shall be
supplementary to such laws, unless the latter should
specially provide the contrary.

Hence, the legal principles developed from the Penal Code may be
applied in a supplementary capacity to crimes punished under
special laws, such as RA 9262, in which the special law is silent on
a particular matter.

Thus, the principle of conspiracy may be applied to RA 9262. For


once conspiracy or action in concert to achieve a criminal design is
shown, the act of one is the act of all the conspirators, and the
precise extent or modality of participation of each of them becomes
secondary, since all the conspirators are principals.

It must be further noted that Section 5 of RA 9262 expressly


recognizes that the acts of violence against women and their
children may be committed through any of the following acts:

xxx

(h) Engaging in purposeful, knowing, or reckless


conduct, personally or through another, that alarms or
causes substantial emotional or psychological
distress to the woman or her child. This shall include,
but not limited to, the following acts:

174
1. Stalking or following the woman or her child in
public or private places;
2. Peering in the window or lingering outside the
residence of the woman or her child;
3. Entering or remaining in the dwelling or on the
property of the woman or her child against
his/her will;
4. Destroying the property and personal
belongings or inflicting harm to animals or
pets of the woman or her child; and
5. Engaging in any form of harassment of
violence; xxx.

In addition, the protection order that may be issued for the purpose
of preventing further acts of violence against the woman or her child
may include individuals other than the offending husband, thus:

SEC. 8. Protection Orders. – xxx The protection


orders that may be issued under this Act shall include
any, some or all of the following reliefs:

a. Prohibition of the respondent from threatening to


committing, personally or through another, any of the acts
mentioned in Section 5 of this Act;
b. Prohibition of the respondent from harassing,
annoying, telephoning, contacting or otherwise
communicating with the petitioner, directly or indirectly; xxx

Finally, Section 4 of RA 9262 calls for a liberal construction


of the law, thus:

SEC. 4. Construction. – This Act shall be liberally


construed to promote the protection and safety of
victims of violence against women and their children.

Therefore, the respondents spouses are included in the Protection


Order pursuant to the principle of conspiracy being applied
suppletorily to RA 9262.

175
176
MODULE 5
JUSTIFYING CIRCUMSTANCES
AND ABSOLUTORY CAUSES

177
CASE TITLE Manaban v. CA G.R NO. G.R. No. 150723
PONENTE CARPIO, J. DATE: 11 July 2006
CONTRIBUTOR GARCIA, LEXANNE O.
DOCTRINE
Unlawful aggression is an indispensable requisite of self-
defense. Self-defense is founded on the necessity on the part of
the person being attacked to prevent or repel the unlawful
aggression. Thus, without prior unlawful and unprovoked attack by
the victim, there can be no complete or incomplete self-defense.

Unlawful aggression is an actual physical assault or \ at least a


threat to attack or inflict physical injury upon a person. Mere
threatening or intimidating attitude not considered unlawful
aggression.

Aggression presupposes that the person attacked must face a real


threat to his life and the peril sought to be avoided is imminent
and actual, not imaginary.
FACT/S
Joselito Bautista had to buy medicines for her sick
daughter. Although he had taken alcoholic drinks earlier, he still
went to the BPI Kalayaan Branch to withdraw some money from its
Automated Teller Machine (ATM).

However, he could not withdraw money due to entering the wrong


PIN and he started kicking and pounding on the machine. The
guard went near him and
advised him to just return the next morning. This
angered Bautista all the more and resumed pounding the
machine. Manaban then urged him to calm down and referred
him to their customer service over the phone. Bautista continued
raging and striking the machine. When Manaban could no longer
pacify him, he fired a warning shot. That diverted the attention of
Bautista. He confronted Manaban. After
some exchange of words, a shot rang out fatally hitting Bautista.
MAIN ISSUE/S
Was there unlawful aggression and was there a valid self-defense?
RULING ON
MAIN ISSUE/S When the accused invokes self-defense, he in effect admits killing
the victim and the burden is shifted to him to prove that he killed
the victim to save his life. Unlawful aggression is an indispensable
requisite of self-defense. Self-defense is founded on the necessity
on the part of the person being attacked to prevent or repel the
unlawful aggression.

178
There must be an actual, sudden, unexpected attack or imminent
danger thereof, which puts the defendant’s life in real peril.
Aggression presupposes that the person attacked must face a real
threat to his life and the peril sought to be avoided is imminent and
actual, not imaginary.

In this case, there was no unlawful aggression on the part of the


victim. First, Bautista was shot at the back as evidenced by the
point of entry of the bullet. Second, when Bautista
was shot, his gun was still inside a locked holster
and tucked in his right waist. Third, when Bautista turned his
back at Manaban, Manaban was already pointing his service
firearm at Bautista.

These circumstances clearly belie Manaban’s claim of


unlawful aggression on Bautista’s part. Thus, self-defense as a
justifying circumstance cannot be appreciated.

179
CONTRIBUTOR Manango, John De Divine B.
MODULE Module 5: Justifying Circumstances and Absolutory Causes –
TOPIC Defense of Self (Art 11 (1))
CASE TITLE Senoja v. People G.R.NO. G.R. No. 160341
PONENTE J. Callejo DATE: 19 October, 2004
DOCTRINE Affirmative defense of self-defense – Unlawful aggression
FACTS Exequiel Senoja, Fidel Senoja (brothers),Jose Calica and Miguel
Lumasac were drinking gin in the hut of Crisanto Reguyal. Leon
Lumasac suddenly barged in, holding a bolo and was looking for his
brother Miguel whom he suspected of drying up the rice field he was
plowing. However, when Senoja (Exequiel) approached Leon, the
latter tried to hack him so he embraced Leon
and Jose took Leon’s bolo. After the confrontation, Leon wanted to
get his bolo back because he wanted to go home. After getting it
back, Leon walked out of the place followed by Senoja. Suddenly,
Senoja stabled Leon at the back. When Leon turned around, Senoja
continued stabbing him until he fell to the ground. Then petitioner
ran towards the barangay road and threw away the knife he used to
stab Leon. Petitioner admitted killing the victim but invoked the
affirmative defense of self-defense. His version said that after the
commotion inside the house, Leon left but with a threat that
something will happen to Senoja. Senoja followed Leon as the latter
was making his way home. When Leon realized that Senoja was
following him, Leon walked back towards him and suddenly hacked
Senoja at the left side of his head and right thigh. Unable to evade
the treacherous attack by Leon, Senoja drew his colonial knife and
stabbed Leon in self-defense, inflicting upon him multiple wounds
which caused his death
ISSUE/S Whether or Not Senoja merely acted in self-defense
RULING/S No. Senoja is guilty of Homicide. In this case, there were two
interrelated events that occurred: 1) The arrival of Leon who was
armed with a bolo and 2) When Leon demanded for his bolo
because he wanted to go home already after the commotion inside
the house, and then eventually left with a threat. Quoting the
appellate court, the SC said that the victim had already left the hut.
At that point in time, the victim was simply walking toward his
home; he had stopped being an aggressor. It was Senoja who
wanted a confrontation this time. It was Senoja who was now the
unlawful aggressor in this second phase of their confrontation.
The question that must be resolved is whether or not the victim
was the unlawful aggressor as the appellant’s testimony pictures
him to be. The Court rules in the negative. The victim had already
left the hut and was ten (10) meters away from it. There is no
showing that the victim, who was drunk, was aware that appellant
was following him, or that the appellant called out to him so that he

180
(the victim) had to turn around and notice him. It is clear that at
that point in time, the victim was simply walking toward his home;
he had stopped being an aggressor. It was the appellant who,
smarting from the earlier incident in the hut where Leon had
threatened him, wanted a confrontation. Appellant stabbed or
poked the victim in the left buttock resulting in the non-fatal wound,
and when the latter turned around, successively stabbed and
hacked the victim in the armpit and chest until he fell. In all, the
victim suffered nine (9) wounds.
It is the well-considered finding of this Court that while Leon
Lumasac had ceased being the aggressor after he left the hut to
go home, accused Exequiel Senoja was now the unlawful
aggressor in this second phase of their confrontation. It bears
mentioning that appellant contradicted himself with respect for the
reason why he left the hut. First, it was to pacify Leon and the
second reason was that he was going home.
As for appellant’s injuries, it is clear that they were sustained in the
course of the victim’s attempt to defend himself as shown by the
lacerated wound on the victim’s left palm, a defensive wound
The affirmative defense of self-defense may be complete or
incomplete. It is complete when all the three essential requisites
are present; it is incomplete if only unlawful aggression on the part
of the victim and any of the two essential requisites were present.
Unlawful aggression on the part of the victim is a condition sine
qua non to self-defense, complete or incomplete. The right of self-
defense proceeds from necessity and limited by it. The right begins
where necessity does, and ends where it ends. There is however,
a perceptible difference between necessity and self-defense. Self-
defense excuses the repulse of a wrong; necessity justifies the
invasion of a right. Hence, it is essential to self-defense that it
should be a defense against a present unlawful attack. Self-
defense is an act to save life; hence, it is a right and not a crime.It
is a settled rule that to constitute aggression, the person attacked
must be confronted by a real threat on his life and limb; and the
peril sought to be avoided in imminent and actual, not merely
imaginary. Hence, when an inceptual / unlawful aggression ceases
to exist, the one making a defense has no right to kill or injure the
former aggressor. After the danger has passed, one is not justified
in following up his adversary to take his life

181
CONTRIBUTOR MARCELINO, Ferilynn T.
MODULE Justifying Circumstances – Defense of Self, Relatives and
TOPIC Strangers
CASE TITLE PEOPLE V. DECENA G.R.NO. 107874
PONENTE CALLEJO, SR., J: DATE: 04 August 1994
DOCTRINE The basic requirement for self-defense, as a justifying circum-
stance, is that there was an unlawful aggression against the person
defending himself. It must be positively shown that there was a
previous unlawful and unprovoked attack that placed the life of the
accused in danger and forced him to inflict more or less severe
wounds upon his assailant, employing therefor reasonable means
to resist said attack.
FACTS On 20 September 1991, George Decena was charged with murder
by the trial court, imposing on him the penalty of reclusion perpetua
& ordering him to indemnify the heirs of the deceased in the amount
of Php50,000.00 plus the additional amounts of Php4,500.00 and
Php2,300.00, for allegedly stabbing to death one Jaime Ballesteros
in San Fabian, Pangasinan on - of all dates - December 25, 1990.
A motion for reconsideration filed by appellant was denied on 26
August 1992 for lack of merit, hence this appellate review wherein
appellant contends, in his assigned errors, that the lower court
blundered in disregarding his claim of self-defense, and not in
appreciating the mitigating circumstance of voluntary surrender in
his favor, granting arguendo that he is guilty.

Testimony of Luzviminda Ballesteros, 14-year old daughter of the


victim: On Christmas Day of 1990, at around 4:00 P.M., said
Luzviminda was playing with her siblings at home. She recalled
being asked by her mother, Teresita Ballesteros, to fetch her father,
Jaime Ballesteros, who was then watching a game in the basketball
court. On her way to the hardcourt, Luzviminda met her father
walking home in an intoxicated state. Suddenly, she saw appellant
rushing towards her father with a long bladed weapon, prompting
Luzviminda to warn her father to run for safety by shouting in the
vernacular "Batik kila, Tatay!” Instead, Jaime simply raised his hand,
thus allowing appellant to stab him on the right chest just below the
nipple. Appellant then fled from the crime scene, while the victim
also managed to run but stumbled and fell to the ground. Finding
that her father was too heavy for her to carry, Luzviminda called for
her mother at their house, which was only fifteen meters away from
the scene of the crime, saying: "Mother, come! My father has been
stabbed by George Decena." Her mother immediately called for a
tricycle and rushed Jaime to the Provincial Hospital where, however,
the victim was declared dead on arrival.

182
Decena’s claim: At about 4:00 P.M. of that day, appellant was
watching a basketball game. The victim, Jaime Ballesteros, went
around the basketball court, walking in a wobbly manner due to
drunkenness. Jaime stopped near the place where appellant was
sitting and, for no apparent reason, held the latter by the neck with
one arm and, at the same time, poking a fork against it with the other
arm. Brgy. Tanod Romeo Decena who was also watching the
basketball game, intervened. He took the fork from Jaime & advised
appellant to go home. The latter left and was followed later by Jaime.
ISSUE/S 1) Whether or not Decena acted in complete self-defense in killing
Jaime Ballesteros. (main issue)
2) Whether or not mitigating circumstance of voluntary surrender
may be appreciated.
RULING/S 1) NO. The basic requirement for self-defense, as a justifying
circumstance, is that there was an unlawful aggression against the
person defending himself. It is necessary that one be assaulted or
that he be attacked, or at least that he be threatened with an attack
in an immediate manner, as, for example, brandishing a knife with
which to stab him or pointing a gun to be discharged against him.
Following Decena’s claim of events that happened in the court,
when appellant and Jaime heeded the advice of the Brgy. Tanod,
the unlawful aggression no longer existed, appellant had no right
whatsoever to kill or even wound the former aggressor. Decena
further contended the continuation of the unlawful aggression:
Jaime persisted in his design to attack Decena while the latter was
already in front of his house. However, the defense failed to
establish this fact. Decena’s support to this claim was that whenever
the victim was drunk, he would look for trouble. Still, the victim’s wife
testified that the latter has no such record in their barangay and,
significantly, her said testimony was never refuted nor objected to
by appellant. Also, witnesses for and against the appellant testified
that throughout the incident Jaime was inebriated and that he was
staggering or wobbling as he walked. If he had such difficulty even
in performing the normal bodily function of locomotion, it could not
be expected that he would muster enough courage to persist in
attacking and attempting to kill appellant, as posited by the defense,
considering that the latter was decidedly stronger than him.
Supposed eyewitness of the defense who is appellant's uncle,
Fernando Biala, impresses us as either an imaginative or a coached
witness - he said that he did not see where Jaime came from and
he could not tell how long the fight transpired but he said that Jaime
attacked first; his reason was that the two were already fighting
when he went up the road.

2) YES. The rule is that the mitigating circumstance of voluntary


surrender may properly be appreciated if the following requisites

183
concur: (a) the offender had not actually been arrested; (b) the
offender surrendered himself to a person in authority or to an agent
of a person in authority; and (c) the surrender was voluntary. We
believe that the mitigating circumstance of voluntary surrender may
be awarded to appellant. The records disclose that appellant was,
evidently with his concurrence, accompanied and surrender by his
father to a person in authority, Sgt. Romeo Diagan, early in the
morning after the incident and before he could actually be arrested.
That mitigating circumstance can, therefore, be properly considered
in his favor to impose the penalty in its minimum period.

Dispositive: WHEREFORE, the appealed judgment of the court a


quo is hereby MODIFIED by finding accused-appellant George
Decena y Rocaberte guilty of the crime of homicide, and imposing
upon him an indeterminate sentence of 8 years of prision mayor, as
minimum, to 14 years and 8 months of reclusion temporal, as
maximum. In all other respects, the said judgment is hereby
AFFIRMED.

The Court rejected the trial court’s ruling that the killing of the victim
was attended by treachery. Any circumstance which would qualify a
killing to murder must be proven as indubitably as the crime itself.
Here, the qualifying circumstance of treachery cannot be
appreciated, for none of the prosecution's arguments can uphold its
allegation that, in the language of the law, appellant committed the
crime by employing means, methods or forms in the execution
thereof which tended directly and especially to insure its execution,
without risk to himself arising from the defense which the offended
party might make. It is true that the attack was sudden, but that fact
per se does not bespeak the circumstance of alevosia. It is further
required that the means, methods or forms were deliberated upon
or consciously adopted by the offender. The crime committed,
therefore, was simple homicide.

184
CONTRIBUTOR MAYUGA, Eunice Allaine G.
MODULE Article 11 (1) of the RPC – Justifying Circumstances
TOPIC
CASE TITLE PEOPLE v. DELA CRUZ G.R.NO. 128359
PONENTE VITUG, J: DATE: 06 DEC 2000
DOCTRINE Self-defense requires that there must be: a) unlawful aggression; b)
reasonable necessity of the means employed to prevent or repel it;
and c) lack of sufficient provocation on the part of the person
defending himself.
FACTS Daniel Macapagal, a married man, had been a live-in partner of Ma.
Luz Perla San Antonio for about two to three years before San
Antonio took Roberto de la Cruz, a widower, as lover and live-in
partner. At the time of the incident on 27 May 1996, Roberto and
San Antonio were resting in their bedroom at around 6:00PM when
they heard a car stopped in front of their house and thereafter,
someone knocks on their door. San Antonio opened the door and
Macapagal made his way inside the house holding a gun, looking
for Roberto. When Macapagal saw a door that is closed, he banged
at the door and yelled “come out, come out”. Roberto opened the
door, greeted by Macapagal’s revolver pointing at him. Roberto then
closed the door and after a few moments, he opened the door again
and was now armed with a .38 caliber revolver. Macapagal and
Roberto grappled for each other firearms and moments after, shots
were heard and Macapagal fell dead on the floor. Roberto told San
Antonio to call the police. When the police arrived, Roberto
surrendered the .38 caliber revolver and told them that he shot
Macapagal in self-defense and went with them to the police station.
The autopsy report showed that Macapagal had a four gunshot
wounds, three of them did not penetrate a vital organ. But the other
gunshot wound penetrated Macapagal’s chest which instantly
caused his death. Moreover, Macapagal had a license to carry a
firearm while Roberto, who denied ownership of the .38 caliber
revolver he used, had no license. Unmoved by the claim of self-
defense, the lower court pronounced a judgment of guilt. Hence, this
petition.

ISSUE/S Whether or not Roberto de la Cruz can claim that he acted in self-
defense in order to extenuate him for the crime.

RULING/S No. When self-defense is invoked, the burden of evidence shifts to


the accused to show that the killing has been legally justified. The
accused must discharge the burden of proving in a clear and
convincing manner that all the requisites for self-defense are
present in order to avail himself of the extenuating circumstance.

185
Unlawful aggression presupposes an actual, sudden and
unexpected attack or imminent danger on the life and limb of a
person. In this case, Macapagal barged into the house of Roberto
and banged at his door. When Roberto opened the door and
thereafter promptly closing the door, he could have stopped there
and prevented the incident but rather, he armed himself and
confronted the victim. At this encounter, it would be quite hard to still
claim self-defense.

The second element of self-defense would demand that the means


employed to quell the unlawful aggression were reasonable and
necessary. The number of the wounds sustained by the deceased
in this case would negate the existence of this indispensable
component of self-defense.

Lastly, there must be lack of sufficient provocation on the part of


Roberto for him to claim self-defense. When Roberto opened the
door the second times, instead of taking precautionary measures,
he can no longer argue that there was no provocation on his part.
Hence, Roberto de la Cruz is guilty of homicide with the use of
unlicensed firearm, an aggravating circumstance as provided by RA
8294.

186
CONTRIBUTOR PALALA, Amer B.
MODULE Justifying circumstances and Absolutory Causes
TOPIC
CASE TITLE PEOPLE V. JAURIGUE G.R.NO. 384
PONENTE DE JOYA, J. DATE: February 21, 1946
DOCTRINE In the defense of their honor, when brutally attacked, women are
permitted to make use of all reasonable means available within their
reach, under the circumstances.
FACTS Defendant Avelina Jarigue and appellant Amado Capino lived in the
same barrio. Prior to the incident at hand, Capino had been courting
Jarigue to no avail. A month prior to the incident, Capino stole a
hanky belonging to Jarigue bearing her nickname “aveling” while I
was being washed. On another night, Jarigue was feeding a dog
under her house, when Capino approached her and spoke to her of
his love, which she flatly refused, and he thereupon suddenly
embraced and kissed her and touched her boobs. She thereafter
kept a long fan knife to protect herself. A few days later, Capino
climbed up the house of Jarigue and entered the room where she
was sleeping. He felt her forehead with the intention of raping her.
She immediately screamed for help, which awakened her parents
and brought them to her side. Capino then came out from where he
was hiding, under the bed, and kissed the hand of Jarigue’s father
to beg for forgiveness. Several days later on the fateful night, her
family went to the local church where it was quite bright. When
Jarigue was left alone in the bench while her father tended to some
business, Capino sat beside Jarigue and placed his hand on top of
her thigh. On observing this highly improper conduct, Jaurigue
stabbed Capino in the neck, fatally causing a single wound from
which he died. Jaurigue surrendered without question.
ISSUE/S 1. Whether or not defendant acted in the legitimate defense of
her honor and should be completely absolved from all
criminal liability.
2. Whether or not there were mitigating and aggravating
circumstances.
RULING/S 1. No, she is not absolved from all criminal liability. If the
defendant had killed Capino when he climbed up her house
to rape her, she could have been perfectly justified in killing
him. However, when the deceased sat beside defendant on
the same bench in a well-lit chapel with several people
inside, including her own father and the barrio lieutenant
where there is no possibility of being raped. She cannot be
legally declared completely exempt from criminal liability for
fatally wounding the deceased since the means employer
by her in the defense of her honor was evidently excessive.

187
2. As shown in the case, there are as mitigating circumstances
in her favor but the aggravating circumstance cannot be legally
sustained. The fact that she voluntarily surrendered to the lieutenant
in the chapel and admitted to stabbing the deceased, and the fact
that she had acted in the immediate vindication of a grave offense
committed against her a few moments before, and upon such
provocation as to produce passion or temporary loss of reason,
should be considered as mitigating circumstances in her favor.
However, the aggravating circumstance that the killing was
done in a place dedicated to religious worship cannot be
legally sustained as there is no evidence to show that
defendant had murder in her heart when she entered the
chapel. She should therefore be charged with homicide
without aggravating circumstances and with mitigating
circumstances.

188
CONTRIBUTOR PANGAN, Gabrielle L.
MODULE Defense of property, Incomplete defense of property
TOPIC
CASE TITLE PEOPLE V. NARVAEZ G.R.NO. L-33466-67
PONENTE MAKASIAR, J: DATE: APRIL 20, 1983
DOCTRINE Justifying Circumstance:
• In order for the defense of one's person or rights to be
appreciated as a justifying circumstance, the following
requisites must occur: Unlawful aggression; Reasonable
necessity of the means employed to prevent or repel it;
Lack of sufficient provocation on the part of the person
defending himself. When the appellant fired his shotgun
from his window, killing his two victims, his resistance was
disproportionate to the attack, inconsistent with the second
requirement.

Mitigating Circumstance:
• Appellant's act in killing the deceased was not justifiable,
since-not all the elements for justification are present. He
should therefore be held responsible for the death of his
victims, but he could be credited with the special mitigating
circumstance of incomplete defense, pursuant to paragraph
6, Article 13 of the Revised Penal Code.
FACTS This incident is intertwined with the long drawn out legal battle
between the Fleischer and Co., Inc. of which deceased Fleischer
was the secretary-treasurer and deceased Rubia the assistant
manager, on the one hand, and the land settlers of Cotabato, among
whom was appellant. During the pendency of this case, appellant
entered into a contract of lease with the company whereby he
agreed to lease the area he occupies from the company for a
consideration of P16.00 monthly. According to him, he signed the
contract although the ownership of the land was still uncertain, in
order to avoid trouble, until the question of ownership could be
decided. He never paid the agreed rental, although he alleges that
the milling job they did for Rubia was considered payment. From a
letter sent to him by the deceased Fleischer, he was given six
months, that is until December 31, 1996, to vacate the property.

At about 2:30 in the afternoon of August 22, 1968, Graciano Juan,


Jesus Verano and Cesar Ibañez, together with the two deceased
Davis Fleischer and Flaviano Rubia, were fencing the land of
George Fleischer, father of deceased Davis Fleischer. The place
was in the boundary of the highway and the hacienda owned by
George Fleischer. This is located in the municipality of Maitum,
South Cotabato. At the place of the fencing is the house and rice
drier of appellant Mamerto Narvaez.

189
At that time, appellant was taking his rest, but when he heard that
the walls of his house were being chiselled, he arose and there he
saw the fencing going on. If the fencing would go on, appellant
would be prevented from getting into his house and the bodega of
his ricemill. So he addressed the group, saying -'Pare, if possible
you stop destroying my house and if possible we will talk it over -
what is good,' addressing the deceased Rubia, who is appellant's
compadre. The deceased Fleischer, however, answered: 'No,
gademit, proceed, go ahead.'
Appellant apparently lost his equilibrium and he got his gun and shot
Fleischer, hitting him. As Fleischer fell down, Rubia ran towards the
jeep, and knowing there is a gun on the jeep, appellant red at Rubia,
likewise hitting him. Both Fleischer and Rubia died as a result of the
shooting.
ISSUE/S Whether or not the appellant acted in defense of property
RULING/S The defense of property is incomplete as to the necessary
elements which would have justified the act, but he may be
credited with the special mitigating circumstance of incomplete
defense, pursuant to paragraph 6, Article 13 of the Revised Penal
Code.
Defense of one's person or rights is treated as a justifying
circumstance under Art. 11, par. 1 of the Revised Penal Code, but
in order for it to be appreciated, the following requisites must
occur: "First. Unlawful aggression; "Second. Reasonable necessity
of the means employed to prevent or repel it; "Third. Lack of
sucient provocation on the part of the person defending himself"
(Art 11, par. 1, Revised Penal Code, as amended).
There is no question that there was aggression on the part of the
victims: Fleischer was ordering angrily the continuance of the
fencing would have resulted in the further chiselling of the walls of
appellant's house as well as the closure of the access to and from
his house and rice mill — which were not only imminent but were
actually in progress, and Rubia was actually participating in the
fencing. This was indeed aggression, not on the person of
appellant, but on his property rights.
The act is also unlawful because at the time of the incident on
August 22, 1968, Civil Case no. 755 for annulment of the order of
award to Fleischer and Company was still pending in the Court of
First Instance of Cotabato. The parties could not have known that
the case would be dismissed over a year after the incident. In any
case, their prior agreement allowed the appellant to stay at his
property until December 31, 1962. The deceased had no right to
destroy or cause damage to appellant's house, nor to close his
accessibility to the highway while he was pleading with them to
stop and talk things over with him. The assault on appellant's

190
property, therefore, amounts to unlawful aggression as
contemplated by law.
The third element of defense of property is also present, i.e., lack
of sufficient provocation on the part of appellant who was
defending his property. As a matter of fact, there was no
provocation at all on his part, since he was asleep at first and was
only awakened by the noise produced by the victims and their
laborers. His plea for the deceased and their men to stop and talk
things over with him was no provocation at all.
The missing element is the second requirement: reasonable
means employed to prevent or repel it. When the appellant fired
his shotgun from his window, killing his two victims, his resistance
was disproportionate to the attack.
Appellant's act in killing the deceased was not justifiable, since not
all the elements for justification are present. He should therefore
be held responsible for the death of his victims, but he could be
credited with the special mitigating circumstance of incomplete
defense, pursuant to paragraph 6, Article 13 of the Revised Penal
Code.

191
CONTRIBUTOR PROVIDO, Gemy Hale A.
MODULE Justifying Circumstance, defense of a relative
TOPIC
CASE TITLE SABANG v. PEOPLE G.R.NO. 168818
PONENTE TINGA, J: DATE: MAR. 9, 2007
DOCTRINE Unlawful aggression is an indispensable requisite in using the
justifying circumstance of defense of a relative and the petitioner
must clearly establish the evidence to use it as a defense..
FACTS During a fiesta in Liloan, Ormoc City, on January 17, 1997, Nicanor
Butad was drinking with spouses Cruz and Andresa Villamor.
Nicanor Butad had a .38-caliber revolved tucked in holster as he
was a civilian agent of the Philippine National Police. He uttered the
words ¨I will shoot you¨ to Randy Sabang, who just happens to
appear in their location. Shortly after, he was shot dead with four (4)
gunshot wounds on his body by the father of Randy Sabang, Nilo
Sabang. Nicanor Butad died from multiple bullets from the autopsy
results. Based from the witnesses of the defense, Butad is already
pointing his gun to Randy Sabang and Nilo Sabang only acted in
defense of his son. However, the trial court contends that based
from the evidence, powder burns were absent in the scene, and its
absence proves that the gunshots were fired from a distance of
more than 10 inches from the victim´s body. The absence of powder
guns makes the statements of Nilo Sabang invalid that he only acted
in defense for his son when Butad pointed a gun to his son.
ISSUE/S Whether or not Nilo Sabang can use the defense of a relative as a
justifying circumstance
RULING/S No, the trial court concluded that Nicanor Butad did not exhibit
unlawful aggression as Nilo Sabang failed to clearly establish the
evidence. To use the justifiying circumstance of defense of a
relative, the following requisites must be present:
1. Unlawful aggression on the part of the person killed or
injured;
2. Reasonable necessity of the means employed to prevent or
repel the means employed to prevent or repel the unlawful
aggression; and
3. The person defending the relative had no part in provoking
the assailant, should any provocation been given by the
relative attacked.
The act when Nicanor Butad uttered the words ¨I will kill you¨ while
pointing a gun at Randy Sabang was not considered as an unlawful
aggression on the part of Nicanor Butad. Since the first requisite is
not present, the instant petiton is denied and Nilo Sabang is charged
with the crime of homicide. Since he voluntarily surrendered for the

192
crime, the mitigating circumstance of voluntary surrender is
appreciated and the penalty shall be imposed in its minimum period.

193
CONTRIBUTOR ROMERO, Ma. Camille Concepcion M
MODULE Justifying Circumstances and Absolutory Causes : Self Defense
TOPIC
CASE TITLE People v. Dagani G.R.NO. 153875
PONENTE AUSTRIA-MARTINEZ, J. DATE: August 16, 2006
DOCTRINE When self-defense is invoked, the burden of evidence shifts to the
accused to show that the killing was legally justified.
FACTS At about 4:45 in the afternoon on September 11, 1989 a group
composed of Javier, Miran and two other individuals were drinking
at the canteen inside the compound of PNR along C.M Recto Ave.,
Tondo, Manila. Appellants who were security guards entered and
approached the group. Dagani shoved Miran causing him to fall
from his chair. Dagani then held Javier while Santiano shot Javier
twice at his left side, killing the latter.

Appellants invoked the justifying circumstances of self-defense and


lawful performance of official duty as PNR security officers. They
also argued that the prosecution failed to establish treachery and
conspiracy.

The RTC held the appellants guilty. Appellants failed to prove that
Javier attempted to squeeze the trigger of .22 caliber gun when he
pointed at Dagani ; they also failed to produce two empty shells as
physical evidence of the gunfire caused by Javier, there was also
no points of entry or bullet markings on the walls of the canteen were
shown ; that no unlawful aggression was present on the part of the
victim ; they failed to prove that the crime was consequence of their
due performance of an official duty ; the appellants were acting in
conspiracy ; qualifying circumstance of treachery attended the
killing, when Javier had been shot while his hands were being held
by Dagani and as his body was out of balance and about to fall; and
that the mitigating circumstance of voluntary surrender should be
appreciated in favor of the appellants.
ISSUE/S Whether or not the lower court erred in appreciating self defense on
the part of the accused
RULING/S No, the appellant failed to prove that there was unlawful aggression
onf the part of Javier, Dagani also failed to provide the two empty
shells proving that the victim fired his gun. Javier also tested
negative on the gun powder residue.
When self defense is invoked the burden of evidence lies on the
accused to show that the killing is legally justified. Self-defense
requires that there be (1) an unlawful aggression by the person
injured or killed by the offender, (2) reasonable necessity of the
means employed to prevent or repel that unlawful aggression, and

194
(3) lack of sufficient provocation on the part of the person defending
himself. All these conditions must concur.
The first element of unlawful aggression is the sudden or
unexpected attack of the victim, in the present case since they were
unable to present evidence that the victim fired his gun at the
appellant. Even if it was established that Javier fired his gun there
is still no unlawful aggression because the imminence of danger to
the appellants had already ceased when the victim had been thrown
off-balance.
The second element that the means employed to neutralize the
unlawful aggression are reasonable and necessary. What the law
requires is rational equivalence between the means of attack and
defense. Dagani grappled the gun with Javier which amounted to
two fatal gunshot wounds on the victim. Dagani has a larger figure
and had SWAT hand-to-hand combat training does not justify the
shooting at the victim twice.
The plea of self-defense cannot be justifiably entertained where it is
not only uncorroborated by any separate competent evidence but is
in itself extremely doubtful.

195
CONTRIBUTOR SAMONTE, Vanessa Antoinette
MODULE JUSTIFYING CIRCUMSTANCES- DEFENSE OF SELF,
TOPIC RELATIVE, AND STRANGERS
CASE TITLE RUJJERIC Z. PALAGANAS A.M. NO. G.R. NO.: 165483
vs. PEOPLE
PONENTE CHICO-NAZARIO, J.: DATE: SEPTEMBER 12, 2006

DOCTRINE In order to constitute unlawful aggression, the person attacked must


be confronted by a real threat on his life and limb; and the peril
sought to be avoided is imminent and actual, not merely imaginary.

FACTS Brothers Servillano, Melton and Michael Ferrer were on a drinking


spree in their house because Melton visited his mother and brothers
in the province. They later decided to proceed to Tidbits Videoke
Bar to continue their drinking spree and to sing. Thereafter, Jaime
Palaganas arrived together with Ferdinand Palaganas and Virgilio
Bautista. When it was Jaime Palaganas’ turn to sing,Melton sang
with him, which the former resented as he felt insulted. Jaime went
to Melton’s table and struck Servillano with the microphone on his
head. A rumble then ensued between the two groups. Ferdinand
went out if the bar and sought help from Rujjeric, his brother. They
went back to the bar and upon seeing the Ferrers instructed Rujjeric
to shoot them. Rujjeric Palaganas shot Servillano, Melton and
Michael. As a result, Melton was killed, Servillano was fatally
wounded and Michael was shot in his right shoulder. The Regional
Trial Court found petitioner guilty beyond reasonable doubt of the
crime of homicide and two (2) counts of frustrated homicide. On
appeal, the Court of Appeals affirmed the decision of the lower court.
Petitioner prayed for the reversal of the decision on the ground of
self-defense.

ISSUE/S Whether or not self-defense can be validly invoked.

RULING/S No. There is an unlawful aggression on the part of the victim when
he puts in actual or imminent peril the life, limb, or right of the person
invoking self-defense. There must be actual physical force or actual
use of weapon. In order to constitute unlawful aggression, the
person attacked must be confronted by a real threat on his life and
limb; and the peril sought to be avoided is imminent and actual, not
merely imaginary.
In the case at bar, it is clear that there was no unlawful aggression
on the part of the Ferrer brothers that justified the act of petitioner in
shooting them. There were no actual or imminent danger to the lives
of petitioner and Ferdinand when they proceeded and arrived at the
videoke bar and saw thereat the Ferrer brothers. It appears that the
Ferrer brothers then were merely standing outside the videoke bar

196
and were not carrying any weapon when the petitioner arrived with
his brother Ferdinand and started firing his gun.

197
CONTRIBUTOR SAMSON, Jessa Viena D.
MODULE Avoidance of greater evil (Art. 11, par. 4, RPC)
TOPIC
CASE TITLE PEOPLE v. RICOHERMOSO G.R.NO. L-30527-28
PONENTE AQUINO, J.: DATE: MAR. 29, 1974
DOCTRINE Paragraph 4, Article 11 of the Revised Penal Code which states that
any person who, in order to avoid an evil or injury, does not act
which causes damage to another, provided that the evil sought to
be avoided actually exists; that the injury feared be greater than that
done to avoid it; and that there be no other practical and less harmful
means of preventing it.
FACTS This is an appeal by Severo and Juan Padernal from the decision of
the Circuit Criminal Court convicting them of murder, sentencing
each of them to reclusion perpetua, and ordering them to pay
solidarily the sum of twelve thousand pesos to the heirs of
Geminiano de Leon and to pay the costs.
Geminiano de Leon, together with his common-law wife Fabiana
Rosales, his son Marianito de Leon, and one Rizal Rosales,
happened to encounter Pio Ricohermoso. Owning a parcel of land
which Ricohermoso cultivated as kaingin, Geminiano asked the
latter if he could have some palay. Relative thereon, Ricohermoso
said that [he] could drop by his house for palay anytime.
When Geminiano returned to the barrio, he stopped at
Ricohermoso’s place to ask about the palay. However, the latter
answered in defiant tone, “whatever happens, I will not give you
palay” to which the former remonstrated, “why did you tell us to pass
by your house, if you were not willing to give the palay?"
Consequently, Ricohermoso unsheathed his bolo and approached
Geminiano from the left, while Severo, who is Ricohermoso's father-
in-law, got an axe and approached Geminiano from the right. The
latter pleaded but was still stabbed by Ricohermoso on the neck with
his bolo. When Geminiano was in a helpless position, he was
hacked on the back with an axe by Severo. At the same time and
place, Ricohermoso’s brother-in-law Juan suddenly embraced
Marianito from behind. They grappled and rolled downhill at which
point the latter passed out. When he regained consciousness, he
saw that the rifle he carried beforehand was gone, and his father
was mortally wounded.
On the contrary, when Ricohermoso refused to give any palay to
Geminiano, because the land tilled by the former was allegedly a
public land, the latter approached former. When Geminiano
unsheathed his bolo, Ricohermoso met him, drew his bolo and
struck Geminiano on the left side of the neck. The latter tried to
parry the blow. He was wounded in the wrist. As Geminiano turned
right to flee, Ricohermoso struck him again on the left side of his
body, causing him to fall on the ground. The former died on the spot

198
due to the bleeding from the wound on his neck. While Geminiano
was being assaulted, his son Marianito tried to shoot with his
rifle. However, Juan disabled him and wrested the gun.
ISSUE/S Whether or not Juan Padernal can invoke the justifying
circumstance of avoidance of greater evil or injury
RULING/S Juan Padernal’s reliance on that justifying circumstance is
erroneous. The act of him in preventing Marianito from shooting
Ricohermoso and Severo, who were the aggressors, was designed
to insure the killing of Geminiano without any risk to his
assailants. [He] was not avoiding any evil when he sought to disable
Marianito. His malicious intention was to forestall any interference
in the felonious assault made by his father and brother-in-law on
Geminiano. This situation is unarguably not the case envisaged in
paragraph 4 of Article 11 of the Revised Penal Code.

199
CONTRIBUTOR TERTE, Karen A.
MODULE Justifying Circumstances; Avoidance of greater evil
TOPIC
CASE TITLE PEOPLE v. NORMA G.R.NO. 22553-R
HERNANDEZ
PONENTE GUTIERREZ DAVID, PRES. J. DATE: APRIL 4, 1959
DOCTRINE Under Article 11, paragraph 4 of the Revised Penal Code, any
person who, in order to avoid an evil or injury, does not act which
causes damage to another, provided that the following requisites
are present; (1) That the evil sought to be avoided actually exists;
(2) That the injury feared be greater than that done to avoid it; and
(3) That there be no other practical and less harmful means of
preventing it, do not incur criminal liability. Any person party has
the right to avoid to himself or herself the evil of going through a
loveless marriage pursuant to Article 11, paragraph 4 of the
Revised Penal Code. A party to an agreement to marry who backs
out can- not be held liable for the crime of slander by deed, for
then that would be an inherent way of compelling said party to go
into a marriage without his or her free consent, and this would
contravene the principle in law that what could not be done
directly could not be done indirectly.
FACTS Vicencio Lascano, the complainant, courted the appellant Norma
Hernandez. After months of courtship, the two have agreed to
marry. Vivencio and his parents went to Norma’s house, they
brought chickens and goats and agreed to buy a wedding dress, 2
vestidas, shoes, P20 for the sponsors and to repair the uncle’s roof.
However, during the celebration of marriage, Norma did not show
up. This has caused Vivencio and his parents’ great shame and
humiliation.

Norma testified that she was not in love with Vivencio and was only
persuaded by her parents to accept the proposal. However, before
Vivencio’s parents visit to their home, she communicated with them
to not bring those chickens but they insisted such that appellant had
to tell them that they and Vivencio should not regret what should
happen later. Hence, she felt that she has no choice but to leave
home as a last recourse to prevent the marriage. The Regional Trial
Court convicted her of serious slander by deed as she purposely
and deliberately fled to prevent celebration of marriage.

ISSUE/S Whether or not Norma Hernandez is guilty of slander by deed.


RULING/S No, Norma Hernandez cannot be punished for the crime of
Slander by deed.
Malice, one of the essential requisites of slander, has not been
proven. There is no malice in the act of the appellant changing her

200
mind. She was merely exercising her right not to give her consent
to the marriage after mature consideration. Furthermore, there
were no strained relations existing between the complainant &
appellant before the incident. There always existed good relations
between them for they were neighbors so it cannot be sustained
that appellant was motivated by spite or ill-will in deliberately
frustrating the marriage.
Pursuant to Article 11 paragraph 4 of the Revised Penal Code, any
person who, in order to avoid an evil or injury, does not act which
causes damage to another, provided that the requisites are
present, do not incur criminal liability. Appellant had the right to
avoid to herself the evil of going through a loveless marriage.
Norma has the privilege to reconsider her previous commitment to
marry and it would be utterly inconsistent to convict her for slander
by deed simply because she desisted in continuing with the
marriage. If she would be liable then that would be tantamount to
compelling her to go into a marriage without her free consent.

201
CONTRIBUTOR TIDALGO, Aimee Diane A.
MODULE Justifying Circumstances; Fulfillment of duty, Defined
TOPIC
CASE TITLE PEOPLE V. DELIMA G.R.NO. L-18660
PONENTE ROMUALDEZ, J. DATE: DEC. 22, 1922
DOCTRINE Fulfillment of a duty- Any person who acts in the fulfillment of a
duty or in the lawful exercise of a right or office
FACTS Lorenzo Napilon had escaped from the jail where he was serving
sentence.
Some days afterwards the policeman Felipe Delima, who was
looking for him, found him in the house of Jorge Alegria, armed with
a pointed piece of bamboo in the shape of a lance, and demanded
his surrender. The fugitive answered with a stroke of his lance. The
policeman dodged, it, and to impose his authority fired his revolver,
but the bullet did not hit him. The criminal ran away, without parting
with his weapon. These peace officer went after him and fired again
his revolver, this time hitting and killing him.
The policeman was tried and convicted for homicide and sentenced
to reclusion temporal and the accessory penalties. He appeals from
that judgment which must be reversed.
ISSUE/S Whether or not Delima acted self-defense in the performance of his
duty
RULING/S YES. The killing was done in the performance of a duty. The
deceased was under the obligation to surrender, and had no right,
after evading service of his sentence, to commit assault and
disobedience with a weapon in the hand, which compelled the
policeman to resort to such an extreme means, which, although it
proved to be fatal, was justified by the circumstances.lawphil.ne

202
CONTRIBUTOR Valdez, Ariane Faye V.
MODULE Module 5- Fulfillment of duty (Art. 11 (5), RPC)
TOPIC
CASE TITLE People v. Lagata G.R.NO. L-1940-42
PONENTE PERFECTO, J. DATE: 24 March 1949
DOCTRINE Mandate of the Gospel "THOU SHALT NOT KILL."— Human life is
valuable. Laws, constitutions, world charters have been written to
protect human life.

Firing at or killing prisoners without absolute necessity is illegal and


improper.
FACTS The accused, Ignacio Lagata, a provincial guard of Catbalogan,
Samar, was in charge of six prisoners charged with murder,
assigned to clean the capitol plaza of Samar. On their return to the
prison compound, he gave said prisoners’ permission to gather
gabi, in the presence of the accused who remained at a distance of
about six meters. Instantly, he discovered that prisoner Epifanio
Labong had escaped. The accused then asked the remaining
prisoners to help in locating him, but in so doing he was led by said
prisoners to places where escape was much easier. The accused
fired his gun in the air in order to stop the fleeing prisoners. Some
of the prisoners were already going to the nearby mountain
apparently in attempt also to escape. Whereupon the accused
decided to aim his gun at those who were fleeing until one of them
was hurt and another was killed.

Appellant was charged with murder, serious physical injuries and


evasion through negligence in three separate cases which have
been tried jointly. The trial court found him guilty of the said charges.
Hence this appeal from a judgment of the CFI of Samar.
ISSUE/S Whether or not the appellant was entitled to the benefit of Art. 11-
Justifying Circumstances under par. 5- Fulfillment of a duty or lawful
exercise of right or office.
RULING/S No. The requisites for fulfillment of duty or lawful exercise of
right or office under Art 11 par. 5 of the RPC includes the following:
1. That the accused acted in the performance of a duty or
in the lawful exercise of a right or office;
2. That the injury caused or the offense committed be the
necessary consequence of the due performance of duty
or the lawful exercise of such right or office.
In the case at bar, we apply the following requisites:
1. It is clear that Lagata had absolutely no reason to fire at
Tipace. Lagata could have fired at him in self-defense or if
absolutely necessary to avoid his escape. The record

203
does not show that Tipace was bent on committing any act
of aggression or that he attempted to escape.

2. The necessary consequence made by Lagata was not in the


performance of a duty or in the lawful exercise of right or
office. It can be remembered from the facts that, the escape
of Labong scared Lagata, according to him, because of the
experience of other guards who were dismissed from office
or even prosecuted because of prisoners who had escaped
under their custody, and that it was his duty to fire against the
prisoners if he wanted to be exempt from any responsibility.
Even if appellant sincerely believed, although erroneously, that in
firing the shots be acted in the performance of his official duty, the
circumstances of the case show that there was no necessity for him
to fire directly against the prisoners, so as to seriously wound one
of them and kill instantaneously another.
While custodians of prisoners should take all care to avoid the
latter's escape, only absolute necessity would authorize them to
fire against them. Theirs is the burden of proof as to such
necessity.
Therefore, accused Lagata is not entitled to the benefit of Art. 11-
Justifying Circumstances under par. 5- Fulfillment of a duty or lawful
exercise of right or office.

204
CONTRIBUTOR VILLANUEVA, Sean Ruthie
MODULE JUSTIFYING CIRCUMSTANCES AND ABSOLUTORY CAUSES
TOPIC
CASE TITLE RUFINO S. MAMANGUN, petitioner, vs. G.R NO. 149152
PEOPLE OF THE PHILIPPINES,
respondent.
PONENTE GARCIA, J DATE: February 2,
2007
DOCTRINE The justifying circumstance of fulfillment of duty under paragraph 5,
Article II, of the Revised Penal Code may be invoked only after the
defense successfully proves that: (1) the accused acted in the
performance of a duty; and (2) the injury inflicted or offense
committed is the necessary consequence of the due performance
or lawful exercise of such duty.

FACTS At about 9:00 o'clock that same evening, the desk officer of the
Meycauayan PNP Police Station, upon receiving a telephone call
that a robbery-holdup was in progress in Brgy. Calvario,
immediately contacted and dispatched the crew to the scene
Petitioner Mamangun, PO2 Diaz and PO2 Cruz went to the rooftop
of the house whereat the suspect was allegedly taking refuge.
According to Ayson, the lone eyewitness for the prosecution, he
accompanied the three policemen to the rooftop of Abacan's house.
He was following petitioner Mamangun who was ahead of the group.
They passed through the second-floor door of the house to the
rooftop. The roof was lighted by an incandescent bulb from an
adjacent house. He was beside Mamangun when they saw, some
four to five arms-length away, a man whom he recognized as
Contreras. Mamangun pointed his .45 cal. pistol at the man, who
instantly exclaimed, "Hindi ako, hindi ako!," to which Mamangun
replied, "Anong hindi ako?" Before he (Ayson) could say anything,
Mamangun fired his gun, hitting the man who turned out to be
Contreras. He (witness) approached the victim who was then lying
on his left side unconscious. He brought down the victim and they
rushed him to the hospital where he died at about 10:00 o'clock that
same evening.

ISSUE/S Whether or not the death of the victim was a necessary


consequence of the petitioner’s fulfillment of his duty

RULING/S Acts in the fulfillment of a duty, without more, do not completely


justify the petitioner's firing the fatal gunshot at the victim.

Although the petitioner, as one of the policemen responding to a


reported robbery then in progress, was performing his duty as a
police officer as well as when he was trying to effect the arrest of

205
the suspected robber and in the process, fatally shoot said suspect,
albeit the wrong man. However, in the absence of the equally
necessary justifying circumstance that the injury or offense
committed be the necessary consequence of the due performance
of such duty, there can only be incomplete justification, a privileged
mitigating circumstance under Articles 13 and 69 of the Revised
Penal Code.

There can be no quibbling that there was no rational necessity for


the killing of Contreras. Petitioner could have first fired a warning
shot before pulling the trigger against Contreras who was one of the
residents chasing the suspected robber.

206
CONTRIBUTOR AGUILAR, Jose Maria L.
MODULE JUSTIFYING CIRCUMSTANCES AND ABSOLUTORY CAUSES
TOPIC
CASE TITLE INTESTATE ESTATE OF MANOLITA G.R.NO. 181409
CONZALES v. PEOPLE
PONENTE CORONA, J: DATE: FEB. 11,
2010
DOCTRINE Ubi lex non distinguit nec nos distinguere debemos. Basic is the rule
in statutory construction that where the law does not distinguish, the
courts should not distinguish. There should be no distinction in the
application of law where none is indicated.
FACTS Mediatrix G. Carungcong, in her capacity as the duly appointed
administratrix of petitioner intestate estate of her deceased mother
Manolita Gonzales vda. de Carungcong, filed a complaint-
affidavit for estafa against her brother-in-law, William Sato, a
Japanese national.

In her conference with her nieces, she was able to learn that prior
to the death of her mother, their father William Sato (accused),
through fraudulent misrepresentations, was able to secure the
signature and thumbmark of my mother on a Special Power of
Attorney whereby her niece, who was then only twenty 20 years old,
was made her attorney-in-fact, to sell and dispose four valuable
pieces of land. Said Special Power of Attorney was signed and
thumbmark by her mother because the accused told her that the
documents she was being made to sign involved her taxes. At that
time, my mother was completely blind. The said Special Power of
Attorney was signed by her mother in the presence of her other
niece, their maid and Governor Ramirez who later became the
second wife of her sister's widower (accused).

Demands have been made for the accused to make an accounting


and to deliver the proceeds of the sales to her as Administratrix of
her mother's estate, but he refused and failed, and continues to
refuse and to fail to do so. Sato moved for the quashal of the
Information, claiming that under Article 332 of the Revised Penal
Code, his relationship to the person allegedly defrauded, the
deceased Manolita who was his mother-in-law, was an exempting
circumstance.
ISSUE/S Whether or not the accused is exempted from criminal liability for
the crime of estafa under Art. 332 of the Revised Penal Code.
RULING/S No. Article 332 provides for an absolutory cause in the crimes of
theft, estafa (or swindling) and malicious mischief. It limits the
responsibility of the offender to civil liability and frees him from
criminal liability by virtue of his relationship to the offended party. In

207
connection with the relatives mentioned in the first paragraph, it has
been held that included in the exemptions are parents-in-law,
stepparents and adopted children.
Affinity is the relation that one spouse has to the blood relatives of
the other spouse. It is a relationship by marriage or a familial relation
resulting from marriage.

The absolutory cause under Article 332 of the Revised Penal Code
only applies to the felonies of theft, swindling and malicious
mischief. Under the said provision, the State condones the criminal
responsibility of the offender in cases of theft, swindling and
malicious mischief. As an act of grace, the State waives its right to
prosecute the offender for the said crimes but leaves the private
offended party with the option to hold the offender civilly liable.

However, the coverage of Article 332 is strictly limited to the felonies


mentioned therein. The plain categorical and unmistakable
language of the provision shows that it applies exclusively to the
simple crimes of theft, swindling and malicious mischief. It does not
apply where any of the crimes mentioned under Article 332 is
complexed with another crime, such as theft through falsification or
estafa through falsification.

In the case at bar, the information given, what the accused


committed is not simple estafa but complex crime of estafa through
falsification of public documents.
Thus, by inducing Manolita to sign the SPA, the accused made it
appear that Manolita granted his daughter Wendy a special power
of attorney for the purpose of selling, assigning, transferring or
otherwise disposing of Manolita's properties when the fact was that
Manolita signed and thumbmarked the document presented by the
accused in the belief that it pertained to her taxes.

The situation would have been different if the accused, using the
same inducement, had made Manolita sign a deed of sale of the
properties either in his favor or in favor of third parties. In that case,
the damage would have been caused by, and at exactly the same
time as, the execution of the document, not prior thereto. On the
other hand, absent any inducement the crime would have only been
the simple crime of falsification.

208
CONTRIBUTOR ALGURA, Nino N.
MODULE Justifying Circumstances and Absolutory Causes
TOPIC
CASE TITLE THE PEOPLE OF THE G.R.NO. 34917
PHILIPPINE ISLANDS
v.
LUA CHU and UY SE TIENG
PONENTE VILLA-REAL, J.: DATE: Sept. 7, 1931
DOCTRINE The entrapment of offenders and instigation to commit crime, as laid
down by the courts of the United States, are summarized in 16
Corpus Juris, page 88, section 57 .The practice of entrapping
persons into crime for the purpose of instituting criminal
prosecutions is to be deplored, and while instigation, as
distinguished from mere entrapment, has often been condemned
and has sometimes been held to prevent the act from being criminal
or punishable, the general rule is that it is no defense to the
perpetrator of a crime that facilitates for its commission were
purposely placed in his way, or that the criminal act was done at the
"decoy solicitation" of persons seeking to expose the criminal, or
that detectives feigning complicity in the act were present and
apparently assisting in its commission.
FACTS The defendants Lua Chu and Uy Se Tieng appeal from the judgment
of the Court of First Instance of Cebu convicting them of the illegal
importation of opium, and sentencing them each to four years'
imprisonment, a fine of P10,000, with subsidiary imprisonment in
case of insolvency not to exceed one-third of the principal penalty,
and to pay the proportional costs.
The accused Uy Se Tieng, an agent of the real owners of the
Shipments of Opium, wrote to his correspondent in Hongkong to
send him a shipment of opium. This opium had been in Hongkong
for some time, awaiting a ship that would go directly to Cebu. The
Collector of Customs of Cebu received information that the accused
was intending to land opium in the port. Juan Samson, a secret
serviceman, pretended to smooth the way for the introduction of the
prohibited drug. Samson then promised the accused that he would
remove all the difficulties in the way, and for this purpose agreed to
receive in exchange P6,000: P2,000 for Juan Samson, P2,000 for
Joaquin Natividad, and the remaining P2,000 would be distributed
among certain employees in the customhouse. Upon arrival of the
shipment of opium in the ports of Cebu, Uy Se Tieng, informed
Samson that the former consulted the real owners on how to
proceed with the payment of P6,000 and will come over to Samson
’s
house on to inform the decision of the owners. On the same day
Samson informed the Constabulary headed by Colonel Francisco
who instructed the provincial commander, Captain Buencosejo to

209
discuss the capture of the opium owners. Jumapao, a stenographer,
through the provincial fiscal and in the presence of Captain
Buencosejo,was asked to take the down the conversation Samson
would have with Uy Se Tieng. Upon the rendezvous, Captain
Buencosejo and Jumapao hid themselves behind the curtains in the
house of Samson to witness the conversation between Samson, Uy
Se Tieng, and Lua Chu. The following morning, Uy Se Tieng and Uy
Ay, a companion, presented papers to Samson. Captain
Buencosejo showed up and arrested the two Chinese. The
Constabulary arrested Lua Chu and seized the 3,252 tins of opium
worth P50,000.
ISSUE/S Whether the trial court erred in excluding Juan Samson as one of
the accused more so an instigator.
RULING/S It is true that Juan Samson smoothed the way for the introduction of
the prohibited drug, but that was after the accused had already
planned its importation and ordered for said drug. Juan Samson
neither induced nor instigated the accused to import the opium in
question, but pretended to have an understanding with the Collector
of Customs, who had promised them that he would remove all the
difficulties in the way of their enterprise so far as the customs house
was concerned. This is not a case where an innocent person is
induced to commit a crime merely to prosecute him, but it is simply
a trap set to catch a criminal. Therefore, the mere fact that the chief
of customs secret service pretended to agree to a plan for smuggling
illegally imported opium through the custom house, in order to better
the seizure of said opium and the arrest of its importers, is no bar to
the prosecution and conviction of the accused.

ENTRAPMENT AND INSTIGATION. — While it has been said that


the practice of entrapping persons into crime for the purpose of
instituting criminal prosecutions is to be deplored, and while
instigation, as distinguished from mere entrapment, has often been
condemned and has sometimes been held to prevent the act from
being criminal or punishable, the general rule is that it is no defense
to the perpetrator of a crime that facilitates for its commission were
purposely placed in his way, or that the criminal act was done at the
"decoy solicitation" of persons seeking to expose the criminal, or
that detectives feigning complicity in the act were present and
apparently assisting in its commission. Especially is this true in that
class of cases where the offense is one of a kind habitually
committed, and the solicitation merely furnishes evidence of a
course of conduct. Mere deception by the detective will not shield
defendant, if the offense was committed by him free from the
influence or the instigation of the detective. The fact that an agent
of an owner acts as supposed confederate of a thief is no defense
to the latter in a prosecution for larceny, provided the original design

210
was formed independently of such agent; and where a person
approached by the thief as his confederate notifies the owner or the
public authorities, and, being authorized by them to do so, assists
the thief in carrying out the plan, the larceny is nevertheless
committed. It is generally held that it is no defense to a prosecution
for an illegal sale of liquor that the purchase was made by a
"spotter," detective, or hired informer; but there are cases holding
the contrary.

211
CONTRIBUTOR AROZA, Maria Minette R.
MODULE Absolutory Causes
TOPIC
CASE TITLE PEOPLE v. DORIA G.R.NO. 125299
PONENTE PUNO, J: DATE: JAN. 22, 1999
DOCTRINE The type of entrapment the law forbids is the inducing of another to
violate the law, the "seduction" of an otherwise innocent person
into a criminal career. Instigation is a defense and is considered an
absolutory cause.
FACTS Florencio Doria and Violeta Gaddao were charged with violation of
Section 4, in relation to Section 21 of the Dangerous Drugs Act of
1972. Members of PNP received an information from civilian
informants that one “Jun” who was later identified to be Florencio
Doria was engaged in illegal drug activities. The Narcom agents
decided to entrap and arrest "Jun" in a buy-bust operation.

During the buy-bust operation "Jun" appeared and the informant


introduced PO3 Manlangit as interested in buying one (1) kilo of
marijuana. “Jun” took out from his bag an object wrapped in plastic
and gave it to PO3 Manlangit. PO3 Manlangit forthwith arrested
“Jun” as SPO1 Badua rushed to help in the arrest. They frisked “Jun”
but did not find the marked bills on him. Upon inquiry, “Jun” revealed
that he left the money at the house of his associate named “Neneth”.
He led the police to “Neneth’s” house.

The team found the door of “Neneth’s” house open and a woman
inside. “Jun” identified the woman as his associate. SPO1 Badua
asked “Neneth” about the P1,600.00 as PO3 Manlangit looked over
“Neneth’s” house. Standing by the door, PO3 Manlangit noticed a
carton box under the dining table. He saw that one of the box’s flaps
was open and inside the box was something wrapped in plastic. The
plastic wrapper and its contents appeared similar to the marijuana
earlier “sold” to him by “Jun.” His suspicion aroused, PO3 Manlangit
entered “Neneth’s” house and took hold of the box. He peeked
inside the box and found that it contained 10 bricks of what
appeared to be dried marijuana leaves.

The prosecution story was denied by accused-appellants.

“Neneth” who was later identified as Violeta Gaddao, on her


testimony, stated that inside her house were her co-accused Doria
and three (3) other persons. She was asked about a box on top of
the table. This was the first time she saw the box. The box was
closed and tied with a piece of green straw. The men opened the
box and showed her its contents. She said she did not know
anything about the box and its contents.

212
She denied the charge against her and Doria and the allegation that
marked bills were found in her person.
ISSUE/S Whether or not the buy-bust operation in the apprehension of
accused-appellant Doria is valid.
RULING/S The Supreme Court ruled that the buy-bust operation is valid.
Accused-appellants were caught by the police in a buy-bust
operation. A buy-bust operation is a form of entrapment employed
by peace officers as an effective way of apprehending a criminal in
the act of the commission of an offense.
Entrapment has received judicial sanction when undertaken with
due regard to constitutional and legal safeguards, however, there is
a certain amount of entrapment used to outwit the persons violating
or about to violate the law. Not every deception is forbidden. The
type of entrapment the law forbids is the inducing of another to
violate the law, the "seduction" of an otherwise innocent person into
a criminal career.
Instigation is a defense and is considered an absolutory cause. To
determine whether there is entrapment or instigation, our courts
have mainly examined the conduct of the apprehending officers, not
the predisposition of the accused to commit the crime. The
"objective" test is applied in some cases.
The "objective" test in buy-bust operations demands that the details
of the purported transaction must be clearly and adequately shown.
This must start from the initial contact between the poseur-buyer
and the pusher, the offer to purchase, the promise or payment of
the consideration until the consummation of the sale by the delivery
of the illegal drug subject of the sale. The manner by which the initial
contact was made, whether or not through an informant, the offer to
purchase the drug, the payment of the "buy-bust" money and the
delivery of the illegal drug, whether to the informant alone or the
police officer, must be the subject of strict scrutiny by courts to
insure that law-abiding citizens are not unlawfully induced to commit
an offense. Criminals must be caught but not at all cost. At the same
time, however, examining the conduct of the police should not
disable courts into ignoring the accused's predisposition to commit
the crime. If there is overwhelming evidence of habitual
delinquency, recidivism or plain criminal proclivity, then this must
also be considered. Courts should look at all factors to determine
the predisposition of an accused to commit an offense in so far as
they are relevant to determine the validity of the defense of
inducement.
In the case at bar, the evidence shows that it was the confidential
informant who initially contacted accused-appellant Doria. At the
pre-arranged meeting, the informant was accompanied by PO3
Manlangit who posed as the buyer of marijuana. PO3 Manlangit
handed the marked money to accused-appellant Doria as advance

213
payment for one (1) kilo of marijuana. Accused-appellant Doria was
apprehended when he later returned and handed the brick of
marijuana to PO3 Manlangit.

214
CONTRIBUTOR Belano, Renato Jr. P.
MODULE Justifying Circumstances and Absolutory Causes
TOPIC
CASE TITLE People v. Lim G.R. NO. 231989
PONENTE Peralta, J. DATE: Sept 4, 2018
DOCTRINE To establish a chain of custody sufficient to make evidence
admissible, the proponent needs only to prove a rational basis
from which to conclude that the evidence is what the party claims it
to be. (Proof beyond reasonable doubt)
FACTS Version of the Prosecution

Around 8:00 p.m. on October 19, 2019, IO1 Orellan and his
teammates were at Regional Office X of PDEA. Based on a report
of a confidential informant that a certain “Romy” has been engaged
in the sale of prohibited drugs in Cabina, Bonbon, Cagayan de Oro
City. The confidential informant knocked at the door and uttered,
“ayo, nong Romy.” Gorres came out and invited them to enter. Lim
was inside sitting on the sofa while watching the television. When
the confidential informant introduced IO1 Carin as a shabu buyer,
Lim nodded and told Gorres to get one inside the bedroom. Gorres
stood up and did as instructed. After he came out, he handed a small
medicine box to Lim, who then took one piece of heat-sealed
transparent plastic of shabu and gave it to IO1 Carin. In tum, IO1
Carin paid him with the buy-bust money.

After examining the plastic sachet, IO1 Carin executed a missed


call to IO1 Orellan which was the pre-arranged signal. The latter,
with the rest of the team members, rushed to Lim’s house. They
then entered the house because the gate was opened. IO1 Orellan
declared that they were PDEA agents and informed Lim and
Gorres, who were visibly surprised, of their arrest for selling
dangerous drug.

During the arrest IO1 Orellan took custody and marked a disposable
lighter and the two seized plastic sachets from Lim. One of which
was from Lim’s pocket.

Despite exerting efforts to secure the attendance of representative


from the media and barangay officials, nobody arrived to witness
the inventory taking.

IO1 Orellan took possession of the seized items and made the
Inventory Receipt of the confiscated items. It was not signed by Lim
and Gorres. Also, there was no signature of an elected public official
and the representatives of the DOJ and the media witness.

215
PSI Caceres got urine samples from Lim and Gorres and conducted
screening and confirmatory tests on them. Based on her
examination, only Lim was found positive for the presence of shabu.
With respect to the two sachets of white crystalline substance, both
were found to be positive of shabu after a chromatographic
examination was conducted by PSI Caceres. PSI Caceres, likewise,
put her own marking on the cellophane containing the two sachets
of shabu. After that, she gave them to the evidence custodian.

Version of the Defense


Around 10:00 p.m. on October 19, 2010, Lim and Gorres were in
their house. Lim was sleeping in the bedroom, while Gorres was
watching the television. When the latter heard that somebody
jumped over their gate, he stood up to verify. Before he could
reach the door, however, it was already forced opened by the
repeated pulling and kicking of men in civilian clothing. They
entered the house, pointed their firearms at him, instructed him to
keep still, boxed his chest, slapped his ears, and handcuffed him.
They inquired on where the shabu was, but he invoked his
innocence. Thereafter, the two were brought to the PDEA Regional
Office and the crime laboratory. During the inquest proceedings,
Lim admitted, albeit without the assistance of a counsel, ownership
of the two sachets of shabu because he was afraid that the police
would imprison him.

Rubenia, Lim's live-in partner and the mother of Gorres, was at her
sister's house in Pita, Pasil, Kauswagan the night when the arrests
were made. The following day, she returned home and noticed that
the door was opened, and its lock was destroyed. She took
pictures of the damage and offered the same as exhibits for the
defense, which the court admitted as part of her testimony.

RTC Ruling
After trial, the RTC handed a guilty verdict on Lim for illegal
possession and sale of shabu and acquitted Gorres for lack of
sufficient evidence linking him as a conspirator.

CA Ruling
On appeal, the CA affirmed the RTC Decision. It agreed with the
finding of the trial court that the prosecution adequately
established all the elements of illegal sale of a dangerous drug as
the collective evidence presented during the trial showed that a
valid buy-bust operation was conducted.
ISSUE/S Whether or not the chain of custody rule was properly complied with
in accordance with R.A. No. 9165.

216
RULING/S No. We have held that the immediate physical inventory and
photograph of the confiscated items at the place of arrest may be
excused in instances when the safety and security of the
apprehending officers and the witnesses required by law or of the
items seized are threatened by immediate or extreme danger such
as retaliatory action of those who have the resources and capability
to mount a counter-assault. The present case is not one of those.

Evident in this case, is the absence of an elected public official and


representatives of the DOJ and the media to witness the physical
inventory and photograph of the seized items. In fact, their
signatures do not appear in the Inventory Receipt.

It is well to note that the absence of these required witnesses does


not per se render the confiscated items inadmissible. However, a
justifiable reason for such failure or a showing of any genuine
and sufficient effort to secure the required witnesses under
Section 21 of RA 9165 must be adduced.

In this case, IO1 Orellan testified that members of the media and
barangay officials were absent at the crime scene because it was
late at night and it was raining, making it unsafe for them to wait at
Lim's house. IO2 Orcales similarly declared that the inventory was
made in the PDEA office considering that it was late in the evening
and there were no available media representative and barangay
officials despite their effort to contact them. He admitted that there
are times when they do not inform the barangay officials prior to
their operation as they might leak the confidential information. The
court is of the view that these justifications are unacceptable as
there was no genuine and sufficient attempt to comply with the
law.

The prosecution likewise failed to explain why they did not secure
the presence of a representative from the DOJ. While the arresting
officer, IO1 Orellan, stated in his Affidavit that they only tried to
coordinate with the barangay officials and the media, the
testimonies of the prosecution witnesses failed to show that they
tried to contact a DOJ representative. The testimonies of the
prosecution witnesses also failed to establish the details of an
earnest effort to coordinate with and secure presence of the
required witnesses.

The judgment of conviction is reversed and set aside, and Lim


should be acquitted based on reasonable doubt.

217
CONTRIBUTOR BILTZ, Aralind Louise A.
MODULE Justifying Circumstances
TOPIC
CASE TITLE VELASQUEZ VS. PEOPLE G.R.NO. 195021
PONENTE LEONEN, J. DATE: MARCH 15, 2017
DOCTRINE An accused who pleads a justifying circumstance under Article 11
of the Revised Penal Code1 admits to the commission of acts,
which would otherwise engender criminal liability. However, he
asserts that he is justified in committing the acts. In the process of
proving a justifying circumstance, the accused risks admitting the
imputed acts, which may justify the existence of an offense were it
not for the exculpating facts. Conviction follows if the evidence for
the accused fails to prove the existence of justifying
circumstances.
FACTS Spoused Jesus and Ana Del Mundo went to their nipa hut to sleep.
Upon arriving at the nipa hut, they saw Ampong and Nora Castillo
in the midst of having sex. Aghast at what he perceive to be a
defilement of his property, Jesus shouted incentives at Ampong and
Nora, who both scampered away. Jesus decided to pursue them
and went to the house of Ampong’s aunt but neither of them were
there. He then went home but was blocked by Ampong and his
fellow accused.

Without provocation, petitioner Nicolas hit Jesus on the forehead


with a stone. Petitioner Victor hit Jesus on the eyebrow with a stone.
Accused Felix did the same while Accused Sonny struck Jesus with
a bamboo. Ampong punched Jesus on his left cheek. They left
Jesus bloodied on the ground. Jesus the hid on the fear that they
might return. He got up and staggered his way back to their house.
ISSUE/S Whether or not there was sufficient evidence to prove that
justifying circumstances existed.
RULING/S None.
To successfully invoke self-defense, an accused must establish:
"(1) unlawful aggression on the part of the victim; (2) reasonable
necessity of the means employed to prevent or repel such
aggression; and (3) lack of sufficient provocation on the part of the
person resorting to self-defense."
Defense of a relative under Article 11 (2) of the Revised Penal
Code requires the same first two (2) requisites as self-defense
and, in lieu of the third "in case the provocation was given by the
person attacked, that the one making the defense had no part
therein.
Petitioners offered nothing more than a self-serving,
uncorroborated claim that Jesus appeared out of nowhere to go
berserk in the vicinity of their homes. They failed to present

218
independent and credible proof to back up their assertions. The
Regional Trial Court noted that it was highly dubious that Jesus
would go all the way to petitioners' residences to initiate an attack
for no apparent reason.

219
CONTRIBUTOR CACHERO, Luis III L.
MODULE Justifying Circumstance
TOPIC
CASE TITLE PEOPLE V REGALARIO G.R.NO. 174483
PONENTE LEONARDO-DE CASTRO, J. DATE: March 31, 2009
DOCTRINE When self-defense is invoked by an accused charged with murder
or homicide, he necessarily owns up to the killing but may escape
criminal liability by proving that it was justified and that he incurred
no criminal liability thereof. The three (3) elements of self-defense,
namely: (a) Unlawful aggression on the part of the victim, (b)
reasonable necessity of the means employed to prevent or repel the
aggression; and (c) lack of sufficient provocation on the part of the
person defending himself. Unlawful aggression is an indispensable
requirement in order for self-defense to be appreciated, either
complete or incomplete.
FACTS On the night of February 22, 1987, a dance and singing contest was
being held in the barangay pavilion of Natasan, Libon, Albay. The
victim Rolando Sevilla and Armando Poblete were enjoying the
festivities when appellant Sotero Regalario approached them. To
avoid trouble, the two distanced themselves from Sotero. After a
while, appellants Sotero and Bienvenido were seen striking Ronaldo
Sevilla with nightsticks several times. The blows caused Sevilla to
fall down in a sitting position but was able to get up after a short
while. Bienvenido and Sotero chased Sevilla. When Sevilla was
already near Marciano’s house, he was waylaid by appellant Ramon
Regalario and at this point, Marciano Regalario and his son Noel
came out of their house. Noel was carrying a seven-inch knife. The
five appellants took turns in hitting the victim until he slumped to the
ground face down. In this position, Sevilla was boxed by Marciano
in the jaw. When Sevilla was no longer moving, Marciano ordered
the others to kill the victim and tie him up. Upon hearing the order,
Bienvenido, with the help of Sotero, tied the neck, hands and feet of
the victim with a nylon rope used by farmers for tying carabao. The
rest of the group just stood by watching.
Accused-appellants Ramon, Marciano, Sotero, Bienvenido, and
Noel, all surnamed Regalario were charged with the crime of Murder
for the death of Rolando Sevilla.
The trial court found the accused-appellants guilty of Murder and
sentenced them with a penalty of reclusion perpetua to death.
Accused-appellant Ramon Regalario argues that the trial court
erred in not appreciating the justifying circumstance of self-defense.

ISSUE/S Whether or not the trial court erred in not finding that the deceased
was killed in self-defense and/or defense of relative.

220
RULING/S The Court did not appreciate accused-appellant’s claim of self-
defense. It was held that when unlawful aggression ceases, the
defender no longer has the right to kill or even wound the former
aggressor. In this case, accused-appellant contends that the victim
Rolando Sevilla committed an act of unlawful aggression with no
provocation on his part. By accused-appellant’s own account, after
he was shot, he hit the victim at the back of the latter’s head and he
continued hitting the victim who retreated backwards. From that
moment, the inceptive unlawful aggression on the part of the victim
ceased to exist.

221
CONTRIBUTOR CALZADO, Anne Valerie L.
MODULE Justifying Circumstances, Self-Defense (Art. 11, RPC)
TOPIC
CASE TITLE PEOPLE v. DULIN G.R.NO. 171284
PONENTE BERSAMIN, J. DATE: JULY 29, 2015
DOCTRINE Unless the victim committed unlawful aggression against the
accused, self-defense, whether complete or incomplete, should not
be appreciated, two other essential elements of self-defense would
have no factual and legal bases without any unlawful aggression to
prevent or repel.
FACTS There has been a long-standing grudge between Batulan and Dulin
and on two occasions Dulin uttered: He (Batulan) will soon have his
day and I will kill him.

On August 22, 1990, accused, Dulin, armed with a sharp bladed


instrument, attacked and stabbed Batulan, inflicting upon him
several stab wounds on the different parts of his body which caused
his death. Tamayao, one of the witnesses, saw Dulin stabbing
Batulan who was already prostrate face down in Danao’s house.
Dulin was holding Batulan by the hair with his left hand, and
thrusting the knife at the latter with his right hand.

In his defense, Dulin testified that Batulan initially stabbed and


attacked him. Dulin, ran to the upper level of Danao’s house,
pursued by Batulan who stabbed him again several times. They
then grappled for the weapon until Dulin was able to wrest it from
Batulan and which he then used to stab Batulan.
ISSUE/S Whether or not the Justifying Circumstance of self-defense is
present in the case
RULING/S No. Self-defense cannot be appreciated in the case.

Unlawful aggression on the part of the victim is the primordial


element of the justifying circumstance of self-defense. Without
unlawful aggression, there can be no justified killing in defense of
oneself. The CA observed that although Batulan had initiated the
attack against Dulin, the unlawful aggression from Batulan
effectively ceased once Dulin had wrested the weapon from the
latter.

Batulan, albeit the initial aggressor against Dulin, ceased to be the


aggressor as soon as Dulin had dispossessed him of the weapon.
Even if Batulan still went after Dulin despite the latter going inside
the house of Danao, where they again grappled for control of the
weapon, the grappling for the weapon did not amount to aggression

222
from Batulan for it was still Dulin who held control of the weapon at
that point.
Whatever Dulin did thereafter – like stabbing Batulan with the
weapon – constituted retaliation against Batulan. In this regard,
retaliation was not the same as self-defense. In retaliation, the
aggression that the victim started already ceased when the accused
attacked him, but in self-defense, the aggression was still continuing
when the accused injured the aggressor. As such, there was no
unlawful aggression on the part of Batulan to justify his fatal stabbing
by Dulin.

223
CONTRIBUTOR CARPIO, Anna Clarissa C.
MODULE Module 5: Justifying Circumstances and Absolutory Causes
TOPIC
CASE TITLE PEOPLE v. FONTANILLA G.R.NO. 177743
PONENTE Bersamin, J. DATE: January 25, 2012
DOCTRINE An indispensable requisite of self-defense is that the victim must be
mounted an unlawful aggression against the accused. Without such
unlawful aggression, the accused cannot invoke self-defense as a
justifying circumstance.
FACTS On October 29, 1996 at around 9:30 p.m., Jose Olais was walking
along the provincial road in Balaoan, La Union when accused-
appellant Alfonso Fontanilla suddenly attacked him with a piece of
wood (bellang) on the head and fell facedown on the ground.
Fontanilla hit him again and only desisted hitting a third time when
Olais’ sons-in-law, Jose and Tirso shouted at him and caused him
to run away. Olais was rushed to a medical clinic but was
pronounced dead on arrival. Thereafter, an information for murder
against Fontanilla was filed in the RTC.

At trial, the prosecution presented the sons-in-law as witnesses who


claimed that they were only several meters away from Olais when
Fontanilla struck him and they were able to see and identify him
because the area was then well-lighted. The autopsy report
revealed that the cause of Olais’ death is a fracture on the left
temporal area of the skull which was caused by a hard object or a
severe force that hit the skull more than once. SPO1 Valdez who
investigated the case and apprehended Fontanilla testified that
Fontanilla’s father denied that he was in the house but were able to
arrest him inside after the search. Valdez further declared that
Fontanilla asserted that he would only speak in court. Accused-
appellant Fontanilla on the other hand, invoked self-defense
claiming that he was standing on the road near his house when
deceased Olais, who was wielding a nightstick and appeared to be
drunk, boxed him in the stomach. Fontanilla tried to talked to Olais
nicely but the latter, being a karate expert, continued hitting him with
his fists, striking him with straight blows and also kicked him with
both his legs; so, he was forced to defend himself by picking up a
stone and hit the right side of the victim’s head which caused the
latter to fall face down to the ground. He then left the scene for his
house upon seeing that Olais was no longer moving. Fontanilla’s
statement was corroborated by his daughter, Marilou.

The RTC rejected Fontanilla’s plea of self-defense and found him


guilty of murder and sentenced him to suffer the penalty of reclusion
perpetua on the ground that he had no necessity to employ a big
stone, inflicting upon him a mortal wound which caused his death

224
as Olais only attacked him with the latter’s bare hands. Further,
Fontanilla did not suffer any injury, did not receive any treatment,
had no medical certificate to attest any injury he might have
suffered, despite his claim that he was mauled. The ruling was
affirmed by the Court of Appeals but given the fact that there was
absence of any aggravating or mitigating circumstance, the decision
was modified to reclusion perpetua.
ISSUE/S Whether or not Fontanilla properly invoked self-defense?
RULING/S NO. The Supreme Court affirmed the conviction. In order for self-
defense to be appreciated as a justifying circumstance, Fontanilla
had to prove by clear and convincing evidence the following
elements: (a) unlawful aggression on the part of the victim; (b)
reasonable necessity of the means employed to prevent or repel it;
and (c) lack of sufficient provocation on the part the person
defending himself. Fontanilla did not discharge his burden.

Unlawful aggression is the indispensable element of self-


defense, for if no unlawful aggression attributed to the victim
is established, self-defense is unavailing, for there is nothing
to repeal. Unlawful aggression on the part of the victim is the
primordial element of the justifying circumstance of self-defense,
and without unlawful aggression, there can be no justified killing in
defense of oneself. The test for the presence of unlawful
aggression is whether the aggression from the victim put in real
peril the life or personal safety of the person defending himself; and
that the peril must not be an imagined or imaginary threat. Thus, the
concurrence of the 3 elements of unlawful aggression: (a) there
must be a physical or material attack or assault; (b) the attack or
assault be actual, or, at least, imminent; and (c) the attack or assault
must be unlawful.

Further, there are 2 kinds of unlawful aggression: (a) actual or


material; and (b) imminent. Actual or material unlawful aggression
is an attack with physical force or with a weapon, an offensive act
that positively determines the intent of the aggressor to cause injury.
While imminent unlawful aggression is an attack that is impending
or at the point of happening. It must be offensive and positively
strong and not a mere threatening attitude of the victim or merely
imaginary.

Fontanilla, by invoking self-defense, admitted to have inflicted the


fatal injuries that caused Olais’ death, thus, once an accused in a
prosecution for murder or homicide admitted his infliction of the fatal
injuries on the deceased, he assumed the burden to prove by clear,
satisfactory and convincing evidence the justifying circumstance
that would avoid his criminal liability. The accused should discharge

225
the burden of proof by relying on the strength of his own evidence,
because the prosecution’s evidence, even if weak, would not be
disbelieved in view of his admission of the killing. Fontanilla did not
discharge his burden as the review of the records reveals that (1)
Olais did not commit unlawful aggression against Fontanilla; and
that (2) Fontanilla’s act of hitting the victim’s head with a stone,
which caused the mortal injury was not proportional to, and
constituted an unreasonable response to the victim’s fistic attack
and kicks. Had Olais really attacked Fontanilla, the latter would have
suffered injuries but there was none of any kind or gravity was found
when he presented himself in the hospital, nor was there any
medical certificate issued, or any medication applied. In contrast,
the cadaver of Olais showed that he has been hit on the head more
than once. Thus, the plea of self-defense was belied for the weapon
used, the location and number of wounds inflicted revealed
Fontanilla’s intent to kill, not merely an effort to repel or prevent an
attack. The gravity of the wounds manifested the determined effort
of the accused to kill his victim and not just to defend himself.

226
CONTRIBUTOR CHUA, Kristy Anne
MODULE JUSTIFYING CIRCUMSTANCES AND ABSOLUTORY CAUSES
TOPIC
CASE TITLE JOSUE V. PEOPLE G.R.NO. 199579
PONENTE REYES, J.: DATE: DEC. 10, 2012
DOCTRINE In invoking self-defense, appellant admitted committing the felonies
for which he was charged albeit under circumstances which, if
proven, would justify his commission of the crimes. Thus, the
burden of proof is shifted to appellant who must show, beyond
reasonable doubt, that the felony committed was attended by the
following circumstances: (1) unlawful aggression on the part of the
victims; (2) reasonable necessity of the means employed to prevent
or repel it; and (3) lack of sufficient provocation on the part of the
person defending himself.
FACTS On May 1, 2004, in the City of Manila, Philippines, Josue killed
Pineda several times with a cal. 45 pistol hitting him on different
parts of his body which should have resulted to Homicide, as a
consequence of Josue’s acts but did not produce it because of the
timely and able medical attendance rendered to Pineda.

On May 1, 2004 in the evening, Pineda, a barangay tanod, was


buying medicine from a store near Josue’s residence in Barrio
Obrero, Tondo, Manila when he saw Josue approaching him while
shouting why Pineda painted Josue’s vehicle. Pineda denied said
accusation but Josue still shot him which hit his elbow and fingers.
Pineda was unarmed and tried to flee but Josue kept firing shots,
causing Pineda to sustain gunshot wound at his back, Pineda was
rushed to the Chinese General Hospital for medical aid which Dr.
Tiongson confirmed the gunshot wounds and also stated that the
injuries were fatal, if not medically treated.

As per Josue’s statement, he, together with his son Rafael, was
watching a television program when a sound indicated that
someone was opening the hood of his jeepney. He went to his
jeepney armed with a .45 caliber pistol. Josue caught Pineda,
Matias and Richard, in the act of removing the locks of his vehicle’s
battery. Josue got their attention which Pineda pointed his .38
caliber gun at Josue and pulled its trigger, luckily the gun jammed
and failed to fire. Josue fired at Pineda, who was hit in the upper
arm. With every attempt of Pineda to fire at Josue with a jammed
gun, Josue fired at Pineda instead and kept his gun in his residence
to which Josue’s son testified in court to corroborate his father’s
testimony.
ISSUE/S Whether or not Ramon Josue y Gonzales is entitled to self-
defense.

227
RULING/S No, Josue is not entitled to self defense. Since Josue invoked self-
defense which means he is admitting that he committed the felony
charged against him. Thus, the burden of proof is shifted to
appellant who must justify, beyond reasonable doubt, that the three
essential requisites of self-defense were present. Pineda was
unarmed at the time of the shooting while Josue had a pistol.
According to witness Villanueva, Josue confronted the victim, who
was only buying medicine, even if Pineda tried stealing Josue’s car
battery, it did not equate to endangering his life. Pineda was even
said to have ran away from Josue but he kept chasing after him and
firing at his direction. With that, it is proven that there was no danger
to his life or personal safety, thus no unlawful aggression transpired
which could justify him shooting Pineda.

228
CONTRIBUTOR CRUZ, Jakielyn Anne O.
MODULE MODULE 5: Justifying Circumstances and Absolutory Causes
TOPIC
CASE TITLE TOLEDO v. PEOPLE G.R.NO. 158057
PONENTE CALLEJO, SR., J: DATE: SEP. 24, 2004
DOCTRINE The essential requisites of self-defense are: (1) unlawful aggression
on the part of the victim; (2) reasonable scrutiny of the means
employed to prevent or repel it; and (3) lack of sufficient provocation
on the part of the person defending himself.
Self-defense under Article 11, paragraph 1 of the RPC necessarily
implies a deliberate and positive overt act of the accused to prevent
or repel an unlawful aggression of another with the use of
reasonable means. The accused has freedom of action. He is
aware of the consequences of his deliberate acts. The defense is
based on necessity which is the supreme and irresistible master of
men of all human affairs, and of the law. From necessity, and limited
by it, proceeds the right of self-defense.
FACTS On September 16, 1995, the accused Noe Toledo went to a black
smith who made the design of his bolo. On his way home, he saw
the group of Lani Famero, Michael Fosana, Rex Cortez and Ricky
Guarte drinking gin at the house of the Spouses Manuel and Eliza
Guarte, Ricky's parents having a drinking spree. The accused
Toledo requested the group to refrain from making noise and went
inside his house to sleep. However, Toledo was awakened at
around 9:30pm by the loud noises coming from Ricky and his
companions.
Ricky, together with Lani, Rex and Michael, went to sleep at the
Guarte house. They had not laid down for long when they heard
stones being hurled at the roof of the house. The stoning was made
three (3) times. Ricky rose from bed and peeped through a window.
He saw Toledo stoning their house. Ricky went out of the house and
proceeded to Toledo's house. Ricky asked Toledo, his uncle, why
he was stoning their house. Toledo did not answer but met Ricky at
the doorstep of his house and, without any warning, stabbed Ricky
on the abdomen with a bolo. Ricky was being followed by his mom
and called for help when she saw his son was stabbed. Ricky was
brought to Romblon Provincial Hospital and died while being
operated.
The accused Toledo claimed that Ricky who was then already
inebriated, was incensed; he pulled out a balisong, pushed the door,
and threatened to stab Toledo. Toledo tried ran to the upper floor of
the house and got hold of his bolo, which accidentally hit Ricky on

229
the stomach, and the latter lost his balance and fell to the floor.
Toledo, thereafter, surrendered to the barangay captain the
following day.
ISSUE/S Whether or not Noe Toledo is guilty beyond reasonable doubt of
homicide based on the evidence on record?
RULING/S YES. It is an aberration for the petitioner to invoke the two defenses
at the same time because the said defenses are intrinsically
antithetical. There is no such defense as accidental self-defense in
the realm of criminal law.
The petitioner failed to prove that the victim was killed by accident,
without fault or intention on his part to cause it.
First. The petitioner failed to adduce real evidence that the door of
his house was destroyed and that he sustained any physical
injuries, considering that he was only five inches away from the
door.
Second. If the door fell to the sala of the house of the petitioner, the
victim must have fallen on top of the door. It is incredible that the
bolo of the petitioner could have hit the stomach of the victim.
Third. When he surrendered to the barangay captain and to the
policemen, he failed to relate to them that his bolo accidentally hit
the stomach of the victim.
Fourth. There is no evidence that the petitioner surrendered either
the bolo that accidentally hit the victim or the balisong held by the
deceased to the barangay captain or the police authorities. Such
failure of the petitioner negates his claim that his bolo accidentally
hit the stomach of the victim and that he acted in self-defense.
Fifth. To prove self-defense, the petitioner was burdened to prove
the essential elements thereof, namely: (1) unlawful aggression on
the part of the victim; (2) lack of sufficient provocation on the part of
the petitioner; (3) employment by him of reasonable means to
prevent or repel the aggression. Unlawful aggression is a condition
sine qua non for the justifying circumstances of self-defense,
whether complete or incomplete. Unlawful aggression presupposes
an actual, sudden, and unexpected attack, or imminent danger
thereof, and not merely a threatening or intimidating attitude.
Sixth. With the failure of the petitioner to prove self-defense, the
inescapable conclusion is that he is guilty of homicide as found by
the trial court and the CA. He cannot even invoke Article 12,
paragraph 4 of the Revised Penal Code.

230
CONTRIBUTOR DELA CRUZ, Ma. Luzelle P.
MODULE Justifying Circumstance Self-defense
TOPIC
CASE TITLE DELA CRUZ v. PEOPLE OF THE G.R.NO. 189405
PHILIPPINES
PONENTE PERALTA, J: DATE: NOV. 19,
2014
DOCTRINE To constitute aggression, the person attacked must be confronted
by a real threat on his life and limb; and the peril sought to be
avoided is imminent and actual, not merely imaginary.
FACTS According to the prosecution, the petitioner went to the office of
Sykes Asia Inc. located at the 25th Floor of Robinson’s Summit
Center, Ayala Avenue, Makati City. When petitioner was already
inside the building, he went to the workstation of the deceased
victim, Jeffrey Wernher L. Gonzales (Jeffrey), who, by the
configuration of the eyewitness Antonette Managbanag’s sketch,
was seated fronting his computer terminal, with his back towards
the aisle. As petitioner approached Jeffrey from the back, petitioner
was already holding a gun pointed at the back of Jeffrey’s head. At
the last second, Jeffrey managed to deflect the hand of petitioner
holding the gun, and a short struggle for the possession of the gun
ensued thereafter. Petitioner won the struggle and remained in
possession of the said gun. Petitioner then pointed the gun at
Jeffrey’s face, pulled the trigger four (4) times, the fourth shot finally
discharging the bullet that hit Jeffrey in the forehead, eventually
killing him. Finally, after shooting Jeffrey, petitioner fled the office.
According to the defense, petitioner, together with his children, went
to Sykes Asia, the workplace of his wife, Darlene Dela Cruz
(Darlene), located at the 25th Floor of Robinson’s Summit Building
in Makati City, to fetch the latter so that their family could spend time
and celebrate together the New Year’s Day. Before entering the
Robinson’s Summit Building, petitioner underwent the regular
security check-up/procedures. He was frisked by the guards-on-
duty manning the main entrance of said building and no firearm was
found in his possession. He registered his name at the security
logbook and surrendered a valid I.D.
Upon reaching the 25th Floor of the same building, Petitioner finding
that Darlene was then not on her table, approached a certain man
(deceased Jeffrey) and asked the latter as to the possible
whereabouts of Darlene. After casually introducing himself as the
husband of Darlene, Jeffrey curtly told him, "Bakit mo hinahanap si
Darlene?"to which he answered, "Nagpapasundo kasi sa akin."The
response given by Jeffrey shocked and appalled petitioner: "Ayaw
na nga ng asawa mo sayo sinusundo mo pa!"
Shocked by the words and reaction of Jeffrey, petitioner tried to
inquire from Jeffrey who he was. But Jeffrey suddenly cursed

231
petitioner. Then, Jeffrey suddenly picked up something in his chair
which happened to be a gun and pointed the same at petitioner’s
face followed by a clicking sound. The gun, however, did not fire.
Seeing imminent danger to his life, petitioner grappled with Jeffrey
for the possession of the gun. While grappling, the gunclicked for
two (2) to three (3) more times. Again, the gun did not fire.
Petitioner was able to wrest away the gun from Jeffrey and tried to
run away to avoid any further confrontation with the latter. However,
Jeffrey immediately blocked petitioner’s path and shouted, "Guard!
Guard!" Immediately then, Jeffrey took hold of a big fire
extinguisher, aimed and was about to smash the same on
petitioner’s head.
Acting instinctively, petitioner parried the attack while still holding
the gun. While in the act of parrying, the gun accidentally fired due
to the reasonable force and contact that his parrying hand had made
with the fire extinguisher and the single bullet discharged hit the
forehead of Jeffrey, which caused the latter to fall on the floor and
die.
Petitioner left the gun and went out ofthe premises of Sykes Asia
and proceeded towards the elevator. On his way to the elevator, he
heard Darlene shout, "Sherwin anong nangyari?", but he was not
able to answer.
ISSUE/S WON the petitioner can invoke justifying circumstance of self-
defense meritoriously.
RULING/S The essential requisites of self-defense are the following: (1)
unlawful aggression on the part of the victim; (2) reasonable
necessity of the means employed to prevent or repel such
aggression; and (3) lack of sufficient provocation on the part of the
person resorting to self-defense. In other words, there must have
been an unlawful and unprovoked attack that endangered the life of
the accused, who was then forced to inflict severe wounds upon the
assailant by employing reasonable means to resist the attack.
In the case at bar, other than petitioner’s testimony, the defense did
not adduce evidence to show that Jeffrey condescendingly
responded to petitioner’s questions or initiated the confrontation
before the shooting incident; that Jeffrey pulled a gun from his chair
and tried to shoot petitioner but failed — an assault which may have
caused petitioner to fear for his life.
Even assuming arguendo that the gun originated from Jeffrey and
an altercation transpired, and therefore, danger may have in fact
existed, the imminence of that danger had already ceased the
moment petitioner disarmed Jeffrey by wresting the gun from the
latter. After petitioner had successfully seized it, there was no longer
any unlawful aggression to speak of that would have necessitated
the need to kill Jeffrey. As aptly observed by the RTC, petitioner had

232
every opportunity to run away from the scene and seek help but
refused to do so.
Accused alleged that the victim was about to smash the fire
extinguisher on his (accused’s) headbut he parried it with his hand
holding the gun. This is doubtful as nothing in the records is or would
be corroborative of it. In contrast, the two (2) Prosecution witnesses
whose credibility was not impeached, both gave the impression that
the victim got the fire extinguisher to shield himself from the accused
who was then already in possession of the gun.
The victim was holding the fire extinguisher while the second was
holding the gun. The gun and the discharge thereof was
unnecessary and disproportionate to repel the alleged aggression
with the use of fire extinguisher. The rule is that the means
employed by the person invoking self-defense contemplates a
rational equivalence between the means of attack and the defense.
The Supreme Court has ruled that neither an imagined impending
attack nor an impending or threatening attitude is sufficient to
constitute unlawful aggression. It is a settled rule that to constitute
aggression, the person attacked must be confronted by a real threat
on his life and limb; and the peril sought to be avoided is imminent
and actual, not merely imaginary.
Petitioner guilty of homicide.

233
CONTRIBUTOR DIMAKUTA, Jasmine A.
MODULE JUSTIFYING CIRCUMSTANCE OF SELF-DEFENSE
TOPIC
CASE TITLE RODOLFO GUEVARRA and JOEY G.R. NO. 170462
GUEVARRA V. PEOPLE OF THE
PHILIPPINES
PONENTE BRION, J. DATE: February 5,
2014
DOCTRINE Unlawful aggression is an actual physical assault, or at least a threat
to inflict real imminent injury, upon a person. The element of
unlawful aggression must be proven first in order for self-defense to
be successfully pleaded. There can be no self-defense, whether
complete or incomplete, unless the victim had committed unlawful
aggression against the person who resorted to self-defense.

FACTS As its rebuttal witness, the prosecution presented the sole testimony
of Erwin who survived the hacking. Erwin narrated that, at around
10:00 to 11:00 p.m., on November 8, 2000, he, his brother David
and Philip went to a birthday party and passed in front of the
petitioners' compound. He was walking 20 meters ahead of his
companions when, suddenly, Philip ran up to him saying that David
was being stabbed by Joey with a bolo. While approaching the
scene of the stabbing, which was 3 meters away from where his
brother David was, Erwin was met by Rodolfo who then hacked him,
hitting his arm and back. Thereafter, Rodolfo and Joey dragged
Erwin inside the petitioners' compound and kept on hacking him. He
was hacked and stabbed 13 times. He became weak and ultimately
fell to the ground.

Erwin denied that he and David threw stones at the petitioners'


house and damaged Rodolfo's tricycle. They did not likewise
destroy the petitioners' gate, which was only damaged when his
brother David clung on to it while he was being pulled by Rodolfo
and Erwin into their compound. While they were being hacked and
stabbed by Rodolfo and Erwin, stones actually rained on them and
people outside the petitioners' gate were saying, "Do not kill the
brothers. Allow them to come out." After the incident, Erwin and
David, both unconscious, were brought to the hospital. David died
in the hospital while being treated for his wounds.

In a decision dated April 16, 2004, the RTC gave credence to the
prosecution's version of the incident and found the petitioners guilty
beyond reasonable doubt of the crimes of frustrated homicide and
homicide. It disbelieved the defense's version of the events due to
material inconsistencies in the testimonies of the defense

234
witnesses. It denied the petitioners' claim of self-defense for lack of
clear, convincing and satisfactory supporting evidence.

The RTC explained in its decision that "[w]hen an accused invokes


the justifying circumstance of self-defense, he loses the
constitutional presumption of innocence and assumes the burden of
proving, with clear and convincing evidence, the justification for his
act"; that self-defense is an affirmative allegation which must be
proven with certainty by sufficient, satisfactory and convincing
evidence that excludes any vestige of criminal aggression on the
part of the person invoking it. The RTC held that the petitioners
miserably failed to prove that there was unlawful aggression on the
part of the victims, Erwin and David.

On appeal, the CA affirmed the RTC's judgment and convicted the


petitioners of the crimes charged. As the RTC did, the CA found that
Erwin and David committed no unlawful aggression sufficient to
provoke the actions of the petitioners; that "aggression, to be
unlawful, must be actual and imminent, such that there is a real
threat of bodily harm to the person resorting to self-defense or to
others whom that person is seeking to defend." The CA further held
that the petitioners' plea of self-defense was belied by the nature
and number of wounds inflicted on Erwin, who sustained 13 stab
wounds on his arm and back, and David, who suffered around 10
stab wounds on his back and stomach causing his death. These
wounds logically indicated that the assault was no longer an act of
self-defense but a determined homicidal aggression on the part of
the petitioners.

ISSUE/S Whether or not the Honorable Court of Appeals erred in failing to


appreciate the presence of the justifying circumstance of self-
defense despite clear and convincing evidence showing the
elements of self-defense

RULING/S No. By invoking self-defense, the petitioners, in effect, admitted to


the commission of the acts for which they were charged, albeit under
circumstances that, if proven, would have exculpated them. With
this admission, the burden of proof shifted to the petitioners to show
that the killing and frustrated killing of David and Erwin, respectively,
were attended by the following circumstances: (1) unlawful
aggression on the part of the victims; (2) reasonable necessity of
the means employed to prevent or repel such aggression; and (3)
lack of sufficient provocation on the part of the persons resorting to
self-defense.

Of all the burdens the petitioners carried, the most important of all
is the element of unlawful aggression. Unlawful aggression is an

235
actual physical assault, or at least a threat to inflict real imminent
injury, upon a person. The element of unlawful aggression must be
proven first in order for self-defense to be successfully pleaded.
There can be no self-defense, whether complete or incomplete,
unless the victim had committed unlawful aggression against the
person who resorted to self-defense.

As the RTC and the CA did, we find the absence of the element of
unlawful aggression on the part of the victims. As the prosecution
fully established, Erwin and David were just passing by the
petitioners' compound on the night of November 8, 2000 when
David was suddenly attacked by Joey while Erwin was attacked by
Rodolfo. The attack actually took place outside, not inside, the
petitioners' compound, as evidenced by the way the petitioners' gate
was destroyed. The manner by which the wooden gate post was
broken coincided with Erwin's testimony that his brother David, who
was then clinging onto the gate, was dragged into the petitioners'
compound. These circumstances, coupled with the nature and
number of wounds sustained by the victims, clearly show that the
petitioners did not act in self-defense in killing David and wounding
Erwin. The petitioners were, in fact, the real aggressors.
WHEREFORE, the petition is DENIED.

236
CONTRIBUTOR FLORES, Precious Eureka D.
MODULE Justifying Circumstance and Absolutory Causes
TOPIC
CASE TITLE PEOPLE v. SEVILLANO G.R.NO. 200800
PONENTE PEREZ, J. DATE: FEBRUARY 9, 2015
DOCTRINE The findings of the trial court on the credibility of witnesses deserve
great weight, as the trial judge is in the best position to assess the
credibility of the witnesses, and has the unique opportunity to
observe the witness first hand and note his demeanor, conduct and
attitude under grueling examination.
FACTS Version of the prosecution:

On March 11, 2007 at around 3:00 pm, Jose Palavorin and


Carmelita Cardona together with the victim, Pablo Maddauin, were
seated on a long bench having their usual chitchat at the vacant lot.
While conversing, they saw Oscar Sevillano coming towards their
direction. Sevillano could not walk straight and appeared to be
drunk. Without warning, appellant pulled out a knife from his waist
and stabbed the victim on the chest. Jose and Carmelita tried to
restrain the appellant from attacking the victim, but Jose
experienced leg cramps and lost his hold on Sevillano. Appellant
turned again on the victim and continued to stab him several times
more, the victim asked him, “Bakit?”. Carmela shouted for help. The
victim’s wife came to the scene and embraced Sevillano as she
wrestled for the knife. The victim was brought to the hospital,
unfortunately, he died that same day.

Version of the Appellant:

Sevillano interposed self-defense to absolve himself from criminal


liability. He averred that on that fateful afternoon, he went to the
vacant lot where the victim and his friends usually hangout. The
victim, whom he described to have bloodshot eyes, walk towards
him and stepped on his injured foot. While he was on his knees
because of the pain, he saw the victim draw a knife. The latter
stabbed him but missed his target. Sevillano and the victim grappled
for the knife wherein the latter was accidentally stabbed. When he
saw the blood oozing out of the victim, he became apprehensive of
the victim’s relative to such extent that he fled the scene and hid to
a far as Bulacan where he was eventually apprehended.
ISSUE/S 1. Whether or not the court erred in finding the accused guilt for the
crime charged has been proven beyond reasonable doubt.

237
2. Whether or not the court erred in convicting the accused of
murder instead of homicide.

3. Whether or not the court erred in not finding that the accused
acted in self-defense.
RULING/S 1. The court finds that there no reason to disbelieve the testimonies
of the witnesses considering that their narration of facts were
straightforward and replete with details that coincide with the
medical examination conducted on the body of the victim. The
court is no persuaded by the appellant’s defense of denial as this
cannot prevail over the eyewitnesses’ positive identification of
him as the perpetrator of the crime. Denial, like alibi, if not
substantiated by clear and convincing evidence, is negative and
self-serving evidence undeserving of weight in law. Therefore,
Sevillano is guilty beyond reasonable doubt.

2. In the prosecution of the crime of murder as defined in Article


248 of the RPC, the following elements must be met:

1. That the person was killed;


2. That the accused killed the person;
3. That the killing was attended by treachery; and
4. That the killing is not infanticide or parricide.

The court found that in the case these elements were clearly met.
The prosecution witnesses positively identified the appellant as
the person who stabbed Pablo several times on the chest which
eventually caused the latter’s death.

Anent the presence of the element of treachery as a qualifying


circumstance, the prosecution was able to establish that the
attack on the unsuspecting victim, who was merely seated on a
bench and talking with his friends, was very sudden. In fact, the
victim was able to utter only "Bakit?". We note that the essence
of treachery is the sudden and unexpected attack on the
unsuspecting victim by the perpetrator of the crime, depriving the
former of any chance to defend himself or to repel the
aggression, thus insuring its commission without risk to the
aggressor and without any provocation on the part of the victim.
Therefore, accused is guilty of the crime of murder.

238
3. Under Article 11, paragraph 11 of the RPC, the following
elements must be present in order that the plea of self-defense may
be validly considered in absolving a person from criminal liability:
1. Unlawful aggression;
2. Reasonable necessity of the means employed to
prevent or repel aggression; and
3. Lack of sufficient provocation on the side of the person
defending himself.

Appellant’s version that it was the victim who was armed with a knife
and threatened to stab him was found by the lower court to be
untenable. Assuming arguendo that there was indeed unlawful
aggression on the part of the victim, the imminence of that danger
had already ceased the moment appellant was able to wrestle
the knife from him. Thus, there was no longer any unlawful
aggression to speak of that would justify the need for him to kill the
victim or the former aggressor. The Court has ruled that if an
accused still persists in attacking his adversary, he can no
longer invoke the justifying circumstance of self-defense. The
fact that the victim suffered many stab wounds in the body that
caused his demise, and the nature and location of the wound
also belies and negates the claim of self-defense. It
demonstrates a criminal mind resolved to end the life of the victim.
Therefore, self-defense cannot be invoke by the accused.

239
CONTRIBUTOR CARPIO, Anna Clarissa C.
MODULE Module 5: Justifying Circumstances and Absolutory Causes
TOPIC
CASE TITLE PEOPLE v. DAGANI G.R.NO. 153875
PONENTE Austria-Martinez, J. DATE: August 16, 2006
DOCTRINE When self-defense is invoked, the burden of evidence shifts to the
accused to show that the killing was legally justified. The accused
must discharge this burden by clear and convincing evidence and
when successful, an otherwise felonious deed would be excused,
mainly predicated on the lack of criminal intent of the accused. To
invoke self-defense successfully, there must be an unlawful and
unprovoked attacked that endangered the life of the accused, who
was then forced to inflict severe wounds upon the assailant by
employing reasonable means to resist the attack.
FACTS Accused-appellants Otello Santiano and Rolando Dagani were
found guilty of murder for the shooting of Ernesto Javier. The
prosecution maintains that in the afternoon of September 1989, a
group composed of deceased Javier and Lincoln Miran, and 2 other
individuals were drinking at the canteen inside the Philippine
National Railways (PNR) when all of a sudden, Santiano and
Dagani, PNR security officers entered the canteen and approached
them. Dagani shoved Miran, causing the latter to fall from his chair
and then held Javier while Santiano shot Javier twice at his left side,
which killed him.

Appellants however invoked the justifying circumstance of self-


defense and lawful performance of official duty, stating that that they
were ordered by their desk officer to investigate a commotion at the
canteen and upon reaching the place, Santiano ordered Dagani to
enter while he waited outside. Dagani approached Javier who had
been striking a bottle of beer on the table. Javier then pulled out a
.22 caliber revolver and attempted to fire at Dagani but the gun failed
to go off. Santiano who was outside heard the gunfire and from his
vantage point saw Javier and Dagani grapping for a gun which
belong to Javier. During the course of the struggle, the gun went off
which forced Santiano to fire a warning shot. Santiano heard
Javier’s gun fire again so he rushed into the canteen and shot Javier
from a distance of less than 4 meters. The appellants further claimed
that the prosecution failed to established treachery and conspiracy.

The RTC found the appellants guilty for the crime of murder with the
presence of mitigating circumstance of voluntary surrender. On
appeal, the CA modified the decision and sentenced them to
reclusion perpetua.
ISSUE/S 1. Whether or not the justifying circumstance of self-defense
should be appreciated?

240
2. Whether or not the justifying circumstance of lawful
performance of an official duty should be appreciated?
3. Whether or not there is conspiracy in the case at bar?
RULING/S NO, to all issues.
1. The appellants argue that there was unlawful aggression on
the part of Javier as he was armed with a revolver at the time
he was struggling with Dagani; and that given the fact that
Javier had been drinking, it is quite probable for him to act
harshly and aggressively towards the officers and could have
easily killed the Dagani. In fact, Javier fired 3 shots of his gun.
These arguments of the appellants are untenable. The
defense presented a bare claim of self-defense without any
proof of the existence of its requisites.

When self-defense is invoked, the burden of evidence shifts to the


accused to show that the killing was legally justified. The accused
must discharge this burden by clear and convincing evidence and
when successful, an otherwise felonious deed would be excused,
mainly predicated on the lack of criminal intent of the accused. All
requisites must concur fir self-defense to be appreciated: (1)
unlawful aggression by the person injured or killed by the offender;
(2) reasonable necessity of the means employed to prevent or repel
that unlawful aggression; and (3) lack of sufficient provocation on
the part of the person defending himself.

Unlawful aggression, a primordial element of self-defense, would


presuppose an actual, sudden and unexpected attack or imminent
danger on the life and limb of a person – not a mere threatening or
intimidating attitude – but most importantly, at the time the defensive
action was taken against the aggressor. To invoke self-defense
successfully, there must be an unlawful and unprovoked attacked
that endangered the life of the accused, who was then forced to
inflict severe wounds upon the assailant by employing reasonable
means to resist the attack.

In the case, the assertions of the appellants that it was quite


probable that Javier could have easily killed the appellants were
uncertain and speculative. There is aggression in contemplation of
the law only when the one attacked faces real and immediate threat
to one’s life, and the peril sought to be avoided must be imminent
and actual, not speculative. The defense was unable to prove
Javier’s unlawful aggression. There were no evidence that victim
actually fired his gun, no spent spells from the pistol were found and
no bullets were recovered from the scene. Javier also tested
negative for gunpowder residue. Further, Dagani’s account of the
incent was incredible and self-serving. Assuming Javier fired his

241
gun, the imminence of the danger to the appellants’ lives had
already ceased the moment Dagani held down the victim and
grappled for the gun. After the victim had been thrown off-balance,
there was no longer unlawful aggression that would have
necessitated the act of killing. When an unlawful aggression that has
begun no longer exists, the one who resorts to self-defense has no
right to kill or even wound the former aggressor.

Further, self-defense demands that the means employed to


neutralize the unlawful aggression are reasonable and necessary
that is, it does not imply material commensurability between the
means of attack and defense but “rational equivalence.” The nature
and the number of gunshot wounds sustained by Javier (2 fatal
wounds), the fact that Dagani was larger than Javier and has
finished Special Weapons and Tactics (SWAT) combat training, and
that Javier was intoxicated at the time of the incident, do not justify
Santiano’s act of fatally shooting the victim twice.

2. Both requisites for the defense of fulfillment of a duty are


absent in the case at bar to wit: (1) the accused must have acted in
the performance of a duty or in the lawful exercise of a right or office;
and (2) the injury caused or the offense committed should have
been the necessary consequence of such lawful exercise.

The defense failed to prove that the appellants were in fact on duty
at the time they were at the canteen. They were unable to submit
their daily time records (DTRs) to show that they were in duty. Their
assertion that they were ordered to go on 24-hour duty was belied
by PNR’s testimony that PNR’s security officers work in two 12-hour
shifts from 7:00 am to 7:00 p.m. and from 7:00 p.m. to 7:00 a.m.
Moreover, since it was not established that Javier fired his gun, the
injury inflicted upon him (gunshot wound) cannot be regarded as
necessary consequence of appellants’ due performance of an
official duty.

The right to kill and offender is not absolute and may be used only
as last resort, and under circumstances indicating that the offender
cannot otherwise be taken without bloodshed.

3. Conspiracy is not present in the case at bar. The facts show


that Javier was shot by Santiano as he was subdued by Dagani. The
lower courts erred in holding that the manner of attack was indicative
of a joint purpose and design by the appellants.

242
Courts must judge the guilt or innocence of the accused based on
facts and not on mere conjectures, presumptions, or suspicions.
Other than the plain fact that Javier was shot by Santino while being
held by Dagani, there is no other evidence that the appellants were
animated in the same purpose or were moved by a previous
common accord. Thus, their liabilities must be determined on an
individual basis. Conspiracy, while no formal agreement is
necessary as it may be inferred from the circumstances attending
the commission of the crime, must be established by clear and
convincing evidence. Even if all malefactors joined in the killing,
such circumstance alone does not satisfy the requisite of
conspiracy. Neither joint nor simultaneous action is per se sufficient
proof of conspiracy. Conspiracy must be shown to exist as clearly
and convincingly as the commission of the offense itself.

No evidence was presented to show that appellants planned to


killed Javier or that Dagani’s overt act facilitated the alleged plan.
The prosecution did not establish that Dagani’s act in trying to
wrestle the gun from Javier and holding the latter’s hands in the
process was for the purpose of enabling Santiano to shoot at Javier.
The fact that Dagani did not expect Santiano to shoot the victim was
established by the latter’s testimony that former seemed shocked
and was standing and looking at the victim as Javier gradually fell
on the ground. As the prosecution failed to prove conspiracy,
Dagani should be acquitted.

Santiano is guilty of homicide. Dagani is Acquited.

243
CONTRIBUTOR CHUA, Kristy Anne
MODULE JUSTIFYING CIRCUMSTANCES AND ABSOLUTORY CAUSES
TOPIC
CASE TITLE PEOPLE V. BERONILLA G.R.NO. L-4445
PONENTE REYES, J.B.L., J.: DATE: FEB. 28, 1955
DOCTRINE To constitute a crime, the act must, except in certain crimes made
such by statute, be accompanied by a criminal intent, or by such
negligence or indifference to duty or to consequence, as, in law, is
equivalent to criminal intent. The maxim is, actus non facit reum, nisi
mens sit rea crime is not committed if the minds of the person
performing the act complained of be innocent.
FACTS Beronilla, Paculdo, Velasco, and Adriatico filed an appeal due to the
Court of First Instance of Abra convicting them of murder for the
death of Borjal on April 18, 1945 in the evening in the town of La
Paz, Province of Abra. The elected mayor of La Paz, Abra was
Borjal, at the outbreak of war, and continuously served as a Mayor
during the Japanese occupation, until March 10, 1943, when he
relocated to Bangued due to an attempt made upon his life by an
unknown party. On December 18, 1944, Beronilla was assigned the
position of a Military Mayor of La Paz by regimental commander of
the 15 Infantry, Philippine Army, operating as a guerilla unit. After
th

said appointment, Beronilla received a copy of a memorandum


issued by Lt. Col. Arnold to all existing Military Mayor authorizing
them to appoint a jury composed of 12 bolomen to try persons
accused of treason, espionage or the aiding and abetting the
enemy. A list of puppet government officials of the province of Abra
was received and Military Mayors were tasked to investigate and
corroborate information regarding those people which included
Borjal.

In March, 1945, Borjal had to return with his family to La Paz in order
to escape the bombing of Bangued. Beronilla placed Borjal under
custody and asked the residents of La Paz to file complaints against
him, it did not take long for people to start charging Borjal. Atty.
Barreras voluntarily appeared and served as Borjal’s counsel. On
April 10, 1945, the jury found Borjal guilty on all accounts and
imposed upon him instructions from his superiors. On April 18,
1945, the records were returned from the 15 Infantry after reviewing
th

and that same day Beronilla ordered the execution of Borjal which
Adriatico acted as an executioner and Palope as grave digger.
Father Luding of the Roman Catholic Church performed last rites, in
return, Beronilla reported the matter to Col. Arnold.

After two years of said execution, numerous individuals including


the accused were indicted in the Court of First Instance of Abra for

244
the murder of Borjal. Later on, the late President Roxas granted
amnesty to all persons who committed acts penalized under the
Revised Penal Code in relation to the resistance to the enemy
against persons aiding in the war efforts of the enemy. Labuguen
applied and was granted amnesty by the Amnesty Commission. The
other defendants filed for amnesty but were denied their application
on the ground that the crime was purely personal in motive. Trial
proceeded by acquitting the jury and the grave digger on the ground
that they did not participate in the killing of Borjal, acquitted three
other people on the gound that there is insufficiency of evidence to
estabhlish their participation but convicted Beronilla, Paculdo,
Velasco and Adriatico for the crime of murder and sentencing them
to suffer imprisonment of from 17 years, 4 months and 1 day of
reclusion temporal to reclusion perpetua and to indemnify the heirs
of Borjal P4,000.
ISSUE/S Whether or not the accused are liable as conspirators and co-
principals of the crime of murder.
RULING/S No, according to the Supreme Court, there are ample records that
sustain the claim of the defense that the arrest, prosecution and trial
of Borja were done under the memorandum of the 15 Infantry th

Headquarters. The accused were simply acting upon the orders of


a superior officer as military subordinates, who should not and could
not question such commands causing them to obey in good faith
without being aware of the illegality of such acts without any fault or
negligence on their own parts. The trial and sentencing of Borja was
not seriously disputed by the prosecution if it was done according to
the instructions of superior military authorities. However, the state’s
evidence is presented by the existence of the radiogram from Col.
Volckmann, overall area commander, to Lt. Col. Arnold, which
focuses on the illegality of Borjal’s conviction and sentence. It was
also stated that there was no satisfactory proof found that Beronilla
received the radiogram of Col. Volckmann on the day of the
execution. Lt. Col. Arnold, for some reason that can not now be
ascertained, did not successfully transmit the Volckmann message
to Beronilla. With this, it caused the accused to be charged with a
criminal conspiracy to do away with Borjal which must be rejected
because there is no conspiracy when a man who was, to their
knowledge, duly sentenced to death. The latin maxim actus non facit
reum, nisi mens sit rea, the act must be accompanied by a criminal
intent. A crime cannot be committed if the minds of the accused
performing the said act complained of are innocent or without a
guilty mind.

245
CONTRIBUTOR CRUZ, Jakielyn Anne O.
MODULE MODULE 5: Justifying Circumstances and Absolutory Causes –
TOPIC Obedience to a lawful order of a superior (Art. 11 (6), RPC)
CASE TITLE TABUENA v. G.R.NO. 103501-03
SANDIGANGBAYAN
PONENTE FRANCISCO, J: DATE: FEB. 17,
1997
DOCTRINE To constitute a crime, the act must, except in certain crimes made
such by statute, be accompanied by a criminal intent, or by such
negligence or indifference to duty or to consequences as, in law, is
equivalent to criminal intent. The maxim is actus non facit reum, nisi
mens sit rea — a crime is not committed if the mind of the person
performing the act complained of is innocent.
For an act to be justified under Article 11(6) of the Revised Penal
Code three requisites must concur: (a) an order must have been
issued by a superior; (b) the order must be for a lawful purpose; and
(c) the means used by the subordinate in carrying out such order
must itself be lawful.
FACTS Luis Tabuena, the General Manager of Manila International Airport
Authority (MIAA) was instructed over the phone by President
Ferdinand Marcos to pay directly to the president’s office and in
cash what the MIAA owes the Philippine National Construction
Corporation (PNCC), to which Tabuena agreed to do so. On
January 8, 1986, Tabuena received a Presidential Memorandum
from private secretary of President Marcos reiterating in black and
white such verbal instruction, for the payment of Php 55,000,000.00
in cash as partial payment of MIAA to PNCC.
In obedience to President Marcos' verbal instruction and
memorandum, Tabuena, with the help of Gerardo Dabao and
Adolfo Peralta, caused the release of P55 Million of MIAA funds by
means of three (3) withdrawals.
The first withdrawal as made on January 10, 1986 for P25 Million,
following a letter of even date signed by Tabuena and Dabao
requesting the PNB extension office at the MIAA — the depository
branch of MIAA funds, to issue a manager's check for said amount
payable to Tabuena.The check was encashed, however, at the PNB
Villamor Branch. Dabao and the cashier of the PNB Villamor branch
counted the money after which Tabuena took delivery thereof. The
P25 Million in cash were then placed in peerless boxes and duffle
bags, loaded on a PNB armored car and delivered on the same day
to the office of Mrs. Gimenez located at Aguado Street fronting
Malacañang.

246
Similar circumstances surrounded the second withdrawal
/encashment and delivery of another P25 Million, made on January
16, 1986.
The third and last withdrawal was made on January 31, 1986 for P5
Million. Peralta was Tabuena's co-signatory to the letter- request for
a manager's check for this amount. Peralta accompanied Tabuena
to the PNB Villamor branch as Tabuena requested him to do the
counting of the P5 Million. After the counting, the money was placed
in two (2) peerless boxes which were loaded in the trunk of
Tabuena's car. Peralta did not go with
Tabuena to deliver the money to Mrs. Gimenez' o􀀾ce at Aguado
Street. It was only upon delivery of the P5 Million that Mrs. Gimenez
issued a receipt for all the amounts she received from Tabuena.
The disbursement of the P55 Million was, as described by Tabuena
and Peralta themselves, "out of the ordinary" and "not based on the
normal procedure". Not only were there no vouchers prepared to
support the disbursement, the P55 Million was paid in cold cash.
As a defense, Tabuena and Peralta claimed that they were acting
in good faith. Tabuena was merely complying with the President
Marcos Memorandum which ordered him to forward immediately to
the Office of the President P55 Million in cash as partial payment of
MIAA's obligations to PNCC, and that he was of the belief that MIAA
indeed had liabilities to PNCC. Peralta also shared the same belief
and he heeded to the request of his supervisor to help him release
the Php 5,000,000.00
ISSUE/S Whether or not the petitioners can raise the defense of good faith
and lawful order, given the fact that it was the President himself who
gave the order?
RULING/S YES. The accused may thus always introduce evidence to show he
acted in good faith and that he had no intention to convert. And this,
to our mind, Tabuena and Peralta had meritoriously shown.
Tabuena had no other choice but to make the withdrawals, for that
was what the MARCOS Memorandum required him to do. He could
not be faulted if he had to obey and strictly comply with the
presidential directive, and to argue otherwise is something easier
said than done. Marcos was undeniably Tabuena's superior — the
former being then the President of the Republic who unquestionably
exercised control over government agencies such as the MIAA and
PNCC. Tabuena therefore is entitled to the justifying circumstance
of "Any person who acts in obedience to an order issued by a
superior for some lawful purpose."
Thus, even if the order is illegal if it is patently legal and the
subordinate is not aware of its illegality, the subordinate is not liable,
for then there would only be a mistake of fact committed in good
faith.

247
There is no denying that the disbursement, which Tabuena admitted
as "out of the ordinary", did not comply with certain auditing rules
and regulations such as those pointed out by the Sandiganbayan.
He did not have the luxury of time to observe all auditing procedures
of disbursement considering the fact that the MARCOS
Memorandum enjoined his "immediate compliance" with the
directive that he forward to the President's Office the P55 Million in
cash. Be that as it may, Tabuena surely cannot escape
responsibility for such omission. But since he was acting in good
faith, his liability should only be administrative or civil in nature, and
not criminal.
Even assuming that the real and sole purpose behind the MARCOS
Memorandum was to siphon-out public money for the personal
benefit of those then in power, still, no criminal liability can be
imputed to Tabuena. There is no showing that Tabuena had
anything to do whatsoever with the execution of the MARCOS
Memorandum. Nor is there proof that he profited from the felonious
scheme. In short, no conspiracy was established between Tabuena
and the real embezzler/s of the P55 Million.
This Court has acknowledged the right of a trial judge to question
witnesses with a view to satisfying his mind upon any material point
which presents itself during the trial of a case over which he
presides. But not only should his examination be limited to asking
"clarificatory" questions, 45 the right should be sparingly and
judiciously used; for the rule is that the court should stay out of it as
much as possible, neither interfering nor intervening in the conduct
of the trial. Here, these limitations were not observed. Hardly in fact
can one avoid the impression that the Sandiganbayan had allied
itself with, or to be more precise, had taken the cudgels for the
prosecution in proving the case against Tabuena and Peralta when
the Justices cross-examined the witnesses, their cross-
examinations supplementing those made by Prosecutor Viernes
and far exceeding the latter's questions in length. The "cold
neutrality of an impartial judge" requirement of due process was
certainly denied Tabuena and Peralta when the court, with its
overzealousness, assumed the dual role of magistrate and
advocate.

248
MODULE 6
EXEMPTING CIRCUMSTANCES

249
CASE TITLE People v. Taneo G.R NO. G.R. No. L-37673
PONENTE AVANCEÑA, C. J. DATE: 31 March 1933
CONTRIBUTOR GARCIA, LEXANNE O.
DOCTRINE
Involuntary acts of the accused, such as during somnambulism,
exempts him from criminal liability.
FACT/S

Before the commission of the crime, the defendant had a quarrel


over a glass of "tuba" with Enrique Collantes and Valentin Abadilla,
who invited him to come down and fight, and when he was about
to go down, he was stopped by his wife and his mother.

When Potenciano Taneo went to sleep and while sleeping, he


suddenly got up, left the room bolo in hand and, upon meeting his
wife who tried to stop him, he wounded her in the abdomen,
Potenciano Taneo attacked Fred Tanner and Luis Malinao and
tried to attack his father after which he wounded himself.
Potenciano's wife who was then seven months pregnant, died five
days later as a result of her wound. and also the fetus which was
asphyxiated in the mother's womb.
MAIN ISSUE/S
Should the defendant be exempted from criminal liability on the
ground of insanity through somnambulism?
RULING ON
MAIN ISSUE/S The defendant acted while in a dream and his acts, with which he
is charged, were not voluntary in the –sense of entailing criminal
liability.

We are taking into consideration the fact that the apparent lack of
a motive for committing a criminal act does not necessarily mean
that there are none, but that simply they are not known to us.

Doctor Serafica stated that, considering the circumstances of the


case, the defendant acted while in a dream, under the influence of
an hallucination and not in his right mind.

We have thus far regarded the case upon the supposition that the
wound of the deceased was a direct result of the defendant's act
performed in order to inflict it. Nevertheless, the evidence does not
clearly show this to have been the case, but that it may have been
caused accidentally. Nobody saw how the wound was inflicted.
The defendant did not testify that he wounded his wife. He only
seemed to have heard her say that she was wounded.

250
Defendant is not criminally liable for the offense with which he is
charged, and it is ordered that he be confined in the Government
insane asylum, until the director thereof finds that his liberty would
no longer constitute a menace.

251
CONTRIBUTOR Manango, John De Divine B.
MODULE Module 6: Exempting circumstances (Art 12(1))
TOPIC
CASE TITLE People v. Bonoan G.R.NO. G.R. No. L-45130
PONENTE J. Laurel DATE: 17 February 1937
DOCTRINE Insanity (Dementia praecox)
FACTS Celestino Bonoan is charged with the crime of murder for stabbing
Carlos Guison with a knife, which caused his death three days
afterwards. An arraignment was then called, but the defense
objected on the ground that the defendant was mentally deranged
and was at the time confined at the Psychopatic Hospital. After
several months of summons for doctors, production of the
defendant’s complete record of mental condition from the hospital
and defendant’s admission to the hospital for personal observation,
assistant alienist Dr. Jose Fernandez finally reported to the court
that Bonoan may be discharged for being a “recovered case”. After
trial, the lower court found Bonoan guilty and sentenced him to life
imprisonment.

The defense now appeals, claiming the lower court made errors in
finding Bonoan suffered dementia only occasionally and
intermittently, did not show any kind of abnormality, that the defense
did not establish the defendant’s insanity and finding accused guilty.
ISSUE/S Whether or Not the lower court erred in finding the accused guilty
RULING/S Yes. The Court finds the accused demented at the time he
perpetrated the crime, which consequently exempts him from
criminal liability, and orders for his confinement in San Lazaro
Hospital or other hospital for the insane. This ruling was based on
the following evidence:
Uncontradicted evidence that accused was confined in the insane
department of San Lazaro Hospital and diagnosed with dementia
praecox long before the commission of the offense and recurrence
of ailments were not entirely lacking of scientific foundation
Persons with dementia praecox are disqualified from legal
responsibility because they have no control of their acts; dementia
praecox symptoms similar to manic depression psychosis
Accused had an insomnia attack, a symptom leading to dementia
praecox, four days prior to act according to Dr. Francisco
Accused was sent the Psychopatic hospital on the same day of
crime and arrest, indicating the police’s doubt of his mental
normalcy
Defendant suffered from manic depressive psychosis according to
Dr. Joson

252
CONTRIBUTOR MARCELINO, Ferilynn T.
MODULE Exempting Circumstances
TOPIC
CASE TITLE PEOPLE V. FORMIGONES G.R.NO. 3246
PONENTE MONTEMAYOR, J: DATE: 29 November 1950
DOCTRINE In order that a person could be regarded as an imbecile within the
meaning of article 12 of the Revised Penal Code so as to be exempt
from criminal liability, he must be deprived completely of reason of
discernment and freedom of the will at the time of committing the
crime.
FACTS This is an appeal from the decision of the Court of First Instance of
Camarines Sur finding the appellant guilty of parricide and
sentencing him to reclusion perpetua, to indemnify the heirs of the
deceased in the amount of Php2,000, and to pay the costs. The
following facts are not disputed.

In the month of November 1946, Abelardo Formigones (hereinafter


refered to as “defendant”) together with his wife, Julia Formigones
and his five children went to live in the house of the defendant’s
brother, Zacarias Formigones in Binahian, Sipocot, Camarines Sur.
They were employed in said barrio as palay harvesters.

After a month’s stay or on the late afternoon of 28 December 1946,


without any previous quarrel or provocation, defendant stabbed his
wife, Julia, at the back penetrating her right lung resulting to her
death thereafter. After such, defendant took the body of his wife into
the living room and laid her on the floor and lay down beside her.
Not long after, they were found in said position by the people who
responded from the cries of help of his eldest daughter, Irene
Formigones.

During the preliminary investigation conducted by the Justice of


peace of Sipocot, defendant admitted that he killed his wife due to
jealousy with the alleged illicit relation with Zacarias. However,
during the trial on the Court of First Instance, he pleaded not guilty
but did not testify. His counsel presented the testimonies of two
guards of the provincial jail where the defendant was confined.
According to them, he behaved like an insane person:

“That sometimes he would remove his clothes and go


stark naked in the presence of his fellow prisoners;
that at times he would remain silent and indifferent to
his surroundings; that he would refuse to take a bath
and wash his clothes until forced by the prison
authorities; and that sometimes he would sing in
chorus with his fellow prisoners, or even alone by

253
himself without being asked; and that once when the
door of his cell was opened, he suddenly darted from
inside into the prison compound apparently in an
attempt to regain his liberty.”||| (People v.
Formigones, G.R. No. L-3246, [November 29, 1950],
87 PHIL 658-665)

His counsel claimed that he was an imbecile therefore exempt from


criminal liability. However, Dr. Francisco Gomez, who examined the
defendant, told that he was only suffering from feeblemindedness
and not imbecility and that he could distinguish right or wrong.
ISSUE/S Whether or not the defendant who is suffering from
feeblemindedness is exempt from criminal liability.
RULING/S NO. In order that exempting circumstance be taken into
consideration, it is necessary that there be complete deprivation of
intelligence in committing the act – that the defendant be deprived
of reason; that there must be no responsibility for his own acts; that
he acts without discernment. In the case at bar, defendant, who was
only found to be feebleminded and not an imbecile, could still
distinguish what was right from wrong. Hence, with the facts that he
was able to raise five children, work for a living dutifully, and provide
for his family, defendant cannot be exempt from criminal liability.

Feeblemindedness of the accused warrants the finding in his favor


of the mitigating circumstance provided for in either paragraph 8 or
paragraph 9 of article 13 of the Revised Penal Code and the fact
that the accused evidently killed his wife in a fit of jealousy, he is,
likewise entitled to the mitigating circumstance in paragraph 6 of the
same article — that of having acted upon an impulse so powerful as
naturally to have produced passion or obfuscation.

The penalty applicable for parricide under article 246 of the Revised
Penal Code is composed only two indivisible penalties, to wit,
reclusion perpetua to death. Although the commission of the act is
attended by some mitigating circumstance without any aggravating
circumstance to offset them, article 63 of the said code is the one
applicable and must be applied.

When the court believes that the appellant is entitled to a lighter


penalty the case should be brought to the attention of the Chief
Executive who, in his discretion may reduce the penalty to that next
lower to reclusion perpetua to the death or otherwise apply
executive clemency in the manner he sees fit.

254
CONTRIBUTOR MAYUGA, Eunice Allaine G.
MODULE Article 12 of the RPC - Exempting Circumstances
TOPIC
CASE TITLE PEOPLE v. PUNO G.R.NO. L-33211
PONENTE AQUINO, J: DATE: 29 JUNE 1981
DOCTRINE Under Article 12 of the RPC, insanity means that the accused must
be deprived completely of reason or discernment and freedom of
the will at the time of the commission of the crime.

Insanity exists when there is complete deprivation of intelligence in


committing the act, that is, the accused is deprived of reason, he
acts without the least discernment because there is a complete
absence of the power to discern or complete deprivation of freedom
of the will. Mere abnormality of the mental faculties will not exclude
imputability.
FACTS Ernesto Puno, a jeepney driver, entered a bedroom in the house of
Aling Kikay. Seeing Aling Kikay sitting on the bed, Ernesto Puno
insulted her by saying "Mangkukulam ka, mambabarang ka,
mayroon kang bubuyog" and then slapped her and struck her
several times on the head with a hammer until she was dead.

The assault was witnessed by Hilaria Dela Cruz, who was in the
bedroom with Aling Kikay and Lina Pajes, a tenant. They both
testified that Ernesto's eyes were reddish. His look was baleful and
menacing. After the killing, Ernesto confessed that he had killed
Aling Kikay and threatened Hilaria and Lina not to say anything but
Lina, nevertheless, notified the police. Ernesto's father surrendered
him to the police and he was charged with murder. Five months after
the killing, Ernesto testified that he did not remember killing Aling
Kikay. He believed that there are mambabarang, mangkukulam and
mambubuyog and when one is victimized by those persons, his feet
might shrink or hands might swan.

The defense presented 3 psychiatrists but instead of testifying that


Ernesto was insane, the medical experts testified that Ernesto acted
with discernment. A doctor, whom Ernesto was referred to for
treatment said that he was already cured. Moreover, even though
Ernesto was suffering from schizophrenic reaction, his symptoms
were not socially incapacitating and that he could adjust to his
environment. Another doctor said that Ernesto was not mentally
deficient otherwise, he would not have reached 3rd year high
school.
ISSUE/S Whether or not Ernesto Puno was insane at the time the crime was
committed.

255
RULING/S No. The Supreme Court ruled that Ernesto was sane or knew that
the killing of Aling Kikay was wrong and that he would be punished
for it, as shown by the threats which he made to Hilaria de la Cruz
and Lina Pajes, the old woman's companions who witnessed his
dastardly deed. Moreover, if Puno was a homicidal maniac who had
gone berserk, he would have killed Hilaria and Lina also. The fact
that he singled out Aling Kikay signified that he really disposed of
her because he thought that she was a witch.
Insanity exists when there is complete deprivation of intelligence in
committing the act that is deprived of reason, acts without the least
discernment and total deprivation of freedom of will. Hence, the
court is led to the conclusion that Ernesto was not legally insane
when he killed the hapless and helpless victim. The facts and the
findings of the psychiatrists reveal that on that tragic occasion he
was not completely deprived of reason and freedom of will.

256
CONTRIBUTOR PALALA, Amer B.
MODULE Exempting Circumstances
TOPIC
CASE TITLE People v. Dungo G.R.NO. 89420
PONENTE PARAS, J. DATE: July 31, 1991
DOCTRINE In order that insanity may relieve a person from criminal
responsibility, it is necessary that there be a complete deprivation of
intelligence in committing the act, that is, that the accused be
deprived of cognition; that he acts without the least discernment;
that there be complete absence or deprivation of the freedom of the
will.
FACTS On March 16, 1987 between 2:00 and 3:00pm, the accused went to
Mrs. Sigua's office at the Department of Agrarian Reform, Apalit,
Pampanga. After a brief talk, the accused drew a knife from the
envelope he was carrying and stabbed Mrs. Sigua several times.
After which he departed from the office with blood stained clothes,
carrying a bloodied bladed weapon. The autopsy report revealed
that the victim sustained 14 wounds, 5 of which were fatal.

Rodolfo Sigua, husband of the deceased, testified that sometime in


February 1987, the accused Rosalino Dungo inquired from him why
his wife was requiring so many documents from him. Rodolfo
explained to him the procedure at the DAR.

The accused, in defense of himself, tried to show that he was insane


at the time of the commission of the offense:
• Two weeks prior to March 16, 1987, Rosalino's wife noticed
that he appears to be in deep thought always, maltreating
their children when he was not used to it before. There were
also times that her husband would inform her that his feet
and head were on fire when in truth they were not.
• On that fateful day, Rosalino complained of stomachache but
they didn't bother to buy medicine as the pain went away
immediately. Thereafter, he went back to the store. But when
Andrea followed him to the store, he was no longer there.
Worried, she looked for him. On her way home, she heard
people saying that a stabbing occurred. She saw her
husband in her parents-in-law's house with people milling
around. She asked her husband why he did the act, to which
Rosalino answered, "That's the only cure for my ailment. I
have cancer of the heart. If I don't kill the deceased in a
number of days, I would die.” That same day, the accused
went to Manila.

Dr. Santiago and Dr. Echavez of the National Center for Mental
Health testified that the accused was confined in the mental

257
hospital, as per order of the trial court dated Aug. 17, 1987. Based
on the reports of their staff, they concluded that Rosalino was
psychotic or insane long before, during and after the commission of
the alleged crime and classified his insanity as an organic mental
disorder secondary to cerebro-vascular accident or stroke. But Dr.
Balatbat who treated the accused for ailments secondary to stroke,
and Dr. Lim who testified that the accused suffered dorm occlusive
disease, concluded that Rosalino was somehow rehabilitated after
a series of medical treatment in their clinic.

ISSUE/S Whether or not the accused was insane during the commission of
the crime charged.
RULING/S No, the accused was not insane during the commission of the crime
charged. For insanity to relieve the person of criminal liability, it is
necessary that there be a complete deprivation of intelligence in
committing the act, that he acts w/o the least discernment and that
there be complete absence or deprivation of the freedom of the will.

Under Philippine jurisdiction, there's no definite test or criterion for


insanity. However, the definition of insanity under Sec 1039 of the
Revised Administrative Code can be applied. In essence, it states
that insanity is demonstrated by a deranged and perverted condition
of the mental faculties, which is manifested in language or conduct.
An insane person has no full and clear understanding of the nature
and consequence of his act.

Evidence of insanity must refer to the mental condition at the very


time of doing the act. However, it is also permissible to receive
evidence of his mental condition for a reasonable period before and
after the time of the act in question. The vagaries of the mind can
only be known by outward acts.

It is not usual for an insane person to confront a specified person


who may have wronged him. But in the case at hand, the accused
was able to Mrs. Sigua. From this, it can be inferred that the accused
was aware of his acts. This also established that the accused has
lucid intervals.

Moreover, Dr. Echavez testified to the effect that the appellant could
have been aware of the nature of his act at the time he committed it
when he shouted (during laboratory examination) that he killed Mrs.
Sigua. This statement makes it highly doubtful that the accused was
insane when he committed the act.

The fact that the accused was carrying an envelope where he hid
the fatal weapon, that he ran away from the scene of the incident

258
after he stabbed the victim several times, that he fled to Manila to
evade arrest, indicate that he was conscious and knew the
consequences of his acts in stabbing the victim.

259
CONTRIBUTOR PANGAN, Gabrielle L.
MODULE Art. 12, RPC: Schizophrenia as exempting circumstance
TOPIC
CASE TITLE PEOPLE VS. RAFANAN G.R.NO. 54135
PONENTE FELICIANO, J: DATE: NOV. 21, 1991
DOCTRINE People vs. Formigiones established two (2) distinguishable tests
(a) the test of cognition — "complete deprivation of intelligence in
committing the [criminal] act," and (b) the test of volition — "or that
there be a total deprivation of freedom of the will." It is complete
loss of intelligence which must be shown if the exempting
circumstance of insanity is to be found. The law presumes every
man to be sane. A person accused of a crime has the burden of
proving his affirmative allegation of insanity.
FACTS Estelita Ronaya who was then only fourteen years old was hired as
a househelper by the mother of the accused, Ines Rafanan alias
'Baket Ines' with a salary of P30.00 a month.

On March 16, 1976, in the evening, after dinner, Estelita Ronaya


was sent by the mother of the accused to help in their store which
was located in front of their house about six (6) meters away.
Attending to the store at the time was the accused. At 11:00 o'clock
in the evening, the accused called the complainant to help him close
the door of the store and as the latter complied and went near him,
he suddenly pulled the complainant inside the store and said,
'Come, let us have sexual intercourse,' to which Estelita replied, 'I
do not like,' and struggled to free herself and cried. The accused
held a bolo measuring 1-1/2 feet including the handle which he
pointed to the throat of the complainant threatening her with said
bolo should she resist. Then, he forced her to lie down on a bamboo
bed, removed her pants and after unfastening the zipper of his own
pants, went on top of the complainant and succeeded having carnal
knowledge of her in spite of her resistance and struggle. After the
sexual intercourse, the accused cautioned the complainant not to
report the matter to her mother or to anybody in the house,
otherwise he would kill her.

The principal submission of appellant is that he was suffering from


a metal aberration characterized as schizophrenia when he inflicted
his violent intentions upon Estelita. The trial court suspended the
trial and ordered appellant confined at the National Mental Hospital
in Mandaluyong for observation and treatment. After a third report
in 1977, the appellant was deemed capable to be tried.

Dr. Jovellano testified for the appellant’s alleged insanity during the
commission of the crime. His statement is; while a person suffers
from insanity, they may still be capable of knowing the difference

260
between what is right and what is wrong. They may be equipped
with the knowledge, but what is lost is the inhibition. He states that
the volition and drive are absent.
ISSUE/S Whether or not the accused may be exempted from criminal
liability due to insanity
RULING/S No. The accused shall not be exempt from criminal liability.
People vs. Formigiones established two (2) distinguishable tests
(a) the test of cognition — "complete deprivation of intelligence in
committing the [criminal] act," and (b) the test of volition — "or that
there be a total deprivation of freedom of the will." It is complete
loss of intelligence which must be shown if the exempting
circumstance of insanity is to be found. The law presumes every
man to be sane. A person accused of a crime has the burden of
proving his affirmative allegation of insanity.
Schizophrenia pleaded by appellant has been described as a
chronic mental disorder characterized by inability to distinguish
between fantasy and reality, and often accompanied by
hallucinations and delusions. Formerly called dementia praecox, it
is said to be the most common form of psychosis and usually
develops between the ages 15 and 30.
Here, appellant failed to present clear and convincing evidence
regarding his state of mind immediately before and during the
sexual assault on Estelita. It has been held that inquiry into the
mental state of the accused should relate to the period
immediately before or at the very moment the act is committed.
The fact that appellant Rafanan threatened complainant Estelita
with death should she reveal she had been sexually assaulted by
him, indicates, to the mind of the Court, that Rafanan was aware of
the reprehensible moral quality of that assault. The testimony of
Dr. Jovellano, in substance, negates complete destruction of
intelligence at the time of commission of the act charged which, in
the current state of our caselaw, is critical if the defense of insanity
is to be sustained.
In People vs. Puno, the Court ruled that schizophrenic reaction,
although not exempting 'because it does not completely deprive
the offender of the consciousness of his acts, may be considered
as a mitigating circumstance under Article 13(9) of the Revised
Penal Code, i.e., as an illness which diminishes the exercise of the
offender's will power without, however, depriving him of the
consciousness of his acts. Appellant should have been credited
with this mitigating circumstance,

261
CONTRIBUTOR PROVIDO, Gemy Hale A.
MODULE Exempting Circumstance, Insanity
TOPIC
CASE TITLE PEOPLE v. MADARANG G.R.NO. 132319
PONENTE PUNO, J: DATE: MAY 12, 2000
DOCTRINE Under Article 12 of the Revised Penal Code, for an accused to
invoke the defense of insanity, he must prove that he was deprived
of reason or acted without discernment at the time of the
commission of the crime.
FACTS On Septmber 3, 1993 at Poblacion, Infante, Pangasinan, Fernando
Madarang killed his wife, Lilia Madarang, using a bladed weapon.
Lilia Madarang suffered from 4 fatal stab wounds. He and his wife
had an argument because of his jealousy towards another guy
which led to the unfortunate event. He was charged with parricide,
however, he pleaded not guilty by using the defense of insanity. He
claims that:
1. He is not able to recall the moments of committing the crime
and was completely unaware of his acts
2. He is diagnosed having schizophrenia and claims that it may
have been present at the commission of the crime
3. That his mental illness may have long occurred before due to
the loss he suffered from his business ventures and his
inability to give financial support to his family
Furthermore, he argues that he and his wife never had an argument
before the day of her death, and claims that it is an effect of his
mental illness. The change in his behavior and aggressiveness is
because he is not of sound mind at the time of the crime.
ISSUE/S Whether or not Fernando Madarang can use insanity to be exempt
from criminal liability
RULING/S No, Fernando Madarang cannot use insanity to be exempt from
criminal liability as he was not deprived of reason when he
committed the crime. In the case at bar, Madarang was only
diagnosed of schizophrenia months after killing his wife. The
instances presented by the witnesses of Madarang was not
sufficient to establish that he was insane at that time. The remarks
made by his mother-in-law further strengthens the evidence that he
is not insane at the commission of the crime. His mother-in-law
claimed that when they were all living together, there was nothing
odd in his actions.
Dr. Wilson S. Tibayan claimed that he might be suffering from the
mental illness even before the commision of the crime. However,
the trial court is still convinced from the facts of the case that
Madarang does not suffer from insanity at the time of the
commission of the crime and from the number of tests to determine
whether sanity can be used as justifying circumstance. Since he

262
failed to prove from the evidence that he acted without discernment,
Madarang is charged with the crime of Parricide.

263
CONTRIBUTOR ROMERO, Ma. Camille Concepcion M
MODULE Exempting Circumstances: Insanity or Unsound Mind
TOPIC
CASE TITLE People v. Robios G.R.NO. 138453
PONENTE PANGANIBAN, J. DATE: May 29, 2002
DOCTRINE For insanity to be an exempting circumstance the burden of proof
shall be established by the defendant, in the absence thereof every
person is presumed to be sane during the commission of the crime.
The evidence must also refer that defendant’s unsound mind was
present prior or during the crime.

When the law prescribes penalties for two indivisible penalties and
there are neither mitigating nor aggravating circumstances, the
lower penalty shall be applied.
FACTS On in March 25 1995 at around seven o’clock in the morning fifteen
year old Robios was in his parents house at Barangay San Antonio
in Camiling, Tarlac. While he was cooking he heard the appellant
Melecio Robios and victim Lorenza quarrelling at the sala.

Lorenzo heard his mother tell appellant 'Why did you come home,
why don't you just leave?' After hearing what his mother said,
Lorenzo, about five meters away saw appellant, with a double-
bladed knife, stab the victim on the right shoulder, blood gushed and
the appellant fell down the floor. Lorenzo left their house and went
to his grandmother’s house.

At eight oclock in the morning of the same day Benjamin, brother of


the victim went to his mothers house to inform them that his uncle
Alejandro was killed by the appellant on the night of March 24 1995
but he received the news that his sister Lorenza had been killed by
the appellant. While Benjamin stayed at his mothers house, the
appellant shouted at him “It’s good you would see how your sister
died.”

Together Benjamin, the police, barangay officials and some


barangay folk went to the scene of the crime where they saw
Lorenza and her dripping blood. The police told appellant to come
out but failed they then detached the bamboo wall from the house.
SPO1 Lugo saw the appellant lying on the floor embracing the victim
while holding the double-headed knife on his right hand. He uttered
the words “I will kill myself” repeatedly. They went upstairs and took
appellant away from the victim’s dead body.

After the incident the Chielf of Police of Camiling, Tarlac prepared a


special report that the victim was The victim Lorenza Robiños was
six months pregnant. She suffered 41 stab wounds on the different

264
parts of her body. While the suspect was found drunk when he
quarreled with his wife, until the suspect got irked, drew a double
knife and delivered forty-one stab blows. Robios also stabbed his
own body and was brought to the hospital.

The appellant pleading exculpation, accused interposed insanity


ISSUE/S Whether or not the lower court erred in not giving probative weight
on appellants Insanity as an exempting circumstance to the crime.

Whether or not rthe lower court erred in imposing death penalty on


the appellant.
RULING/S The court found appellants appeal to be meritorious. Insanity
presupposes that the accused was deprived of discernment of
freedom of will at the time of the commission of the crime. The
defendant has the burden of establishing the fact of Insanity at the
time of the commission for the crime be considered as an exempting
circumstance. In the absence of proof every person is presumed to
be of sound mind. The testimonies of the witnesses shows no
substantial evidence that appellant was completely deprived of
discernment when he killed his wife. The evaluation testimonies
points to his alleged insanity after the crime.The court explained that
insanity must have existed at the time of the commission of the
offense, or the accused must have been deranged even prior
thereto. Otherwise he would still be criminally responsible. Verily,
his alleged insanity should have pertained to the period prior to or
at the precise moment when the criminal act was committed, not at
anytime thereafter.
The imposition of death penalty on the defendant was contrary to to
the rules of application of penalties as provide in the RPC. Since
appellant was convicted of the complex crime of parricide with
unintentional abortion, the penalty to be imposed on him should be
that for the graver offense which is parricide. This is in accordance
with the mandate of Article 48 of the Revised Penal Code, which...
states: "When a single act constitutes two or more grave or less
grave felonies, x x x, the penalty for the most serious crime shall be
imposed.
When the law prescribes penalties for two indivisible penalties and
there are neither mitigating nor aggravating circumstances, the
lower penalty shall be applied. The crime parricide is not a capital
crime and it is not punishable by death, therefore, the imposable
penalty is reclusion perpetua.

265
CONTRIBUTOR SAMONTE, Vanessa Antoinette
MODULE CIRCUMSTANCES WHICH EXEMPT FROM CRIMINAL
TOPIC LIABILITY- INSANITY
CASE TITLE PEOPLE v. ANACITO A.M. NO. G.R. NO. 147674-75
OPURAN
PONENTE DAVIDE, JR., C.J.: DATE: MARCH 17, 2004
DOCTRINE When insanity is used as a defense, the burden is on the defense
as the appellant has to prove that the perpetrator is insane
immediately before the commission of the crime or at the moment
of its execution. There should be proof that the accused acted
without discernment.

FACTS Accused appellant Opuran stabbed Allan Dacles who was lying on
a bench. Subsequently, while Demetrio Patrimonio was walking on
the national highway, the accused emerged from where he was
hiding and stabbed Patrimonio. Accused appellant was found guilty
of the crimes of homicide and murder respectively. Accused
appellant appealed contending that he was suffering from a
psychotic disorder and was, therefore, completely deprived of
intelligence when he stabbed the victims.

ISSUE/S Whether or not accused-appellant has clearly and convincingly


proven his defense of insanity to exempt him from criminal liability.

RULING/S NO. Insanity is one of the exempting circumstances enumerated in


Article 12 of the Revised Penal Code.
To be exempting from criminal responsibility, insanity is the
complete deprivation of intelligence in committing the criminal act.
Mere abnormality of the mental faculties does not exempt from
criminal responsibility. It must be stressed that an inquiry into the
mental state of an accused should relate to the period immediately
before or at the precise moment of the commission of the act which
is the subject of the inquiry. His mental condition after that crucial
period or during the trial is inconsequential for purposes of
determining his criminal liability.
A careful scrutiny of the records, however, indicates that Anacito
failed to prove by clear and convincing evidence the defense of
insanity. Tested against the stringent criterion for insanity to be
exempting, such deportment of Anacito, his occasional silence, and
his acts of laughing, talking to himself, staring sharply, and stabbing
his victims within a 15-minute interval are not sufficient proof that he
was insane immediately before or at the time he committed the
crimes. Such unusual behavior may be considered as mere
abnormality of the mental faculties, which will not exclude
imputability.

266
CONTRIBUTOR SAMSON, Jessa Viena D.
MODULE Exempting Circumstances (Art. 12, RPC)
TOPIC
CASE TITLE VERDADERO v. PEOPLE G.R.NO. 216021
PONENTE MENDOZA, J.: DATE: MAR. 02, 2016
DOCTRINE An imbecile or an insane person is exempt from criminal liability,
unless the latter had acted during a lucid interval. The defense of
insanity or imbecility must be clearly proved for there is a
presumption that the acts penalized by law are
voluntary. Moreover, insanity exists when there is a complete
deprivation of intelligence.
FACTS This petition for review on certiorari seeks to reverse and set aside
the decision and resolution of the Court of Appeals which affirmed
the judgment of the Regional Trial Court finding accused Solomon
Verdadero y Galera (Verdadero) guilty beyond reasonable doubt of
the crime of homicide, defined and penalized under Article 249 of
the Revised Penal Code (RPC).
Verdadero, armed with a Rambo knife, assaulted and stabbed
Romeo Plata thereby inflicting upon him stab wounds on the
different parts of his body which caused his death.
Before the said incident, Maynard Plata and his father Romeo were
at the Baggao Police Station. Together with Ronnie Elaydo, they
went there to report that Verdadero had stolen the fan belt of their
irrigation pump. After the confrontation with Verdadero at the police
station, the three men made their way home on a tricycle, but
stopped at a drugstore as Maynard intended to buy some baby
supplies. Romeo proceeded towards a store near the drugstore
while Ronnie stayed inside the tricycle. From the drug store,
Maynard saw Verdadero stabbing Romeo, after he was alerted by
the shouts of Ronnie. Maynard tried to help his father; however,
Verdadero attempted to attack him as well. He defended himself
using a small stool, which he used to hit Verdadero in the
chest. Meanwhile, Ronnie ran towards the police station to seek
assistance, and the responding police officers arrested
Verdadero. On the other hand, Romeo was rushed to the Medical
Center, but unfortunately, he was dead on arrival.
Prior to the incident, Verdadero had been an outpatient of CVMCs
Psychiatric Department. He was also diagnosed with schizophrenia
and was given medications to address his mental illness.
Relative to this, after Miriam, Verdadero’s sister, heard of the
stabbing incident, she proceeded to CVMC and saw his brother
removing the IV tubes connected to his body and, thereafter, locked
himself inside the comfort room. In another instance, Doctor
Andres-Juliana opined that Verdadero had suffered a relapse as
evidenced by his violent behaviour. Thus, the latter suffered a
relapse on the day of the stabbing incident.

267
In view thereof, the trial court opined that Verdadero failed to
establish insanity as an exempting circumstance. Moreover, it was
posited that Verdadero was unsuccessful in establishing that he was
not in a lucid interval when he stabbed Romeo or that he was
completely of unsound mind prior to or coetaneous with the
commission of the crime. The Court of Appeals (CA) affirmed the
trial court’s judgment.
ISSUE/S Whether or not accused’s defense of insanity will prosper
RULING/S Under Article 12 of the Revised Penal Code, an imbecile or an
insane person is exempt from criminal liability, unless the latter had
acted during a lucid interval. The defense of insanity or imbecility
must be clearly proved for there is a presumption that the acts
penalized by law are voluntary.
In the case at bar, there were instances where Verdadero was
brought to Psychiatric Department of CVMC for treatment;
diagnosed with depression and schizophrenia; confined in the
psychiatric ward due to a relapse; in and out of psychiatric care from
the time of his first confinement until the stabbing incident; and
diagnosed to have suffered a relapse. Thus, it is without question
that he was suffering from schizophrenia and the only thing left to
be ascertained is whether he should be absolved from responsibility
in killing Romeo because of his mental state.
Schizophrenia is a chronic mental disorder characterized by inability
to distinguish between fantasy and reality, and often accompanied
by hallucinations and delusions. A showing that an accused is
suffering from a mental disorder, however, does not automatically
exonerate him from the consequences of his act. Mere abnormality
of the mental faculties will not exclude imputability.
Relative to the aforementioned provision, it exists when there is a
complete deprivation of intelligence in committing the act, that is the
appellant is deprived of reason; he acts without the least
discernment because of complete absence of the power to discern;
or there is a total deprivation of freedom of the will.
In raising the defense of insanity, Verdadero admits to the
commission of the crime because such defense is in the nature of a
confession or avoidance. As such, he is duty bound to establish
with certainty that he was completely deprived, not merely
diminished, of intelligence at the time of the commission of the
crime. Failing such, Verdadero should be criminally punished for
impliedly admitting to have stabbed Romeo to death.
To prove insanity, clear and convincing circumstantial evidence
would suffice. Various doctors have testified on the mental
conditions of Verdadero. The Court finds that Verdadero sufficiently
proved that he was insane and deprived of intelligence at the time
of the commission of the offense. Thus, the Court takes a view
different from that of the CA.

268
However, in exonerating Verdadero on the ground of insanity, the
Court does not totally free him from the responsibilities and
consequences of his acts.
Wherefore, the Court grants the petition and acquits accused-
appellant Solomon Verdadero y Galera of homicide by reason of
insanity. He is ordered confined at the National Center for Mental
Health for treatment and shall be released only upon order of the
RTC acting on a recommendation from his attending physicians
from the institution. He is also ordered to pay the heirs of Romeo B.
Plata, accordingly.

269
CONTRIBUTOR TERTE, Karen A.
MODULE Justifying Circumstances; Battered Woman Syndrome
TOPIC
CASE TITLE PEOPLE v. GENOSA G.R.NO. 13598
PONENTE PANGANIBAN, J: DATE: JANUARY 15, 2004
DOCTRINE To establish all the elements of self-defense arising from the
battered woman syndrome, the following are needed: (a) each of
the phases of the cycle of violence must be proven to have
characterized at least two battering episodes between the
appellant and her intimate partner; (b) the final acute battering
episode preceding the killing of the batterer must have produced in
the battered persons mind an actual fear of an imminent harm from
her batterer and an honest belief that she needed to use force in
order to save her life; and (c) at the time of the killing, the batterer
must have posed probable – not necessarily immediate and actual
– grave harm to the accused, based on the history of violence
perpetrated by the former against the latter. Taken altogether,
these circumstances could satisfy the requisites of self-defense.
FACTS Marivic Genosa, the appellant, was the wife of the deceased Ben
Genosa. In their first year of marriage, Marivic and Ben lived happily
but soon thereafter, the couple would quarrel often and their fights
would become violent. Appellant testified that every time her
husband came home drunk, he would provoke her and sometimes
beat her. Whenever beaten by her husband, she consulted medical
doctors who also testified during the trial. She had tried to leave her
husband at least five times, but Ben would always follow her and
they would reconcile.

On the night of the killing, appellant, who was then eight months
pregnant, and the victim quarreled. The latter beat her, however,
she was able to run to another room. Appellant admitted having
killed the victim with the use of a gun. The information for parricide
against appellant, however, alleged that the cause of death of the
victim was by beating through the use of a lead pipe. Appellant
invoked self defense and defense of her unborn child. After trial, the
Regional Trial Court found appellant guilty beyond reasonable doubt
of the crime of parricide with an aggravating circumstance of
treachery and imposed the penalty of death.

Experts opined that Marivic fits the profile of a battered woman


syndrome and at the time she killed her husband, her mental
condition was that she was re-experiencing the trauma, together
with the imprint of all the abuses that she had experienced in the
past. Appellant filed an urgent omnibus motion praying that the
Honorable Court allow (1) the exhumation of Ben Genosa and the
re-examination of the cause of his death; (2) the examination of

270
Marivic Genosa by qualified psychologists and psychiatrists to
determine her state of mind at the time she killed her husband; and
finally, (3) the inclusion of the said experts’ reports in the records of
the case for purposes of the automatic review or, in the alternative,
a partial re-opening of the case a quo to take the testimony of said
psychologists and psychiatrists.

ISSUE/S Whether or not Marivic Genosa can validly invoke the Battered
Woman Syndrome as self-defense.
RULING/S No, Marivic failed to prove that she has the Battered Woman
syndrome hence, self-defense cannot be invoked.
Appellant failed to establish all the elements of self-defense arising
from the battered woman syndrome. A battered woman has been
defined as a woman who is repeatedly subjected to any forceful
physical or psychological behavior by a man in order to coerce her
to do his bidding without concern for her rights. Battered women
include wives or women in any form of intimate relationship with
men. Furthermore, in order to be classified as a battered woman,
the couple must go through the battering cycle at least twice. Any
woman may find herself in an abusive relationship with a man
once. If it occurs a second time, and she remains in the situation,
she is defined as a battered woman. The battered woman
syndrome is characterized by the so-called “cycle of violence,”
which has three phases: (1) the tension-building phase; (2) the
acute battering incident; and (3) the tranquil, loving (or, at least,
nonviolent) phase.
However, the existence of the syndrome in a relationship does not
in itself establish the legal right of the woman to kill her abusive
partner. Evidence must still be considered in the context of self-
defense. In the facts presented, the defense fell short of proving all
three phases of the “cycle of violence” supposedly characterizing
the relationship of Ben and Marivic Genosa. No doubt there were
acute battering incidents but appellant failed to prove that in at
least another battering episode in the past, she had gone through
a similar pattern. Neither did appellant proffer sufficient evidence in
regard to the third phase of the cycle. Additionally, there was a
sufficient time interval between the unlawful aggression of Ben and
her fatal attack upon him. She had already been able to withdraw
from his violent behavior and escape to their children’s bedroom.
During that time, he apparently ceased his attack and went to bed.
The reality or even the imminence of the danger he posed had
ended altogether. He was no longer in a position that presented an
actual threat on her life or safety.

271
CONTRIBUTOR TIDALGO, Aimee Diane A.
MODULE Exempting Circumstances- Minor who acted with discernment
TOPIC
CASE TITLE PEOPLE V. DOQUEÑA G.R.NO. 46539
PONENTE DIAZ, J, DATE: SEPT. 29, 1939
DOCTRINE The discernment that constitutes an exception to the exemption
from criminal liability of a minor under fifteen years of age but over
nine, who commits an act prohibited by law, is his mental capacity
to understand the difference between right and wrong
FACTS Between 1 and 2 o'clock in the afternoon of November 19, 1938, the
now deceased Juan Ragojos and one Epifanio Rarang were playing
volleyball in the yard of the intermediate school of the municipality
of Sual, Pangasinan. Accused Valentin Doqueña, who was also in
said yard, intervened and, catching the ball, tossed it at Ragojos,
hitting him on the stomach. For this act of the accused, Ragojos
chased him around the yard and, upon overtaking him, slapped him
on the nape. Doqueña then turned against Rarang assuming a
threatening attitude, for which reason Rarang struck him on the
mouth with his fist, returning immediately to the place where Rarang
was in order to continue playing with him. The accused, offended by
what he considered an abuse on the part of Ragojos, who was taller
and more robust than he, looked around the yard for a stone with
which to attack Ragojos, but finding none, he approached a cousin
Romualdo Cocal, to ask the latter to lend him his knife. Rarang, who
had heard what the accused had been asking his cousin, told the
latter not to give the accused his knife because he might attack
Ragojos with it. The accused, however, succeeded in taking
possession of the knife which was in a pocket of his cousin's pants.
Once in possession of the knife, Doqueña approached Ragojos and
challenged the latter to give him another blow with his fist, to which
Ragojos answered that he did not want to do so because Ragojos
was bigger than the accused. Ragojos, ignorant of the intentions of
Doqueña, continued playing and, while he was thus unprepared and
in the act of stopping the ball with his two hands, the accused
stabbed him in the chest with the knife which he carried.
ISSUE/S Whether or not a minor who committed a homicide acted with
discernment shall be held liable
RULING/S YES. Taking into account the fact that when, the accused Valentin
Doqueña, who was exactly thirteen years, nine months and five
days old at the date of the crime, was one of the brightest 7th grade
pupil in the intermediate school and was a captain of a company of
the cadet corps thereof, and during the time he was studying therein
he always obtained excellent marks, this court is convinced that the
accused, in committing the crime, acted with discernment and was
conscious of the nature and consequences of his act, and so also

272
has this court observed at the time said accused was testifying in
his behalf during the trial of this case.

The proven facts, as stated by the lower court in the appealed order,
convinces us that the appeal taken from said order is absolutely
unfounded, because it is error to determine discernment by the
means resorted to by the attorney for the defense, as discussed by
him in his brief. He claims that to determine whether or not a minor
acted with discernment, we must take into consideration not only
the facts and circumstances which gave rise to the act committed
by the minor, but also his state of mind at the time the crime was
committed, the time he might have had at his disposal for the
purpose of meditating on the consequences of his act, and the
degree of reasoning he could have had at that moment. It is clear
that the attorney for the defense mistakes the discernment referred
to in article 12, subsection 3, of the Revised Penal Code, for
premeditation, or at least for lack of intention which, as a mitigating
circumstance, is included among other mitigating circumstances in
article 13 of said Code.

273
CONTRIBUTOR Valdez, Ariane Faye V.
MODULE Module 6- Art. 12, RPC- Exempting Circumstances
TOPIC
CASE TITLE Ortega v. People G.R.NO. 151085
PONENTE NACHURA, J. DATE: 20 August 2008
DOCTRINE “Favorabilia suntamplianda adiosa restrigenda”- Penal laws
which are favorable to the accused are given retroactive effect.
FACTS On February 27, 1990, AAA was born to spouses FFF and MMM.
Among her siblings CCC, BBB, DDD, EEE and GGG, AAA is the
only girl in the family. Before these disturbing events, AAA’s family
members were close friends of petitioner’s family, aside from the
fact that they were good neighbors. However, BBB caught petitioner
raping his younger sister AAA inside their own home. BBB then
informed their mother MMM who in turn asked AAA. There, AAA
confessed that the petitioner raped her three (3) times on three (3)
separate occasions.

1. The first occasion happened sometime in August 1996. MMM


left her daughter AAA, then 6 years old and son BBB, then
10 years old, in the care of Luzviminda Ortega (Luzviminda),
mother of petitioner, for two (2) nights because MMM had to
stay in a hospital to attend to her other son who was sick.
2. The second occasion occurred the following day, again at the
petitioner’s residence. AAA confessed that petitioner inserted
his penis into her vagina and she felt pain.
3. The third and last occasion happened in the evening of
December 1, 1996 inside the own home of the victim. While
AAA’s siblings were busy watching, petitioner called AAA to
come to the room of CCC and BBB. AAA obeyed. While
inside the said room, petitioner pulled AAA behind the door,
removed his pants and brief, removed AAA’s shorts and
panty, and in a standing position inserted his penis into the
vagina of AAA.

Due to the confession made by AAA, MMM, together with


Luzviminda, brought the latter to Dr. Katalbas, the Rural Health
Officer of the locality who examined AAA and found no indication
that she was molested. Refusing to accept such findings, on
December 12, 1996, MMM went to Dr. Jocson, of the Bacolod City
Health Office. Dr. Jocson made an unofficial written report showing
that there were “abrasions on both right and left of the labia minora
and a small laceration at the posterior fourchette.” She also found
that the minor injuries she saw on AAA’s genitals were relatively
fresh; and that such abrasions were superficial and could disappear
after a period of 3 to 4 days.

274
Subsequently, an amicable settlement was reached between the
two families through the DAWN Foundation, an organization that
helps abused women and children. Part of the settlement required
petitioner to depart from their house and stayed with a certain priest
in the locality to avoid contact with AAA.

However, petitioner went home for brief visits. At the sight of


petitioner, AAA’s father FFF was infuriated and confrontations
occurred. At this instance, AAA’s parents went to the National
Bureau of Investigation (NBI) which assisted them in filing the crime
of rape.

On May 13, 1999, the RTC finds the accused guilty beyond
reasonable doubt as Principal by Direct Participation of the crime of
RAPE. On October 26, 2000, the CA affirmed in toto the ruling of
the RTC. Hence, this petition.
ISSUE/S 1. Whether or not petitioner is guilty beyond reasonable doubt
of the crime of rape as found by both the RTC and the CA.

2. Whether or not the pertinent provisions of R.A. No. 9344


apply to petitioner’s case, considering that at the time he committed
the alleged rape, he was merely13 years old.
RULING/S 1. Yes. Jurisprudence provides in most cases of rape
committed against young girls like AAA who was only 6 years
old then, total penetration of the victim’s organ is improbable
due to the small vaginal opening. Thus, it has been held that
actual penetration of the victim’s organ or rupture of the
hymen is not required.
Therefore, it is not necessary for conviction that the petitioner
succeeded in having full penetration, because the slightest
touching of the lips of the female organ or of the labia of the
pudendum constitutes rape.
2. Yes. Section 6 of R.A. No. 9344 clearly and explicitly
provides:
“SECTION 6. Minimum Age of Criminal
Responsibility. —A child fifteen (15) years of age or
under at the time of the commission of the offense
shall be exempt from criminal liability. However, the
child shall be subjected to an intervention program
pursuant to Section 20 of this Act.
A child above fifteen (15) years but below eighteen
(18) years of age shall likewise be exempt from
criminal liability and be subjected to an intervention

275
program, unless he/she has acted with discernment,
in which case, such child shall be subjected to the
appropriate proceedings in accordance with this Act.
The exemption from criminal liability herein
established does not include exemption from civil
liability, which shall be enforced in accordance with
existing laws.”
It bears stressing that the petitioner based from the facts
stated above, was only 13 years old at the time of the
commission of the alleged rape. This was duly proven by the
certificate of live birth, by petitioner’s own testimony, and by
the testimony of his mother. Indubitably, petitioner, at the
time of the commission of the crime, was below 15 years of
age. Under R.A. No. 9344, he is exempted from criminal
liability.
However, while the law exempts petitioner from criminal
liability for the two (2) counts of rape committed against AAA,
Section 6 thereof expressly provides that there is no
concomitant exemption from civil liability. This award is in the
nature of actual or compensatory damages, and is
mandatory upon a conviction for rape.
Likewise, Section 64 of the law categorically provides that
cases of children 15 years old and below, at the time of the
commission of the crime, shall immediately be dismissed and
the child shall be referred to the appropriate local social
welfare and development officer (LSWDO).
Therefore, the pertinent provision of R.A. No. 9344 was
applicable in the case of the petitioner. The latter is referred
to the local social welfare and development officer of the
locality for the appropriate intervention program. Moreover,
he is ordered to pay private complainant AAA, civil indemnity
in the amount of One Hundred Thousand Pesos
(P100,000.00) and moral damages in the amount of One
Hundred Thousand Pesos (P100,000.00).

276
CONTRIBUTOR VILLANUEVA, Sean Ruthie
MODULE Republic Act No. 9344 as amended by Republic Act No. 10630
TOPIC
CASE TITLE PEOPLE V. MANTALABA G.R.NO. 186227,
PONENTE PERALTA, J: DATE: July 20 2011
DOCTRINE If said child in conflict with the law has reached eighteen (18) years
of age while under suspended sentence, the court shall determine
whether to discharge the child in accordance with the Act, to order
execution of sentence, or to extend the suspended sentence for a
certain specified period or until the child reaches the maximum age
of twenty-one (21) years.
FACTS The Task Force Regional Anti-Crime Emergency Response
(RACER) received a report that a certain Allen Mantalaba, who was
seventeen (17) years old at the time, was selling shabu. Thus, a
buy-bust team was organized, composed of PO1 Randy Pajo, PO1
Eric Simon and two (2) poseur-buyers who were provided with two
(2) pieces of ₱100 marked bills to be used in the purchase.

The team, armed with the marked money, proceeded to conduct the
buy-bust operation. The operation was successful. The items
recovered were positively identified as illegal drugs
(methamphetamine hydrochloride).

Thereafter, two separate Informations were filed before the RTC of


Butuan City against appellant for violation of Sections 5 and 11 of
RA 9165, selling and possession of drugs respectively.

The Regional Trial Court found the accused Allen Mantalaba y


Udtojan guilty for selling shabu, a dangerous drug, penalizing him
with reclusion perpetua. For the illegal possession of drugs, he is
penalized with six (6) years and one (1) day, as minimum, to eight
(8) years, as maximum of prision mayor.
ISSUE/S Whether or not the accused is entitled to the retroactive application
of Republic Act No. 9344
RULING/S Yes, but such application is already moot and academic.
The appellant was seventeen (17) years old when the buy-bust
operation took place or when the said offense was committed, but
was no longer a minor at the time of the promulgation of the RTC's
Decision.
It must be noted that RA 9344 took effect on May 20, 2006, while
the RTC promulgated its decision on this case on September 14,
2005, when said appellant was no longer a minor. The RTC did not
suspend the sentence in accordance with Article 192 of P.D. 603,
The Child and Youth Welfare Cod and Section 32 of A.M. No. 02-1-
18-SC, the Rule on Juveniles in Conflict with the Law, the laws that

277
were applicable at the time of the promulgation of judgment,
because the imposable penalty for violation of Section 5 of RA 9165
is life imprisonment to death.
Generally, the RA 9344 provides that if said child in conflict with the
law has reached eighteen (18) years of age while under suspended
sentence, the court shall determine whether to discharge the child
in accordance with the Act, to order execution of sentence, or to
extend the suspended sentence for a certain specified period or until
the child reaches the maximum age of twenty-one (21) years.
Hence, the appellant, who is now beyond the age of twenty-one (21)
years can no longer avail of the provisions of Sections 38 and 40 of
RA 9344 as to his suspension of sentence, because such is already
moot and academic. It is highly noted that this would not have
happened if the CA, when this case was under its jurisdiction,
suspended the sentence of the appellant. The records show that the
appellant filed his notice of appeal at the age of 19 (2005), hence,
when RA 9344 became effective in 2006, appellant was 20 years
old, and the case having been elevated to the CA, the latter should
have suspended the sentence of the appellant because he was
already entitled to the provisions of Section 38 of the same law,
which now allows the suspension of sentence of minors regardless
of the penalty imposed as opposed to the provisions of Article 192
of P.D. 603. Nevertheless, the appellant shall be entitled to
appropriate disposition under Section 51 of RA No. 9344, which
provides for the confinement of convicted children.
Consequently, the privileged mitigating circumstance of minority
shall also be appreciated in fixing the penalty. The RTC imposed the
penalty of reclusion perpetua without considering the minority of the
appellant. Thus, the proper penalty should be one degree lower than
reclusion perpetua, which is reclusion temporal, the privileged
mitigating circumstance of minority having been appreciated.
Necessarily, also applying the Indeterminate Sentence Law
(ISLAW), the minimum penalty should be taken from the penalty
next lower in degree which is prision mayor and the maximum
penalty shall be taken from the medium period of reclusion
temporal, there being no other mitigating circumstance nor
aggravating circumstance. The ISLAW is applicable in the present
case because the penalty which has been originally an indivisible
penalty (reclusion perpetua to death), where ISLAW is inapplicable,
became a divisible penalty (reclusion temporal) by virtue of the
presence of the privileged mitigating circumstance of minority.
Therefore, a penalty of six (6) years and one (1) day of prision
mayor, as minimum, and fourteen (14) years, eight (8) months and
one (1) day of reclusion temporal, as maximum, would be the proper
imposable penalty.

278
CONTRIBUTOR AGUILAR, Jose Maria L.
MODULE EXEMPTING CIRCUMSTANCES
TOPIC
CASE TITLE PEOPLE v. RETUBADO G.R.NO. 124058
PONENTE CALLEJO, SR., J: DATE: DEC. 10, 2003
DOCTRINE It is indispensable that the state of necessity must not be brought
about by the intentional provocation of the party invoking the same.
FACTS At sometime, someone inserted a lighted firecracker in a cigarette
pack and gave it to Edwin Retubado, the appellant’s younger
brother. He brought the cigarette home. Momentarily, the firecracker
exploded. The suspect was the victim, but it turned out that the
victim was not the culprit.
Later on, the victim, drove his pedicab and stopped at the junction.
The victim ignored the appellant’s provocation during that time. The
appellant was incensed and ran after the victim. He overtook the
victim, grabbed and pushed the pedicab. The victim again ignored
the appellant and pedaled on until he reached his house.
Undeterred, the appellant continued following Emmanuel.
Shortly after the victim had entered his house, the appellant arrived
and tarried at the porch. The victim suddenly opened the door and
demanded to know why he was being followed. The appellant told
the victim that he just wanted to talk but the victim told the appellant
that his son was already asleep. The appellant forthwith pulled out
a handgun from under his T-shirt and shot the victim on the
forehead. The latter fell to the floor as the appellant walked away
from the scene.
The appellant admitted shooting the victim but claimed that he was
merely performing a lawful act with due care; hence, cannot be held
criminally liable for the victim’s death. He testified that the victim
went to his room and emerged therefrom holding a handgun with his
right hand. Emmanuel’s trigger finger was outside the trigger guard,
and he held the firearm with the muzzle facing downward. Fearing
that he would be shot, the appellant took hold of Emmanuel and
pulled the gun towards the victim’s stomach. The appellant pulled
the gun to the level of the victim’s forehead, and the gun suddenly
went off. The bullet hit the victims’s forehead.
ISSUE/S Whether or not the accused cannot be criminally liable for he was
merely performing a lawful act with due care.
RULING/S No. The defense of a state of necessity is a justifying circumstance
under Article 12, paragraph 4 of the Revised Penal Code. It is an
affirmative defense that must be proved by the accused with clear
and convincing evidence. By admitting causing the injuries and
killing the victim, the accused must rely on the strength of his own
evidence and not on the weakness of the evidence of the
prosecution because if such evidence is weak but the accused fails

279
to prove his defense, the evidence of the prosecution can no longer
be disbelieved. Whether the accused acted under a state of
necessity is a question of fact, which is addressed to the sound
discretion of the trial court. The legal aphorism is that the findings of
facts by the trial court, its calibration of the testimony of the
witnesses of the parties and of the probative weight thereof as well
as its conclusions based on its own findings are accorded by the
appellate court high respect, if not conclusive effect, unless the trial
court ignored, misconstrued or misapplied cogent facts and
circumstances of substance which, if considered, will change the
outcome of the case.

In the case at bar, when the wife of the victim heard her husband
and the appellant arguing with each other in the porch of their house,
she went down from the balcony towards her husband and placed
her hand on the latter’s shoulders. She was shocked when the
appellant pulled out his handgun and deliberately shot the victim on
the forehead. After shooting the victim, the appellant fled from the
situs criminis. He surrendered to the police authorities but failed to
surrender the gun he used to kill the victim and he never presented
the police officer to whom he confessed that he killed the victim in a
state of necessity. The appellant had the motive to shoot and kill the
victim. The victim ignored the appellant as the latter talked to him at
the junction but he was been attacked The appellant, exasperated
at the victim’s intransigence, pulled out a gun from under his shirt
and shot the victim on the forehead. It was impossible for the victim
to survive. Therefore, the requisites for the accused to invoke state
of necessity as a defense were not present in this case.

280
CONTRIBUTOR ALGURA, Nino N.
MODULE Justifying Circumstances and Absolutory Causes
TOPIC
CASE TITLE ROWENO POMOY G.R.NO. 150647
v.
PEOPLE OF THE PHILIPPINES
PONENTE PANGANIBAN, J.: DATE: Sept. 29, 2004
DOCTRINE Exemption from criminal liability proceeds from a finding that the
harm to the victim was not due to the fault or negligence of the
accused, but to circumstances that could not have been foreseen or
controlled. Thus, in determining whether an "accident" attended
17

the incident, courts must take into account the dual standards of lack
of intent to kill and absence of fault or negligence.
FACTS Tomas Balboa was a master teacher of the Concepcion College of
Science and Fisheries in Concepcion, Iloilo.
On January 4, 1990, about 7:30 in the morning, some policemen
arrived at the Concepcion College to arrest Balboa, allegedly in
connection with a robbery which took place in the municipality in
December 1989. With the arrest effected, Balboa and the policemen
passed by the Concepcion Elementary School where his wife,
Jessica, was in a get-together party with other School
Administrators. When his wife asked him, 'Why will you be
arrested?' [H]e answered '[Even I] do not know why I am arrested.
That is why I am even going there in order to find out the reason for
my arrest.
At that time, petitioner had a gun, a .45 caliber pistol, tucked in a
holster which was hanging by the side of his belt. The gun was fully
embedded in its holster, with only the handle of the gun protruding
from the holster.
petitioner was seen still holding a .45 caliber pistol, facing Balboa,
who was lying in a pool of blood,. about two (2) feet away. When
the Commanding Officer of the Headquarters arrived, he disarmed
petitioner and directed that Balboa be brought to the hospital.
He was one of the investigators of their outfit; about 2 o'clock or past
that time of January 4, 1990 he got Tomas Balboa... from their
stockade for tactical interrogation; as he was already holding the
door knob of their investigation room and about to open and enter
it, all of a sudden he saw Tomas Balboa approach him and take hold
or grab the handle of his gun; Tomas Balboa was a suspect in a...
robbery case who was apprehended by the police of Concepcion
and then turned over to them (PC) and placed in their stockade; he
asked the sergeant of the guard to let Balboa out of the stockade for
interrogation; from the stockade with Balboa walking with him, he
had his .45... caliber pistol placed in his holster attached to his belt
on his waist; then as he was holding the doorknob with his right hand
to open the door, the victim, who was two meters away from him,

281
suddenly approached him and grabbed his gun, but all of a sudden
he held the handle of... his gun with his left hand... his gun was
already loaded in its chamber and cocked when he left his house,
and it was locked when it fired; during the grappling he used his left
hand to prevent Balboa from holding his gun, while the victim used
his right hand in trying to reach the... gun; after the gun fired, they
were separated from each other and Balboa fell; he is taller than
Balboa though the latter was bigger in build; he cannot say nor
determine who of them was stronger.
ISSUE/S Whether or not the Court of Appeals committed serious and
reversible error in affirming petitioner's conviction despite the
insufficiency of the prosecution's evidence to convict the petitioner,
in contrast to petitioner's overwhelming evidence to support his
theory/defense of accident.

Whether or not the Court of Appeals committed grave and reversible


error in affirming the conviction of the petitioner on a manifestly
mistaken inference that when the gun fired, the petitioner was in full
control of the handle of the gun, because what the testimonies of
disinterested witnesses and the petitioner reveal was that the gun
fired while petitioner and Balboa were both holding the gun in
forceful efforts to wrest the gun from each other.

RULING/S Timeless is the legal adage that the factual findings of the trial court,
when affirmed by the appellate court, are conclusive. Both courts
16

possess time-honored expertise in the field of fact finding. But where


some facts are misinterpreted or some details overlooked, the
Supreme Court may overturn the erroneous conclusions drawn by
the courts a quo. Where, as in this case, the facts in dispute are
crucial to the question of innocence or guilt of the accused, a careful
factual reexamination is imperative.
Accident is an exempting circumstance under Article 12 of the
Revised Penal Code:
"Article 12. Circumstances which exempt from criminal liability. - The
following are exempt from criminal liability.
'4. Any person who, while performing a lawful act with due care,
causes an injury by mere accident without fault or intent of causing
it.'
Exemption from criminal liability proceeds from a finding that the
harm to the victim was not due to the fault or negligence of the
accused, but to circumstances that could not have been foreseen or
controlled. Thus, in determining whether an "accident" attended
17

the incident, courts must take into account the dual standards of lack

282
of intent to kill and absence of fault or negligence. This
determination inevitably brings to the fore the main question in the
present case: was petitioner in control of the .45 caliber pistol at the
very moment the shots were fired?
Petitioner advanced self-defense as an alternative. Granting
arguendo that he intentionally shot Balboa, he claims he did so to
protect his life and limb from real and immediate danger.
Self-defense is inconsistent with the exempting circumstance of
accident, in which there is no intent to kill. On the other hand, self-
defense necessarily contemplates a premeditated intent to kill in
order to defend oneself from imminent danger. Apparently, the fatal
shots in the instant case did not occur out of any conscious or
premeditated effort to overpower, maim or kill the victim for the
purpose of self-defense against any aggression; rather, they
appeared to be the spontaneous and accidental result of both
parties' attempts to possess the firearm.
Since the death of the victim was the result of an accidental firing of
the service gun of petitioner - - an exempting circumstance as
defined in Article 12 of the Revised Penal Code - - a further
discussion of whether the assailed acts of the latter constituted
lawful self-defense is unnecessary.

283
CONTRIBUTOR AROZA, Maria Minette R.
MODULE Exempting Circumstances – Irresistible Force
TOPIC
CASE TITLE US v. CABALLEROS G.R.NO. 1352
PONENTE MAPA, J: DATE: MAR. 29, 1905
DOCTRINE The Penal Code exempts from liability any person who performs
the act by reason of irresistible force.
FACTS The defendants have been sentenced by the Court of First Instance
to the penalty of seven years of presidio mayor as accessories to
the murder of four American school teachers for they took part in the
burial of the corpses of the victim in order to conceal the crime.
ISSUE/S Whether or not the defendant-appellants are criminally liable.
RULING/S The Supreme Court reversed the judgement of the CFI, acquitting
the defendant-appellants of the crime of murder. As the evidence
shows, Roberto Baculi, one of the defendant-apellants, confessed
to having assisted in the burial of the corpses, however he only did
so because he was compelled to bury them. Teodoro Sabate, the
prosecution witness stated that Baculi was not a member of the
group who killed the Americans, that he was in a banana plantation
on his property gathering some bananas when he heard the shots.
He began to run, however, Damaso and Isidoro, the leaders of the
band saw him, called to him and strike him with the butts of their
guns. The Penal Code exempts from liability any person who
performs the act by reason of irresistible force Baculi acted,
doubtless, under such circumstances when he executed the acts
which are charged against him. As to Caballeros, there is no proof
that he took any part in any way in the execution of the crime with
which he has been charged.

284
CONTRIBUTOR Belano, Renato Jr. P.
MODULE Exempting Circumstances
TOPIC
CASE TITLE People v. Loreno G.R. NO. L-54414
PONENTE Concepcion Jr., J. DATE: July 9, 1984
DOCTRINE A person who acts under the compulsion of an irresistible force, like
one who acts under the impulse of uncontrollable fear of equal or
greater injury is exempt from criminal liability because he does not
act with freedom. The force must be irresistible to reduce him to a
mere instrument who acts not only without will but against his will.
The duress, force, fear or intimidation must be present, imminent
and impending and of such a nature as to induce a well-grounded
apprehension of death or serious bodily harm if the act is not done.
FACTS In the evening of January 7, 1998, Barangay Captain Elias Monge
was at his house with his two daughters including the victim, Monica
Monge who was then single at 14 years of age, and were preparing
to attend a dance to be held in the barrio proper that evening.

During their preparation, Francisco Fabie, their farm helper, saw at


first two men with flashlights approaching. When they came near,
he heard one of them call Elias Monge saying that there was a letter
from the chief. Fabie called Elias Monge and later invited the two
men outside, one wearing red clothes and the other a dark sweater,
because it was dark.

On reading the letter, Elias and Monica read “kami mga NPA” which
caused Monica to run for her mother in fear because of what she
discovered about their visitors. The men poked guns at the family
members and ordered each one of them to stay in the sala where
Fabie recognized the man in red clothes to be Eustaquio Loreno.
Elias Monge and his two daughters also recognized the Estaquio
Loreno as he entered the sala.

They were then tied with their hands behind their backs as ordered
by the man in dark sweater to Loreno, after which he ordered Loreno
to silence the dog downstairs. Loreno held Fabie and brought him
downstairs to drive the barking dog away. Where upon reaching the
corner of the house below the flashlight used by Loreno happened
to focus on the person of Jimmy Marantal. Fabie immediately
recognized Jimmy Marantal as one of the visitors who remained on
the ground as lookouts.

After Loreno and Fabie returned to the sala the man in dark sweater
got hold of Monica Monge and dragged her up to a room located
above the balcony. The man in dark sweater then seized Monica

285
and forcibly removed her pants and panty. He then proceeded to
have sexual intercourse with her. While this was happening Loreno
brought Beata Monge to open the aparador and the trunk
respectively with her keys and he got the contents which he brought
to the sala. All the things he got from the two rooms were poured on
the floor of the sala.

After returning Monica Monge to the sala, not long thereafter, he


turned his attention to Cristina Monge and dragged her to the room
which was then rented by a schoolteacher Miss Olitoquit. Inside said
room, the man in the dark sweater forced his lewd designs on her
where he was successful in proceeding to satisfy his lust on her.

After the malefactors have left and being untied by Elias who was
able to untie himself, the family discovered they had been robbed of
items totaling P10,305.00 more or less.

The family, including Fabie had known some of the malefactors


because of the activities they do with them in the barrio.

After Elias’ report and the positive identification by the police,


Loreno and Marantal claimed they acted under the compulsion of
an irresistible force and/or under the impulse of an uncontrollable
fear of an equal or greater injury. They admitted being in the house
of Elias Monge that the night of January 7, 1978, but were only
forced by a man in a black sweater with five companions who
claimed to be members of the NPA.
ISSUE/S Whether or not bot the accused, Loreno and Marantal, can interpose
the defense of having acted under the compulsion of an
uncontrollable fear of an equal or greater injury.
RULING/S No. A person who acts under the compulsion of an irresistible force,
like one who acts under the impulse of uncontrollable fear of equal
or greater injury is exempt from criminal liability because he does
not act with freedom. The force must be irresistible to reduce him to
a mere instrument who acts not only without will but against his will.
The duress, force, fear or intimidation must be present,
imminent and impending and of such a nature as to induce a
well-grounded apprehension of death or serious bodily harm if
the act is not done. A threat of future injury is not enough. The
compulsion must be of such a character as to leave no opportunity
to the accused for escape or self-defense in equal combat.

A perusal of the appellants statement of the robbery-rape incident


as summarized in their joint brief, showed that they admitted their
participation in the commission of the crimes of robbery and rape

286
against Elias Monge and his family on January 7, 1978. Further
established were facts inconsistent with appellant s claim of having
acted under the compulsion of an irresistible force and/or under the
impulse of an uncontrollable fear of equal or greater injury, to wit:

1. Appellant Eustaquio Loreno was armed with a short firearm


when he and the man in dark sweater went up the house of
Elias Monge. While inside the house, Loreno pointed the gun
to the victims which enabled the malefactors to ransack the
house

2. When Eustaquio Loreno and the man in dark sweater


reached the balcony, Loreno positioned himself next to the post in
the balcony, while the man in dark sweater delivered the letter to
Elias Monge. Loreno admitted that, without prior instructions, he
immediately positioned himself near the post of the balcony, an act
which showed his voluntary participation in the criminal acts.
3. Eustaquio Loreno himself tied the victim with rattan and
thereafter, with ropes of the hammock. Loreno in fact admitted that
he was the one who furnished the rattan which he got from inside
the house

4. When Monica Monge was struggling and shouting for help


from inside the room where she was earlier dragged by the man in
dark sweater, Loreno's immediate reaction was to point his gun to
the victims who were then lying on the floor, telling them not to rise
if they wanted to live

All the facts presented demonstrate the voluntary participation and


the conspiracy of the appellants. The foregoing acts, though
separately performed from those of their unidentified companions,
clearly showed their community of interest and concert of criminal
design with their unidentified companions which constituted
conspiracy without the need of direct proof of the conspiracy itself.

287
CONTRIBUTOR BILTZ, Aralind Louise A.
MODULE Exempting Circumstances
TOPIC
CASE TITLE PEOPLE vs. DEL ROSARIO G.R.NO. 127755
PONENTE BELLOSILLO, J. DATE: APRIL 14, 1999
DOCTRINE A person who acts under the compulsion of an irresistible force,
like one who acts under the impulse of an uncontrollable fear of
equal or greater injury, is exempt from criminal liability because he
does not act with freedom. Actus me invito factus non est meus
actus. An act done by me against my will is not my act. The force
contemplated must be so formidable as to reduce the actor to a
mere instrument who acts not only without will but against his will.
The duress, force, fear or intimidation must be present, imminent
and impending, and of such nature as to induce a well-grounded
apprehension of death or serious bodily harm if the act be done. A
threat of future injury is not enough. The compulsion must be of
such a character as to leave no opportunity for the accused for
escape or self-defense in equal combat.
FACTS Accused-appellant Joselito was hired by Boy Santos, his co-
accused. Their original agreement was that he would drive him to
the cockpit at the Blas Edward Coliseum. However, despite their
agreement, Boy Santos directed him to proceed to the market place
to fetch Jun Marquez and Dodong Bisaya. Marquez and Bisaya
boarded and he was asked to stop at the corner of Burgos and
General Luna Streets. A certain Alonzo, also a tricycle driver, was
parked at a distance of about one and a half meter from Del
Rosario’s tricycle and saw the incident.

Bisaya accosted the victim Virginia Bernas and grappled with her
for the possession of her bag. Jun Marquez alighted from the tricycle
to help Dodong. Accused del Rosario tried to leave but Boy Santos
prevented him by threatening to shoot him.

Dodong Bisaya succeeded in takin the bag but before boarding the
tricycle, Jun Marquez shot the victim on the head while she was
lying on the ground. They then drove off and witness Alonzo chase
them to get the plate number of the tricycle. The three men alighted
and warned del Rosario not to inform to the police about the incident
otherwise his family would be harmed.
ISSUE/S Whether or not del Rosario is exempt from criminal liability as he
acted under the compulsion of an irresistible force
RULING/S YES.
The conviction of del Rosario must be set aside. His claim for
exemption from criminal liability under Art. 12, par. 5, Revised
Penal Code as he acted under the compulsion of an irresistible

288
force must be sustained. He was then unarmed and unable to
protect himself when he was prevented at gunpoint by his co-
accused from leaving the crime scene during the perpetration of
the robbery and killing, and was only forced to help them escape
after the commission of the crime.
As a rule, it is natural for people to be seized by fear when
threatened with weapons, even those less powerful than a gun,
such as knives and clubs. People will normally, usually and
probably do what an armed man asks them to do, nothing more,
nothing less. In the instant case, del Rosario was threatened with a
gun. He could not therefore be expected to flee nor risk his life to
help a stranger. A person under the same circumstances would be
more concerned with his personal welfare and security rather than
the safety of a person whom he only saw for the first time that day.

289
CONTRIBUTOR CACHERO, Luis III L.
MODULE Exempting Circumstances
TOPIC
CASE TITLE PEOPLE V JOSEFINA G.R.NO. 45186
BANDIAN
PONENTE DIAZ, J. DATE: September 30, 1936
DOCTRINE A woman who has mistaken a childbirth to a response to call of
nature, if by doing so she caused a wrong as that of giving birth to
her child in that same place and later abandoning it, not because of
imprudence or any other reason than that she was overcome by
strong dizziness and extreme debility, she should not be blamed
therefore because it all happened by mere accident, from liability
any person who so acts and behaves under such circumstances (Art
12, par. 4, Revised Penal Code).
FACTS At about 7 o’clock in the morning of January 31, 1936, Valentin
Aguilar, the appellant’s neighbor, saw the appellant go to the thicket
about four or 5 brazas from her house, apparently to respond to a
call of nature. A few minutes later, he again saw her emerge from
the thicket with her clothes stained with blood both in the front and
back, staggering and showing signs of not being able to support
herself. Valentin then came to her to assist her having noted that
she was very weak and dizzy. Valentin helped the accused go up to
her house and placed her in her own bed. Valentin then sought the
help of Adriano Comcom, another neighbor to help them. Comcom
had scarcely gone about five brazas when he saw the body of a
newborn baby near a path adjoining the thicket where the appellant
had gone a few moments before. Comcom informed Aguilar of it and
the latter told him to bring the body to the appellant’s house. Upon
being asked whether the baby which had just been shown to her
was hers or not, the appellant answered in the affirmative.

Accused-appellant Josefina Bandian was charged with the crime of


infanticide.

ISSUE/S Whether or not appellant is exempt from criminal liability given the
fact that she perfectly thought that her giving birth of a newborn child
was just a call of nature due to lack of childbirth experience.
RULING/S She is exempt from criminal liability. The act performed by the
appellant in the morning in question, by going to the thicket,
according to her, to respond to call of nature, notwithstanding the
fact that she had fever for a long time, was perfectly lawful. The
evidence certainly does not show that the appellant, in cause her
child’s death in one way or another, or in abandoning it in the thicket,
did so willfully, consciously or imprudently. The appellant enjoys the

290
fourth exempting circumstance of performance of lawful act with due
care, causes an injury by mere accident without fault or intention of
causing it, and the seventh exempting circumstance which is any
person who fails to perform an act required by law, when prevented
by some lawful insuperable cause.

291
CONTRIBUTOR CALZADO, Anne Valerie L.
MODULE Exempting Circumstances, Minority (R.A. 9344)
TOPIC
CASE TITLE MADALI v. PEOPLE G.R.NO. 180380
PONENTE CHICO-NAZARIO, J. DATE: AUGUST 4, 2009
DOCTRINE Republic Act No. 9344 “Juvenile Justice and Welfare Act of 2006” is
given retroactive effect in favor of the accused who is not a habitual
criminal.

An accused 15 of age or under at the time he committed the crime


is exempt from criminal liability and should be released to the
custody of his parents or guardian.

Discernment is that mental capacity of a minor to fully appreciate


the consequences of his unlawful act, which capacity may be known
and should be determined by taking into consideration all the facts
and circumstances afforded by the records in each case.
FACTS Raymond, 14 years old, Rodel 16 years old, along with Jojo
Bernardino, were charged with homicide for killing AAA.

Jovencio, cousin of the victim, who was at close range during the
incident, stand as witness during the trial and recounted the events
as follows: That on the night of April 13, 1999, Jovencio, the victim,
along with Rodel, Raymund and Bernardino went to a place near
Romblon National High School. Upon reaching the top of the stairs
leading to the reservoir, Bernardino blindfolded the victim with a
handkerchief and told the latter to “Join the rugby boys!”.

The victim responded, “That’s enough!” Bernardino then hit the


victim thrice, using a green and hard coconut frond. Unable to
withstand the beatings, the victim hit the ground and was lifted to his
feet by Bernardino, Raymund and Rodel. With the same coconut
frond, Raymund hit the victim on his right thigh. Rodel followed by
punching the body and the head of the victim with a brass knuckle
wrapped around the former’s right fist. Feeling for his cousin,
Jovencio shouted “Tama na! Tama na!” Bernardino responded,
“Yari na ini, ideretso na,” (We have come this far, we have to finish
it.) The victim’s strength was no match to the injuries he received.
He passed out. Raymund then tied a handkerchief around the
victim’s neck, fastened a dog chain to the ends of the said
handkerchief and, with the aid of Raymund and Rodel, hoisted the
victim’s body to and hanged it from a nearby tree.

The perpetrators warned Jovencio not to divulge to anyone what he


saw, or he would be the next victim. Then they all left the place,
leaving the victim’s body hanging from a tree.

292
ISSUE/S Whether or not the petitioners should be exempted from criminal
liability
RULING/S Yes. The petitioners should be exempted from criminal liability by
virtue of RA 9344.

Petitioners should be exempted from criminal liability due to the


provisions of RA 9344 which exempts an accused 15 of age or
under at the time of the commission of the crime from criminal
liability and should be released to the custody of his parents or
guardian. Although it was only passed on 2006, it can be applied to
petitioners since laws favorable to the accused can have retroactive
effect.

The exemptions however, differ. Raymond’s case was dismissed for


being only 14 years old at the time of the commission of the crime.
Rodel’s case was sustained since he was 16 at the time of the
commission.

RA 9344 states that, “A child above fifteen (15) years but below
eighteen (18) years of age shall likewise be exempt from criminal
liability and be subjected to an intervention program, unless he/she
has acted with discernment, in which case, such child shall be
subjected to the appropriate proceedings in accordance with this
Act.”

Rodel was found to have acted with discernment. Rodel, together


with his cohorts, warned Jovencio not to reveal their hideous act to
anyone; otherwise, they would kill him. Rodel knew, therefore, that
killing AAA was a condemnable act and should be kept in secrecy.
He fully appreciated the consequences of his unlawful act. And
since he acted with discernment, he shall be under an intervention
program. Section 38 of the act also provides for the automatic
suspension of his sentence.

293
CONTRIBUTOR CARPIO, Anna Clarissa C.
MODULE Module 6: Exempting Circumstances
TOPIC
CASE TITLE PEOPLE v. AGACER G.R.NO. 177751
PONENTE Del Castillo, J. DATE: January 7, 2013
DOCTRINE Regardless of the fact that the accused’s minority was not proved
during trial, and that his birth certificate was belatedly presented for
the court’s consideration, the rule will not adversely affect the rights
of the state, the victim and his heirs. The rationale of the law in
extending such leniency and compassion is that because of his age,
the accused is presumed to have acted with less discernment.
FACTS Accused-appellants Florencio, Eddie, Elynor, Franklin and Eric all
surnamed Agacer were convicted for the murder of Cesario Agacer
by RTC Aparri Cagayan and was affirmed by the Court of Appeals
and now seeks resolution from the Supreme Court through a Motion
for Reconsideration (MR). Appellants assert that their mere
presence at the crime scene is not evidence of conspiracy; that
there was no treachery since a heated argument preceded the
killing of the victim; and that even assuming that their guilt was duly
established, the privileged mitigating circumstance of minority
should have been appreciated in favor of appellant Franklin who
was only 16 years and 106 days old at the time of the incident (April
2, 1998), having been born on December 21, 1981.

The Office of the Solicitor General (OSG) was required to comment


on the MR, particularly on the issue of Franklin’s minority. The OSG
stated that there exists no cogent reason to disturb the findings of
the court as the facts and evidence clearly established conspiracy
and treachery. However, the OSG agreed that minority should be
appreciated as privileged mitigating circumstance in favor of
Franklin as it was supported by the latter’s NSO Certificate of Live
Birth.

Meanwhile, appellant Florencio died of cardio pulmonary arrest


while awaiting the resolution.
ISSUE/S 1. Whether or not the privileged mitigating circumstance of
minority should be appreciated in the case at bar?
2. What is the effect of Florencio’s death on his criminal
liability?
RULING/S 1. YES, Franklin’s Certificate of Live Birth shows that he was
born on December 20, 1981, thus, he was merely 16 years
old at the time of the commission of the crime on April 2,
1998; therefore, is entitled to the privileged mitigating
circumstance of minority.

294
As embodied in Article 68(2) of the RPC, when a minor is over 15
and under 18 years, the penalty next lower than that prescribed by
law shall be imposed on the accused but always in the proper
period. Regardless of the fact that the accused’s minority was not
proved during trial, and that his birth certificate was belatedly
presented for the court’s consideration, the rule will not adversely
affect the rights of the state, the victim and his heirs. The rationale
of the law in extending such leniency and compassion is that
because of his age, the accused is presumed to have acted with
less discernment.

As to the effect of the privileged mitigating circumstance of minority,


Franklin’s penalty must be modified. The penalty for murder is
reclusion perpetua to death and a degree lower is reclusion
temporal. As there is no aggravating and ordinary mitigating
circumstance, the penalty imposed on Franklin should be reclusion
temporal in its medium period as maximum (ranges from 14 years,
8 months and 1 day to 17 years and 4 months). Applying the
Indeterminate Sentence Law, the penalty next lower in degree is
prision mayor in its medium period (8 years and 1 day to 10 years).
Due to the seriousness of the crime and the manner it was
committed, the penalty must be imposed at its most severe range.

2. The death of appellant Florencio prior to the court’s final


judgment extinguishes his criminal liability and civil liability ex
delicto.

Pursuant to Article 89(1) of the RPC which provides that criminal


liability is totally extinguished, “by the death of the convict, as to the
personal penalties; and as to pecuniary penalties, liability therefor is
extinguished only when the death of the offender occurs before final
judgment.” Thus, it is settled that upon the death of the accused
pending appeal of his conviction, the criminal action is extinguished
inasmuch as there is no longer a defendant to stand as accused;
and the civil action instituted for recovery of civil liability ex delicto is
ipso facto extinguished, grounded as it is on the criminal.

As the court was made aware of the demise of Florencio only after
the promulgation of the December 14, 2011 decision despite his
death way back February 7, 2007, the 2011 judgment for his
conviction was set aside insofar as Florencio is concerned and his
criminal and civil ex delicto liabilities were extinguished by his death
prior to final judgment.

295
CONTRIBUTOR CHUA, Kristy Anne
MODULE EXEMPTING CIRCUMSTANCES
TOPIC
CASE TITLE TY V. PEOPLE G.R.NO. 149275
PONENTE TINGA, J.: DATE: SEP. 27, 2004
DOCTRINE Defense of uncontrollable fear is an exempting circumstance and in
order to invoke it successfully, the following requisites must concur:
(1) existence of an uncontrollable fear; (2) the fear must be real and
imminent; and (3) the fear of an injury is greater than or at least
equal to that committed.
FACTS A petition for review was filed by Petitioner Vicky V. Ty regarding
the decision of the appellate court affirming the decision of the lower
court finding her guilty of seven counts of violation of B.P. 22, in
other words the Bouncing Checks Law. On May 30, 1993, Ty
knowing that at the time of issue she did not have sufficient funds in
or credit with Metrobank issued checks for payment upon
presentment, which check when presented for payment within 90
days from the date, it was subsequently dishonored by the drawee
bank for “Account Closed” and even if they received a notice of such
dishonor, Ty still failed to pay Manila Doctors Hospital the amount
of the check or pay the full amount of the same within five days.

From October 30, 1990 until June 4, 1992, Ty’s mother Chua Lao
So Un was confined at the Manila Doctor’s Hospital which Ty had to
sign an “Acknowledgment of Responsibility for Payment'', a
Contract of Admission on October 30, 1990. On July 4, 1992, the
Statement of Account shows the total liability of the confined mother
amounting to P657,182.40 also as for Ty’s sister, Judy’s
confinement also amounted to a liability of P418,410.55.” On June
2, 1992, Ty promised through a promissory note that she will
assume the payment of obligation in installments. In order to provide
assurance on the payment, she issued several postdated checks
against Metrobank payable to the hospital with each check covering
P30,000 which were all deposited on the due dates but were
dishonored and returned due to insufficiency of funds. Demand
letters were sent by the hospital to Ty.

Ty claimed that she issued the postdated checks because of


“uncontrollable fear of a greater injury.” She claimed that she issued
the checks to obtain release of her mom whom the hospital
inhumanely and harshly treated but would not discharge unless
obligations were paid. She alleged that her mother was deprived of
room facilities.She pointed out that the “debasing treatment”
affected her mother’s mental, psychological and physical health that
the mother contemplated committing suicide if not discharged from
the hospital, thus compelling Ty to sign a promissory note, open an

296
account with Metrobank and issue checks for her mother’s
discharge.
ISSUE/S Whether or not the defense of uncontrollable fear is a justifiable
claim to exempt Ty from criminal liability.
RULING/S No, as stated by the Supreme Court, in order to invoke the defense
of uncontrollable fear successfully, this three requisites must occur:
(1) existence of an uncontrollable fear; (2) the fear must be real and
imminent; and (3) the fear of an injury is greater than or at least
equal to that committed. It must seem that the threat could cause
uncontrollable fear with gravity and imminence that an individual
would have succumbed to it. It should be real, imminent or
reasonable fear for one’s life or limb, a mere future injury as a threat
is not enough, the threat should not be speculative, fanciful or
remote. It should appear that the individual is not only acting without
will but against his will also and it must leave no opportunity to the
accused to escape. Speculative fear as seen in the case at bar is
different from uncontrollable fear presented by the Revised Penal
Code.

297
CONTRIBUTOR CRUZ, Jakielyn Anne O.
MODULE MODULE 6: Exempting Circumstances
TOPIC
CASE TITLE PEOPLE v. ARPON G.R.NO. 183563
PONENTE LEONARDO-DE CASTRO, J: DATE: DEC. 14, 2011
DOCTRINE Guidelines in appreciating age, either as an element of the crime or
as a qualifying circumstance, as follows:
1. The best evidence to prove the age of the offended party is
an original or certified true copy of the certificate of live birth
of such party.
2. In the absence of a certificate of live birth, similar authentic
documents such as baptismal certificate and school records
which show the date of birth of the victim would suffice to
prove age.
3. If the certificate of live birth or authentic document is shown
to have been lost or destroyed or otherwise unavailable, the
testimony, if clear and credible, of the victim's mother or a
member of the family either by affinity or consanguinity who
is qualified to testify on matters respecting pedigree such as
the exact age or date of birth of the offended party pursuant
to Section 40, Rule 130 of the Rules on Evidence shall be
sufficient under the following circumstances:
1. If the victim is alleged to be below 3 years of age and
what is sought to be proved is that she is less than 7
years old;
2. If the victim is alleged to be below 7 years of age and
what is sought to be proved is that she is less than 12
years old;
3. If the victim is alleged to be below 12 years of age and
what is sought to be proved is that she is less than 18
years old.
4. In the absence of a certificate of live birth, authentic
document, or the testimony of the victim's mother or relatives
concerning the victim's age, the complainant's testimony will
suffice provided that it is expressly and clearly admitted by
the accused.
5. It is the prosecution that has the burden of proving the age
of the offended party. The failure of the accused to object to
the testimonial evidence regarding age shall not be taken
against him.

Republic Act No. 9344, Section 6:

298
SEC. 6. Minimum Age of Criminal Responsibility. — A child fifteen
(15) years of age or under at the time of the commission of the
offense shall be exempt from criminal liability. However, the child
shall be subjected to an intervention program pursuant to Section
20 of the Act.
A child above fifteen (15) years but below eighteen (18) years of
age shall likewise be exempt from criminal liability and be subjected
to an intervention program, unless he/she has acted with
discernment, in which case, such child shall be subjected to the
appropriate proceedings in accordance with this Act.
The exemption from criminal liability herein established does not
include exemption from civil liability, which shall be enforced in
accordance with existing laws.
FACTS The accused Henry Arpon first raped the victim AAA which was his
niece when she was around eight years old. According to AAA’s
testimony she felt pain but she did not know if his organ penetrated
her vagina. AAA was again raped in July 1999 for five times on
different nights. The accused was drinking alcohol with AAA’s
stepfather when came to AAA’s house, took off his panty and went
on top of AAA. The accused even made AAA to hold his penis. AAA
was once again raped twice in August 1999 at nighttime. He kissed
her and then he took off his shirt, went on top of her and pumped.
She felt pain in her vagina and in her chest because he was heavy.
She did not know if his penis penetrated her vagina.
AAA did not tell anybody about the rapes because the accused
threatened to kill her mother if she did. She only filed a complaint
when he proceeded to also rape her younger sister, DDD.
The accused denied the allegations and stated that he was only 13
years of age when the first rape incident happened in 1995, further
claimed that he was a houseboy in Tacloban City. The accused also
claims that in July 1999, he was living in Tacloban City and working
as a dishwasher in a restaurant until around September 1999.
ISSUE/S Whether or not the accused Henry Arpon criminally liable of all
counts of rape charged against him?
RULING/S NO. For the first count of rape herein established, the accused-
appellant Henry Arpon y Juntilla is hereby EXEMPTED from
criminal liability.
In the instant case, we have thoroughly scrutinized the testimony of
AAA and we found no cogent reason to disturb the finding of the
RTC that the accused-appellant indeed committed the first incident
of rape charged. AAA positively identified the accused-appellant as
the perpetrator of the dastardly crimes.

299
Anent the five incidents of rape that were alleged to have been
committed in July 1999, the Court disagrees with the ruling of the
trial court that all five counts were proven with moral certainty. From
the testimony, AAA merely described a single incident of rape. She
made no reference whatsoever to the other four instances of rape
that were likewise supposedly committed in the month of July 1999.
The same is also true for the two (2) counts of rape allegedly
committed in August 1999.
"It is settled that each and every charge of rape is a separate and
distinct crime that the law requires to be proven beyond reasonable
doubt. The prosecution's evidence must pass the exacting test of
moral certainty that the law demands to satisfy the burden of
overcoming the appellant's presumption of innocence." Thus,
including the first incident of rape, the testimony of AAA was only
able to establish three instances when the accused-appellant had
carnal knowledge of her.
The RTC and the Court of Appeals failed to consider in favor of the
accused-appellant the privileged mitigating circumstance of
minority. Although this matter was not among the issues raised
before the Court, we still take cognizance of the same in accordance
with the settled rule that "[i]n a criminal case, an appeal throws open
the entire case wide open for review, and the appellate court can
correct errors, though unassigned, that may be found in the
appealed judgment."
The Court deemed sufficient the testimonial evidence regarding the
minority and age of the accused provided the following conditions
concur, namely: "(1) the absence of any other satisfactory evidence
such as the birth certificate, baptismal certificate, or similar
documents that would prove the date of birth of the accused; (2) the
presence of testimony from accused and/or a relative on the age
and minority of the accused at the time of the complained incident
without any objection on the part of the prosecution; and (3) lack of
any contrary evidence showing that the accused's and/or his
relatives' testimonies are untrue."
In the instant case, the accused-appellant testified that he was born
on February 23, 1982 and that he was only 13 years old when the
first incident of rape allegedly happened in 1995. Other than his
testimony, no other evidence was presented to prove the date of his
birth. However, the records of this case show neither any objection
to the said testimony on the part of the prosecution, nor any contrary
evidence to dispute the same. Thus, the RTC and the Court of
Appeals should have appreciated the accused-appellant's minority
in ascertaining the appropriate penalty.
Accordingly, for the first count of rape, which in the information in
Criminal Case No. 2000-01-46 was allegedly committed in 1995,
the testimony of the accused-appellant sufficiently established that

300
he was only 13 years old at that time. In view of the failure of the
prosecution to prove the exact date and year of the first incident of
rape, i.e., whether the same occurred in 1995 or in 1998 as
previously discussed, any doubt therein "should be resolved in favor
of the accused, it being more beneficial to the latter." 76 The Court,
thus, exempts the accused-appellant from criminal liability for the
first count of rape pursuant to the first paragraph of Section 6 of
Republic Act No. 9344. The accused-appellant, nevertheless,
remains civilly liable therefor.

301
CONTRIBUTOR DELA CRUZ, Ma. Luzelle P.
MODULE R.A. 9344: Juvenile Justice and Welfare Act
TOPIC
CASE TITLE PEOPLE OF THE PHILIPPINES v. G.R.NO. 193507
MONTICALVO
PONENTE PEREZ, J.: DATE: JAN. 30,
2013
DOCTRINE Persons who have been convicted and are serving sentence at the
time of the effectivity of this Act, and who were below the age of
eighteen (18) years at the time of the commission of the offense for
which they were convicted and are serving sentence, shall likewise
benefit from the retroactive application of this Act.
FACTS AAA is a mental retardate and was 12 years and 11 months old at
the time of the rape incident. She and appellant, who was then 17
years old, are neighbors − their respective houses are adjoining
each other.
In the afternoon of 9 December 2002, AAA and her friend, Analiza,
were in front of the sari-sari store of AAA’s mother, BBB, while
appellant was inside the fence of their house adjacent to the said
sari-sari store. Shortly, thereafter, appellant invited AAA to go with
him to the kiln at the back of their house. AAA acceded and went
ahead.
Upon seeing appellant and AAA going to the kiln, Analiza,
pretending to look for her one peso coin, followed them until she
reached a papaya tree located three and a half meters away from
the place. Analiza hid under the papaya tree and from there she saw
appellant undress AAA by removing the latter’s shorts and panty.
Appellant, however, glanced and saw Analiza. Frightened, Analiza
ran away and went back to the sari-sari store of BBB without telling
BBB what she saw.
Appellant proceeded to satisfy his bestial desire. After undressing
AAA, appellant made her lie down. He then placed himself on top of
AAA and made push and pull movements. Afterwards, appellant
stopped, allowed AAA to sit down for a while and then sent her
home.
When AAA arrived at their house around 7:30 p.m., she was asked
by her mother, BBB, where she came from and why she came home
late. AAA replied that she was at the back of their house as appellant
brought her there and had sexual intercourse with her.
The following day, BBB brought AAA to the police station and then
to the Northern Samar Provincial Hospital where AAA was
examined by Dr. Nochete.
Appellant denied having raped AAA. He claimed that on 9
December 2002, at around 1:00 p.m., he, together with Pio and a
certain Dinnes Samson, was having a drinking spree in the house
of one Adolfo Congayao (Adolfo). They finished drinking at around

302
6:00 p.m. As he was too drunk, Pio assisted him in going home. He
went to sleep and woke up only at 12:00 midnight as he needed to
urinate. He went back to sleep and woke up at 6:00 a.m. of the
following day, i.e., 10 December 2002. He was surprised that AAA
charged him with rape. He was then arrested at around 3:00 p.m. of
10 December 2002.
Both Pio and his father, Cesar, confirmed that on 9 December 2002,
they brought appellant to his bedroom and let him sleep there
because he was too drunk. Thereafter, Pio and Cesar engaged in a
drinking spree inside the latter’s house, particularly at the kitchen
that is more than two (2) meters away from appellant’s bedroom,
which lasted until 11:00 p.m. Pio and Cesar likewise stated that
there was no moment that appellant went out of his bedroom since
the time they brought him there.
Alexander, another defense witness, presented appellant’s
Certificate of Live Birth to prove that the latter was only 17 years old
during the commission of the crime.
ISSUE/S WON Monticalvo is rightfully convicted for the crime of rape of a
demented person.
WON the trial court failed to appreciate Monticalvo's age at the time
of the commission of the crime.
RULING/S As to penalty. Under Article 266-B in relation to Article 266-A(1) of
the Revised Penal Code, as amended, simple rape is punishable by
reclusion perpetua. However, when rape is committed by an
assailant who has knowledge of the victim’s mental retardation, the
penalty is increased to death. But this circumstance must be alleged
in the information being a qualifying circumstance which increases
the penalty to death and changes the nature of the offense from
simple to qualified rape. In the case at bench, while appellant
categorically admitted that he knew AAA to be suffering from mental
abnormalities, the prosecution failed to allege this fact in the
information. As such, even if it was proved, it cannot be appreciated
as a qualifying circumstance. Thus, appellant’s conviction is only for
simple rape for which he should be meted the penalty of reclusion
perpetua.
SC finds merit in appellant’s assertion that he was a minor during
the commission of the crime charged. During trial, upon order of the
trial court, the Local Civil Registrar of Bobon, Northern Samar,
brought before it their office records, particularly appellant’s
Certificate of Live Birth containing the fact of birth of the latter.
Appellant’s Certificate of Live Birth shows that he was born on 23
February 1985. Indeed, at the time of the commission of the crime
charged on 9 December 2002, appellant was only 17 years old, a
minor. Thus, he is entitled to the privileged mitigating circumstance
of minority pursuant to Republic Act No. 9344, otherwise known as
the "Juvenile Justice and Welfare Act of 2006," which took effect on

303
06 May 2006. Section 68 thereof specifically provides for its
retroactive application, thus:
SEC. 68. Children Who Have Been Convicted and are Serving
Sentence. – Persons who have been convicted and are serving
sentence at the time of the effectivity of this Act, and who were
below the age of eighteen (18) years at the time of the
commission of the offense for which they were convicted and
are serving sentence, shall likewise benefit from the retroactive
application of this Act. They shall be entitled to appropriate
dispositions provided under this Act and their sentences shall be
adjusted accordingly. They shall be immediately released if they are
so qualified under this Act or other applicable law.
Clearly, Republic Act No. 9344 is applicable in this case even
though the crime was committed four (4) years prior to its enactment
and effectivity.
Section 38 of Republic Act No. 9344 warrants the suspension of
sentence of a child in conflict with the law notwithstanding that
he/she has reached the age of majority at the time the judgment of
conviction is pronounced. It reads, thus:
SEC. 38. Automatic Suspension of Sentence. – Once the child who
is under eighteen (18) years of age at the time of the commission of
the offense is found guilty of the offense charged, the court shall
determine and ascertain any civil liability which may have resulted
from the offense committed. However, instead of pronouncing the
judgment of conviction, the court shall place the child in
conflict with the law under suspended sentence, without need
of application: Provided, however, That suspension of
sentence shall still be applied even if the juvenile is already
eighteen (18) of age or more at the time of the pronouncement
of his/her guilt.
Upon suspension of sentence and after considering the various
circumstances of the child, the court shall impose the appropriate
disposition measures as provided in the Supreme Court Rule on
Juveniles in Conflict with the Law.
At present, appellant is already 27 years of age, and the judgment
of the trial court was promulgated prior to the effectivity of Republic
Act No. 9344.
Be that as it may, to give meaning to the legislative intent of Republic
Act No. 9344, the promotion of the welfare of a child in conflict
with the law should extend even to one who has exceeded the
age limit of 21 years, so long as he/she committed the crime
when he/she was still a child. The offender shall be entitled to the
right to restoration, rehabilitation and reintegration in accordance
with Republic Act No. 9344 in order that he/she is given the chance
to live a normal life and become a productive member of the
community. The age of the child in conflict with the law at the time

304
of the promulgation of the judgment of conviction is not material.
What matters is that the offender committed the offense when
he/she was still of tender age.
The appellant, therefore, shall be entitled to appropriate disposition
under Section 51 of Republic Act No. 9344, which provides for the
confinement of convicted children as follows:
SEC. 51. Confinement of Convicted Children in Agricultural
Camps and Other Training Facilities. – A child in conflict with the
law may, after conviction and upon order of the court, be made to
serve his/her sentence, in lieu of confinement in a regular penal
institution, in an agricultural camp and other training facilities that
may be established, maintained, supervised and controlled by the
BUCOR, in coordination with the DSWD.
To conform to this Court’s ruling in People v.Sarcia, the case shall
be remanded to the court of origin to effect appellant’s confinement
in an agricultrual camp or other training facility.

305
CONTRIBUTOR DELA CRUZ, Ma. Luzelle P.
MODULE R.A. 9344 Juvenile Justice and Welfare Act
TOPIC
CASE TITLE SPARK v. TULLETT PREBON G.R.NO. 225442
PONENTE PERLAS-BERNABE, J: DATE: AUG. 08, 2017
DOCTRINE Ordinances enacted by local governments concerning juvenile
status offenses such as, but not limited to, curfew violations,
truancy, parental disobedience, anti-smoking and anti-drinking
laws, as well as light offenses and misdemeanors against public
order or safety such as, but not limited to, disorderly conduct, public
scandal, harassment, drunkenness, public intoxication, criminal
nuisance, vandalism, gambling, mendicancy, littering, public
urination, and trespassing, shall be for the protection of children.
No penalty shall be imposed on children for said violations, and they
shall instead be brought to their residence or to any barangay official
at the barangay hall to be released to the custody of their parents.
Appropriate intervention programs shall be provided for in such
ordinances.
FACTS Following the campaign of President Rodrigo Roa Duterte to
implement a nationwide curfew for minors, several local
governments in Metro Manila started to strictly implement their
curfew ordinances on minors through police operations which were
publicly known as part of "Oplan Rody."
Among those local governments that implemented curfew
ordinances were respondents: (a) Navotas City, through
Pambayang Ordinansa Blg. 99-02, dated August 26, 1999, entitled
"Nagtatakda ng 'Curfew' ng mga Kabataan na Wala Pang Labing
Walong (18) Taong Gulang sa Bayan ng Navotas, Kalakhang
Maynila," as amended by Pambayang Ordinansa Blg. 2002-13,
dated June 6, 2002 (Navotas Ordinance); (b) City of Manila, through
Ordinance No. 8046 entitled "An Ordinance Declaring the Hours
from 10:00 P.M. to 4:00A.M. of the Following Day as 'Barangay
Curfew Hours' for Children and Youths Below Eighteen (18) Years
of Age; Prescribing Penalties Therefor; and for Other Purposes"
dated October 14, 2002 (Manila Ordinance); and (c) Quezon City,
through Ordinance No. SP-2301, Series of 2014, entitled "An
Ordinance Setting for a [sic] Disciplinary Hours in Quezon City for
Minors from 10:00 P.M. to 5:00A.M., Providing Penalties for
Parent/Guardian, for Violation Thereof and for Other Purposes"
dated July 31, 2014 (Quezon City Ordinance; collectively, Curfew
Ordinances).
ISSUE/S WON the Curfew Ordinances are unconstitutional.
RULING/S The Navotas Ordinance and, to a greater extent, the Manila
Ordinance still do not account for the reasonable exercise of the
minors' rights of association, free exercise of religion, rights to
peaceably assemble, and of free expression, among others.

306
The Navotas City Ordinance effectively prohibits minors from
attending traditional religious activities (such as simbang gabi) at
night without accompanying adults. It also does not accommodate
avenues for minors to engage in political rallies or attend city council
meetings to voice out their concerns in line with their right to
peaceably assemble and to free expression which the Court finds
no reason to prohibit them from during curfew hours.
The Manila Ordinance, particularly Section 4[19] thereof,
contravenes Section 57-A[20] of RA 9344, as amended, given that
the cited curfew provision imposes on minors the penalties of
imprisonment, reprimand, and admonition.
Only the Manila Ordinance imposed various sanctions to the minor
based on the age and frequency of violations, to wit:
SEC. 4. Sanctions and Penalties for Violation. Any child or youth
violating this ordinance shall be sanctioned/punished as follows:
(a) If the offender is Fifteen (15) years of age and below, the
sanction shall consist of a REPRIMAND for the youth offender and
ADMONITION to the offender's parent, guardian or person
exercising parental authority.
(b) If the offender is Fifteen (15) years of age and under Eighteen
(18) years of age, the sanction/penalty shall be:
For the FIRST OFFENSE, Reprimand and Admonition;
For the SECOND OFFENSE, Reprimand and Admonition, and a
warning about the legal impostitions in case of a third and
subsequent violation; and
For the THIRD AND SUBSEQUENT OFFENSES, Imprisonment of
one (1) day to ten (10) days, or a Fine of TWO THOUSAND PESOS
(Php2,000.00), or both at the discretion of the Court, PROVIDED,
That the complaint shall be filed by the Punong Barangay with the
office of the City Prosecutor."
Sections 57 and 57-A of RA 9344, as amended, prohibit the
imposition of penalties on minors for status offenses such as curfew
violations, viz.:
SEC. 57. Status Offenses. — Any conduct not considered an
offense or not penalized if committed by an adult shall not be
considered an offense and shall not be punished if committed by a
child.
SEC. 57-A. Violations of Local Ordinances. — Ordinances
enacted by local governments concerning juvenile status
offenses such as, but not limited to, curfew violations, truancy,
parental disobedience, anti-smoking and anti-drinking laws, as well
as light offenses and misdemeanors against public order or safety
such as, but not limited to, disorderly conduct, public scandal,
harassment, drunkenness, public intoxication, criminal nuisance,
vandalism, gambling, mendicancy, littering, public urination, and
trespassing, shall be for the protection of children. No penalty shall

307
be imposed on children for said violations, and they shall
instead be brought to their residence or to any barangay
official at the barangay hall to be released to the custody of
their parents. Appropriate intervention programs shall be
provided for in such ordinances. The child shall also be recorded
as a "child at risk" and not as a "child in conflict with the law." The
ordinance shall also provide for intervention programs, such as
counseling, attendance in group activities for children, and for the
parents, attendance in parenting education seminars.
WHEREFORE, the petition is PARTLY GRANTED. The Court
hereby declares Ordinance No. 8046, issued by the local
government of the City of Manila, and Pambayang Ordinansa Blg.
No. 99-02, as amended by Pambayang Ordinansa Blg. 2002-13
issued by the local government of Navotas City,
UNCONSTITUTIONAL and, thus, NULL and VOID; while Ordinance
No. SP-2301, Series of 2014, issued by the local government of the
Quezon City is declared CONSTITUTIONAL and, thus, VALID in
accordance with this Decision.

308
CONTRIBUTOR DIMAKUTA, Jasmine A.
MODULE EXEMPTING CIRCUMSTANCE OF ACCIDENT
TOPIC
CASE TITLE THE UNITED STATES V. G.R. NO. L-5418
CECILIO TAÑEDO
PONENTE MORELAND, J. DATE: February 12,
1910
DOCTRINE The court quoted State vs. Legg: "Where accidental killing is relied
upon as a defense, the accused is not required to prove such a
defense by a preponderance of the evidence, because there is a
denial of intentional killing, and the burden is upon the state to show
that it was intentional, and if, from a consideration of all the
evidence, both that for the state and the prisoner, there is a
reasonable doubt as to whether or not the killing was accidental or
intentional, the jury should acquit."

FACTS The accused was a landowner. On the morning of the 26th of


January, 1909, he, with Bernardino Tagampa, Casimiro Pascual,
Valeriano Paulillo, and Juan Arellano, went to work on a malecon or
dam on his land. The defendant took with him a shotgun and a few
shells, with the intention to hunt wild chickens after he had set his
laborers at work. He remained with his laborers an hour or so and
then went a short distance away across a stream to see how the
alteration which he had made in the malecon affected the flow of
water from the rice filed on the other side of the stream. He carried
his shotgun with him across the stream. On the other side of the
stream he met the deceased, who, with his mother and uncle, had
been living in a small shack for a month or so during the rice-
harvesting season. The accused asked the uncle of the deceased
where he could find a good place in which to hunt wild chickens.
The uncle was lying on the floor in the interior of the shack sick of
fever. The deceased, a young man about 20 years of age, was
working at something under a manga tree a short distance from the
shack. Although the accused directed his question to the uncle
inside of the shack, the deceased answered the question and
pointed out in a general way a portion of the forest near the edge of
which stood the shack. There is some contradiction between the
testimony of the accused and the Government witnesses just at this
point. The uncle of the deceased testified that the boy and the
accused invited each other mutually to hunt wild chickens and that
the accused accepted the invitation. The accused, however,
testified that he did not invite the deceased to go hunting with him,
neither did the deceased go with him, but that he remained under
the manga tree "trying something." At any rate the accused went
into the forest with his gun. What took place there is unknown to

309
anybody except the accused. Upon that subject he testified as
follows:
“And after Feliciano Sanchez pointed out that place to me, that
place where the wild chickens were to be found, I proceeded to
hunt, because, in the first place, if I could kill some wild chickens
we would have something to eat on that day. So when I arrived at
that place I saw a wild chicken and I shot him. And after I shot that
chicken I heard a human cry. I picked up the chicken and went
near the place where I heard the noise, and after I saw that I had
wounded a man I went back toward the malecon, where my
companions were working, running back, and when I arrived there
I left my shotgun behind or by a tree not far from where my
companions were working; and I called Bernardino Tagampa to tell
him about the occurrence, and to him I told of that occurence
because he is my friend and besides that he was a relative of the
deceased, and when Tagampa heard of this he and myself went
together to see the dead body.”

Only one shot was heard that morning and a chicken was killed by
gunshot wound. Chicken feathers were found in considerable
quantities at the point where the chicken was shot and where the
accident occurred. The defendant within a few minutes after the
accident went out of the woods to the malecon where he had left his
laborers at work, carrying the dead chicken with him. The accused
called Bernardino Tagampa, one of the laborers, to go with him and
they disappeared for some time. Tagampa says that they went a
little way toward the woods and came back. The accused says that
they went to the place where the body of the deceased lay and
removed it to a place in the cogon grass where it would not be easily
observed. It is certain, however, that the body was concealed in the
cogon grass. During the afternoon Tagampa left the malecon, where
his fellow laborers were working, probably to hunt for a place in
which to hide the body. The rest of the laborers saw the witness
Yumul take the chicken which had been killed by the accused. He
delivered it to the wife of the accused, who testified that she received
the chicken from Yumul and that it had been killed by a gunshot
wound. That evening the accused and Tagampa went together to
dispose of the body finally. They took it from the cogon grass where
it lay concealed and carried it about 17 or 18 hundred meters from
the place where it had originally fallen, and buried it in an old well,
covering it with straw and earth and burning straw on top of the well
for the purpose of concealing it. Tagampa said that he helped the
accused dispose of the body because he was afraid of him, although
he admits that the accused in no way threatened or sought to
compel him to do so. The defendant prior to the trial denied all
knowledge of the death of the deceased or the whereabouts of the

310
body. On the trial, however, he confessed his participation in the
death of the deceased and told the story substantially as above.

ISSUE/S Whether or not the accused is guilty

RULING/S No. So far as can be ascertained from the evidence the prior
relations between the accused and the deceased had been normal.
The deceased was a tenant on land belonging to a relative of the
accused. There was no enmity and no unpleasant relations between
them. No attempt was made to show any. There appears to have
been no motive whatever for the commission of the crime. The
Government has not attempted to show any. The only possible
reason that the accused could have for killing the deceased would
be found in the fact of a sudden quarrel between them during the
hunt. That idea is wholly negative by the fact that the chicken and
the man were shot at the same time, there having been only one
shot fired.

In this case there is absolutely no evidence of negligence upon the


part of the accused. Neither is there any question that he was
engaged in the commission of a lawful act when the accident
occurred. Neither is there any evidence of the intention of the
accused to cause the death of the deceased. The only thing in the
case at all suspicious upon the part of the defendant are his
concealment and denial. We are of the opinion that the evidence is
insufficient to support the judgment of conviction. The judgment of
conviction is, therefore, reversed, the defendant acquitted, and his
discharge from custody ordered.

311
CONTRIBUTOR FLORES, Precious Eureka D.
MODULE Exempting Circumstance
TOPIC
CASE TITLE PEOPLE v. CASTILLO G.R.NO. 172695
PONENTE YNARES-SANTIAGO, J. DATE: JUNE 29, 2007
DOCTRINE Conviction can be had on the basis of circumstantial evidence
provided that: (1) there is more than one circumstance; (2) the facts
from which the interferences are derived are proven; and (3) the
combination of all the circumstances is such as to produce a
conviction beyond reasonable doubt.
FACTS In the evening of 5 November 1993, the accused-appellant came
home drunk and angry. His father-in-law (Guillermo) tried to subdue
him but to no avail which caused the former to leave the house. As
he was leaving, Guillermo saw him take out his sling and arrow.
Consorcia, the accused’s wife, was heard crying and screaming.
Thereafter, the accused-appellant was seen carrying the bloodied
body of Consorcia out of the house and was later taken to the
hospital but to no avail. Cause of death was the cut jugular vein
caused by a fatal weapon which could have been a “pointed
instrument like a nail.”
ISSUE/S 1. Whether or not the court failed to sufficiently establish the guilt
of the accused beyond reasonable doubt.
2. Whether or not the husband is exempt from criminal liability.
RULING/S 1. Direct evidence of the commission of the offense is not the only
matrix wherefrom a trial court may draw its conclusions and
finding of guilt. Conviction can be had on the basis of
circumstantial evidence provided that: (1) there is more than
one circumstance; (2) the facts from which the inferences are
derived are proven; and (3) the combination of all the
circumstances is such as to produce a conviction beyond
reasonable doubt. While no general rule can be laid down as to
the quantity of circumstantial evidence which will suffice in a
given case, all the circumstances proved must be consistent
with each other, consistent with the hypothesis that the accused
is guilty, and at the same time inconsistent with the hypothesis
that he is innocent, and with every other rational hypothesis
except that of guilt. The circumstances proved should constitute
an unbroken chain which leads to only one fair and reasonable
conclusion that the accused, to the exclusion of all others, is the
guilty person. Proof beyond reasonable doubt does not mean
the degree of proof excluding the possibility of error and
producing absolute certainty. Only moral certainty or "that
degree of proof which produces conviction in an unprejudiced
mind" is required.

312
The essential requisites are present for circumstantial evidence to
sustain conviction:

1. Consortia would often confide to her sister Leticia about the


violent behavior of her (Consortia) husband, herein accused-
appellant. And even if Consortia would not tell Leticia about
the beatings, the latter would see her face with black eyes as
evident proofs of maltreatment.
2. On the night of the incident, accused-appellant arrived at
their house drunk and displaying violent behavior, kicking the
door and table.
3. Accused-appellant was last seen holding and practicing his
sling and arrow.
4. Immediately afterwards, Consortia was heard crying and
shouting.
5. Accused-appellant was thereafter seen carrying Consortia,
bloodied and unconscious, to be brought to the hospital
where she later died.
6. The autopsy findings indicate that Consortia sustained a
punctured wound in the neck which fatally lacerated her
jugular vein. The cause of the wound was a pointed object.
7. While detained, accused-appellant wrote letters to the
parents and sister of Consortia asking for forgiveness.

Also notable is accused-appellant’s behavior immediately after the


incident. He disappeared and did not enter the clinic where
Consortia was rushed for treatment. And when Consortia’s sister
later sought police assistance in searching for accused-appellant,
the latter was found by the police hiding inside a toilet at a nearby
barangay. Therefore, he is found guilty of the crime of Parricide with
a penalty of reclusion perpetua.

2. Article 12. Circumstances which exempt from criminal liability


– The following are exempt from criminal liability:

Xxx

(4) Any person who, while performing a lawful act with due
care, causes an injury by mere accident without fault or
intention of causing it.

"Accident" is an affirmative defense which the accused is burdened


to prove, with clear and convincing evidence. The defense
miserably failed to discharge its burden of proof. The essential
requisites for this exempting circumstance, are:
1. A person is performing a lawful act;

313
2. With due care;
3. He causes an injury to another by mere accident;
4. Without fault or intention of causing it.
By no stretch of imagination could playing with or using a deadly
sling and arrow be considered as performing a "lawful act." Thus,
on this ground alone, appellant’s defense of accident must be struck
down because he was performing an unlawful act during the
incident. As correctly found by the trial court:
Furthermore, mere possession of sling and arrow is punishable
under the law. In penalizing the act, the legislator took into
consideration that the deadly weapon was used for no legal
purpose, but to inflict injury, mostly fatal, upon other persons. Let it
be stressed that this crude weapon cannot attain the standards as
an instrument for archery competitions. To sustain the accused’s
assertion that he was practicing the use of said weapon at the time
of the incident is patently absurd. The defense even failed to rebut
Guillermo Antiporta’s testimony that the accused was keeping said
sling and arrow inside his house.

314
MODULE 7
MITIGATING CIRCUMSTANCES

315
CONTRIBUTOR DIMAKUTA, Jasmine A.
MODULE JUSTIFYING CIRCUMSTANCE OF SELF-DEFENSE,
TOPIC MITIGATING CIRCUMSTANCE OF NO INTENT TO COMMIT SO
GRAVE A WRONG AS THAT COMMITTED AND VOLUNTARY
SURRENDER
CASE TITLE THE PEOPLE OF THE PHILIPPINES V. C.A. NO. 384
NICOLAS JAURIGUE and AVELINA
JAURIGUE
PONENTE DE JOYA, J. DATE: February 21,
1946
DOCTRINE The attempt to rape a woman constitutes an unlawful aggression
sufficient to put her in a state of legitimate defense, inasmuch as a
woman's honor cannot but be esteemed as a right as precious, if
not more, than her very existence; and it is evident that a woman
who, thus imperiled, wounds, nay kills the offender, should be
afforded exemption from criminal liability, since such killing cannot
be considered a crime from the moment it became the only means
left for her to protect her honor from so great an outrage.

FACTS At about 8 PM of the same day, September 20, 1942, Nicolas


Jaurigue went to the chapel of the Seventh Day Adventists of which
he was the treasurer, in their barrio, just across the provincial road
from his house, to attend religious services, and sat on the front
bench facing the altar with the other officials of the organization and
the barrio lieutenant, Casimiro Lozada. Inside the chapel it was quite
bright as there were electric lights. Defendant and appellant Avelina
Jaurigue entered the chapel shortly after the arrival of her father,
also for the purpose of attending religious services, and sat on the
bench next to the last one nearest the door. Amado Capina was
seated on the other side of the chapel. Upon observing the presence
of Avelina Jaurigue, Amado Capina went to the bench on which
Avelina was sitting and sat by her right side, and, without saying a
word, Amado, with the greatest of impudence, placed his hand on
the upper part of her right thigh. On observing this highly improper
and offensive conduct of Amado Capina, Avelina Jaurigue,
conscious of her personal dignity and honor, pulled out with her right
hand the fan knife, which she had in a pocket of her dress, with the
intention of punishing Amado's offending hand. Amado seized
Avelina's right hand, but she quickly grabbed the knife with her left
hand and stabbed Amado once at the base of the left side of the
neck, inflicting upon him a wound about 4 1/2 inches deep, which
was necessarily mortal. Nicolas Jaurigue, who was seated on one
of the front benches, saw Amado bleeding and staggering towards
the altar, and upon seeing his daughter still holding the bloody knife,
he approached her and asked: "Why did you do that," and
answering him Avelina said: "Father, I could not endure anymore."

316
Amado Capina died from the wound a few minutes later. Barrio
lieutenant Casimiro Lozada, who was also in the same chapel,
approached Avelina and asked her why she did that, and Avelina
surrendered herself, saying: "Kayo na po ang bahala sa
aquin.”Fearing that Amado's relatives might retaliate, barrio
lieutenant Lozada advised Nicolas Jaurigue and herein defendant
and appellant to go home immediately, to close their doors and
windows and not to admit anybody into the house, unless
accompanied by him. That father and daughter went home and
locked themselves up, following instructions of the barrio lieutenant,
and waited for the arrival of the municipal authorities; and when
three policemen arrived in their house, at about 10 PM, and
questioned them about the incident, defendant and appellant
immediately surrendered the knife, and informed said policemen
briefly of what had actually happened in the chapel and of the
previous acts and conduct of the deceased, as already stated
above, and went with said policemen to the police headquarters.

ISSUE/S 1. Whether or not the lower court erred in not holding that said
appellant had acted in the legitimate defense of her honor and
that she should be completely absolved of all criminal
responsibility;
2. Whether or not the lower court erred in not finding in her favor
the additional mitigating circumstances that (a) she did not have
the intention to commit so grave a wrong as that actually
committed, and that (b) she voluntarily surrendered to the agents
of the authorities; and
3. Whether or not the trial court erred in holding that the
commission of the alleged offense was attended by the
aggravating circumstance of having been committed in a sacred
place

RULING/S 1. No. The attempt to rape a woman constitutes an unlawful


aggression sufficient to put her in a state of legitimate defense,
inasmuch as a woman's honor cannot but be esteemed as a right
as precious, if not more, than her very existence; and it is evident
that a woman who, thus imperiled, wounds, nay kills the offender,
should be afforded exemption from criminal liability, since such
killing cannot be considered a crime from the moment it became the
only means left for her to protect her honor from so great an outrage.

In the case, however, in which a sleeping woman was awakened at


night by someone touching her arm, and, believing that some
person was attempting to abuse her, she asked who the intruder
was and receiving no reply, attacked and killed the said person with
a pocket knife, it was held that, notwithstanding the woman's belief
in the supposed attempt, it was not sufficient provocation or

317
aggression to justify her completely in using deadly weapon.
Although she actually believed it to be the beginning of an attempt
against her, she was not completely warranted in making such a
deadly assault, as the injured person, who turned out to be her own
brother-in-law returning home with his wife, did not do any other act
which could be considered as an attempt against her honor.

In the instant case, if defendant and appellant had killed Amado


Capina, when the latter climbed up her house late at night on
September 15, 1942, and surreptitiously entered her bedroom,
undoubtedly for the purpose of raping her, as indicated by his
previous acts and conduct, instead of merely shouting for help, she
could have been perfectly justified in killing him, as shown by the
authorities cited above.

2. Yes. According to the facts established by the evidence and found


by the learned trial court in this case, when the deceased sat by the
side of defendant and appellant on the same bench, near the door
of the barrio chapel and placed his hand on the upper portion of her
right thigh, without her consent, the said chapel was lighted with
electric lights, and there were already several people, about ten of
them, inside the chapel, including her own father and the barrio
lieutenant and other dignitaries of the organization; and under the
circumstances, there was and there could be no possibility of her
being raped. And when she gave Amado Capina a thrust at the base
of the left side of his neck, inflicting upon him a mortal wound 4 1/2
inches deep, causing his death a few moments later, the means
employed by her in the defense of her honor was evidently
excessive; and under the facts and circumstances of the case, she
cannot be legally declared completely exempt from criminal liability.

But the fact that defendant and appellant immediately and


voluntarily and unconditionally surrendered to the barrio lieutenant
in said chapel, admitting having stabbed the deceased, immediately
after the incident, and agreed to go to her house shortly thereafter
and to remain there subject to the order of the said barrio lieutenant,
an agent of the authorities; and the further fact that she had acted
in the immediate vindication of a grave offense committed against
her a few moments before, and upon such provocation as to
produce passion and obfuscation, or temporary loss of reason and
self-control, should be considered as mitigating circumstances in
her favor.

Defendant and appellant further claims that she had not intended to
kill the deceased but merely wanted to punish his offending hand
with her knife, as shown by the fact that she inflicted upon him only

318
one single wound. And this is another mitigating circumstance which
should be considered in her favor.

3. Yes. The claim of the prosecution, sustained by the learned trial


court, that the offense was committed by the defendant and
appellant, with the aggravating circumstance that the killing was
done in a place dedicated to religious worship, cannot be legally
sustained; as there is no evidence to show that the defendant and
appellant had murder in her heart when she entered the chapel that
fatal night. Avelina is not a criminal by nature. She happened to kill
under the greatest provocation.

In the mind of the court, there is not the least doubt that, in stabbing
to death the deceased Amado Capina, in the manner and form and
under the circumstances above indicated, the defendant and
appellant committed the crime of homicide, with no aggravating
circumstance whatsoever, but with at least three mitigating
circumstances of a qualified character to be considered in her favor;
and, in accordance with the provisions of article 69 of the Revised
Penal Code, she is entitled to a reduction by one or two degrees in
the penalty to be imposed upon her. And considering the
circumstances of the instant case, the defendant and appellant
should be accorded the most liberal consideration possible under
the law.

319
CONTRIBUTOR FLORES, Precious Eureka D.
MODULE Mitigating Circumstances
TOPIC
CASE TITLE PEOPLE v. NARVAEZ G.R.NO. L-33466-67
PONENTE MAKASIAR, J. DATE: APRIL 20, 1983
DOCTRINE Illegal aggression is equivalent to assault or at least threatened
assault of immediate and imminent kind.
FACTS In the afternoon of August 22, 1968, Graciano Juan, Jesus Verano
and Cesar Ibanez together with Davis Fleischer and Flaviano Rubia
were fencing the land of George Fleischer, father of Davis Fleischer
which is located in the municipality of Maitum, South Cotabato.

During that time, Narvaez was taking a rest, but when he heard that
the walls of his house were being chiseled, he arose and there he
saw the fencing going on. If the fencing would go on, it would have
an effect of shutting off the accessibility to appellant’s house and
rice mill from the highway, since the door of the same opens to the
Fleischer’s side. So he addressed the group, saying, “Pare, if
possible you stop destroying my house and if possible we will talk it
over what is good,” addressing Rubia, who is also the appellant’s
compadre. However, Fleischer answered, “No goddamit, proceed,
go ahead.” Upon hearing, Narvaez, lost his equilibrium and he got
his gun and shot Fleischer, hitting him. As Fleischer fell down, Rubia
ran towards the jeep, and knowing there is a gun on the jeep,
appellant fired at Rubia, likewise hitting him. Both Fleischer and
Rubia died as result of the shooting.
ISSUE/S 1. Whether or not the lower court erred in convicting Narvaez
despite the fact that he acted in defense of his person.
2. Whether or not the lower court erred in convicting Narvaez
although he acted in defense of his rights.
RULING/S In order for defense of one’s person or rights to be treated as a
justifying circumstance, the following requisites must occur:

1. Unlawful aggression;
2. Reasonable necessity of the means employed to
prevent or repel aggression;
3. Lack of sufficient provocation on the side of the person
defending himself.

Appellant admitted having shot them from the window of his house
with the shotgun which he surrendered to the police authorities. He
claims, however, that he did so in defense of his persons and his
rights.

320
The aggression referred to in this case is the angry utterance by
Fleischer, in answer to his request addressed to his compadre.
Fleischer’s remarks caused a reaction as if, he lost his senses and
unknowingly took the gun on the bed and unknowingly shot the
victims. The action of the victim in angrily ordering the continuance
of the fencing which my result to the further chiseling of the walls as
well as the closure of the access to and from his house and rice mill
which were not only imminent but were actually in progress. The
said action was clearly an aggression on the side of the victim not
on the person but on the property rights of the appellant.

To consider the act as unlawful is based on the following provisions


of the Civil Code of the Philippines:

Art. 536. In no case may possession be acquired


through force or intimidation as long as there is a
possessor who objects thereto. He who believes that
he has an action or a right to deprive another of the
holding of a thing must invoke the aid of the competent
court, if the holder should refuse to deliver the thing.

Art. 539. Every possessor has a right to be respected


in his possession; and should he be disturbed therein
he shall be protected in or restored to said possession
by the means established by the laws and the Rules of
Court

Conformably to the foregoing provisions, the deceased had no right


to destroy or cause damage to appellant's house, nor to close his
accessibility to the highway while he was pleading with them to stop
and talk things over with him. The assault on appellant's property,
therefore, amounts to unlawful aggression as contemplated by law.

However, the reasonableness of the resistance was not met when


Narvaez fired a shotgun from his window which killed the victims,
his resistance was disproportionate to the attack.

The third element of defense of property is present. There is no


provocation on the side of the accused, since he was asleep in the
first place and was only awakened by the noise produced by the
victims and their laborers. His plea for the deceased and their men
to stop and talk things over with him was no provocation at all.

On the other hand, the qualifying circumstance of treachery cannot


be appreciated because of the presence of provocation on the part
of the victim. Likewise, the aggravating circumstance of evident

321
premeditation cannot be applied since it was not sufficiently
established.

Voluntary surrender is appreciated as a mitigating circumstance


since Narvaez surrendered to the authorities soon after the
shooting. In addition, passion or obfuscation was attendant to the
crime since the appellant awoke to find his house being damaged
and its accessibility to the highway and rice mill being closed.

Therefore, the appellant was only guilty of two (2) counts of


homicide, the killing not being attended by any qualifying
circumstance, but extenuated by the privileged mitigating
circumstance of incomplete self-defense and by two generic
mitigating circumstance of voluntary surrender and passion and
obfuscation.

322
CASE TITLE People v. Ulep G.R NO. G.R. No. 132547
PONENTE BELLOSILLO, J. DATE: 20 September 2000
CONTRIBUTOR GARCIA, LEXANNE O.
DOCTRINE
Fulfillment of a duty under Art. 11, par. 5, of The Revised Penal
Code may be successfully invoked, the accused must prove the
presence of two (2) requisites – first, that the act was done in
performance of a duty and second, the crime was a necessary
consequence of such duty.

The fatal wound in the head of the victim was a necessary


consequence of accused appellant’s due performance of a duty or
the lawful exercise of a right or office.
FACT/S
Buenaventura Wapili went berserk in Cotabato. Leydan then went
to the house of policewoman Norma Plando, a neighbor, and
asked for assistance. Plando then contacted SPO1 Ernesto Ulep,
SPO1 Edilberto Espadera and SPO2 Crispin Pillo. The three were
on board an Anfra police service jeep, all armed with M-16 rifles.
As Wapili was approaching them, Ulep fired a warning shot and
told Wapili to put down his weapons or they would shoot him. But
Wapili retorted “fire!” and continued advancing towards the police
officers. When Wapili was near them, SPO1 Ulep shot the victim
with his M-16 rifle, hitting him in various parts of his body. Ulep
came closer and pumped another bullet into his head and literally
blew his brains out.

Ulep now claims that the killing of the victim was in the course of
the performance of his official duty as a police officer, and in self-
defense.
MAIN ISSUE/S
Would the defense of fulfillment of a duty under Art. 11, par. 5 of
the RPC prosper?
RULING ON
MAIN ISSUE/S Fulfillment of a duty under Art. 11, par. 5, of The Revised Penal
Code may be successfully invoked, the accused must prove the
presence of two (2) requisites. The second requisite is lacking in
the instant case.

To perform a legal duty is present as they sought to render police


assistance, and restore peace and order at Mundog Subdivision
where the victim was then running amuck.

However, he cannot be exonerated from overdoing his duty during


the second stage of the incident —when he fatally shot the victim
in the head. Sound discretion and restraint dictated that accused-

323
appellant, a veteran policeman should have ceased firing at the
victim the moment he saw the latter fall to the ground. The victim
at that point no longer posed a threat and was already incapable
of mounting an aggression against the police officers. Shooting
him in the head was obviously unnecessary.

It cannot therefore be said that the fatal wound in the head of the
victim was a necessary consequence of accused appellant’s due
performance of a duty or the lawful exercise of a right or office.

The presence of unlawful aggression is a condition sine qua non.


The records show that the victim was lying in a prone position on
the ground—bleeding from the bullet wounds he sustained, and
possibly unconscious—when accused-appellant shot him in the
head. We find in favor of accused-appellant the incomplete
justifying circumstance of fulfillment of a duty or lawful exercise of
a right.

324
CONTRIBUTOR CARPIO, Anna Clarissa C.
MODULE Module 7: Mitigating Circumstances
TOPIC A(b)(d): Articles 64 and 69, RPC
CASE TITLE GUILLERMO v. PEOPLE G.R.NO. 153287
PONENTE Brion, J. DATE: June 30, 2008
DOCTRINE Reasonableness is a function of the nature or severity of the attack
or aggression confronting the accused, the means employed to
repel his attack. It is necessary that the surrounding circumstances
of the attack such as the place and occasion, must show rational
equivalence between the attack and defense. That is, the means
employed by a person invoking self-defense must be reasonably
commensurate to the nature and extent of the attack sought to
be averted. The nature and number of wounds inflicted on the victim
as important indicia material to a plea of self-defense. Nonetheless,
the penalty imposed when the crime committed was not wholly
excusable is one to two degrees than that prescribed by law by
reason of the lack of some of the conditions required to justify the
same or to exempt from criminal liability, provided that the majority
of such conditions be present.
FACTS Petitioner Noel Guillermo assails the decision of the Roxas City
Regional Trial Court convicting him, along with 2 others, Arnaldo
Socias and Joemar Palma of the crime of homicide for the death of
Winnie Alon. The prosecution and the defense gave different
versions on what transcribed.

The prosecution presented witnesses Vicente Alon and Eddie


Roque who were both with the deceased at the time of the incident.
According to Vicente, in the afternoon of July 1996, he and the
deceased together with 2 others were at the Cuartero public market
at a restaurant to eat while petitioner Guillermo and his co-accused
were drinking beer. Vicente avers that petitioner and Socias were
known to him since childhood as they came from the same
barangay. While at the restaurant, an altercation between Socias
and deceased Winnie regarding the cutting of wood by a chain saw
transpired. Guillermo suddenly took hold of Winnie and stabbed the
latter at the neck 3 times. Co-accused Palma went to the kitchen
and got a knife. Socias hit Vicente with a bottle of beer by the head
and the latter fell down and lost consciousness. In cross-
examination, witness Vicente admitted that he and Winnie were
already drunk before they went to the restaurant where the stabbing
took place. Witness Roque corroborated Vicente’s testimony and
also admitted the state of intoxication. Roque avers that the 3
accused were already drinking beer at the restaurant when and they
were invited to join. Before each of them could fully consume a bottle
served upon each of them, deceased Winne and accused Socias
argued about the cutting of wood and the argument was so heated

325
that each of them stood up and Socias took 2 bottles which were
thrown to Vicente, hitting the latter at the forehead. Petitioner
hugged or embraced Winnie and stabbed him 3 times on the neck
with a Batangueño knife and Socias, using a pointed object which
cannot be determined, stabbed Winnie from behind on the left side
of his body just below the armpit. Palma also helped in stabbing
Winnie once, at the right side of his body. Roque avers that Winnie
resisted trying to struggle but could not move because he was
ganged up by the 3. In post mortem examination, it was found that
the cause of death was massive hemorrhage due to multiple stab
wounds and that the 3 stab wounds were probably caused by a
sharp-bladed instrument like a knife.

For the defense, Petitioner Guillermo claimed that they have only
consumed half a bottle of the beer when the group of the deceased
arrived. Winnie came to him and requested to join them in their table
so they affirmed. Winnie and Socias had an altercation regarding
the labtik and it was the deceased who challenged Socias to a
contest on clean or straight cutting of wood. When Socias declined
the challenge, he was called “gago ka!” (you’re stupid) by the
deceased to which Socias responded along the same lines. Winnie
suddenly stood up and pointing his finger at Socias telling the latter
to don’t ever call him stupid. Guillermo claims that he tried to pacify
the two as they were friends but Winnie was so furious and grabbed
Socias by the collar. Socias tried to release the hold and as
Guillermo was trying to pacify the two telling them to settle the
matters peacefully, Winnie turned to him and said, “you also,” then
struck him with a bottle of beer which hit him at the right top of his
head thrice. Guillermo stood up and boxed Winnie who picked up a
bottle broke it against the wall and struck him. He stepped back,
pulled his knife and stabbed him 3 times but cannot remember which
parts of Winnie’s body was hit. A waitress at the restaurant narrated
that she saw the group of the deceased transfer to the table of
Guillermo and thereafter, an altercation occurred. She saw Winnie
strike the petitioner on the head with a bottle and then they grappled
with each other. At that point, she hid behind the refrigerator and did
not see what happened next but thereafter saw the bloodied body
of Winnie lying outside the restaurant and the petitioner’s shirt
splattered with blood. Dr. Betita, who conducted the postmortem
examination, stated that the contusion and hematoma suffered by
petitioner Guillermo was caused by a hard object like a bottle of beer
while the linear abrasion could have been caused by a fingernail.
Socias and Palma also testified in his defense, corroborating
Guillermo’s statements but averred that he did not see who stabbed
Winnie because while Guillermo and the deceased were grappling
he was busy fighting with Vicente. Palma averred that Vicente tried

326
to struck Socias but the latter managed to duck so he took the hit
instead and thereafter, he and Socias engaged Vicente.

The trial court convicted petitioner Guillermo of the crime of


homicide but acquitted Socias and Palma. Petitioner now assails the
decision of the RTC invoking self-defense.
ISSUE/S Whether or not petitioner Guillermo properly invoked the justifying
circumstance of self-defense enough to acquit him of the crime
charged?
RULING/S NO, there is no rational equivalence between the means of the
attack and the means of defense sufficient to characterize the latter
as reasonable. However, he is entitled to the mitigating
circumstance of incomplete self-defense.

The justifying circumstance of self-defense may be invoked by


anyone who acts in the defense of his person or rights provided if
the following circumstances concur: (1) unlawful aggression; (2)
reasonable necessity of the means employed to prevent or repel it;
and (3) lack of sufficient provocation on the part of the person
defending himself. Self-defense may either be complete or
incomplete. It is complete when all the three essential requisites
were represent; and it is incomplete, when the mandatory element
of unlawful aggression by the victim is present plus any of the two
essential requisites.

In the case, the first and third requisite were properly established. It
is undisputable that victim started the fight as he struck the petitioner
on the head when the latter intervened to pacify the quarrel between
the deceased and Socias. Indeed, Winnie was the unlawful
aggressor while petitioner was in the lawful act of pacifying the
quarreling parties. Likewise, as Guillermo did not provoke the fight,
being a third party to the quarrel between the original protagonists,
thus, lack of sufficient provocation was satisfied.

However, the element of reasonable means to repel the aggression


is disputable. The petitioner claims that his response to Winnie’s
attack was reasonable as he used a knife to repel an attacker armed
with a broken beer bottle. Reasonableness is a function of the
nature or severity of the attack or aggression confronting the
accused, the means employed to repel his attack. It is necessary
that the surrounding circumstances of the attack such as the place
and occasion, must show rational equivalence between the
attack and defense. That is, the means employed by a person
invoking self-defense must be reasonably commensurate to the
nature and extent of the attack sought to be averted. The nature

327
and number of wounds inflicted on the victim as important indicia
material to a plea of self-defense.

First, there is intrinsic disproportion between a Batangas knife and


a broken beer bottle. Although this disproportion is not conclusive
and may yield a contrary conclusion depending on the
circumstances. There is disproportionality because the
circumstances of the case dictate the contrary conclusion. Second,
physical evidence shows that petitioner Guillermo suffered a
contusion hematoma and 2 linear abrasions and the weapon
caused these were a beer bottle and quite possibly, fingernails as
the victim and the appellant grappled with each other. In contrast,
the victim suffered 3 stab wounds at the neck, abdomen and chest.
As such, the more plausible conclusion is that petitioner received
only one blow, not 3 as he claimed. Guillermo could not have been
a defender reeling from successive head blows inflicted by the
victim. Third, the victim and the 2 others were already drunk when
they arrived at the restaurant before the fatal fight. This state of
intoxication while not critically material to the stabbing is still material
for the purposes of defining its surrounding circumstances,
particularly the fact that a broken beer bottle might not have been a
potent weapon in the hands of a drunk wielder. Fourth, the knife
wounds were all aimed at vital parts of the body in contrast to the
petitioner’s claim that he was simply warding off broken beer bottle
thrusts and used his knife as a means commensurate to the thrusts
he avoided. The depth of these wounds shows that the force exerted
in the petitioner’s thrusts while the locations are indicative that the
thrusts were all meant to kill, not merely to disable the victim and
thereby avoid his drunken thrusts. In summary, there was no
rational equivalence between the means of the attack and the
means of defense sufficient to characterize the latter as reasonable.
Nonetheless, Guillermo is entitled for the mitigating circumstance of
incomplete self-defense.

Article 69 of the RPC provides that the penalty imposed when the
crime committed was not wholly excusable is one to two degrees
than that prescribed by law by reason of the lack of some of the
conditions required to justify the same or to exempt from criminal
liability, provided that the majority of such conditions be present.
The courts shall impose the penalty in the period which may be
deemed proper, in view of the number and conditions of the
exemption present or lacking. Since Guillermo’s please of self-
defense lacks only the element of “reasonable means”, Guillermo is
therefore entitled to the priviledged mitigating circumstance of
incomplete self-defense. Consequently, the penalty for homicide
may be lowered by one or two degrees, at the discretion of the court.

328
Thus, the penalty imposed by the RTC as affirmed by the CA was
reclusion temporal will be lowered yielding the penalty of prision
mayor. From this, the maximum of the indeterminate penalty will be
determined by taking into account modifying circumstances
applying Article 64 of same Code, which, among others provide that
(1) when neither mitigating nor aggravating circumstance are
attendant, the penalty imposed by law shall be in its medium period,
that is, from 8 years and 1 day to 10 years.

To determine the minimum of the indeterminate penalty, prision


mayor has to be reduced by one degree taking into account the
attendant modifying circumstances. Thus, prision correccional, 6
months and 1 day to 6 years; and the court is given the widest
discretion to fix the minimum of the penalty provided that the same
is within the range of the prision correccional. Thus, the
indeterminate penalty of 6 years prision correccional as minimum to
ten 10 years of prision mayor as maximum is the legally correct and
proper penalty to be imposed upon Guillermo.

329
CONTRIBUTOR MARCELINO, Ferilynn T.
MODULE Mitigating Circumstances
TOPIC
CASE TITLE PEOPLE V. URAL G.R.NO. 30801
PONENTE AQUINO, J: DATE: 27 March 1974
DOCTRINE A circumstance is considered as mitigating when the offender had
no intention to commit so grave a wrong as that committed.
FACTS Appeal of judgment from the CFI of Zamboanga del Sur convicting
appellant of murder by means of fire (incendio) (Par. 3, Art. 248,
RPC) and sentencing him to reclusion perpetua for the murder of
Felix Napola, which judgment was based on the testimony of Brigido
Alberio, who saw appellant Ural, a policeman, boxing the prisoner
Felix Napola inside the jail. As a consequence of the fistic blows,
Napola collapsed on the floor. Ura stepped on his prostrate body.
Ural went out of the cell, then went back a while later and poured
the contents of a bottle on Napola’s body. Then he ignited it with a
math and left the cell. Nobody came in to help him.

Perturbed by what he just saw, Alberto left. Before his departure,


Ural warned him to keep quiet about what he has done. A few days
after separating, Agustina contracted new relations with Wallace
Current, a corporal in the Army then went on to live with him at
Agustina’s residence.

Dr. Luzonia R. Bakil, the municipal health officer, certified that the
victim sustained 2 degree burns on the arms, neck, left side of the
nd

face, and one-half of the body including the back. She testified that
if the burns were not properly treated, death would ensue from
toxemia and tetanus infection.

Napola died on August 25, 1966, with “burn” as the cause of death.
ISSUE/S Whether or not there was a mitigating circumstance
RULING/S Yes, but the trial court failed to appreciate it – the mitigating
circumstance "that the offender had no intention to commit so grave
a wrong as that committed" (Par. 3, Art. 13, Revised Penal code). It
is manifest from the proven facts that appellant Ural had no intent to
kill Napola. His design was only to maltreat him may be because in
his drunken condition he was making a nuisance of himself inside
the detention cell. When Ural realized the fearful consequences of
his felonious act, he allowed Napola to secure medical treatment at
the municipal dispensary. Lack of intent to commit so grave a wrong
offsets the generic aggravating, circumstance of abuse of his official
position. The trial court properly imposed the penalty of reclusion
perpetua which is the medium period of the penalty for murder.

330
CONTRIBUTOR MAYUGA, Eunice Allaine G.
MODULE Article 13 - Mitigating Circumstances
TOPIC Article 14 - Aggravating Circumstances
CASE TITLE PEOPLE v. GONZALES G.R.NO. 139542
PONENTE GONZAGA-REYES, J: DATE: 21 JUNE 2001
DOCTRINE Treachery to be appreciated, two elements must concur: 1) The
employment of the means of execution that would insure the safety
of the accused from retaliatory acts of the intended victim and
leaving the latter without an opportunity to defend himself; and 2)
The means employed were deliberately or consciously adopted by
the offender.

Passion and obfuscation must be shown that: 1. an unlawful act


sufficient to produce passion and obfuscation was committed by the
intended victim; 2. the crime was committed within a reasonable
length of time from the commission of the unlawful act that produced
the obfuscation in the accused’s mind; and 3. "the passion and
obfuscation arose from lawful sentiments and not from a spirit of
lawlessness or revenge".
FACTS Noel Andres and Inocencio Gonzales were on their way to the exit
of the Loyola Memorial Park. Inocencio was driving a white Isuzu
Esteem with his grandson and three housemaids while Noel was
driving a maroon Toyota FX with his pregnant wife, Feliber Andres,
his two year old son, Kenneth, his nephew, Kevin, and his sister-in-
law, Francar Valdez. At the intersection, Inocencio was turning left
and Noel was headed straight along the road where the two vehicles
almost collided but Noel was able to timely step on the brakes.
Inocencio continued driving along his way but Noel drove behind
Inocencio’s vehicle and cut him off when he found the opportunity
to do so. Because of this incident, Noel got out of his vehicle and
knocked on Inocencio’s car window.

The prosecution’s version was that Noel cut Inocencio and got out
of the car to tell Inocencio to be careful with his driving since he is
with his family. However, Inocencio replied “Accidents are
accidents, what’s your problem.” Noel claimed that he saw
Inocencio turned red in anger so he decided to go back to his vehicle
when he was blocked by Inocencio’’s son, Dino and said “Anong
problema mo sa erpat ko.” and all of a sudden, a passenger said
“Binaril kami.”. Noel turned to his wife and saw her bloodied and
unconscious and also his son and nephew were also wounded. Noel
admitted that he and Dino were shouting at each other so he did not
hear the shot.

331
The defense’s version, however, was that Noel, upon knocking to
Inocencio’s car window, repeatedly cursed Inocencio “Putang ina
mo, ang tanda tanda mo na hindi ka pa marunong mag maneho.
Ang bobo bobo mo.” Dino, Inocencio’s son, who rode in another
vehicle decided to go back when he did not see his father’s car
behind him and when he arrived at the crime scene, he confronted
Noel and they had an altercation. When Noel suddenly reached
something inside his vehicle, Dino froze on the spot where he stood.
This prompted Inocencio to get his gun and got ready to shoot but
when he saw that Noel did not have a weapon, he put down his gun.
Trisha, Inocencio’s daughter, hugged her father and in the process
held his hand holding the gun. Inocencio tried to free his hand and
with Trisha’s substantial body weight Inocencio lost his balance and
the gun accidentally fired. Inocencio stated that he did not know he
shot somebody until Francar got out of the vehicle carrying a
bloodied small boy.

The lower court rendered judgment finding that the shooting was
attended by the qualifying circumstance of treachery and held
Inocencio guilty of the complex crime of murder for the death of
Feliber Andres and two counts of frustrated murder for the injuries
sustained by Kenneth Andres and Kevin Valdez. Hence, this
petition.
ISSUE/S 1. Whether or not treachery should be attended in this case.
2. Whether or not passion or obfuscation, lack of intention to
commit so grave a wrong, provocation or threat on the part
of the offended party immediately preceded the act,
incomplete self-defense of relative and voluntary surrender
should be appreciated in this case.
RULING/S 1. No. Treachery is defined under Article 14 par. 16 of the RPC
as the deliberate employment of means, methods or forms in
the execution of a crime against persons which tend directly
and specially to insure its execution without risk to the
offender arising from the defense which the intended victim
might raise. The suddenness of the attack, the infliction of the
wound from behind the victim, the vulnerable position of the
victim at the time the attack was made or the fact that the
victim was unarmed do not by themselves render the attack
as treacherous. For the rules on treachery to apply, the
sudden attack must have been preconceived by the accused,
unexpected by the victim and without provocation on the part
of the latter. In this case, the encounter between Noel and
Inocencio was a chance encounter. They were total
strangers before their vehicles almost collided. Thus, the
sudden attack made by Inocencio due to his infuriation by

332
reason of the victim's provocation was held to be without
treachery.
2. The mitigating circumstances of voluntary surrender, passion
and obfuscation, incomplete self defense of a relative and
lack of intent to commit so grave a wrong were not
convincingly proved. In voluntary surrender, the testimony of
prosecution said that Inocencio drove away while he was
questioning him after the shooting hence, voluntary cannot
be appreciated. Passion and obfuscation cannot be obtained
since Noel's act of shouting at Dino is not sufficient to
produce passion and obfuscation as claimed by Inocencio.
The same holds true for Inocencio's claim of provocation on
the part of Noel. Incomplete self defense, on the other hand,
is also unmeritorious since the act of Noel in cursing and
shouting at Inocencio and Dino do not amount to unlawful
aggression. And lastly, the lack of intent to commit so grave
a wrong is likewise devoid of merit. This mitigating
circumstance is obtained when there is a notable disparity
between the means employed by the accused to commit a
wrong and the resulting crime committed. Whereas, the
mitigating circumstance is not applied in this case.

333
CONTRIBUTOR PALALA, Amer B.
MODULE Mitigating Circumstances
TOPIC
CASE TITLE People v. Pagal G.R.NO. L-32040
PONENTE CONCEPCION, JR. J. DATE: October 25, 1977
DOCTRINE Provocation, in order to be a mitigating circumstance, must be
sufficient and immediately preceding the act.
FACTS Pedro Pagal and Jose Torcelino were employees Gau Guan.
Allegedly, Gau Guan maltreated them during their employment with
the latter. One night, Pedro and Jose robbed and stabbed their
employer with an ice pick and took the valuables of the victim. They
were both convicted of murder and were sentenced to suffer the
extreme penalty of death. Hence this automatic review by the
Supreme Court, when the case was called for arraignment, counsel
de oficio for the accused informed said court of their intention to
enter a plea of guilty provided that they be allowed afterwards to
prove the mitigating circumstances of sufficient provocation or threat
on the part of the offended party immediately preceding the act, and
that of having acted upon an impulse so powerful as to produce
passion and obfuscation.

ISSUE/S Whether or not the trial court erred in not appreciating the mitigating
circumstances of sufficient provocation, and passion or obfuscation
as claimed via evidence by the accused.
RULING/S No, the court held that the appellants contention is devoid of merit.
Firstly, since the alleged provocation which caused the obfuscation
of the appellants arose from the same incident, that is, the alleged
maltreatment and/or ill-treatment of the appellants by the deceased,
these two mitigating circumstances cannot be considered as two
distinct and separate circumstances but should be treated as
one. Secondly, the circumstance of passion and obfuscation cannot
be mitigating in a crime which as in the case at bar is planned and
calmly meditated before its execution. Thirdly, the maltreatment that
appellants claim the victim to have committed against them
occurred much earlier than the date of the commission of the crime.
Provocation in order to be a mitigating circumstance must be
sufficient and immediately preceding the act.

After reviewing the case, the court held that there was only one
generic aggravating circumstance, instead of four, and this was then
offset by the only accepted mitigating circumstance of the guilty
plea. Through this, the appellants were each imposed upon with the
lesser penalty of reclusion perpetua.

334
CONTRIBUTOR PANGAN, Gabrielle L.
MODULE Classes of Mitigating Circumstances
TOPIC
CASE TITLE URBANO V. PEOPLE G.R.NO. 182750
PONENTE VELASCO, JR., J: DATE: JAN 20, 2009
DOCTRINE I. Sufficient provocation on the part of the victim
When the law speaks of provocation either as a mitigating
circumstance or as an essential element of self-defense, the
reference is to an unjust or improper conduct of the offended party
capable of exciting, inciting, or irritating anyone; it is not enough
that the provocative act be unreasonable or annoying; the
provocation must be sufficient to excite one to commit the wrongful
act and should immediately precede the act.

II. Praeter Intentionem


The mitigating circumstance that petitioner had no intention to
commit so grave a wrong as that committed should also be
appreciated in his favor. While intent to kill may be presumed from
the fact of the death of the victim, this mitigating factor may still be
considered when attendant facts and circumstances so warrant.

III. Effect of two mitigating circumstances and no aggravating


circumstances
When there are two or more mitigating circumstances and no
aggravating circumstances are present, the court shall impose the
penalty next lower to that prescribed by law, in the period that it
may deem applicable, according to the number and nature of such
circumstances. (Art. 64, pg 5)
FACTS On September 28, 1993, at around 8:00 p.m., the victim Brigido
Tomelden and petitioner arrived from a picnic in the nearby where,
with some other co-workers, they drunk beer in a restaurant.

While inside the compound, the two had a heated altercation in the
course of which Tomelden hurled insulting remarks at petitioner.
Tomelden allegedly slapped and hurled insults at him, calling him
"sip-sip" just to maintain his employment as Navarro's tricycle driver.
Tomelden allegedly challenged him to a fistfight and then delivered
several fist and kick blows at petitioner, couple of which hit him
despite his evasive actions. Then petitioner delivered a "lucky
punch", as described by eyewitness Orje Salazar, on Tomelden's
face, which made Tomelden topple down. Tomelden was on the
verge of hitting his head on the ground had their companions not
caught him and prevented the fall. The blow, however, caused
Tomelden's nose to bleed and rendered him unconscious.

335
Petitioner and his other co-workers brought Tomelden to the office
of the LIWAD general manager where he spent the night. He
remained in the compound the following day. Upon arriving home at
around 6:00 p.m. of that day, Tomelden informed his wife of the fight
the previous night and of his having been rendered unconscious. He
complained of pain in his nape, head, and ear. He was diagnosed
as suffering from "brain injury, secondary to mauling to consider
cerebral hemorrhage". He had several visits to the hospital, as he
was in a state of drowsiness and vomiting.

On October 10, 1993, he seemed to be recovering, but upon


returning home, the symptoms manifested severely. They
immediately returned to the hospital. There, Tomelden died due to
"cardio-respiratory arrest secondary to cerebral concussion with
resultant cerebral hemorrhage due to mauling incident".

The defense presented petitioner who denied having any intention


to kill, asserting that hypertension, for which Tomelden was
receiving treatment, was the cause of the latter's death.
ISSUE/S Whether or not the mitigating circumstances of sufficient
provocation on the part of the victim and lack of intent to commit so
grave a wrong in favor of the petitioner should be appreciated
RULING/S The petition is meritorious where the mitigating circumstances of
sufficient provocation on the part of the victim and praeter
intentionem is concerned.
When the law speaks of provocation either as a mitigating
circumstance or as an essential element of self-defense, the
reference is to an unjust or improper conduct of the offended party
capable of exciting, inciting, or irritating anyone; it is not enough
that the provocative act be unreasonable or annoying; the
provocation must be sufficient to excite one to commit the wrongful
act and should immediately precede the act.
In the instant case, Tomelden's insulting remarks directed at
petitioner and uttered immediately before the fist fight constituted
sufficient provocation. Petitioner was the one provoked and
challenged to a fist fight by the drunken victim when he called him
“sipsip” and other derogatory remarks even before they reached
the compound. It is abundantly clear in the unrebutted testimony of
the petitioner that the provocation came from Tomelden. In fact,
petitioner, being very much smaller in height and heft, had the
good sense of trying to avoid a fight. But as events turned out, a
fisticuff still ensued, suddenly ending when petitioner's lucky punch
found its mark.
The mitigating circumstance that petitioner had no intention to
commit so grave a wrong as that committed should also be
appreciated in his favor. While intent to kill may be presumed from

336
the fact of the death of the victim, this mitigating factor may still be
considered when attendant facts and circumstances so warrant, as
in the instant case.
Petitioner tried to avoid the fight, being very much smaller than
Tomelden. He tried to parry the blows of Tomelden, albeit he was
able, during the scuffle, to connect a lucky punch that ended the
fight. Petitioner also helped carry the unconscious victim to the
office of the LIWAD's general manager. Surely, such gesture
cannot reasonably be expected from, and would be unbecoming
of, one intending to commit so grave a wrong as killing the latter. A
bareknuckle fight as a means to parry the challenge issued by
Tomelden was commensurate to the potential violence petitioner
was facing. It was just unfortunate that Tomelden died from that
lucky punch, an eventuality that could have possibly been averted
had he had the financial means to get the proper medical attention.
Thus, it is clear that the mitigating circumstance of "no intention to
commit so grave a wrong as that committed" must also be
appreciated in favor of petitioner while finding him guilty of
homicide. That petitioner landed a lucky punch at Tomelden's face
while their coworkers were trying to separate them is a compelling
indicium that he never intended so grave a wrong as to kill the
victim.
The prescribed penalty for homicide under Art. 249 of the RPC is
reclusion temporal or from 12 years and one day to 20 years. With
the appreciation of two mitigating circumstances of no intention to
commit so grave a wrong as that committed and of sufficient
provocation from the victim, and the application of par. 5 of Art. 64,
RPC, the imposable penalty would, thus, be the next lower penalty
prescribed for homicide and this should be prision mayor or from
six years and one day to 12 years.

337
CONTRIBUTOR PROVIDO, Gemy Hale A.
MODULE Mitigating Circumstance, Immediate vindication of a grave offense
TOPIC
CASE TITLE PEOPLE v. BENITO G.R.NO. L-32042
PONENTE AQUINO, J: DATE: DEC. 17, 1976
DOCTRINE To use mitigating circumstance of immediate vindication of a grave
offense, the grave offense must be done directly to the accused and
not to other people.
FACTS Alberto Benito, 26 years old and a native of Naga City, was a former
employee of the Civil Service Commision in Sampaloc, Manila and
was suspended from his office for ¨Dishonesty¨. After two months of
suspension, he was back in the office and was then charged with
Qualified Theft, Malversation of Public Funds, Estafa and
Falsification of Public Documents. He was said to have taken money
between P4000.00 to P5000.00 from the sales of examination fee.
On Dec 12, 1969, Pedro Moncayo Jr. , 36 years old and a certificied
public accountant, made a remark in front of everyone the words of
¨Nagiistambay pala dito ang magnanakaw.¨ With these words, the
petitioner then waited for him outside of their office compound later,
around 5:25 P.M. Alberto Benito followed the car of Pedro Moncayo
Jr. and when they were both in full stop, Benito shot Moncayo eight
(8) times on the head and on the diffferent parts of the body. The
multiple shots of Benito lead to the death of Moncayo. Benito used
a revolver in shooting the victim and he surrendered it to the police.
Benito contends that he did the act only because he was humiliated
by the words spoken by Moncayo.
ISSUE/S Whether or not Alberto Benito can use the mitigating circumstance
of immediate vindication of a grave offense
RULING/S No, Alberto Benito cannot use the mitigating circumstance of
immediate vindication of a grave offense. From his defense, he
contends that the reason as to why he shot Moncayo, the victim, is
due to the words or remarks he uttered on December 12, 1969 ‘-
¨Nagiistambay pala dito ang magnanakaw.¨ He argued that it was
an insult made against him. However, the Soliticor General argued
that the mitigating circumstance of immediate vindication of a grave
offense cannot be used in this case as the remarks was not made
directly against Benito and was uttered in front of everyone in the
room. Even if the court is to sssume that the remarks were directed
against Benito, it would still not be used as a mitigating circumstance
as Benito ¨had more than sufficient time to supress his emotion over
said remark if he ever did resent it.¨ from the six-hour interval of the
utterance of the words and the commission of the crime. It was also
found from the facts of the case that the reason as to why Benito
assasinated Moncayo is because of one circumstance when

338
Moncayo refused to help Benito from exposing the anomalies done
by him. Wherefore, the motion for reconsideration is denied.

339
CONTRIBUTOR ROMERO, Ma. Camille Concepcion M
MODULE Mitigating Circumstances: Immediate Vindication of a Grave
TOPIC Offense
CASE TITLE Bacabac v. People G.R.NO. 149372
PONENTE CARPIO MORALES, J. DATE: SEP. 11, 2007
DOCTRINE To appreciate mitigating circumstance of Immediate vindication of a
grave offense the act should be committed in the immediate
vindication of a grave offense to the one committing the felony
(delito), his spouse, ascendants, descendants, legitimate, natural or
adopted brothers or sisters, or relatives by affinity within the same
degree."
FACTS In the evening on December 23, 1990 Jonathan and Edzel left the
dance hall. The victim and his companion also left while on their way
they encountered Jonathan and Edzel and had a misunderstanding.

On his way home Jesus saw the commotion, he soon saw Melchor
was hugging Edzel and later tying the victim with his hands. Later
on the victim hit Edzel with a stick. He then told the victim that Edzel
was the councilor Jose’s son. Eduardo told Jesus to leave or he will
shoot him, he then left and told the councilor about the what
happened. Jonathan and Edzel flee.

While the victim and companions were headed home they met the
petitioner together with his nephews and Edzel’s family. Petitioner
and Jose were carrying M-16 armalites, while Edzel and Jonathan
were carrying a piece of wood and revolver.

Jesus pointed at the victim and his companions who manhandles


Edzel and Jonathan. The victim apologizes that he mistook them as
other persons. Jesus said "You are just bragging that you are brave.
You are only bullying small children. Bacabac fired his armalite into
the air, while Jose fired his armalite as if spraying left to right towards
the victim and Eduardo and hit Jonathan in the thigh as he was
about to strike the victim with a piece of wood. Eduardo then fell.
And so did Quidato who was in a kneeling position and raised his
hands in surrender but Jose shot him again.

Melchor was able to escape, the victim was pronounced dead on


arrival at the hospital and Eduardo died two hours later after.

The RTC convicted them of qualified treachery with mitigating


circumstance of immediate vindication for Jose and Jesus and
voluntary surrender to Bacabac.
ISSUE/S Whether or not there was conspiracy between Bacabac and Jose.

340
Whether or not the mitigating circumstance of immediate vindication
absolves him of criminal liability.
RULING/S Yes, there was conspiracy because Bacabac failed to assist the
victims after the shooting which shows conspiracy between him and
his co-accused to harm the victims. That it was he who first officially
reported the shooting
A conspirator who wants to free himself from criminal liability usually
performs an overt act to dissociate or detach himself from the felony
while the commission of the felony is in progress. Voluntary
surrender and non-flight do not conclusively prove innocence.
Bacabac’s invocation of the mitigating circumstance of immediate
vindication of a grave offense failed because for it to be credited
based on Art. 13 par. 5, the act should be committed in the
immediate vindication of a grave offense to the one committing the
felony, his spouse, ascendants, descendants, legitimate, natural or
adopted brothers or sisters, or relatives by affinity within the same
degree
The offense committed on Edzel was "hitting" his ear with a stick
(according to Jesus), a bamboo pole (according to Edzel). By
Edzel's own clarification, "[he] was hit at [his] ear, not on [his] head."
That act would certainly not be classified as "grave offense." And
Edzel is petitioner's nephew, hence, not a relative by affinity "within
the same degree" contemplated in Article 13, paragraph 5 of the
Revised Penal Code.

341
CONTRIBUTOR SAMONTE, Vanessa Antoinette
MODULE ORDINARY MITIGATING CIRCUMSTANCES- PASSION OR
TOPIC OBFUSCATION
CASE TITLE US v. AUGUSTUS HICKS A.M. NO. G.R. NO.: 4971
PONENTE TORRES, J.: DATE: SEPTEMBER 23,1909

DOCTRINE The only causes which mitigate the criminal responsibility for the
loss of self-control are such as originate from legitimate feelings, not
those which arise from vicious, unworthy, and immoral passions.

FACTS Accused appellant and the deceased victim Sola illicitly lived
together for about 5 years, until Sola left and subsequently had a
relation with another man. Appellant went to the house where Sola
and her new lover lived, and conversed with them. The latter told
appellant that Sola did not want to live with Hicks anymore, which
prompted appellant to pull out his revolver and fired at Sola, which
caused her death.

ISSUE/S Whether or not appellant can avail of the mitigating circumstance of


passion and obfuscation.

RULING/S NO. A mitigating circumstance, under the Revised Penal Code,


under Article 13 is passion or obfuscation.When such a
circumstance is present, the penalty to be imposed against the
accused may be lowered by the court if evidence so warrants.
In the case at bar, appellant’s disappointment and deliberate anger
engendered by the refusal of the woman to continue to live in illicit
relations with him and had elected to live with another man caused
alleged "passion and obfuscation.” To wit, loss of reason and self-
control produced by jealousy as alleged by the defense, inasmuch
as the only causes which mitigate the criminal responsibility for the
loss of self-control are such as originate from legitimate feelings, not
those which arise from vicious, unworthy, and immoral passions.
Thus, appellant is not entitled to the mitigating circumstance of
passion and obfuscation.

342
CONTRIBUTOR SAMSON, Jessa Viena D.
MODULE Classes of mitigating circumstances
TOPIC
CASE TITLE US v. DELA CRUZ G.R.NO. 7094
PONENTE CARSON, J.: DATE: MAR. 29, 1912
DOCTRINE Causes, which mitigate the criminal responsibility for the loss of self-
control, are such as originate from legitimate feelings, not those
which arise from vicious, unworthy, and immoral passions.
FACTS The defendant, in the heat of passion, killed his querida upon
discovering her in flagrante in carnal communication with a mutual
acquaintance.
ISSUE/S Whether or not the mitigating circumstance of passion and
obfuscation can be appreciated
RULING/S In the case at bar, the defendant was entitled to the mitigating
circumstance of passion or obfuscation.
In the case of U.S. v. Hicks, which was distinguished from the
present case, the Court held that held that the "causes which
mitigate the criminal responsibility for the loss of self-control are
such as originate from legitimate feelings, not those which arise
from vicious, unworthy, and immoral passions," and declined to give
the benefit of the provisions of this article to the convict in that case
on the ground that the alleged causes for his loss of self-control did
not "originate from legitimate feelings."
Whereas in the present case, the Court mentioned the insight taken
by the Supreme Court of Spain which states that a man who kills a
woman for having caught her in underclothes with another party and
afterwards shoots himself, inflicting a serious wound should be
responsible for that crime, but with the extenuating circumstance for
he acted as such due to strong emotion. Thus, this presents a
sufficient impulse in the natural and ordinary course to produce
violent passion and obfuscation.

343
CONTRIBUTOR TERTE, Karen A.
MODULE Mitigating Circumstance; Passion and Obfuscation
TOPIC
CASE TITLE PEOPLE v. GELAVER G.R.NO. 95357
PONENTE QUIASON, J: DATE: JUNE 9, 1993
DOCTRINE There is passional obfuscation when the crime was committed due
to an uncontrollable burst of passion provoked by prior unjust or
improper acts, or due to a legitimate stimulus so powerful as to
overcome reason. However, Act which produced the
passion/obfuscation must not be far removed from the commission
of the crime by a considerable length of time during which the
accused might have regained his normal equanimity
FACTS Appellant Eduardo Gelaver was married to Victoria Pacinabao with
whom he begot four children. They lived together at their conjugal
home until July 3, l987 when she abandoned her family to live with
her paramour. He did not know the name of his wife's paramour nor
the name of the owner of the house where his wife and her
paramour had lived together. On March 24, 1988, after he was
informed by his daughter that his wife and paramour were living at
a house in front of the Sto. Niño Catholic Church, appellant
immediately repaired to that place. Upon entering the house, he saw
his wife lying on her back and her paramour on top of her, having
sexual intercourse. Her wife’s paramour immediately stood up, took
a knife placed on top of the bedside table and attacked appellant.
The latter was able to wrest possession of the knife and then used
it against the paramour, who evaded the thrusts of the appellant by
hiding behind the victim. Thus, it was the victim who received the
stab intended for the paramour.

On March 24, 1988, accused-appellant was seen having a heated


argument with his wife, Victoria Pacinabao. Shortly after, Gelaver
dragged his wife and with a knife on his right hand, stabbed the latter
three times on the breast. Appellant then went out of the gate and
fled in the direction of the public market of Sto Niño. Appellant
confided that he continuously stabbed his wife as his mind had been
"dimmed" or overpowered by passion and obfuscation by the sight
of his wife having carnal act with her paramour.

ISSUE/S Whether or not the mitigating circumstance of passion and


obfuscation was present in this case.
RULING/S No, only the mitigating circumstance of voluntary surrender may be
appreciated.
For the mitigating circumstance of passion and obfuscation to be
considered, it is necessary to establish the existence of an
unlawful act sufficient to produce such a condition of mind. The act

344
producing the obfuscation must not be far removed from the
commission of the crime by a considerable length of time, during
which the accused might have recovered his equanimity. In the
present case, the crime was committed almost a year after the
victim had abandoned the conjugal dwelling. The act which
produced the passion/obfuscation must not be so far removed
from the commission of the crime as it is assumed that he should
have regained his reasoning.

345
CONTRIBUTOR TIDALGO, Aimee Diane A.
MODULE Mitigating Circumstances; Passion and Obfuscation
TOPIC
CASE TITLE PEOPLE V. BELLO G.R.NO. L-18792
PONENTE REYES, J.B.L., J. DATE: FEB. 28, 1964
DOCTRINE When there are causes naturally producing in a person powerful
excitement, he loses his reason and self-control, thereby
diminishing the exercise of his will power
FACTS On September 17, 1954, the accused Guillermo Bello, a widower,
who at that time was about 54 years of age, took a young peasant
lady named Alicia Cervantes, about 24 years old. However, due to
lack of substantial livelihood, Bello induced Cervantes to accept an
employment as entertainer at Maring’s Place, but entered as a
public hostess. One day, he saw Cervantes enter the Gumaca
theater with a man whom he found later was caressing his wife.
Being in love with her, he took her out from the theater and warned
her to be more discreet in her personal conduct in Gumaca.

Then, Bello went to Maring’s Place to ask for some money from
Cervantes, but Maring and Cervantes refused to give money,
Maring telling him to forget Alicia completely because he was
already an old man, an invalid and should stop bothering Alicia.

On his way home, he met the brothers Justo Marasigan and Luis
Marasigan who greeted him, Luis saying to his brother Justo the
following: 'So this is the man whose 'wife is being used by Maring
for white slave trade'. These remarks of Luis naturally brought grief
to the accused, to drown which he sought Paty's place in Gumaca
where he drank 5 glasses of tuba. From Paty's place, he went to
Realistic, in front of Maring's Place, and from there he watched the
movements of Alicia. At about 9:00PM, he entered Maring's Place
and without much ado held Alicia from behind with his left hand in
the manner of a boa strangulating its prey and with his right hand
stabbed Alicia several times with a balisong. Seeing Alicia fallen on
the ground and believing her to be mortally wounded, he fled and
went to the municipal building and there surrendered himself to the
police of Gumaca.
ISSUE/S Whether or not the victim should be given the benefit of the
mitigating circumstance of passion or obfuscation
RULING/S YES, the defense maintains that the accused is entitled to the
additional mitigating circumstance of passion and obfuscation,
because the deceased's flat rejection of petitioner's entreaties for
her to quit her calling as a hostess and return to their former relation,
aggravated by her sneering statement that the accused was

346
penniless and invalid (baldado), provoked the appellant, as he
testified, into losing his head and stabbing the deceased.

The accused's insistence that his common-law wife abandon her


work as hostess and live with him again, and his rage at her rejection
of the proposal, cannot be properly termed as arising from immoral
and unworthy passions, and therefore the accused in the case at
bar can be given the benefit of the mitigating circumstance of having
acted on a provocation sufficiently strong to produce passion and
obfuscation.

347
CONTRIBUTOR Valdez, Ariane Faye V.
MODULE Module 7- Art. 13, RPC- Mitigating Circumstances
TOPIC
CASE TITLE People v. Amaguin G.R.NO. 54344-45
PONENTE BELLOSILLO ,J. DATE: 10 January 1994
DOCTRINE For voluntary surrender to be appreciated as a mitigating
circumstance, the following elements must be present:
(a) the offender has not been actually arrested;
(b) the offender surrendered himself to a person in authority;
(c) the surrender must be voluntary.
FACTS Hernando Oro, the younger brother of Pacifico and Diosdado
narrated that in the afternoon of 24 May 1977, he and his brothers
Diosdado and Danilo, brother-in-law Rafael Candelaria, and first
cousin Sergio Argonzola were invited by their eldest brother Pacifico
to the latter’s house in Iloilo City for a small gathering to celebrate
the town fiesta. At about five o’clock in the afternoon of 14 May
1977, while walking along Divinagracia Street on their way to the
plaza for a ride home with his three brothers and two others, they
were waylaid by Celso, Willie and Gildo, their cousin Danny, all
surnamed Amaguin, and several others.
Without any provocation, Celso, with a butcher’s knife in hand,
rushed towards Pacifico. Gildo, Celso’s younger brother, with a knife
tucked to his waist, followed with a slingshot known as “Indian pana”
or “Indian target”. While Gildo aimed a dart from his slingshot at
Danilo, which hit the latter on the chest, Celso hacked Pacifico.
Gildo then stabbed Diosdado with a knife. Thereafter, Willie, the
eldest of the Amaguin brothers, appeared with a handgun and
successively shot the brothers Pacifico, Diosdado and the fleeing
Danilo. Diosdado, pleading for his life, was again shot by Willie who
next fired anew at Pacifico. Meanwhile, Gildo and Celso repeatedly
stabbed Pacifico who was already lying prostrate and defenseless.
The bloodbath resulted in the brothers Willie, Gildo and Celso, all
surnamed Amaguin, being charged with the murder of the Oro
brothers Pacifico and Diosdado. Willie and Gildo went through trial
while Celso to this date remains a fugitive.

After a joint trial, the Court of First Instance of Iloilo, found the
accused Gildo Amaguin, also known as “Tigib,” “guilty beyond
reasonable doubt of the crime of Murder. As regards Willie Amaguin
alias “Tikboy,” the trial court found him guilty “as accomplice in both
Criminal cases. Hence, this appeal.
ISSUE/S 1. Whether or not the mitigating circumstances of voluntary
surrender can be appreciated in the case at bar.

348
2. Whether or not the lower court erred in categorizing the
offense as murder there being no treachery since “the combatants
were face to face” and “confronting each other frontally x x x that
each will know each other’s next move.”

3. Whether or not there was conspiracy between Gildo and


Celso.
RULING/S 1. Yes. For voluntary surrender to be appreciated as a
mitigating circumstance, the following elements must be
present:
a. The offender has not been actually arrested;
b. The offender surrendered himself to a person in
authority; and,
c. The surrender must be voluntary.
While it may have taken both Willie and Gildo a week before
turning themselves in, the fact is, they voluntarily surrendered
to the police authorities before arrest could be effected. It can
be recalled from the case at bar that when the accused found
out that they are being looked for by the law enforcers, they
turned their selves in after five days.
Therefore, the mitigating circumstances of voluntary
surrender can be appreciated in the case at bar.
2. Yes. For treachery to be appreciated:
Treachery- the offender must employ means,
methods, or forms in the commission of the crime
which tend directly and specially to insure its
execution without risk to himself arising from the
defense which the offended party might make.
The killing of Pacifico and Diosdado cannot be qualified by
treachery. While the jurisprudence provides that, even a
frontal attack can be treacherous, as when it is sudden and
unexpected and the victim is unarmed, it appears that the
aggressors did not employ means tending directly and
specially to insure the execution of the crime without risk to
themselves arising from the defense which the offended
parties might make.
It must be noted that the assailants attacked a group of six
(6) individuals who could have been armed. It is highly
probable that at least one of those attacked could offer
resistance and could put the lives of the aggressors in
danger, as what indeed happened when accused-appellant
Gildo Amaguin and his cousin Danny suffered injuries as a

349
result of the fight which, from all indications, ended in a free-
for-all. That Pacifico sustained 15 stab wounds and a
gunshot wound, and Diosdado, ten stab wounds and a bullet
wound, does not necessarily mean that treachery attended
the killings.
Therefore, the lower court erred in categorizing the offense
as murder there being no treachery in the case at bar.
3. Yes. Under Art. 8 par. 2 of the RPC provides that:
Art. 8 par. 2- A conspiracy exists when two or more
persons come to an agreement concerning the
commission of a felony and decide to commit it.
The evidence shows how Celso and Gildo simultaneously
assaulted the Oro brothers. While Celso lunged at Pacifico,
Gildo aimed his slingshot at Danilo who was hit by its dart,
and immediately attacked Pacifico with a knife.
Under the circumstances, it is evident that Gildo and Celso
acted in unison and cooperated with each other toward the
accomplishment of a common felonious objective.
Therefore, there was conspiracy between the brothers Gildo
and Celso, and it was not necessary to prove a previous
agreement to commit the crime since from their overt acts, it
was clear that they acted in concert in the pursuit of their
unlawful design.

350
CONTRIBUTOR VILLANUEVA, Sean Ruthie
MODULE MITIGATING CIRCUMSTANCES
TOPIC
CASE TITLE THE PEOPLE OF THE PHILIPPINE G.R 45284
ISLANDS, plaintiff-appellee, vs. NO.
FRANCISCO DE LA CRUZ, ET AL.,
defendants. FRANCISCO DE LA CRUZ,
appellant.
PONENTE AVANCEÑA, C.J DATE: December
29, 1936
DOCTRINE The confession of guilt, although subsequent to the consummation
of the crime and entirely alien to its development, constitutes a
cause for the mitigation of the penalty because, as an act of
repentance and respect for the law, it indicates a moral disposition
in the accused favorable to his reform. These benefits are not
deserved by the accused who submits to the law only after the
presentation of some evidence for the prosecution, believing that in
the end the trial will result in his conviction by virtue thereof.

FACTS That on or about the 30th day of May, 1936, in the City of Manila,
Philippine Islands, the said accused Francisco de la Cruz, Fernando
Legaspi and three other persons whose identities are still unknown,
confederating together and helping one another, did then and there
wilfully, unlawfully and feloniously, and with intent of gain, attack,
assault and use personal violence upon one Yu Wan, by then and
there giving him blows with his fist on the face and other parts of the
body, thereby inflicting upon him physical injuries which have
required and will require medical attendance for a period of more
than one but less than nine days and have prevented and will
prevent the said Yu Wan from engaging in his customary labor for
the same period of time; and afterwards took, stole and carried away
with him without the consent of the owner twenty-six (P26) pesos in
cash.
During the trial and after two witnesses for the prosecution had
testified, the accused withdrew for the prosecution had testified, the
accused withdrew their plea of not guilty, substituting it by that of
guilty.

ISSUE/S Whether or not the appellant's plea of guilty constitutes a mitigating


circumstance under article 13, subsection 7, of the Revised Penal
Code

RULING/S No. The appellant's plea of guilty does not constitute a mitigating
circumstance under article 13, subsection 7, of the Revised Penal
Code, which requires that this plea be spontaneous and that it be

351
made prior to the presentation of evidence by the prosecution. The
confession of guilt, although subsequent to the consummation of the
crime and entirely alien to its development, constitutes a cause for
the mitigation of the penalty, not because it is a circumstance
modifying criminal responsibility already incurred and in the
evolution of which it has not intervened absolutely, but because, as
an act of repentance and respect for the law, it indicates a moral
disposition in the accused favorable to his reform. It is clear that
these benefits are not deserved by the accused who submits to the
law only after the presentation of some evidence for the prosecution,
believing that in the end the trial will result in his conviction by virtue
thereof.

Wherefore, eliminating the additional penalty by reason of habitual


delinquency, considering the presence of an aggravating
circumstance in the commission of the crime without any mitigating
circumstance, and applying the Indeterminate Sentence Law, the
appellant is sentenced to the penalty of from six months of arresto
mayor, as minimum, to six years, ten months and one day of prision
mayor, as maximum, affirming the appealed sentence in all other
respects, with the costs.

352
CONTRIBUT MARCELINO, Ferilynn T.
OR
MODULE Mitigating Circumstance
TOPIC
CASE TITLE MARIANO v. PEOPLE G.R.NO. 178145
PONENTE BERSAMIN, J: DATE: 07 July 2014
DOCTRINE The rationale behind the law is that the carelessness, imprudence, or
negligence may vary from one situation to another, in nature, extent,
and resulting consequences, and in order that there may be a fair and
just application of the penalty, the courts must have ample discretion
in its imposition, without being bound by what we may call the
mathematical formula provided for in Article 64 of
the Revised Penal Code.
FACTS De Leon was driving his owner type jeep along Barangay Engkanto,
Angat, Bulacan. With him were his wife, and two-year-old son. His uncle
was also driving his owner type jeep along with them. Mariano was
driving his Toyota red pickup with his wife and helper. Then Mariano
overtook De Leon’s pickup and almost bumped the latter’s car. De Leon
got mad, overtook Mariano and blocked his way. De Leon alighted his
car and approached Mariano and they had an altercation. De Leon’s
uncle tried to pacified them to which he succeeded. Instead of going
home, De Leon decided to go to his mother’s house to pick up some
items. He parked his car in front of his mother’s house and alighted the
car. However, he was bumped by a moving vehicle and thrown 4 meters
away and lost consciousness. The vehicle was identified as the same
pickup being driven by Mariano. De Leon was brought to the hospital in
Bustos, Bulacan and was later on transferred in St. Luke’s Medical
Center in QC. Subsequently, Reynaldo went to Camp Alejo S. Santos
in Malolos to surrender and report the incident.

Mariano was charged with frustrated homicide. The RTC found him
guilty of the crime charged. The CA modified his conviction to Reckless
Imprudence Resulting in Serious
Physical Injuries and failed to consider the mitigating circum
stance of voluntary surrender.
ISSUE/S Whether or not the mitigating circumstance of voluntary surrender
should be considered in the case
RULING/S NO. The mitigating circumstance of voluntary surrender cannot be
appreciated. Under Article 365 (5), it expressly states that in the
imposition of the penalties, the courts shall have their sound discretion,
without regard to the rules prescribed in Art. 64 of the RPC. In the case
of People v. Medroso, Jr., the Court explained that the rationale behind
the law is
that the carelessness, imprudence, or negligence may vary from
one situation to another, in nature, extent, and resulting

353
consequences, and in order that there may be a
fair and just application of the penalty, the courts must have am
ple discretion in its imposition, without being bound by what We may
call the mathematical formula provided for in Article 64 of the Revised
Penal Code. On the basis of this particular provision, the trial court was
not bound to apply paragraph 5 of Article 64 in the instant case even if
appellant had two mitigating circumstances in his favor with no
aggravating circumstance to offset them.

354
CONTRIBUTOR MAYUGA, Eunice Allaine G.
MODULE Article 13 - Mitigating Circumstances
TOPIC
CASE TITLE PEOPLE v. IGNAS G.R.NO. 140514-15
PONENTE QUISIMBING, J: DATE: 30 SEPT 2003
DOCTRINE Vindication of a grave offense and passion and obfuscation cannot
be claimed at the same time if they arise from the same facts or
motive. Moreover, for passion and obfuscation to be well founded,
the following requisites must concur: (1) there should be an act both
unlawful and sufficient to produce such condition of mind; and (2)
the act which produced the obfuscation was not far removed from
the commission of the crime by a considerable length of time, during
which the perpetrator might recover his moral equanimity.
FACTS June Ignas is an elementary school graduate. He resided at
Benguet where operated a bakery. Wilma Ignas, on the other hand,
is the wife of June. Wilma used to be the cashier of Windfield
Enterprise, which is owned by Pauline Gumpic who has a brother,
Nemesio Lopate. Sometime in September 1995, Wilma confided to
her close friend, Romenda Foyagao, that she was having an affair
with Nemesio. Wilma, Romenda and Nemesio went to Manila. They
were sending off Wilma as she was leaving for Taiwan to work as a
domestic helper. The three of them checked at Dangwa Inn, with
Nemesio and Wilma sharing a room until Wilma left for Taiwan.
Thereafter, Romenda received letters from Wilma although two of
them were meant to be read by Nemesio. In the other letter, Wilma
instructed Romenda to reveal her affair with Nemesio to June, her
husband. Romenda followed Wilma’s instruction and told June and
even informed him that Wilma and Nemesio even spent the night
together in a room. June became furious and declared “Addan to
aldaw na dayta nga Nemesio, patayek dayta nga Nemesio” which
means that there will be a day that he will kill Nemesio. June had a
talk with Alfred Mayamnes, the elder of Kankanaey tribe, since he
wanted to confirm whether Nemesio was having an affair with his
wife.

At around 10:00PM of 10 March 1996, according to a witness, she


suddenly heard two gunshots shattered the quiet evening and that
she saw a person falling to the ground and another person who
tucked a handgun into his waistband and casually walked away. The
witness then recognized him as June as he was her townmate and
had known him for several years. Moreover, another witness
testified that Jose disclosed to her that he had just shot his wife’s
paramour.

Police Officer Arthur declared on the stand that June voluntarily


admitted to him that he shot Nemesio with a .38 caliber revolver.

355
The trial court found June guilty of the crime of murder. The
appellant contends that the lower court should have at least
considered the mitigating circumstance of immediate vindication of
a grave offense as well as that of passion and obfuscation and lastly,
his voluntary surrender. Hence, this petition.
ISSUE/S Whether or not June Ignas is entitled to the benefits of any mitigating
circumstances.
RULING/S The court agreed with the Solicitor General which contends that
there was no immediate vindication to speak of in this case. June
Ignas has sufficient time to recover his serenity after his discovery
of his wife’s infidelity. Nor could passion and obfuscation be
appreciated since the killing was not proximate to the time of the
offense considering that there is an interval between the revelation
of his wife’s adultery and the fatal shooting was ample and sufficient
for reason and self-control to reassert in June’s mind. And lastly, as
to the mitigating circumstance of voluntary surrender, June’s
surrender at Kayapa, Nueva Vizcaya was actually due to the law
enforcers who came looking for him. There, he did not resist, but
lack of resistance alone is not tantamount to voluntary surrender,
which denotes a positive act and not merely passive conduct. In
other words, if the appellant attacked his victim in proximate
vindication of a grave offense, he could no longer claim in the same
breath that passion and obfuscation also blinded him.

Hence, the Supreme court ruled that June Ignas is guilty beyond
reasonable doubt of the crime of homicide with no aggravating or
mitigating circumstance.

356
CONTRIBUTOR PALALA, Amer B.
MODULE Mitigating Circumstances
TOPIC
CASE TITLE Bongalon v People G.R.NO. 169533
PONENTE BERSAMIN, J.. DATE: March 20, 2013
DOCTRINE Not every instance of the laying of hands on a child constitutes the
crime of child abuse under Section 10 (a) of Republic Act No.
7610. Only when the laying of hands is shown beyond reasonable
doubt to be intended by the accused to debase, degrade or demean
the intrinsic worth and dignity of the child as a human being should
it be punished as child abuse. Otherwise, it is punished under the
Revised Penal Code.
FACTS On June 26, 2000, the Prosecutor’s Office of Legazpi City charged
the petitioner in the Regional Trial Court (RTC) in Legazpi City with
child abuse, an act in violation of Section 10(a) of Republic Act No.
7610.

Petitioner was charged with child abuse, an act in violation of


Section 10(a) of RA 7610, for allegedly committing acts of physical
abuse and/or maltreatment by striking Jayson Dela Cruz (12 year
old) with his palm hitting the latter at his back and by slapping said
minor hitting his left cheek and uttering derogatory remarks to the
latter’s family, which acts of the accused are prejudicial to the child’s
development and which demean the intrinsic worth and dignity of
the said child as a human being.

Petitioner denied having physically abused or maltreated Jayson.


He explained that he only talked with Jayson and Roldan after his
minor daughters, Mary Ann Rose and Cherrylyn, had told him about
Jayson and Roldan’s throwing stones at them and about Jayson’s
burning Cherrylyn’s hair. He denied shouting invectives at and
challenging Rolando to a fight, insisting that he only told Rolando to
restrain his sons from harming his daughters. RTC and CA found
and declared petitioner guilty of child abuse.
ISSUE/S Whether or not petitioner was guilty of child abuse and that even
assuming that he was guilty, his liability should be mitigated
because he had merely acted to protect her two minor daughters.
RULING/S No, the petitioner is not guilty of child abuse. The court disagrees
with their holding that his acts constituted child abuse within the
purview of Section 3 (b) of Republic Act No. 7610.

The records did not establish beyond reasonable doubt that his
laying of hands on Jayson had been intended to debase the
“intrinsic worth and dignity” of Jayson as a human being, or that he
had thereby intended to humiliate or embarrass Jayson. The

357
records showed the laying of hands on Jayson to have been done
at the spur of the moment and in anger, indicative of his being then
overwhelmed by his fatherly concern for the personal safety of his
own minor daughters who had just suffered harm at the hands of
Jayson and Roldan. With the loss of his self-control, he lacked that
specific intent to debase, degrade or demean the intrinsic worth and
dignity of a child as a human being that was so essential in the crime
of child abuse.

Hence, the petitioner is only liable for slight physical injuries under
Article 266 (1) of the Revised Penal Code, considering that Jayson’s
physical injury required 5-7 days of medical attention.

358
CONTRIBUTOR PANGAN, Gabrielle L.
MODULE Classes of mitigating circumstances
TOPIC
CASE TITLE NIZURTADO V. G.R.NO. 107383
SANDIGANBAYAN
PONENTE VITUG, J: DATE: DEC. 7, 1994
DOCTRINE The mitigating circumstance of no intention to commit so grave a
wrong may be appreciated in the case of malversation of public
funds thru falsification of public document.
FACTS Accused-appellant was charged with the crime of malversation thru
falsification of public document.

Nizurtado, public officer, having been the Barangay Captain of


Panghulo, Malabon, Metro Manila, received and later encashed a
check for P10,000.00, specifically intended by way of a loan to the
barangay for its livelihood program; and the funds had come from
the Ministry of Human Settlements, the Metro Manila Commission
and "Kilusang Kabuhayan at Kaunlaran.

Petitioner was able to encash the check on the basis of a resolution


of the Barangay Council, to the effect that a livelihood project which
was allegedly the "T-shirt manufacturing”. The money, however,
instead of its being used for the project, was later lent to, along with
petitioner, the members of the Barangay Council. Undoubtedly, the
act constituted "misappropriation" within the meaning of the law.

Accused-appellant sought to justify the questioned act in that it was


only when the members of the Barangay Council had realized that
P10,000.00 was not enough to support the T-shirt manufacturing
project, that they decided to distribute the money in the form of loans
to themselves.

He submitted, in support thereof, a belated certification issued Chief


of District IV of the Support Staff and Malabon Sub-District Officer
of KKK, to the effect that Barangay Captains were given
discretionary authority to invest the money in any viable project not
falling within the list of project modules provided by the MHS-NCR
Management.
ISSUE/S Whether or not the mitigating circumstances of lack of intent to
commit so grave a wrong, voluntary surrender and restitution of
the amount in favor of the petitioner should be appreciated
RULING/S The petition is meritorious. Voluntary surrender may be treated as a
modifying circumstance independent and apart from restitution of
the questioned funds by petitioner. Furthermore, the petitioner had

359
no intention to commit so grave a wrong as that committed, entitling
him to three distinct mitigating circumstances.
Under Article 48 of the Revised Penal Code, when a single act
constitutes two or more grave or less grave felonies, or when an
offense is a necessary means for committing the other, the penalty
for the most serious crime shall be imposed, the same (the penalty)
to be applied in the maximum period. The penalty prescribed for the
offense of malversation of public funds in this case is prision mayor
in its maximum period to reclusion temporal in its minimum period.
Under the Indeterminate Sentence Law, the court is to impose an
indeterminate sentence, the minimum of which shall be anywhere
within the range of the penalty next lower in degree (i.e., prision
correccional in its medium period to prision correccional in its
maximum period or anywhere from two years, four months and one
day to six years) and the maximum of which is that which the law
prescribes after considering the attendant modifying circumstances.
In view of the mitigating circumstances present in this case, the fine
of P10,000.00 may also be reduced (Art. 66, Revised Penal Code)
and, since the principal penalty is higher than prision correccional,
subsidiary imprisonment would not be warranted.

360
CASE TITLE Canta v. People G.R NO. G.R. No. 140937
PONENTE MENDOZA, J. DATE:
CONTRIBUTOR GARCIA, LEXANNE O.
DOCTRINE
To be given the benefit of mitigating circumstance analogous to
voluntary surrender, there must be an intent to submit oneself
unconditionally to the authorities, showing an intention to save the
authorities the trouble and expense that his search and capture
would require.
FACT/S
Narciso Gabriel acquired from his half-sister Erlinda Monter a cow,
subject of the case, upon its birth on March 10, 1984. It appears
that at 5 o’clock in the afternoon of March 13, 1986, Agapay took
the cow to graze in the mountain of Pilipogan in Barangay
Candatag, about 40 meters from his hut. However, when he came
back for it at past 9 o’clock in the morning of March 14, 1986,
Agapay found the cow gone. He found hoof prints which led to the
house of Filomeno Vallejos. He was told that petitioner
Exuperancio Canta had taken the animal.

They were informed that petitioner had delivered the cow to his
father, Florentino Canta, who was at that time barangay captain of
Laca. Petitioner told Gardenio and Maria he would call them the
next day so that they could talk the matter over with his father.
However, petitioner never called them.

Narciso presented a certificate of ownership issued on March 9,


1986, signed by the municipal treasurer, in which the cow was
described as two years old and female.

All four caretakers of the cow identified the cow as the same one
they had taken care of. Petitioner claimed that the cow in question
was his share and that it was born on December 5, 1984. This cow,
however, was lost on December 2, 1985.

Petitioner’s Certificate of Ownership was, however, denied by the


municipal treasurer, who stated that petitioner Exuperancio Canta
had no Certificate of Ownership of Large Cattle in the municipality
of Padre Burgos.
MAIN ISSUE/S
Could the petitioner be granted a mitigating circumstance of
analogous circumstance of voluntary surrender?
RULING ON
MAIN ISSUE/S There is no question that petitioner is guilty by taking, without the
consent of the owner, the cow from the custody of the caretaker.
Petitioner falsified his Certificate of Ownership of Large Cattle.

361
Accused-appellant should be given the benefit of mitigating
circumstance analogous to voluntary surrender.
Petitioner Exuperancio Canta had not actually been arrested. In
fact, no complaint had yet been filed against him when he
surrendered the cow to the authorities. There must be an intent to
submit oneself unconditionally to the authorities, showing an
intention to save the authorities the trouble and expense that his
search and capture would require. He voluntarily took the cow to
the municipal hall of Padre Burgos to place it unconditionally in the
custody of the authorities and thus saved them the trouble of
having to recover the cow from him.

362
MODULE 8
AGGRAVATING
CIRCUMSTANCES

363
CONTRIBUTOR AGUILAR, Jose Maria L.
MODULE AGGRAVATING CIRCUMSTANCES
TOPIC
CASE TITLE PEOPLE v. GAPASIN G.R.NO. 73489
PONENTE QUIASON, J: DATE: APRIL 25, 1994
DOCTRINE Taking advantage of public position means that the offender use the
prestige, influence or ascendency of his office in the commission of
the crime or to facilitate the commission of the crime.
FACTS According to the witness, he and Rodrigo Ballad left the house of
Enteng Teppang at about 2:00 after attending the "pamisa" for the
deceased father of Teppang. Jerry Calpito (victim) followed them.
While they were walking, Calpito was shot by the appellant with an
armalite rifle. Thereafter, accused Amor Saludares planted a .22
caliber to Calpito. Upon hearing the shots, the wife of the victim ran
to succor her fallen husband.

Accused Nicanor Saludares pointed his gun at Faustina while


accused Soriano fired his gun upwards. Saludares warned that he
would kill any relative of Jerry Calpito who would come near him.
The body of Calpito was autopsied by Dr. Bernardo Layugan, who
found that the victim sustained four bullet wounds.

Appellant invoked self-defense. He testified that he was issued a


mission order to investigate a report regarding the presence of
unidentified armed men. The following day, he was instructed to get
in touch with Nicanor Saludares. When appellant met the latter, he
was informed that the victim had an unlicensed firearm.

Thereafter, when Calpito was about meters away from him,


appellant asked him what was bulging in his waist. Instead of
answering, Calpito took a step backward, drew his firearm from the
waist and fired twice at appellant. He missed because appellant
dropped to the ground simultaneously firing his armalite.
ISSUE/S Whether or not there is/are aggravating circumstance/s present in
the case.
RULING/S Yes. The supreme Court stated that treachery attended the
commission of the crime. The two conditions to constitute treachery
were present in the case at bench, to wit: (a) the employment of
means of execution that gives the person who is attacked no
opportunity to defend himself or to retaliate; and (b) the means of
execution were deliberately or consciously adopted. Appellant
deliberately executed the act in such a way that his quarry was
unaware and helpless. This can be gleaned from his act of waiting
for the victim behind the hollow-block fence of Nicanor Saludares

364
and shooting the victim from his right side. The Court also held that
treachery absorbed abuse of superior strength in this case.

Evident premeditation was indubitably proven by the evidence


showing that the execution of the criminal case was preceded by
cool thought and reflection. In view of the presence of treachery
which qualified the killing as murder, the evident premeditation
should be considered only as a generic aggravating circumstance

The Court appreciated taking advantage of public position as an


aggravating circumstance. Appellant, a member of the Philippine
Constabulary, committed the crime with an armalite which was
issued to him when he received the mission order.

Voluntary surrender may be considered in appellant's favor, but this


is offset by the aggravating circumstance of taking advantage of
public position. Therefore, only the generic aggravating
circumstance of evident premeditation may be appreciated against
appellant. Hence, the proper penalty is reclusion perpetua.

365
CONTRIBUTOR ALGURA, Nino N.
MODULE Aggravating Circumstances
TOPIC
CASE TITLE THE PEOPLE OF THE G.R.NO. L-35123-24
PHILIPPINES
V
RUDY TIONGSON
PONENTE CONCEPCION JR., J.: DATE: July 25, 1984
DOCTRINE The aggravating circumstance that the crimes were committed in
contempt of or with insult to the public authorities cannot also be
appreciated since Pat. Gelera and PC Constable Canela were the
very ones against whom the crime were committed. Besides, Pat.
Gelera and PC Constable Canela are not persons in authority, but
merely agents of a person in authority.
FACTS 1. Crim. Case No. R-DJC-243:
That on the 26th day of October, 1971, at 6:00 o'clock
in the evening, more or less, at Rizal, of the
Municipality of Bulalacao, Province of Oriental
Mindoro, Philippines, and within the jurisdiction of this
Honorable Court, the above-named accused, RUDY
TIONGSON, conspiring and confederating with
Rolando Santiago and George de la Cruz, who are
both at large by reason of their forced escape, and
with treachery, wilfully, unlawfully and feloniously
waited in ambush, waylaid and shot one C2C
AURELIO M. CANELA, a member of the local
Philippine Constabulary Command, while the latter
was in hot pursuit of said accused who had earlier
escaped from custody, thus fatefully resulting to the
instantaneous death of the victim.
That the commission of the offense was qualified by
the circumstance of treachery, and aggravated by the
circumstances of evident premeditation, in contempt
of or with ingult to the public authorities, nocturnity,
committed in an uninhabited place and with abuse of
superior strength.
2. Crim. Case No. R-DJC-244
That on the 26th day of October, 1971, at 5:30 o'clock
in the afternoon, more or less, inside of the Municipal
Building, of the Municipality of Bulalacao, Province of
Oriental Mindoro, Philippines, and within the
jurisdiction of this Honorable Court, the above-named
accused, RUDY TIONGSON, conspiring and
confederating with George de la Cruz and Rolando

366
Santiago, and under the pretext that they would
answer the call of nature, convinced Police First Class
Patrolman Zosimo Gelera to allow them to go out from
their being confined and detained in the Municipal Jail
of same Municipality by virtue of a previous offense,
and while still hardly out of said jail ganged up said
Zosimo Gelera, took the latter's service pistol and with
it, with treachery, shot point blank said police officer at
his right cheek, tragically resulting to the victim's
instantaneous death and thereafter, made good their
escape.
That the offense is qualified by the circumstance of
treachery, and aggravated by the circumstances of
evident premeditation, in contempt of or with insult to
the public authorities and with abuse of superior
strength.
ISSUE/S Whether or not there is treachery.
Whether or not there is an aggravating circumstance.
RULING/S The treachery cannot be appreciated in this case and had been
sufficiently forewarned of the presence of the appellant in the vicinity
and that he was not completely deprived of an opportunity to
prepare and repel or avoid the aggression. Aggravating
circumstances of evident premeditation, in contempt of or with insult
to public authorities, uninhabited place, and abuse of superior
strength were not present in the commission of the crimes. Evident
premeditation must be ruled out in view of the absence of sufficient
proof that a plan to kill the victims existed, the execution of which
was preceded by deliberate thought and reflection. Besides, with
respect to the killing of PC Constable Canela, only ten minutes
passed from the time the accused escaped from the Municipal Jail
up to the time he shot PC Constable Canela near the cemetery, 14
so that there was no lapse of time during which he could have
deliberately planned the killing of the said PC Constable and
meditated on the consequences of his act.

The lower court also found that the killing of PC Constable Canela
was committed in an uninhabited place. It has not been shown,
however, that the offense was committed in an isolated place, far
from human habitation. In order that the aggravating circumstance
of the commission of a crime in an uninhabited place may be
considered, it is necessary that the place of occurrence be where
there are no houses at all, a considerable distance from the village
or town, or where the houses are a great distance apart.

367
Finally, the aggravating circumstance of abuse of superior strength
must also be ruled out since there is no direct evidence that the
accused employed superior strength in the killing of Pat. Gelera.
The accused was then a detainee and was unarmed while Pat.
Gelera had his service pistol with him. With respect to PC
Constable Canela, the accused was alone against three armed
pursuers, namely: PC Sgt. Saway, PC Constable Canela, and Pat
Nicandro Garcia, and a civilian by the name of Fred Barcelona.
The accused is guilty only of the crime of Homicide in the killing of
PC Constable Canela and Pat. Gelera.

368
CONTRIBUTOR AROZA, Maria Minette R.
MODULE Aggravating Circumstances - Treachery
TOPIC
CASE TITLE PEOPLE v. MAGDUEÑO G.R.NO. L-68699
PONENTE PER CURIAM DATE: SEPT. 22, 1986
DOCTRINE There is treachery when the manner of the execution was such
that the appellant deliberately and consciously adopted means and
ways of committing the crime and insured its execution without risk
to himself arising from any defense from the victim.
FACTS On October 15, 1980, around 8:00 o’clock in the morning, while
Fiscal Fernando M. Dilig had placed himself at the driver’s seat
inside his jeep parked near his house in Puerto Princesa City, all of
a sudden, two successive gunshots burst into the air inflicting two
fatal wounds at the neck and lower back of the victim. Before the
Accused-appellant shot the victim, he called out “Fiscal” and
immediately proceeded with the act rendering the victim no
opportunity to defend himself.
Three eyewitnesses positively identified the perpetrator as
Hermogenes Magdueño:
1. Elena Adion Lim, she was sitting at the gate of her fence,
about 20 to 30 meters away from the house of Fiscal Dilig,
saw the gunman bringing a short gun in his right hand and a
clutch bag;
2. Ernesto Mari Y Gonzales, security guard who was on board
a tricycle, passing in front of the house of Fiscal Dilig, on his
way home, likewise heard the two gunshots coming from the
direction of Fiscal Dilig’s house, prompting him to order the
driver to stop. He described the gunman as wearing a white
polo shirt, blue pants and a hat, and when the gunman turned
to his left side, he saw a scar on his left temple below his left
eyebrow. The man was still holding the gun in his right hand
while walking in a limping manner;
3. Cynthia Canto, a taxi dancer, while in front of a store near the
victim’s house, waiting for a tricycle, saw the gunman firing
two successive shots to the victim.
Magdueño also executed an extra-judicial confession wherein he
admitted that he killed Fiscal Dilig for a price or reward and
implicated Leonardo Senas and Mauricio de Leon as the
mastermind to the commission of the crime but were later dropped
from the amended information for lack of a prima facie case against
them. A former inmate in Muntinlupa also testified that the accused-
appellant was known as a killer while in prison and that he saw him
sometime in October at the gate of Palawan Apitong.
ISSUE/S Whether or not the court a quo erred in convicting the accused for
murder qualified by treachery.

369
RULING/S The Supreme Court held that the lower court did not err in convicting
the accused for murder. The appellant fired two successive shots at
the defenseless Fiscal Dilig while the latter was still seated in his
jeep, hitting him at the neck and lumbar region, both wounds were
fatal. The manner of the execution was such that the appellant
deliberately and consciously adopted means and ways of
committing the crime and insured its execution without risk to
himself arising from any defense Fiscal Dilig might make. The two
conditions necessary for treachery to exist are present. The fact that
the appellant called out, "Fiscal" before shooting the victim killer, he
wanted to insure that he was shooting the correct person. When
Dilig turned his face to find out who was calling him, the appellant
fired immediately rendering no opportunity for Dilig to defend
himself.

370
CONTRIBUTOR Belano, Renato Jr. P.
MODULE Aggravating Circumstances
TOPIC
CASE TITLE People v. Tac-an G.R. NO. 76338-89
PONENTE Feliciano, J. DATE: Feb 26, 1990
DOCTRINE/ Treachery - The cumulative effect of the circumstances
PRINCIPLES underscored by the trial court was that the attack upon the victim
had been carried out in a manner which disabled the victim from
defending himself or retaliating against the accused. The
circumstance that the accused, having been informed that the
victim was still alive, re-entered the room and fired again at the
victim who lay on the floor and bathed with his own blood,
manifested Renato's conscious choice of means of execution
which directly and especially ensured the death of his victim
without risk to himself.

Evident Premeditation - In order that evident premeditation may be


taken into account, there must be proof of (a) the time when the
offender formed his intent to commit the crime; (b) an action
manifestly indicating that the offender had clung to his
determination to commit the crime; and (c) the passage of a
sufficient interval of time between the determination of the offender
to commit the crime and the actual execution thereof, to allow him
to reflect upon the consequences of his act.

In contempt or with insult to public authorities - Because a penal


statute is not to be given a longer reach and broader scope than is
called for by the ordinary meaning of the ordinary words used by
such statute, to the disadvantage of an accused, we do not believe
that a teacher or professor of a public or recognized private school
may be regarded as a "public authority" within the meaning of
paragraph 2 of Article 14 of the Revised Penal Code.

P.D. No 1866 (Illegal Possession of Firearms and Bullets) - There


is nothing in P.D. No. 1866 which suggests that it was intended to
remain in effect only for the duration of the martial law imposed
upon the country by former President Marcos. Neither does the
statute contain any provision that so prescribes its lapsing into
non-enforceability upon the termination of the state or period of
martial law.

The killing having been done under the influence of a dangerous


drug - In the absence of competent medical or other direct
evidence of ingestion of a dangerous drug, courts may be wary
and critical of indirect evidence, considering the severe

371
consequences for the accused of a finding that he had acted while
under the influence of a prohibited drug.
FACTS Appellant Renato Tac-an was Francis Ernest were classmates from
the Divine Word College in Tagbilaran City. They were close friends
and members of the same gang, the Bronx gang.

Francis withdrew from the Bronx gang. The relationship between


Renato and Francis turned sour. They would sometime quarrel with
each other wherein one leading both of them being brought to the
high school Principal’s Office. Their strained relationship was later
aggravated when Francis had learned that Renato, together with
other members of the Bronx gang, was looking for him, apparently
with the intention of beating him up. Further deterioration when a
graffiti appeared on the wall of their high school classroom and on
the armrest of a chair in that classroom deprecating the Bronx gang
and describing Renato as “bayot” (homosexual). Renato attributed
the graffiti to Francis.

At about 2:00 o'clock in the afternoon of 14 December 1984, Renato


entered Room 15 of the high school
building to attend his English III class. Renato placed his scrapbook
for their Mathematics class on his chair and approached the
teacher. Upon returning to his chair, he found Francis sitting on his
scrapbook. Renato was angered by what he saw and promptly
kicked the chair on which Francis was seated. A fistfight would have
ensued, but some classmates and two teachers prevented them
from assaulting each other. The two quieted down and shook hands
and the teacher resumed her English III class. While the English III
class was still going on Renato slipped out of the classroom and
went home to get a gun. He was back in the classroom
approximately fifteen minutes later.

While the Mathematics class was on going Renato suddenly burst


into the room, shut the door and with both hands raised, holding a
revolver shouted, “where is Francis”. Renato fired at Francis three
times with his fourth shot hitting Francis on the head. After the
shooting the teacher from the other room without knowing Renato
was the shooter asked Renato to assist Francis. Renato entered the
room knowing Francis was still alive and shot him again from the
back below the right shoulder and exited on his front chest just
above the right nipple.

Renato then proceeded to the faculty room where he found some


teachers and students. Renato ordered them to lock the room and
close the windows, in effect holding them as hostages. He also
reloaded his gun with five bullets. After some time, a team of the

372
Philippines Constabulary troopers arrived and ordered him to
surrender. His brother was successful in convincing him to do so.
ISSUE/S Whether or not the aggravating circumstances of treachery, evident
premeditation, in contempt of or insult to public authorities, illegal
possession of firearms and bullets and the killing having done under
the influence of a dangerous drug should be appreciated against the
accused
RULING/S On treachery (yes)
The trial court made a finding of treachery taking explicit account of
the following factors:
• Room 15 of the Divine Word College, High School
Department, Tagbilaran City, is situated in the second floor
of the building. It is a corner room and it has only one (1)
door which is the only means of entry and exit;
• At the time of the attack, the deceased was seated on his
chair inside his classroom and was writing on the armrest of
his chair and also talking to Ruel Ungab and while their
teacher, Mr. Damaso Pasilbas was checking the
attendance. The deceased was not aware of any impending
assault neither did he have any means to defend himself.
• The accused used an airweight Smith & Wesson .38 caliber
revolver in shooting to death the defenseless and helpless
Francis Ernest Escaño;
• The attack was so sudden and so unexpected. The accused
consciously conceived that mode of attack;
• The accused fired at Francis again and again and did not
give him a chance to defend himself. After the deceased
was hit on the head and fell to the floor while he was
already sprawled and completely defenseless the accused
fired at him again and the deceased was hit on the chest;
• The deceased was not armed. He was totally defenseless.
He was absolutely not aware of any coming attack."

The cumulative effect of the circumstances underscored by the


trial court was that the attack upon Francis had been carried out in
a manner which disabled Francis from defending himself or
retaliating against Renato. The circumstance that Renato, having
been informed that Francis was still alive, re-entered Room
15 and fired again at Francis who lay on the floor and bathed with
his own blood, manifested Renato's conscious choice of means of
execution which directly and especially ensured the death of his
victim without risk to himself. We are compelled to agree with the
trial court that treachery was here present and that, therefore, the
killing of Francis Ernest Escaño III was murder. We are compelled

373
to agree with the trial court that treachery was here present and
that, therefore, the killing of Francis Ernest Escaño III was murder.

On evident premeditation (no)

In order that evident premeditation may be taken into


account, there must be proof of

a. the time when the offender formed his intent to commit the
crime;
b. an action manifestly indicating that the offender had clung to
his determination to commit the crime; and
c. the passage of a sufficient interval of time between the
determination of the offender to commit the crime and the actual
execution thereof, to allow him to reflect upon the consequences of
his act.
While there was testimony to the fact that before that fatal day
of 14 December 1984, anger and resentment had welled up
between Francis and Renato, there was no evidence adequately
showing when Renato had formed the intention and determination
to take the life of Francis. Accordingly, we must discard evident
premeditation as an aggravating circumstance.

On contempt of or with insult to the public authorities (no)

We believe the trial court erred in so finding the presence of a


generic aggravating circumstance.

Article 152 of the Revised Penal Code, as amended by Republic Act


No. 1978 and Presidential Decree No. 299, provides
as follows:
xxx
Art. 152. Persons in authority and agents of persons in authority.
Who shall be deemed as such. In applying the provisions of the
preceding and other articles of this Code, any person directly vested
with jurisdiction, whether as an individual or as a member of some
court or government corporation, board, or commission, shall be
deemed a person in authority. A barrio captain and a barangay
chairman shall also be deemed a person in authority.
xxx
In applying the provisions of Articles 148 and 151 of this Code,
teachers, professors and persons charged with the supervision of
public or duly recognized private schools, colleges and universities,
and lawyers in the actual performance of their professional duties or

374
on the occasion of such performance, shall be deemed persons in
authority.
xxx

Careful reading of the last paragraph of Article 152 will show that
while a teacher or professor of a public or recognized private school
is deemed to be a "person in authority," such teacher or professor
is so deemed only for purposes of application of Articles 148 (direct
assault upon a person in authority), and 151 (resistance and
disobedience to a person in authority or the agents of such person)
of the Revised Penal Code. In marked contrast, the first paragraph
of Article 152 does not identify specific articles of the Revised Penal
Code for the application of which any person "directly vested with
jurisdiction, etc." is deemed "a person in authority." Because a penal
statute is not to be given a longer reach and broader scope than is
called for by the ordinary meaning of the ordinary words used by
such statute, to the disadvantage of an accused, we do not believe
that a teacher or professor of a public or recognized private
school may be regarded as a "public authority" within the
meaning of paragraph 2 of Article 14 of the Revised Penal Code.

On illegal possession of Firearms and Bullets [P.D. No. 1866]


(yes)

Appellant urges that P.D. No. 1866 is inapplicable to him


"considering that the reason for its [P.D. No. 1866] issuance no
longer exists." He argues that P.D. No. 1866 was enforceable only
during the existence of martial law, and that when martial law was
"lifted in 1979,".

There is nothing in P.D. No. 1866 which suggests that it was


intended to remain in effect only for the duration of the martial
law imposed upon the country by former President Marcos.
Neither does the statute contain any provision that so prescribes its
lapsing into non-enforceability upon the termination of the state or
period of martial law. On the contrary, P.D. No. 1866 by its own
terms purported to "consolidate, codify and integrate" all prior laws
and decrees penalizing illegal possession and manufacture of
firearms, ammunition and explosives in order "to harmonize their
provisions," as well as to update and revise certain provisions and
prior statutes "in order to more effectively deter violators of the law
on firearms, ammunitions and explosives."

On the killing being done under the influence of a dangerous


drug

375
The prosecution had presented Orlando Balaba, a student at the
Divine Word College, High School Department, who testified that he
found Renato and one Jaime Racho inside the men's room of the
High School Department sucking smoke from a hand-rolled thing
that look like a cigarette, that he had asked Renato what that was
and that Renato had replied "damo" (marijuana). While the
testimony of Orlando Balaba was corroborated by two (2) other
prosecution witnesses, we believe that Orlando Balaba's testimony
was incompetent to show that what Renato and Jaime Racho were
smoking inside the men's room was indeed marijuana. It was
pointed out by appellant that Orlando Balaba had never smoked nor
smelled marijuana.

In the absence of medical evidence, the trial court took into account
certain detailed factors as circumstantial evidence supporting the
testimony of Orlando Balaba. These circumstances were:
"The circumstance of place where the killing was committed, the
circumstance of the manner of the attack, the circumstance of
holding hostage some teachers and students inside the faculty
room, the circumstance of terrifying an entire school, the
circumstance that sitting on a scrapbook is too insignificant as to
arouse passion strong enough to motivate a killing, are
circumstantial evidences that gave the court no room for doubt that
prosecution witnesses Orlando Balaba, Benjamin Amper and Allan
de la Serna truthfully told the court that they saw the accused
smoking marijuana inside the comfort room at 1:45 in the afternoon
of December 14, 1984 . . .""

The above circumstances pointed to by the trial court may be


indicative of passionate anger on the part of Renato; we do not
believe that they necessarily show that Renato had smoked
marijuana before entering his English III class. In the absence of
competent medical or other direct evidence of ingestion of a
dangerous drug, courts may be wary and critical of indirect
evidence, considering the severe consequences for the accused of
a finding that he had acted while under the influence of a prohibited
drug.

376
CONTRIBUTOR BILTZ, Aralind Louise A.
MODULE Aggravating Circumstances
TOPIC
CASE TITLE PEOPLE vs. DIAZ G.R.NO. L-24002
PONENTE AQUINO, J. DATE: JANUARY 21, 1974
DOCTRINE A person who acts under the compulsion of an irresistible force, like
one who acts under the impulse of an uncontrollable fear of equal
or greater injury, is exempt from criminal liability because he does
not act with freedom. Actus me invito factus non est meus actus. An
act done by me against my will is not my act. The force
contemplated must be so formidable as to reduce the actor to a
mere instrument who acts not only without will but against his will.
The duress, force, fear or intimidation must be present, imminent
and impending, and of such nature as to induce a well-grounded
apprehension of death or serious bodily harm if the act be done. A
threat of future injury is not enough. The compulsion must be of such
a character as to leave no opportunity for the accused for escape or
self-defense in equal combat.
FACTS Remegia Carasos, 14-year old, and her cousin Anita Pacaira, 11-
years old were gathering camotes in a farm. Suddenly, Fracisco
Diaz, a 24-year old unmarried farmer whom the two girls had known
for many years, embraced Remegia from behind and against her
will, held her breast. Remegia shouted and asked for help from
Anita. Reacting to Remegia’s cry for help, Anita struck Francisco on
the head and hands with a Bolo and he released Remegia.

The two girls left the farm and reported the incident to Quintin Tadia,
their grandfather. Tadia immediately reported the incident to the
barrio lieutenant.

On the following day, Tadia, accompanied by his granddaughters,


was on his was to the población to file a complaint. He was unarmed
and while they were ascending the hill, Diaz and his brother Gerardo
appeared on the hill. Gerardo was armed with a bardog, a locally-
made shotgun. He fired sidewise at Tadia hitting him in the neck. He
rolled down then the brothers jumped to the lower part of the cliff.
Gerardo told his brother: “Go ahead, Francisco, stab that fellow”.
Francisco stabbed Tadia on different parts of his body with a bolo.
ISSUE/S Whether treachery and evident premeditation should qualify the
killing as murder
RULING/S YES.
The crime committed by the appellants is murder qualified by
treachery as alleged in the information. There was treachery
(alevosia) because the brothers made a deliberate surprise or
unexpected assault on Tadia. They literally ambushed him. They

377
waited for him on the cliff, a high ground which rendered it difficult
for him to flee or maneuver in his defense. Tadia was shot sidewise
while he was ascending the hill or cliff burdened by his catopis or
food basket. That was another circumstance which handicapped
him in resisting the assault. The initial attack was successful. Tadia
fell and rolled down the cliff and landed near the creek below. In that
helpless state, he was ruthlessly stabbed by Francisco Diaz.
The appellants resorted to means of execution which directly and
specially insured the killing without any risk to themselves arising
from any defense which the victim could have made. Actually, he
was not able to make any defense, unarmed and attacked unaware
as he was. The treacherous mode of attack is incontrovertible (Par.
16, Art. 14 and Art. 248, Revised Penal Code).
The attack was also attended with abuse of superiority. Two armed
young men unexpectedly assaulted an unarmed sexagenarian.
However, abuse of superior strength is merged with treachery.

378
CONTRIBUTOR CACHERO, Luis III L.
MODULE Aggravating Circumstances
TOPIC
CASE TITLE PEOPLE V CLARITO G.R.NO. 135051-52
ARIZOBAL (AT LARGE), ET
AL.
PONENTE PER CURIAM DATE: December 14,
2000
DOCTRINE Aggravating circumstance of by band; Whenever more than three
armed malefactors shall have acted together in the commission of
an offense, it shall be deemed to have committed by a band.
FACTS On August 12, 1994, two (2) separate Informations were filed before
the Regional Trial Court of Cataingnan, Masbate, charging Clarito
Arizobal, Erly Lignes, Rogelio Gemino and two (2) John Does with
Robbery In Band with Homicide for robbing and killing Laurencio
Gimenez and his son Jimmy. The two (2) Informations filed against
Rogelio Gemino were dismissed due to lack of evidence.

Clemencia Gimenez, wife of victim Laurencio Gimenez testified that


on March 24, 1994, She, together with her husband and a
grandchild were sound asleep in their house in Tuybo, Cataingan,
Masbate. She was suddenly confronted by three (3) armed men
pointing their guns at her. She recognized two (2) of them as Clarito
Arizobal and Erly Lignes but failed to recognize the third person who
was wearing a mask. The man wearing the mark stoof guard at the
door, Clarito and Lignes barged into the master’s bedroom and
forcibly opened the aparador. The intruders ransacked their cabinet
and scattered everything on the floor until they found Php 8,000
among sheets of paper. Before leaving with their loot, they ordered
Laurencio to go with them to Jimmy’s house because they had
something to talk about, and Laurencio complied against his will.
Clementina recalled that shortly after the group left she heard
gunshots. Her grandchild could only mutter in fear “Lolo is already
dead!”
Alleged in the Information were the following aggravating
circumstances: 1) By a band because the malefactors were more
than three armed robbers acting together; 2) Treachery because the
robbers tied the hands of the victims before killing them; 3)
Nocturnity; 4) Dwelling
The lower court found both accused Clarito Arizobal and Erly Lignes
guilty of Robbery with Homicide and sentenced them the supreme
penalty of Death.
ISSUE/S Whether or not the aggravating circumstances alleged in the
Information shall be appreciated.

379
RULING/S The trial court did not appreciate the aggravating circumstances of
band, nocturnity, and treachery, but correctly appreciated dwelling.
Robbery by band was not appreciated because there were only
three armed men in the commission of the crime. Paragraph 6 of
Article 12 of the Revised Penal Code specifically states that
“Whenever more than three armed malefactors shall have acted
together in the commission of an offense, it shall be deemed to have
been committed by a band”. Thus, in order for this aggravating
circumstance to be appreciate, there shall be the presence of at
least four armed malefactors.
Nocturnity was likewise not appreciated because it must be shown
that nighttime was deliberately and intentionally sought by accused-
appellants to help them realize their evil intentions. On the contrary,
the locus criminis was well lighted and nighttime was merely and
incidental element to the whole drama.
Treachery was also not appreciated by the High Court. Robbery with
Homicide is a special complex crime and is primarily classified as a
crime against property, and not against persons. Treachery can only
be appreciated if the crime is against persons.
Dwelling was correctly appreciated in this case. The robbers
demonstrated an imprudent disregard of the inviolability of the
victims’ abode when they forced their way in, looted their houses,
intimidated and coerced their inhabitants into submission, disabled
Laurencio and Jimmy by tying their hands before dragging them out
of the house to be killed.
The Supreme Court affirmed the lower court’s ruling with regard to
the imposed penalty of death.

380
CONTRIBUTOR CALZADO, Anne Valerie L.
MODULE Aggravating Circumstances
TOPIC
CASE TITLE PEOPLE v. DANIEL G.R.NO. L-40330
PONENTE MUÑOZ-PALMA, J. DATE: NOVEMBER 20, 1978
DOCTRINE It is not necessary, under the law, that the victim owns the place
where he lives or dwells. Be he a lessee, a boarder, or a bed-spacer,
the place is his home the sanctity of which the law seeks to protect
and uphold.
FACTS On September 20, 1965, Margarita, a 13-year old native of Mt.
Province, arrived in Baguio City from Tublay in a Dangwa bus. She
was then en route to her boarding house in Guisad as she was a
highschool student at the Baguio Eastern High school. While she
was waiting inside the bus, the accused Daniel came and started
molesting her by inquiring her name and getting hold of her bag.
This prompted Margarita to transfer to a jeepney where the accused
followed her until she alighted in Guisad.

Reaching her boarding house, she opened the door and was about
to close it when the accused dashed in and closed the door behind
him. He pulled a dagger 8 inches long and threatened her saying,
“If you talk, I will kill you.” From thereon, the crime of rape was
committed.
ISSUE/S Whether or not a boarding house falls within the definition of the
aggravating circumstance of committing a crime in a “dwelling”
RULING/S Yes.

Although Margarita was merely renting a bed space in a boarding


house, her room constituted for all intents and purposes a “dwelling”
as the term used in Art. 14 (3) RPC. The Court said that it is not
necessary under the law, that the victim owns the place where he
lives or dwells. Be he a lessee, a boarder, or a bed-spacer, the place
is his home the sanctity of which the law seeks to protect and
uphold.

381
CONTRIBUTOR CARPIO, Anna Clarissa C.
MODULE Module 8: Aggravating Circumstances
TOPIC
CASE TITLE PEOPLE v. APDUHAN G.R.NO. L-19491
PONENTE Castro, J. DATE: August 30, 1968
DOCTRINE All trial judges must refrain from accepting with alacrity an accused’s
plea of guilty, for while justice demands a speedy administration,
judges are duty bound to be extra solicitous in seeing to it that when
an accused pleads guilty, he understands fully the meaning of his
plea and the import of an inevitable conviction.
FACTS Defendant-appellant Apolonio a.k.a. Junior Apduhan was convicted
of robbery with homicide and was sentenced to death of Geronimo
Miano. During his arraignment, Apduhan and his co-accused
Rodulfo Huiso and Felipe Quimson pleaded “not guilty”. The
Information provides at about 7:00 p.m. in May 1961 at Bohol,
Apduhan, Huisio and Quimson, together with 5 other persons whose
real names are still unknown and remain at large, all of them armed
with different unlicensed firearms, daggers and other deadly
weapons, in conspiracy, entered by means of violence, the dwelling
of spouses Honorato and Antonia Miano, shot Geromino and
Norberto Aton who happened to be also inside, inflicting physical
injuries which caused their death; and thereafter, took and carried
away the spouses’ cash amounting to Php 320.00. The act was
committed with the following aggravating circumstances: by a band,
with the use of unlicensed firearms, dwelling, nightime, and abuse
of superior strength.

During trial, Atty. Tirol declared that he is only appearing as counsel


de officio for Apduhan and thereafter manifested that Apduhan will
change his former plea of “not guilty” to a plea of “guilty”. The record
shows that after trial, the judge repeatedly apprised Apduhan of the
severity of the offense and the strong possibility that the capital
penalty might be imposed upon him despite his plea of guilt but
Apduhan persisted with the request that death penalty be not
imposed upon him. After hearing the prosecutor’s arguments, the
judge advised Apduhan anew but the latter persisted and reiterated
his specific condition of life imprisonment. Noteworthy is Apduhan’s
inconsistent replies when asked, (1) if it was true that he was
withdrawing his “not guilty” plea; and (2) if he was conscious of the
fact that despite his guity plea, the death penalty may still be
imposed, both to which he reverted, “I will JUST enter the plea of
guilty.” Eventually, Apduhan desisted the guilty plea and let his
previous “not guilty” plea stand on record.

The RTC found the Apduhan’s guilty plea was ambiguous and not
definite hence, the case was reopened and the latter again entered

382
a categorical plea of guilty stating “I enter the plea of guilty.”
Thereafter, the defense presented 3 mitigating circumstances to wit:
(1) voluntary plea of guilty; (2) intoxication; and (3) lack of intent to
commit so grave a wrong [praeter intentionem]. Subsequently, the
defense withdrew praeter intentionem after the prosecution had
withdrawn the aggravating circumstance of abuse of superior
strength. The court rendered a decision finding Apduhan guilty of
the complex crime of robbery with homicide and sentenced him to
suffer the death penalty but given the voluntary plea of guilty by
Apduhan, the trial judge recommended to the President of the
Republic for the commutation of the death sentence imposed to life
imprisonment. The Solicitor General supported the
recommendation for executive clemency.
ISSUE/S Whether or not Apduhan’s voluntary plea of “guilty” is spontaneous
and insistent?
RULING/S NO, the Supreme Court found no compelling reason to justify such
recommendation. Contrary to the trial court’s observation,
Apduhan’s plea of guilty was far from spontaneous and insistent.
His initial plea was one of “not guilty” which he later changed to
“guilty” but with the persistent condition that he be sentenced to life
imprisonment, not death. It was only after much equivocation that
he finally decided to “just” plead guilty. As the plea was found to be
ambiguous, the case was reopened. Conceding that such was
“spontaneous” and “insistent”, such manifestation of sincere
repentance cannot severe to obliterate the attendant aggravating
circumstances which patently reveal the accused’s criminal
perversity.

Judge Alo was commended for his earnest and patient efforts to
forestall the entry of an improvident plea of guilty by accused
Apduhan, notwithstanding that the latter was already represented
by a counsel de officio and hence presumed to have been advised
properly. The judge made sure the accused understood clearly and
fully the seriousness of the offense charged and the severity of the
penalty attached to it, he was warned that death penalty might be
imposed despite his guilty plea and when the plea was ambiguous,
the case was reopened to determine the definitiveness of the nature
of his plea.

The virtue of Judge Alo’s efforts in ascertaining whether Apduhan


pleaded guilty with full knowledge of the significance and
consequences of his act, recommends itself to all trial judges who
must refrain from accepting with alacrity an accused’s plea of guilty,
for while justice demands a speedy administration, judges are duty
bound to be extra solicitous in seeing to it that when an accused

383
pleads guilty, he understands fully the meaning of his plea and the
import of an inevitable conviction.

While the Supreme Court ruled that there is no justification at all for
executive clemency due to the language of the presiding judge was
a mere conjecture and the employment of a firearm in subduing the
lawful resistance of innocent persons is a criminal act by any
standard, Apduhan’s death sentence was reduced to reclusion
perpetua for failure to secure the required number of votes by the
court a quo.

384
CONTRIBUTOR CHUA, Kristy Anne
MODULE AGGRAVATING CIRCUMSTANCES
TOPIC
CASE TITLE PEOPLE V. MANDOLADO G.R.NO. L-51304-05
PONENTE GUERRERO, J.: DATE: JUNE 28, 1983
DOCTRINE In order that abuse of confidence be deemed as aggravating, it is
necessary that "there exists a relation of trust and confidence
between the accused and one against whom the crime was
committed and the accused made use of such a relationship to
commit the crime."
To prove that advantage of his being a sergeant in the Philippine
Army was present in committing the crime, there must be evidence
that the accused already intended to shoot the occupants of the
vehicle. The mere fact that he was in fatigue uniform and had an
army rifle at the time is not sufficient to establish that he misused his
public position in the commission of the crimes.
FACTS On October 3, 1977, in the Municipality of Sultan Kudarat, Province
of Maguindanao, Philippines, Mandolado and Ortillano shot and
killed Mendoza and Tenorio. Ortillano, Mandolado, Erinada and
Simon, trainees of the Armed Forces of the Philippines and
assigned to the 3 Infantry Battalion of the Philippine Army, were
rd

riding a bus heading to Midsayap, North Cotabato. Wearing


uniforms, armed and belonging to the same military outfit, they
decided to acquaint through drinking ESQ rum at the bus terminal.
After an hour of drinking, Mandolado being drunk went to the public
market, after a while, returned and grabbed his .30 caliber machine
gun and started firing. His companions tried to stop him but failed.
Realizing that trouble was arising, Simon and Enrinada ran away
and rode a passing Ford Fiera which Mandolado and Ortillano
followed and boarded the same. Appellants forced the driver to bring
them to Midsayap crossing. Mondolado even tried to attack the
driver with his knife.

Upon reaching the Midsayap crossing, Mondolado fired his .30


caliber machine gun at the speeding vehicle that brought them to
the crossing, hitting the right side of the back of the driver’s sister.
While waiting for a ride, a privately owned jeep, driven by Tenorio
which boarded Mendoza and two others heading to Cotabato City
but the latter left the jeep while Enrinada and Simon boarded the
jeep. Appellants ran after the jeep and boarded the keep, on their
way to their destination, the appellants kept firing shots which
caused Tenorio to threaten them in order to stop their acts. The jeep
was stopped by Mandolado pointing a gun at Tenorio upon knowing
that the destination was wrong, Simon and Enrinada jumped off the
keep and ran towards their detachment camp. Appellants alighted

385
from the jeep, Mandolado instantly fired his gun at the occupants in
the jeep while Ortillano was firing at the ground.

Appellants ran away from the scene and again boarded a vehicle
and arrived in Pinaring crossing. Mandolado went to a house to
leave his belongings and change his wet uniform then after they
rode a bus bound for Midsayap but alighted in the Midsayap
crossing as per advice of the driver. Appellants rode a sand and
gravel truck which took them to Pikit, North Cotabato. Upon arriving,
they returned their guns and did not report the incident. Two days
after the shooting, Sgt. Villanueva informed the appellants that they
were suspects of a killing. Thus, they bought a ship ticket to Manila
but were stopped from boarding and they were apprehended by a
team led by Lt. Licas. Mandolado admitted to killing Tenorio and
Mendoza while Ortillano admitted his presence and that he was
firing at the ground.
ISSUE/S Whether or not abuse of confidence or obvious ungratefulness and
advantage was taken of his being a draftee in the Philippine Army
are the aggravating circumstances present.
RULING/S No, according to the Supreme Court, there are no aggravating
circumstances present in the case at bar. First, as for the advantage
taken as a draftee in the Philippine Army, there was no evidence
proving that when they stopped the jeep, the accused already
intended to kill the occupants even with uniforms and firearms on
them, they could easily hitch hike with private vehicles. There is no
definite proof that the appellants took advantage of his sergeant
position in the Philippine Army in order to commit the crime, the
mere fact of uniforms and firearms are not sufficient to establish this
aggravating circumstance. Second, regarding the abuse of
confidence which is also not present is shown through the victims
lack of confidence. In order to invoke this aggravating circumstance,
there should exist a relation of trust and confidence between the
accused and one against whom the crime was committed and the
accused must have used this said relationship in order to
accomplish the crime. It is important to note that the confidence
between the parties must be immediate and personal. Lastly, there
is no obvious ungratefulness because the requisite trust of the
victims upon the accused prior the criminal act and the breach are
not present.
In finality, the crime charged against Mandolado is murder which
was qualified by treachery while Ortillano was charged with being
an accomplice of the crime of murder with both having no
aggravating circumstance but appreciating drunkenness as
mitigating circumstance for both of the accused.

386
CONTRIBUTOR CRUZ, Jakielyn Anne O.
MODULE MODULE 8: Aggravating Circumstances
TOPIC
CASE TITLE PEOPLE v. GARCIA G.R.NO. L-30449
PONENTE ABAD SANTOS, J: DATE: OCT. 31, 1979
DOCTRINE The Revised Penal Code, Article 14, Provides that it is an
aggravating circumstance when the crime is committed in the
nighttime, whenever nocturnity may facilitate the commission of the
offense. There are two tests for nocturnity as an aggravating
circumstance: the objective test under which nocturnity is
aggravating because it facilitates the commission of the offense;
and the subjective test, under which nocturnity is aggravating
because it was purposely sought by the offender. These two tests
should be applied in the alternative.
FACTS The victim Apolonio Dioquino, Jr. was engaged in a drinking spree
with his gang in Bill’s Place in Pasay City. Upon learning the
whereabouts of Apolonio, his sister Corazon Dioquino Paterno
obtained the permission from her husband to leave the house at
3:00 am and fetch her brother. Corazon wanted to fetch her brother
Apolonio because she wanted him to escape the untoward
influence of his gang.
When Corazon was on her way, she saw that her brother Apolonio
fleeing a group of about seven persons, including the two accused,
Antonio Garcia and Reynaldo Arviso. Corazon saw that the chase
was led by the two accused, with Antonio carrying a long sharp
instrument. Corazon heard a gun shot which caused her to seek
cover. From where she was hiding, about 20 meters away, she saw
the group catch up with her brother and maltreat him. Some beat
him with pieces of wood, while others boxed him. Immediately
afterwards, the group scampered away in different directions.
Antonio was left behind, sitting astride the prostrate figure of
Apolonio, stabbing the latter in the back with his long knife.
When Corazon mustered the courage to approach her brother, she
saw that he was bathed in a pool of his own blood. The incident
threw her in a state of nervous confusion, and she resolved to report
the incident to her younger sister.
The accused denied the allegations and claimed that were in
Pacita’s Canteen during the incident.
ISSUE/S Whether or not the aggravating circumstances of nighttime and
superior strength must be absorbed in treachery?

387
RULING/S NO. In this case we believe that the correct qualifying circumstance
is not treachery, but abuse of superiority. Here we are confronted
with a helpless victim killed by assailants superior to him in arms
and in numbers. But the attack was not sudden nor unexpected,
and the element of surprise was lacking. The victim could have
made a defense; hence, the assault involved some risk to the
assailants. There being no showing when the intent to kill was
formed, it cannot be said that treachery has been proven.
We find that abuse of superiority attended the offense, following a
long line of cases which made this finding on parallel facts. Our
jurisprudence is exemplified by the holding that where four persons
attacked an unarmed victim but there was no proof as to how the
attack commenced and treachery was not proven, the fact that there
were four assailants would constitute abuse of superiority.
However, the information does not allege the qualifying
circumstance of abuse of superiority; hence, this circumstance can
only be treated as generic aggravating.
The Revised Penal Code, Article 14, Provides that it is an
aggravating circumstance when the crime is committed in the
nighttime, whenever nocturnity may facilitate the commission of the
offense. There are two tests for nocturnity as an aggravating
circumstance: the objective test under which nocturnity is
aggravating because it facilitates the commission of the offense;
and the subjective test, under which nocturnity is aggravating
because it was purposely sought by the offender. These two tests
should be applied in the alternative.
In this case, the subjective test is not passed because there is no
showing that the accused purposely sought the cover of nighttime.
Next, we proceed and apply the objective test, to determine whether
nocturnity facilitated the killing of the victim. A group of men were
engaged in a drinking spree, in the course of which one of them
fled, chased by seven others. The criminal assault on the victim at
3:00 a.m. was invited by nocturnal cover, which handicapped the
view of eyewitnesses and encouraged impunity by persuading the
malefactors that it would be difficult to determine their identity
because of the darkness and the relative scarcity of people in the
streets. These circumstances combine to pass the objective test,
and we find that nocturnity is aggravating because it facilitated the
commission of the offense. Nocturnity enticed those with the lust to
kill to follow their impulses with the false courage born out of the
belief that they could not be readily identified.

388
CONTRIBUTOR DELA CRUZ, Ma. Luzelle P.
MODULE Qualifying Aggravating Circumstance
TOPIC
CASE TITLE PEOPLE THE PHILIPPINES v. G.R.NO. 175881
RODAS
PONENTE CHICO-NAZARIO, J.: DATE: AUG. 28, 2007
DOCTRINE Treachery is present when the suddenness of the attack, the
number of the accused and their use of weapons against the
unarmed victim prevent the possibility of any defense or retaliation
by the victim.
FACTS A benefit dance at Milaub, which was sponsored by Boboy
Raquilme, was being held. Among those roaming in the vicinity of
the dance hall were Alberto Asonda and Ernie Anggot. They
stopped and hung out near the fence to watch the affair. Titing
Asenda was standing near them. They saw Charlito Rodas,
Armando Rodas, Jose Rodas, Jr., and Jose Rodas, Sr. surround
Titing Asenda. Suddenly, without a word, Charlito Rodas, armed
with a hunting knife, stabbed Titing at the back. Armando Rodas
then clubbed Titing with a chako hitting him at the left side of the
nape causing him to fall. Thereafter, Jose Rodas, Sr. handed to
Jose Rodas, Jr. a bolo which the latter used in hacking Titing, hitting
him on the left elbow. Alberto Asonda and Ernie Anggot tried to help
Titing but Armando Rodas prevented them by pointing a gun at them
and firing it towards the sky.
After the assailants left, Alberto Asonda and Ernie Anggot
approached Titing Asenda who was already dead. They informed
Danilo Asenda that his brother was killed. The police arrived the
following day after being informed of the incident.
On the part of the defense, accused-appellants Armando Rodas and
Jose Rodas, Sr., and Vilma Rodas, the former’s wife, took the
witness stand. The defense rested its case without marking and
offering any documentary evidence.
Defense evidence showed that only Charlito Rodas and Jose
Rodas, Jr. killed Titing Asenda. Appellant Jose Rodas, Sr. denied
any participation in the killing of Titing Asenda claiming he was not
present in the benefit dance and that he was in his home with his
wife and infant granddaughter when the killing happened. He
revealed that on the night of the killing, his son, Charlito Rodas, who
was carrying a hunting knife, arrived and told him he killed
somebody. He then brought his son to the municipal building of
Siayan to surrender him to the police authorities.
Appellant Armando Rodas likewise denied he was one of those who
killed Titing Asenda. He claimed that at the time of the killing, he
was in his house sleeping with his children. He denied using a chako
and firing a gun. He insisted it was his brothers, Charlito and Jose
Jr., who killed Titing Asenda because they pleaded guilty.

389
To bolster the testimony of the appellants, Vilma Rodas testified that
she was at the benefit dance when the killing happened. Armando
and Jose Sr., she claimed, did not participate in the killing. She said
Charlito stabbed Titing while Jose Jr. merely punched the victim.
On 9 July 1998, the trial court promulgated its decision finding
accused-appellants Armando Rodas and Jose Rodas, Sr. guilty of
the crime of Murder.
ISSUE/S WON the Appellants are guilty of the crime of Murder.
RULING/S The Court finds that Alberto Asonda and Ernie Anggot witnessed
the killing of Titing Asenda by Charlito Rodas, Armando Rodas,
Jose Rodas, Jr. and Jose Rodas, Sr. When Titing was killed,
Asonda and Anggot were near him. Contrary to the claim of the
defense that the place where the killing occurred was not lighted
enough for the assailants to be identified, the place was sufficiently
lighted by a Petromax as testified to by Vilma Rodas.
In the case at bar, both appellants claimed that on the night Titing
Asenda was killed, they were one kilometer away. Thus, it was not
possible for them to have been at the scene of the crime when the
crime was committed. The defense witnesses, however, gave
conflicting testimonies. Appellant Armando said his residence was
more or less one kilometer away from the crime but Jose Sr. said it
was only 50 meters away. Jose Sr. said the house of Charlito was
only 50 meters away from the crime scene but Armando said it was
one kilometer away. Armando said his wife was in Dipolog City
when the killing happened, but his wife said she witnessed the
killing. Armando said he and all the other accused lived in separate
houses, but his wife revealed that Charlito lives with Jose Sr. Vilma
Rodas said after the killing, she immediately went home and told
Armando that his brothers killed somebody but her husband said he
only learned of it the next morning. What is more incredible is the
fact that despite the testimony of Vilma Rodas that she informed
Armando of the killing, the latter never testified to this effect. All
these negate appellants’ claim that they were not at the crime scene
when the killing took place.
The victim was completely unaware that he was going to be
attacked. He was not forewarned of any danger to himself as there
was no altercation or disagreement between the accused and the
victim. If treachery may be appreciated even when the victim was
forewarned, more so should it be appreciated when the victim was
not, as in the case at bar. The suddenness of the attack, the
number of the accused and their use of weapons against the
unarmed victim prevent the possibility of any defense or
retaliation by the victim. The fact that the victim was already
sprawled on the ground and still Jose Jr. hacked him with a bolo
clearly constitutes treachery.

390
For evident premeditation to be appreciated, the following elements
must be established: (1) the time when the accused decided to
commit the crime; (2) an overt act manifestly indicating that he has
clung to his determination; and (3) sufficient lapse of time between
decision and execution to allow the accused to reflect upon the
consequences of his act. In the case at bar, the prosecution failed
to show the presence of any of these elements.
The aggravating circumstance of nocturnity cannot be considered
against appellants. This circumstance is considered aggravating
only when it facilitated the commission of the crime, or was
especially sought or taken advantage of by the accused for the
purpose of impunity. The crime scene was sufficiently lighted by a
Petromax which led to the identification of all the accused.
The circumstance of abuse of superior strength cannot be
appreciated separately, it being necessarily absorbed in treachery.
Appellants Armando Rodas and Jose Rodas, Sr. are found GUILTY
beyond reasonable doubt of murder.

391
CONTRIBUTOR DIMAKUTA, Jasmine A.
MODULE AGGRAVATING CIRCUMSTANCES
TOPIC
CASE TITLE THE PEOPLE OF THE PHILIPPINES V. G.R. L-30116
FAUSTO DAMASO, VICTORIANO NO.
EUGENIO, alias TURING, ESTANISLAO
GREGORIO alias ISLAO, LORENZO
ALVIAR alias ORING AND BONIFACIO
ESPEJO alias MARCIA
PONENTE DATE: November
20, 1978
DOCTRINE There is treachery where the victim, who was unarmed, while seated
on a chair at the investigator’s table, stooping and smoking, was
shot by the accused immediately after he uttered the words “I will
shoot you now” as the victim was completely unaware of the
impending attack. (People vs. Reyes, 69 SCRA 474).
For robbery with homicide to exist it is enough that a homicide would
result by reason of or on the occasion of the robbery. It is immaterial
that death supervened by mere accident as long as it was produced
by reason of or on the occasion of the robbery. What is to be
considered is the result obtained, without reference or distinction as
to the circumstance causes or modes or persons intervening in the
commission of the crime. (People vs. Saliling, 72 O.G. 5290; People
vs. Mangulabuan, 99 Phil. 992.)

Accused is guilty of homicide, not robbery with homicide, where the


thought of taking away the money of the victim came after the killing.
(People vs. Fontanilla, 30 SCRA 242.)
Homicide, not murder, was committed where alevosia is absent.
(People vs. De la Cruz, 36 SCRA 452.) The sudden and unexpected
assault perpetrated by the seven malefactors insured the killing of
the three defenseless victims. Hence, the killings should be
regarded as murder qualified by treachery. (People vs. Pascual, 81
SCRA 548; People vs. Lopez, 80 SCRA 18.)
The four assailants unexpectedly grabbed the unarmed victim and
brought him to the warehouse so that they could kill him with
impunity. They utilized their combined strength to over-power the
helpless victim, (People vs. Sabater, 81 SCRA 564.)
There is treachery where from the accounts of witness it appears
that the firing of the guns of the accused was simultaneous and
sudden, just immediately after the chief of police asked whether the
deceased and companions, who were then riding in a jeep which

392
was moving slowly after being flagged to stop, were carrying
firearms. (People vs. Peralta, 39 SCRA 396.)

FACTS Donata Rebolledo and her son-in-law, Victoriano de la Cruz were


residents of Barrio Bangar, municipality of Victoria, province of
Tarlac. At about 9 P.M. of November 21, 1959, Donata and
Victoriano heard the barkings of dogs outside their house. Shortly,
two men armed with guns, entered, pointed their weapons at them,
tied up the hands of Victoriano, covered him with a blanket and
asked Donata for the whereabouts of her daughter Catalina
Sabado. Stricken by fear, Donata kept silent and blocked the door
leading to her daughter’s room but was promptly pushed aside.
Donata was then ordered to open an “aparador” from which the two
men took valuables like jewelry, clothing, documents, and cutting
instruments. All the while, Donata and Victoriano could hear the
movements and voices of some three to four other persons beneath
the house. The two men brought Catalina Sabado down from the
house and then asked where they could find Susana Sabado,
Donata’s other daughter who was then in her store located about
five meters away in the same house. Thereafter, Donata heard the
men opening the door to Susana’s store. After several minutes,
feeling that the intruders had left, Donata untied the hands of
Victoriano and asked him to go to the store to see if her daughters
were there. When the two women could not be found, Donata sent
Victoriano to the barrio lieutenant to report the incident. Accordingly,
Victoriano went to the barrio lieutenant and the two later went to
town to inform the police of the occurrence.
On the same night, Chief of Police Pedro Valdez with the aid of
several policemen and a handful of civilians went out in search for
the Sabado sisters. It was only the following morning when the two
women were found already dead with wounds in several parts of
their bodies. They were found in a sugar plantation belonging to one
Ignacio Fabros, located about one hundred meters from Donata
Rebolledo’s house. Dr. Carlos Briones, Municipal Health Officer of
Victoria performed the autopsy on the two bodies and reported that
the deaths were caused by profuse hemorrhage due to a fatal, big,
wide, gaping and deep lacerated wound just above the Adam’s
apple. He also testified in court that the death weapon must have
been a sharp instrument with a pointed tip, like a scythe.
A few days after the incident, Donata Rebolledo singled out the
accused Fausto Damaso from a police line-up as one of the men
who went up to her house on that evening. She and Victoriano had
recognized Damaso because of the light coming from a kerosene
lamp placed on a small table near the “aparador.” Later, the PC
rounded up four other suspects in the persons of co-accused
Gregorio, Eugenio, Alviar and Espejo.

393
As further evidence, the prosecution presented separate
extrajudicial statements, sworn to before Municipal Judge Conrado
de Gracia of Paniqui, Tarlac, wherein all the five accused admitted
having participated in the crime. Fausto Damaso stated that he was
with his co-accused Gregorio, Eugenio, Alviar and Espejo on the
night the Sabado sisters were killed; that he never went into the
house of Donata Rebolledo as Eugenio and Gregorio were the ones
who did; that it was Gregorio and Eugenio who actually did the killing
while he, Alviar and Espejo merely stood by; that the victims were
stabbed and their throats cut with a reaping knife (pangapas or lait);
that the killing was motivated by the failure of the older woman
(Catalina) to pay for a carabao bought from Gregorio; and that on
that evening, Gregorio, Eugenio, Alviar and Espejo were carrying
caliber .45 pistols while he was unarmed.
Damaso reiterated his claim that it was Gregorio who actually
stabbed and cut the throats of the victims in the presence of all the
accused; that Catalina was killed ahead of Susana; that Gregorio
killed Susana as she was being held by Eugenio; and that while still
in the house, they were able to get P15 from Susana’s store.
Contrary to what he confessed in his previous sworn statements, he
admitted that it was he and Eugenio who went up to Donata
Rebolledo’s house and not Eugenio and Gregorio. He also changed
his theory as to the motive for the killings, declaring this time that
the two women were killed because the latter had already
recognized them. He further stated that on that night, he was armed
with a caliber .22 (paltik) revolver, Eugenio with a 12-gauge paltik,
Gregorio with two reaping knives (lait), Lorenzo with a long firearm
and Espejo with two stones.
Victoriano Eugenio likewise admitted that he was a party to the
commission of the offense; that it was Gregorio who conceived of
the plot to commit the crime; that it was also Gregorio who killed the
two women with a reaping knife; that after Catalina was killed he
held Susana by the arms as Gregorio stabbed her and cut her
throat; that Alviar, Damaso and Lorenzo were also with them that
night; that he did not know what motivated Gregorio to kill the
victims; that he had no previous agreement with his co-accused to
kill the two women; that he and Damaso were the ones who entered
Donata’s house, took P15 from the “aparador,” brought down
Catalina and also got Susana from another portion of the house;
that he was then armed with a 12-gauge paltik, Damaso with a
caliber .22 paltik revolver, Alviar with a Springfield caliber .30 rifle,
Gregorio with a reaping knife and Espejo with two stones; and that
he was with the group that night because at about 7 P.M., Gregorio
dropped by his house and invited him to Barrio Bangar where the
crime was committed.

394
Estanislao Gregorio narrated that in the afternoon of November 21,
1959, his four co-accused came and informed him of a plan to rob
the Sabado sisters, to which plan he agreed; that Damaso and
Eugenio went up Donata Rebolledo’s house, got P15 in cash and
brought out Catalina and Susana by force; that he stabbed and cut
the throats of the victims with all his co-accused present; that
Eugenio held Catalina while Damaso held Susana as he killed them
both with a reaping knife; that the two women were killed because
they had recognized Eugenio and Damaso and might testify against
them in court.
Bonifacio Espejo declared that he happened to be with the group
because Damaso and Eugenio invited him to Barrio Bangar and
they dropped by the houses of Alviar and Gregorio before actually
proceeding to the barrio; that they had a previous agreement to
commit the crime; that they planned the same in a lot owned by a
certain Don Juan Garcia in Barrio Bangar; that it was Damaso and
Eugenio who entered Donata Rebolledo’s house while he, Alviar
and Gregorio were left downstairs to keep watch.
Substantially similar were the admissions of Lorenzo Alviar in his
sworn statement. He likewise declared that he and his co-accused
took P15 from the house of the victims; that it was Gregorio who
stabbed and cut the throats of the victims with a reaping knife; that
the killing was done in a sugarcane plantation between 10:00 and
11:00 P.M. of November 21, 1959; that Catalina was killed before
Susana. He claimed, however, that he was only forced and
intimidated by his co-accused to join the group.
At the trial, the five accused set up the defense of alibi and
repudiated their respective sworn statements alleging that these
were obtained from them through duress, force and intimidation.
Instances of the use of third degree methods like boxing, pouring of
“7-up” into the nostrils, stripping of clothes, pricking of the penis,
kicking and slapping of the ears were narrated by the accused on
the witness stand, all of which were not believed by the trial court.

ISSUE/S It is argued that (a) there is no evidence of the alleged robbery; (b)
that the homicide was not committed by reason or on occasion of
the robbery; and c) that the crime was not attended by the
aggravating circumstances of armed band, treachery and
uninhabited place.

RULING/S There is no merit to appellants’ submittal. That robbery was


committed is evident from the declaration of prosecution witness
Donata Rebolledo who testified that the two men who barged into
her house, one of whom she recognized as Fausto Damaso,
ordered her to open her “aparador” and then they took therefrom the
following items with their respective values. Moreover the appellants
admitted in their separate statements that they were able to get P15

395
from Donata’s house. What is important is that the culprits carried
away personal property belonging to another by the use of force,
intimidation or violence.

Counsel points out that because there was a motive, at least on the
part of Gregorio, for the killing of the Sabado sisters, the double
homicide could not have been “committed by reason or on occasion
of the robbery” as the law contemplates. He calls Our attention to
the sworn statement wherein Fausto Damaso declared that
Gregorio killed Catalina and Susana because Catalina bought a
carabao from him and did not pay for it. Harping further on this
motive theory, counsel mentions such circumstances as why the
accused specifically asked for Catalina and Susana upon entering
Donata Rebolledo’s house and why Donata and Victoriano were not
killed together with the sisters if the purpose was to remove all
opposition to the robbery or to eliminate witnesses thereto.

Even assuming, however, that such a motive for vengeance existed


on the part of Gregorio, it does not necessarily exclude the fact that
he and co-accused also intended, when they went to Donata’s
house that night, to rob the family. In a complex crime of robbery
with homicide, while an intent to commit robbery must precede the
taking of human life, the fact that the intent of the culprit was
tempered with a desire also to avenge grievances against the
person killed does not prevent the punishment of the accused for
the complex crime.

Counsel for appellants also argues that the trial court erred in its
appreciation of the aggravating circumstances of armed band,
treachery and uninhabited place. It is clear from the above, that
Damaso was armed during the night of the commission of the crime,
and it is immaterial what kind of firearm he carried, the only
important thing being that he was armed. In this case, the presence
of an armed band is to be considered as a generic aggravating
circumstance under Article 14(6) of the Revised Penal Code
inasmuch as the crime committed was that provided for and
penalized in Article 294, paragraph 1 and not under Article 295,
Revised Penal Code.

Treachery is present if the victim is killed while bound in such a


manner as to be deprived of the opportunity to repel the attack or
escape with any possibility of success. The fact that the bodies of
Catalina and Susana were found dead with their arms tied behind
their backs as well as the admission of Gregorio in his confession
that he killed the sisters while their arms were held by Eugenio and

396
Damaso lead Us to conclude that the killing of the two women was
done under treacherous circumstances.

Anent the circumstances of uninhabited place, counsel disclaims its


existence by pointing to the proximity of the sugarcane field where
the victims were killed to the national highway as well as to certain
houses in the barrio. The uninhabitedness of a place is determined
not by the distance of the nearest house to the scene of the crime,
but whether or not in the place of commission, there was reasonable
possibility of the victim receiving some help. Considering that the
killing was done during nighttime and the sugarcane in the field was
tall enough to obstruct the view of neighbors and passersby, there
was no reasonable possibility for the victims to receive any
assistance. That the accused deliberately sought the solitude of the
place is clearly shown by the fact that they brought the victims to the
sugarcane field although they could have disposed of them right in
the house of Donata Rebolledo where they were found.

The trial court considered separately the three circumstances of


armed band, treachery and uninhabited place where under other
situations one may be considered absorbed or inherent in the other.
There is ample justification for this. The elements of each
circumstance subsist independently and can be distinctly perceived
thereby revealing a greater degree of perversity on the part of the
accused. In conclusion, the crime committed by appellants in
Criminal Case No. 2253 is robbery with homicide defined in Article
294, paragraph 1, Revised Penal Code, to wit:

Robbery with violence against or intimidation of persons —


Penalties — Any person guilty of robbery with the use of violence
against or intimidation of any person shall suffer:
1. The penalty of reclusion perpetua to death, when by reason or on
occasion of the robbery, the crime of homicide shall have been
committed.

The penalty is to be imposed in its maximum period by reason of the


presence of three aggravating circumstances found by the trial
court, to wit: that the robbery was committed by a band, with
treachery, and in an uninhabited place. There is likewise the
additional aggravating circumstance that the robbery was
committed in the dwelling of the victim, Donata Rebolledo which
although not alleged in the Information is however established by
the evidence. IN VIEW OF THE FOREGOING CONSIDERATIONS,
We hereby affirm in toto the decision of the trial court in the two
cases.

397
CONTRIBUTOR FLORES, Precious Eureka D.
MODULE Aggravating Circumstance
TOPIC
CASE TITLE PEOPLE v. BALDERA G.R.NO. L-2390
PONENTE REYES, J. DATE: APRIL 24, 1950
DOCTRINE Where one of several codefendants turns state’s evidence on a
promise of immunity, but later retracts and fails to keep his part of
the agreement, his confession made under such promise may then
be used against him.
FACTS The accused was guilty of robbery with homicide and serious and
less serious physical injuries.

On December 23, 1947, a Casa Manila bus loaded with passenger


left Batangas, Batangas, bound for Manila. On the highway in barrio
Calansayan, municipality of San Jose, same province, it was held
up by a group of five or six armed men. One of these, later identified
as herein appellant Pedro Baldera, who was then armed with a .45
caliber pistol, fired a shot, and this was followed by a hail of bullets
coming from different directions. As a result, several passengers,
among them Jose Cabrera, Jose Pastor and Francisco Mendoza,
were wounded. After the firing had ceased, appellant got on the bus
and, threatening the passengers with his gun, took P90 from Jose
Pastor and P34 from Ponciana Villena. Another passenger named
Francisco Mendoza was also relieved of his P3. Appellant then
alighted and ordered the bus to proceed, whereupon the driver
headed for the municipal building of San Jose and there reported
the incident to the authorities. The wounded were taken to the
hospital, where Jose Cabrera died from his wounds on the following
day. Jose Pastor, who was wounded in the left leg, was cured in two
months, while Francisco Mendoza's gunshot wound in the right
shoulder healed in 15 days.
ISSUE/S 1. Whether or not the accused is guilty of robbery with homicide and
serious and less serious physical injuries.
2. Whether or not Baldera’s confession is admissible to the court.
3. Whether or not the lower court erred in holding that the crime
committed is robbery in band.
4. Whether or not the lower court erred in appreciating the
aggravating circumstance of recidivism.
RULING/S 1. Baldera was positively identified by one of the passengers as the
one who relieved her of her money at gunpoint, even stating that
she had a good look at his face for she was watching him closely
for fear that he might fire at her. Such statement has not shown
any motive for falsely testifying against the accused therefore,
even his prostitute alibi cannot stand against the above-
mentioned testimony.

398
2. Baldera’s confession was allegedly made for him to be given
protection from his co-accused, and to use him as a government
witness. But he later on said the confession was taken through force
and use him as a government witness. But, he later on said the
confession was taken through force and intimidation. However, it
was not proved. But his confession will still stand since the
confession made under a promise of immunity, but later retracts and
fails to keep his part of the agreement, may be used against him in
the court of law.

3. It is a fact that there were more than three (3) armed men in
the group that held up the bus as stated in Baldera’s confession and
was established by the uncontradicted testimony of one of the
government witnesses. However, in the case at bar, it is immaterial
since in the crime of robbery with homicide it is not essential that the
robbery be in band, but it may count as an aggravation in the
imposition of the penalty.

Even if it is not considered, there is still another aggravating


circumstance such as that the robbery was perpetrated by
attacking a vehicle, which is not offset by any mitigating
circumstance.

4. The lower court erred in appreciating the aggravating


circumstance of recidivism since the recent case against him was
charged seven (7) days before his conviction of the crime of theft.

399
CASE TITLE People v. Melendrez G.R NO. G.R. No. 39913
PONENTE AVANCEÑA, C. J. DATE: 19 December 1933
CONTRIBUTOR GARCIA, LEXANNE O.
DOCTRINE
The aggravating circumstance of recidivism should be taken into
account in imposing the principal penalty in its corresponding
degree, notwithstanding the fact that the defendant is also
sentenced to suffer an additional penalty as a habitual delinquent.
FACT/S
In a trial for a case on robbery, Melendrez was also acknowledged
by the court as a habitual delinquent, he having been previously
convicted by final judgment of competent courts twice of the crime
of theft and once of the crime of estafa and having been last
convicted of the crime of estafa on September 3, 1932.

The fiscal contends that the aggravating circumstance of


recidivism should be taken into account against the appellant.
MAIN ISSUE/S
Should the petitioner be considered as a recidivist?
RULING ON
MAIN ISSUE/S The aggravating circumstance of recidivism should be taken into
account in imposing the principal penalty in its corresponding
degree, notwithstanding the fact that the defendant is also
sentenced to suffer an additional penalty as a habitual delinquent.

The facts alleged in the information constitute the crime of robbery


committed without the use of arms in an inhabited house, the value
of the articles taken being less than P250. In accordance with
article 299 of the Revised Penal Code, the penalty prescribed for
said crime is prisión correccional in its medium degree. Inasmuch
as there is a concurrence therein of one mitigating and one
aggravating circumstance, this penalty should be imposed in its
medium degree.

400
CONTRIBUTOR Manango, John De Divine B.
MODULE Module 8: Aggravating Circumstances
TOPIC
CASE TITLE US v. Manalinde G.R.NO. G.R. No. 5292
PONENTE J. Torres DATE: 28 August 1909
DOCTRINE Aggravating Circumstances – Evident premeditation
FACTS On January 19, 1909 Manalinde has attacked a Spaniard in the
head and a Chinese in the shoulder causing his death. Afterwards,
Manalinde has flee the town. On the following month, the trial court
has rendered judgment sentencing the accused to the ultimate
penalty of death.

The accused pleaded guilty and confessed that Datu Mupuck


directed him to go juramentado and kill the two persons he would
meet in town in exchange for a pretty woman the former would give
him to replace the latter’s wife that died about one hundred days
ago.
ISSUE/S What are the aggravating circumstances present in the case of
Manalinde?
RULING/S The judgment from the Trial court has been affirmed by SC with
costs to the defendant.
In the commission of the crime of murder the presence of
aggravating circumstances 3 and 7 of article 10 of the Penal Code
should be taken into consideration in that promise of reward and
premeditation are present, which in the present case are held to be
generic, since the crime has already been qualified as committed
with the treachery, because the accused confessed that he
voluntarily obeyed the order given him by Datto Mupuck to go
juramentado and kill someone in the town of Cotabato, with the
promise that if he escaped punishment he would be rewarded with
a pretty woman. Upon complying with the order, the accused
undoubtedly acted of his own volition and with the knowledge that
he would inflict irreparable injury on some of his fellow-beings,
depriving them of life without any reason. The fact that the
arrangement between the instigator and the tool considered the
killing of unknown persons, does not bar the consideration of the
circumstance of premeditation.

401
CONTRIBUTOR MARCELINO, Ferilynn T.
MODULE Aggravating Circumstances
TOPIC
CASE TITLE PEOPLE V. ILAOA G.R.NO. 94308
PONENTE BELOSILLO, J: DATE: 16 June 1994
DOCTRINE Abuse of superior strength, cruelty and evident premeditation must
be sufficiently proved to be appreciated as aggravating
circumstances.
FACTS Pfc. Reynaldo P. Angeles was dispatched in the early morning of 5
November 1987 to Tinio St., Sta. Maria Phase I, Balibago, Angeles
City, where the decapitated body of a man, later identified through
his voters identification card as Nestor de Loyola, was found in a
grassy portion thereof. Apart from the decapitation, the deceased
bore 43 stab wounds in the chest as well as slight burns all over the
body. The head was found some 2 feet away from the corpse. Five
persons were charged for the gruesome murder of Nestor de
Loyola. However, only the brothers Ruben and Rogelio stood trial
since the other accused escaped and were never apprehended.

As found by the trial court, in the late evening of 4 November 1987,


appellant Ruben Ilaoa was engaged in a drinking session with the
deceased Nestor de Loyola together with several others. Ruben
was heard arguing with Nestor. A few moments later, Ruben mauled
and kicked the deceased with the help of their drinking companions
just outside Rubens apartment. As the deceased cried "Aray! Aray!"
and "Pare, bakit nyo ako ginaganito? Hirap na hirap na ako!",
appellant dragged the deceased with the help of Julius Eliginio to
the apartment from where a man’s cries were continued to be heard
later. To further seal the case against him, Ruben borrowed Alex
Villamils tricycle at two o’ clock in the morning of 5 November 1987
on the pretext that a neighbor was about to give birth and had to be
rushed to the hospital. However, he was seen driving the tricycle
alone with a sack placed in the sidecar. The sack looked as if it
contained a human body. Then, an hour later, or at three o’ clock in
the morning, the tricycle was returned with bloodstains on the floor.

RTC found Ruben and Rogelio guilty of murder with the attendant
circumstances of evident premeditation, abuse of superior strength
and cruelty, and imposed upon them the penalty of "life
imprisonment." In this appeal, brothers Ruben and Rogelio Ilaoa
argue for their acquittal. Appellants contention: Specifically, they
assail the finding of evident premeditation, abuse of superior
strength and cruelty as totally unwarranted.
ISSUE/S Whether the aggravating circumstances were correctly appreciated.

402
RULING/S NO. The appellant was liable only for homicide, not murder, on the
ground that the qualifying circumstances alleged in the information,
namely, abuse of superior strength, cruelty and evident
premeditation, were not sufficiently proved to be appreciated
against appellant. Abuse of superior strength cannot be considered
because there was no evidence whatsoever that appellant was
physically superior to the deceased and that the former took
advantage of such superior physical strength to overcome the
latter’s resistance to consummate the offense. Meanwhile, the fact
that Nestor de Loyola’s decapitated body bearing 43 stab wounds,
24 of which were fatal was found dumped in the street is not
sufficient for a finding of cruelty where there is no showing that
appellant Ruben Ilaoa for his pleasure and satisfaction, caused
Nestor de Loyola to suffer slowly and painfully and inflicted on him
unnecessary physical and moral pain. Number of wounds alone is
not the criterion for the appreciation of cruelty as an aggravating
circumstance. Neither can it be inferred from the mere fact that the
victims dead body was dismembered.

Evident premeditation cannot likewise be considered. There is


nothing in the records to show that appellant, prior to the night in
question, resolved to kill Nestor de Loyola, nor is there proof to show
that such killing was the result of meditation, calculation or
resolution on his part. On the contrary, the evidence tends to show
that the series of circumstances which culminated in the killing
constitutes an unbroken chain of events with no interval of time
separating them for calculation and meditation. Absent any
qualifying circumstance, Ruben Ilaoa should only be held liable for
homicide.

403
CONTRIBUTOR MAYUGA, Eunice Allaine G.
MODULE Article 14 of the RPC - Aggravating Circumstances
TOPIC
CASE TITLE PEOPLE v. BIBAT G.R.NO. 124319
PONENTE PURISIMA, J: DATE: 13 MAY 1998
DOCTRINE There is evident premeditation when: 1. The time when the offender
determined to commit the crime; 2. An act manifestly indicating that
the culprit has clung to his determination; and 3. A sufficient lapse
of time between the determination and execution to allow him to
reflect upon the consequences of his act.
FACTS On 12 October 1992, at around 1:30PM, Lloyd Del Rosario was on
his way to school waiting for a ride where he was stabbed by the
accused, Gari Bibat. The accused fled and Lloyd was brought to the
hospital where he was pronounced dead on arrival.

The incident was witnessed by Nona Avila Cinco, a laundry woman,


who testified that she saw a person about one meter away talking
to Bibat. Said person told Bibat "O pare, anduon na. Puntahan mo
na. Siguruhin mo lang na itumba mo na." to which Bibat answered
"Oo ba. Ganito ba, ganito ba?". After hearing the conversation,
Nona went inside a house as she was going to another bettor but
went outside after a while. As she was walking, she saw Bibat
hurried towards Lloyd and took a pointed object from the notebook
then stabbed the victim in the left chest twice. Bibat fled but when
Lloyd shouted for help, Bibat returned and stabbed the victim again
in the middle part of the chest. Nona only reported this incident 9
months after because she was afraid.

The lower court finds Gari Bibat, the accused, guilty of the crime of
murder with intent to kill and with evident premeditation. Hence, this
petition.
ISSUE/S Whether or not the circumstance of evident premeditation is present
in this case.
RULING/S Yes. The court held that the essence of premeditation is that the
execution of the criminal act is preceded by cool thought and
reflection upon the resolution to carry out the criminal intent during
the space of time sufficient to arrive at a calm judgment.
The requisites of evident premeditation appears to have been
thoroughly and sufficiently established. The determination or
conception of the plan to kill the victim could be deduced from the
outward circumstances that happened on the fateful day of 14
October 1992. At around 1:30 in the afternoon, it was then Nona
saw Bibat hurry towards Lloyd and stabbed the latter. These overt
acts clearly evinced that the appellant clung to his resolution to kill
the victim. From the time Nona heard the plan to kill someone at

404
11:30 up to the killing incident at 1:30 in the afternoon of the same
day, there was a sufficient lapse of time for appellant to reflect on
the consequences of his dastardly act.
In the case of People v. Dumdum, it was held that "the killing of the
deceased was aggravated by evident premeditation, because the
accused conceived of the assault at least one hour before its
perpetration." In the case under examination, two hours had elapsed
from the time Bibat clung to his determination to kill the victim up to
the actual perpetration of the crime. Hence, the decision of the lower
court was affirmed in this case.

405
CONTRIBUTOR PALALA, Amer B.
MODULE Aggravating Circumstances
TOPIC
CASE TITLE People v. Empacis G.R.NO. 95756
PONENTE NARVASA, C.J. DATE: May 14, 1993
DOCTRINE Aggravating circumstances; craft or fraud; appreciated in case at
bar
FACTS Around 9pm, Fidel was about to close his small store when Romuldo
and Crisologo came and asked to buy sardines and rice. Camila,
Fidel's wife, served them and made a meal of rice and sardines.
Romualdo bought cigarettes from Fidel and as Fidel hands it over
to him, Romualdo announced "hold-up." Fidel then gave him
P12,000 but then Fidel fought back to keep his money. Romualdo
was able to stab Fidel about 3 times then Crisologo later on joined
and stabbed Fidel. Peter rushed to his father's defense and was
able to inflict 2 wounds on Crisologo, one on the right shoulder and
the other on the neck. Fidel died and Crisologo was arrested the
following day. However, he denied attacking Fidel, rather even tried
to stop Romulado from using the knife against Fidel. The trial court
found Crisologo guilty of robbery with homicide attended by several
aggravating circumstances: dwelling, nigh time, employment of craft
or fraud, and advantage being taken of superior strength

ISSUE/S Whether or not the aggravating circumstance of craft, fraud or


disguise be appreciated in the case.
RULING/S Yes, the aggravating circumstance of craft or fraud was properly
appreciated against Empacis. Both men pretended to be bona fide
customers of the victim’s store and on this pretext gained entry into
the latter’s store and later, into another part of his dwelling. In
previous cases, the Court held the presence of fraud or craft when
one pretended to be constabulary soldiers to gain entry into a
residence to rob and kill the residents, pretended to be needful of
medical treatment only killing the owner of the house, and pretended
to be wayfarers who had lost their way to enter into a house.

Nighttime was also properly appreciated as an aggravating


circumstance as nocturnity was deliberately and purposely sought
to facilitate the commission of the crime. For superior strength to be
deemed present, it does not suffice to prove superiority in number
on the part of the malefactors but that they purposely employed
excessive force, force out of proportion to the means of defense
available to the person attacked w/c was present in this
case. Empacis and his companion took advantage of their
combined strength and their bladed weapons to overcome their
unarmed victim and assure the success of their felonious design to
take the money. Dwelling was also correctly appreciated.

406
CONTRIBUTOR PANGAN, Gabrielle L.
MODULE Classes of Aggravating Circumstances
TOPIC
CASE TITLE PEOPLE V. BIGCAS G.R.NO. 94534
PONENTE REGALADO, J: DATE: July 20, 1992
DOCTRINE For abuse of superior strength to be considered, it is not sufficient
that there be superiority in number or strength; it is necessary that
the accused must have cooperated and intended to use or secure
advantage from such superior strength.
FACTS Appellants Butron and Bigcas were charged with murder qualified
by abuse of superior strength with treachery and aggravating
circumstances of and nocturnity.

Two eyewitnesses testified to the killing of the victim. Allegedly,


three persons were involved in a commotion. It was not so dark then
as the moon was shining brightly. From behind tall cogon grass,
appellant Butron stroke Ambrocio Palapar two times with a piece of
wood on the latter's back. Palapar attempted to flee from his
aggressor but was chased by appellant Bigcas who, upon catching
up with the former stabbed him twice with a bolo at the back. The
chase continued until Bigcas was able to stab the victim again at the
back of the latter's right knee. The victim fell on the ground, after
which he uttered, "Long, stop because I will die of these wounds."
Butron shouted at him saying, "I will kill you, Boyax." He then
approached Palapar and hit him twice with a piece of wood on the
right jaw. Bigcas, on his part, stabbed the supine victim several
times. Thereafter, both appellants left the victim, with Butron telling
Bigcas. "You own the killing and these two bolos and I will be with
you anywhere."

The defense contends that the death of the victim was due to self-
defense. They claimed that the appellants were at a store when the
victim, intoxicated, came in and drank ‘tuba’ with the appellants. A
commotion ensued which was responded and pacified by a
policeman who required Bigcas and Butron to stay a while and let
Palapar leave ahead. Bigcas testified that on his way home as he
was passing Palapar, the latter suddenly stabbed him with a bolo,
hitting his stomach. He backtracked but the victim followed him and
gave him three stab thrusts which he parried. He was able to take
hold of the victim's hand holding the bolo and wrestled the same
from him. Butron then repeatedly stabbed Palapar until the latter fell.
When he went to the police, he also surrendered the bolo used in
the alleged killing. Butron was brought by Bigcas and the police to
the hospital where he was treated and confined for four days.

407
ISSUE/S Whether or not the aggravating circumstances of treachery, abuse
of superior strength, nocturnity should be appreciated.
RULING/S Abuse of superior strength cannot be appreciated in the case.
In People vs. Cabiling abuse of superior strength may be
considered not only when there is an inequality of force between
the victim and the aggressor but there must be a situation of
superiority of strength notoriously selected or taken advantage of
by him in the commission of the crime. For this qualifying
circumstance to be considered, it is not sufficient that there be
superiority in number or strength; it is necessary that the accused
must have cooperated and intended to use or secure advantage
from such superior strength.
It was established that the victim was ordered by a policeman to
leave the store ahead of the others, with appellants directed to
stay behind for about fifteen minutes, so that the parties would not
encounter each other again shortly after the incident at said store.
It cannot, therefore, be said that when the fight took place more
than fifteen minutes later, because that victim instead of going
straight home obviously waited for appellants to catch up with him,
appellants could have anticipated such an unexpected contingency
and had accordingly conceived of taking advantage of their
combined strength and weapons.
Treachery cannot be appreciated as a qualifying circumstance
against appellants is correct, since there is no evidence that in the
commission of the crime they deliberately adopted means,
methods or forms considered in law as treacherous.
Likewise, nocturnity may not be appreciated in the case as it
neither facilitated the commission of the crime nor was it purposely
sought by appellants in order to afford impunity. During the
incident, the testimony given by the eyewitnesses established that
the moon was shining brightly. The light was bright enough to see
what was going on and to recognize the assailants. Moreover, It,
therefore, does not qualify as an aggravating circumstance under
either the subjective or objective tests laid down by this Court for it
to be considered as such.
As for the claim of self-defense, the same cannot be appreciated
because unlawful aggression, an element of self-defense that is
sine qua non is found wanting. Even if the testimony of the
appellant is to be taken credence, the unlawful aggression is
deemed to have ceased from the moment the victim was disarmed
of his weapon and tried to flee. The fact that the doctor of the
appellant also testified that the wounds of the latter were
superficial, the locations of which is inconsistent with his testimony
and may be self-inflicted also disarms the credibility of his claims.
The accused-appellants are declared guilty of homicide, to serve
the indeterminate sentence of prision mayor as minimum and

408
reclusion temporal as maximum as there are no aggravating
circumstances that would qualify the crime as murder and no
generic aggravating circumstances, with due extenuation by
voluntary surrender.

409
CONTRIBUTOR PROVIDO, Gemy Hale A.
MODULE Aggravating Circumstance – Qualifying Circumstance of Treachery
TOPIC
CASE TITLE PEOPLE v. SANGALANG G.R.NO. L-32914
PONENTE AQUINO, J: DATE: AUG. 30, 1974
DOCTRINE Article 14, Paragraph 16 of the Revised Penal Code. The act be
committed with treachery. There is treachery when the offender
commits a crime against a person employing means, methods or
forms which makes the victim defenseless and insures its execution
of the crime to be committed.
FACTS In Sitio Adlas, Barrio Biluso, Silang, Cavite on June 9, 1968, Ricardo
Cortez left his nipa hut to gather tua from a coconut tree. From there,
he was shot and fell to the ground leading to his death. There were
two testimonies of prosecution eyewitnesses on what happened on
this unfortunate day.
According to Flora Sarno (the wife of the victim), Ricardo Cortez was
gathering tuba from atop a coconut tree when he was shot by five
armed men, each armed with a long firearm. One of them was
Laureano Sangalang, the respondent, and the others are Conrado
Gonzales, Irineo Canuel, Perino Canuel and Eleuterio Cuyom. She
tried to go near where her husband was lying on the ground, but the
assailants shot at her. Hence, she just retreated to the hut for cover.
After five minutes, the armed men left and Flora found her husband
to be dead lying on the ground.
According to Flora’s brother, Ricardo Sarno, the respondent
Laureano Sangalang, used a Garand carbine. He was able to
witness the scene of the crime as he was inside his nipa hut which
is 10 meters away from the spouses’ hut. He tried to go to her sister
but the assailants also shot at him and it was when the assailants
left when he went to his sister.
The victim sustained twenty three (23) gunshot wounds on the
different parts of his body, fourteen (14) were entrance-wounds and
nine (9) were exit-wounds, from which is the reason of his death.
The respondent, Laureano Sangalang, presented an alibi that he
was not at the scene as he was in Julian Gatdula’s place at
Sampaloc, Manila.
ISSUE/S Whether or not the aggravating circumstances of treachery, band
and evident premeditation will be appreciated in the case
RULING/S Laureano Sangalang´s alibi was not enough to prove that he was
not in the scene of the crime and failed to show that the two
testimonies were only done to falsely incriminate hm, hence, he is
found to be gulty.
The aggravating circumstance to be appreciated in the case is the
qualifying circumstance of treachery which absorbs the aggravating
circumstance of a band. The requisites of treachery are present in

410
the case as the victim was shot unarmed and defenseless, there
was a swift and surprise attack of the offenders which made the
victim unable to defend himself, and provided that the victim did not
give any provocation as the victim was on top of the coconut tree
when he was shot.
Since the qualifying circumstance of treachery is established in the
case, the aggravating circumstance of band will be absorbed.
However, the aggravating circumstance of evident premeditation
was not established and is not appreciated in the case.
Wherefore, the respondent is guilty for the crime of murder and is
charged with the penalty of reclusion perpetua.

411
CONTRIBUTOR ROMERO, Ma. Camille Concepcion M.
MODULE Aggravating Circumstances
TOPIC
CASE TITLE People vs. San Pedro G.R.NO. L-44274
PONENTE AQUINO, J. DATE: JAN. 22, 1980
DOCTRINE Craft has a very distinct application to the crime of robbery, separate
and independent of the homicide.
FACTS In the afternoon of June 2, 1970 dead body of Felimon Rivera a
jeepney driver was found in between Barrio Masaya and Paciano
Rizal. Municipality of Laguna. The result of autopsy was deceased
died of profuse hemorrhage due to 23 lacerated and stab wounds
and multiple abrasions found on the different parts of the body of
the deceased.

The jeepney is owned by Pablito delos Reyes a fruit vendor. Earlier


that day was the last time Rivera was driving the jeep, and the said
jeep was not recovered after his death.

Rodrigo Esguerra was interviewed by the police and a written


statement was done, he admitted his participation and named his
companions. Artemio Banasihan was apprehended, he was
investigated. He gave a statement confessing his participation in the
robbery and killing of Felimon Rivera. He recounted the last four
days before June 2, 1970 he and his co-accused met and planned
to get the jeep driven by the deceased. He and Luisito San Pedro
approached Rivera to hire him on hauling coconuts. They were
joined joined by Salvador Litan and Rodrigo Esguerra. Esguerra
was then carrying a water pipe wrapped in paper. Upon reaching a
river between the barrios of Mainit and Puypuy San Pedro ordered
Rivera to stop. Whereupon, at Esguerra's signal, Litan hit Rivera at
the nape with the water pipe. Rivera jumped out of the jeep but was
chased by San Pedro and Litan who stabbed him at the back several
times with a dagger. Esguerra then drove the jeep and the group
proceeded to Makati, Rizal, He then joined Nelson Piso and Antonio
Borja. The jeep was brought to Cavite City where it was sold for
P2,000.00. Four days later, Piso went to Los Baños and gave San
Pedro, Litan and Banasihan P50.00 each, with the promise that the
balance would be given later. However, the promised balance was
not given them.
ISSUE/S Whether or not the aggravating circumstance of craft is absorbed by
treachery
RULING/S No, the court cannot subscribe to the theory of craft being absorbed
by treachery, as nighttime and abuse of superior strength may be
so absorbed, as held in numerous decisions of this Court. Craft was
employed not with a view to making treachery more effective as

412
nighttime and abuse of superior strength would in the killing of the
victim. It was directed actually towards facilitating the taking of the
jeep in the robbery scheme as planned by the culprits. From the
definition of treachery, it is manifest that the element of defense
against bodily injury makes treachery proper for consideration only
in crimes against person as so explicitly provided by the Revised
Penal Code (Art. 14[16]).

413
CONTRIBUTOR SAMONTE, Vanessa Antoinette
MODULE AGGRAVATING CIRCUMSTANCES- TREACHERY
TOPIC
CASE TITLE PEOPLE vs ROBERT CASTILLO A.M. NO. G.R. NO. 120282
PONENTE PANGANIBAN, J.: DATE: APRIL 20, 1998

DOCTRINE In order for treachery to be properly appreciated, two elements must


be present: (1) at the time of the attack, the victim was not in a
position to defend himself; and (2) the accused consciously and
deliberately adopted the particular means, methods, or forms of
attack employed by him.

FACTS Eulogio Velasco, the floor manager of the Cola Pubhouse was
sitting outside the Pubhouse talking with his co-worker, Dorie. After
some time, Antonio "Tony" Dometita, one of their customers, came
out of the pubhouse. As he passed by, he informed Eulogio that he
was going home. When Tony Dometita was about an arms length
from Eulogio, appellant Robert Castillo suddenly appeared and,
without warning, stabbed Tony with a fan knife on his left chest.
Appellant stabbed him once more, hitting him on the left hand. As
Tony cried out for help, Eulogio placed a chair between them to stop
appellant from further attacking Tony. Tony ran towards the other
side of EDSA, but appellant pursued him. Tony died and the medico-
legal officer who autopsied Tony's cadaver, testified that the
proximate cause of Tony's death was the stab wound on his left
chest. Tony also suffered several incised wounds and abrasions,
indicating that he tried to resist the attack. The trial court found that
the killing was qualified by abuse of superior strength, because "the
accused used a deadly weapon in surprising the victim who was
unarmed."
Although treachery was present, the trial court held that this was
absorbed by abuse of superior strength.

ISSUE/S Whether or not the killing was attended by treachery, which qualified
the crime to murder.

RULING/S Yes. We hold that the killing was qualified by treachery. "Treachery
is committed when two conditions concur, namely, that the means,
methods, and forms of execution employed gave the person
attacked no opportunity to defend himself or to retaliate;and that
such means, methods, and forms of execution were deliberately and
consciously adopted by the accused without danger to his person.”
These requisites were evidently present in this case when the
accused appeared from nowhere and swiftly and unexpectedly
stabbed the victim just he was bidding goodbye to his friend,
Witness Velasco. Said action rendered it difficult for the victim to
defend himself. The presence of "defense wounds" does not negate

414
treachery because, as testified to by Velasco, the first stab, fatal as
it was, was inflicted on the chest. The incised wounds in the arms
were inflicted when the victim was already rendered defenseless.

415
CONTRIBUTOR SAMSON, Jessa Viena D.
MODULE Classes of aggravating circumstances
TOPIC
CASE TITLE PEOPLE v. ARIZOBAL G.R.NO. 135051-52
PONENTE PER CURIAM: DATE: DEC. 14, 2000
DOCTRINE Generally, dwelling is considered inherent in the crimes which can
only be committed in the abode of the victim, such as trespass to
dwelling and robbery in an inhabited place. However, in robbery
with homicide, the authors thereof can commit heinous crime
without transgressing the sanctity of the victim’s domicile. Thus, it
can be appreciated as aggravating circumstance.
FACTS Two separate Informations were filed were filed before the Regional
Trial Court charging Clarito Arizobal, Erly Lignes, Rogelio Gemino
and two John Does with Robbery in Band with Homicide for robbing
and slaying Laurencio Gimenez, and his son Jimmy Gimenez.
In the evening of March 24, 1994, Laurencio roused his wife
Clementina from sleep and told the latter to open the door for there
were persons outside their house. Since it was pitch-dark, she lit a
kerosene lamp and stood up to open the door. Suddenly, she was
confronted by three armed men pointing their guns at her. She
recognized two of them as Clarito Arizobal and Erly Lignes, but
failed to recognize the third person who was wearing a
maskara. Subsequently, these [two] barged into the master’s
bedroom and forcibly opened the aparador. They ransacked the
cabinet and scattered everything on the floor until they found
₱8,000.00 among sheets of paper. Before leaving with their loot,
they ordered Laurencio to go with them to Jimmy's house for they
have something to talk about. Against his will, Laurencio went with
them. Clementina recalled that shortly after the group left, she
heard a volley of shots.
On the same night, as narrated by Erlinda, wife of Jimmy, while the
latter was in the process of skinning a chicken for their supper, three
men suddenly appeared and ordered them to lie face down. One of
them pushed her to the ground while the others tied Francisco and
Jimmy as they whipped the latter with an armalite rifle. [She] noticed
one of them wearing a mask, another wearing a hat, and still
another, a bonnet. Realizing the utter helplessness of their victims,
the robbers took the liberty of consuming the food and cigarettes
Erlinda was selling in her sari-sari store. Thereafter, [they]
proceeded to ransack the household in search for valuables. They
took around ₱1,000.00 from her sari-sari store and told them to
produce ₱100,000.00 in exchange for Jimmy's life. Unfortunately,
the couple not produce such a big amount. In lieu of the said
amount, Erlinda offered to give their certificate of large
cattle. However, the culprits did not accept it and threw the
document back to her. Consequently, the three men dragged

416
Jimmy outside the house and together with Laurencio, brought them
some fifty meters away while leaving behind Clarito and Erly to
guard Francisco and Erlinda's son. Moments later she heard a burst
of gunfire which reverberated through the stillness of the night.
In view thereof, the lower court ruled that the robbery with killing was
aggravated by a band for the malefactors were more than three
armed robbers acting together; with treachery for the robbers tied
the hand of the victims before killing them; by night time or nocturnity
for the accused took advantage of the night; and by dwelling for the
robbery was committed with violence against or intimidation of
persons, and the commission of the crime begun in the dwelling.
ISSUE/S Whether or not the aggravating circumstance of dwelling may be
appreciated
RULING/S The Court affirmed the trial court in appreciating dwelling as an
aggravating circumstance. Generally, dwelling is considered
inherent in the crimes which can only be committed in the abode of
the victim, such as trespass to dwelling and robbery in an inhabited
place. However, in robbery with homicide, the authors thereof can
commit heinous crime without transgressing the sanctity of the
victim’s domicile. In the said case, the robbers demonstrated an
impudent disregard of the inviolability of the victims' abode when
they forced their way in, looted their houses, intimidated and
coerced their inhabitants into submission, disabled Laurencio and
Jimmy by tying their hands before dragging them out of the house
to be killed.
However, with regard to other issues on aggravating circumstances,
treachery cannot be appreciated in this case. The accused tried
and convicted of robbery with homicide. This special complex crime
is primarily classified in this jurisdiction as a crime against property,
and not against persons, homicide being merely an incident of
robbery with the latter being the main purpose and object of the
criminals. Also, band cannot be appreciated for there were no
records that more than three of the robbers were armed. Moreover,
the aggravating circumstance of night time did not attend the
commission of the crime. To be properly so considered, it must be
shown that nocturnidad was deliberately and intentionally sought by
accused-appellants to help them realize their evil intentions. In the
case at bar, the houses of the victims were adequately lighted by
kerosene lamps when the robbers entered and went about their
looting spree, and they also , lingered in the locus criminis and even
conversed with their intended victims for an appreciable period of
time inside the well-lit houses.

417
CONTRIBUTOR TERTE, Karen A.
MODULE Aggravating circumstances; Treachery
TOPIC
CASE TITLE PEOPLE v. ESCOTE G.R.NO. 140756
PONENTE CALLEJO, SR. J.: DATE: APRIL 4, 2003
DOCTRINE Mandated in Rule 110 Sec. 8 of the Revised Rules on Criminal
Procedures that the aggravating and qualifying circumstances of
an offense should be stated in the complain or information. Even if
treachery is proven but it is not alleged in the information,
treachery cannot aggravate the penalty for the crime.

Treachery is a generic aggravating circumstance to robbery with


homicide although said crime is classified as a crime against
property and a single and indivisible crime. Treachery is a generic
aggravating circumstance in robbery with homicide when the victim
of homicide is killed by treachery. There is treachery when the
following essential elements are present, viz: (a) at the time of the
attack, the victim was not in a position to defend himself; and (b)
the accused consciously and deliberately adopted the particular
means, methods or forms of attack employed by him.
FACTS Past midnight on September 28, 1996, Rodolfo Cacatian, driver of
Five Star Passenger Bus, drove the bus from its terminal at Pasay
City to its destination in Bolinao, Pangasinan. Also on board was
Romulo Digap, the regular conductor of the bus, as well as some
passengers. At Camachile, Balintawak, six passengers boarded the
bus, including Victor Acuyan and Juan Gonzales Escote, Jr.

While the bus was travelling along the highway in Plaridel, Bulacan,
Juan and Victor suddenly stood up, whipped out their handguns and
announced a holdup. Juan fired his gun upward to awaken and
scare off the passengers. Victor followed suit and fired his gun
upward. Juan and Victor then accosted the passengers and
divested them of their money and valuables. SPO1 Manio, another
passenger, was approached by the two to demand from him his
wallet and ID. When they found that SPO1 Manio is a member of
the police force, they used his own service gun to shoot him despite
Manio’s plea for safety. He sustained six entrance wound: mouth,
right ear, chest and right side of the body.

The robbers assured Rodolfo that if the latter will follow their
instructions, he will not be harmed. Victor and Juan ordered Rodolfo
to stop the bus along the overpass in Mexico, Pampanga where they
alighted from the bus. The robbery was over in 25 minutes. Upon
reaching Dau, Pampanga, Rodolfo and Romulo reported the
incident to the police.

418
Barely a month after the incident, Escote was stopped at a
checkpoint along the national highway of Tarlac. Escote claimed
that he was a policeman and handed SPO1 Manio’s I.D. The police
manning the checkpoint became suspicious as the ID has already
expired. They, then, asked Escote for a new payslip which the latter
could not provide. Escote confessed that he was not a policeman
ans was brought to the station where he also admitted of the robbery
that he and Acuyan committed.
ISSUE/S Whether or not the aggravating circumstance of Treachery was
present in this case.
RULING/S No, Treachery cannot be appreciated against Juan and Victor.
Treachery was present in the commission of the crime however, it
cannot be appreciated in this case because it was not alleged in
the information as mandated by Section 8, Rule 110 of the
Revised Rules on Criminal Procedures. Although at the time the
crime was committed, generic aggravating circumstances need not
be alleged in the Information, however, the general rule had been
applied retroactively because if it is more favorable to the accused.
Even if treachery is proven but it is not alleged in the information,
treachery cannot aggravate the penalty for the crime.
The essence of treachery is the sudden and unexpected attack by
an aggressor on the unsuspecting victim, depriving the latter of
any chance to defend himself and thereby ensuring its commission
without risk of himself. Treachery may also be appreciated even if
the victim was warned of the danger to his life where he was
defenseless and unable to flee at the time of the infliction of the
coup de grace. In the case at bar, both perpetrators were armed.
On the other hand, when the victim was shot, he was defenseless.
He was shot at close range, thus ensuring his death. The victim
was on his way to rejoin his family after a hard day's work. Instead,
he was mercilessly shot to death, leaving his family in grief for his
untimely demise. The killing is a grim example of the utter
inhumanity of man to his fellowmen
Although Treachery applies to crimes against persons only and
robbery with homicide is a crime against persons, it can still be
appreciated as a generic aggravating circumstance. Treachery is a
generic aggravating circumstance in robbery with homicide when
the victim of homicide is killed by treachery.

419
CONTRIBUTOR TIDALGO, Aimee Diane A.
MODULE Aggravating Circumstance; Treachery and Abuse of superior
TOPIC strength
CASE TITLE PEOPLE V. VILLONEZ G.R.NO. 122976-77
PONENTE DAVIDE, JR., J. DATE: NOV. 16, 1998
DOCTRINE Treachery may still be appreciated even when the victim was
forewarned of danger to his person
FACTS On May 3 1994, while Edgar Jimenez while resting inside his store,
a certain Tonton informed him that his Gerardo Longasa had a
fistfight with Rudy “Dede”. Edgar proceeded to the area to mediate,
since Longasa and Rudy were both his friends. Edgar passed
through Javier II Street in going to Liwayway Street. At Javier II
Street, a group of seven armed men attacked Edgar. Ruel hit Edgar
on his forehead and back with a bottle. Edgar was able to escape
from his attackers. While fleeing, he ran past Longasa, who seemed
drunk. When Edgar called Longasa, the attackers were already
upon Longasa. While he was about eight arms’ length away from
Longasa, Edgar saw Emerlito hit Longasa with a 2 x 2 inches piece
of wood. Simultaneously, Regando and Ruel struck Longasa with
bottles. Rudy Santos and Eddie Santos then stabbed Longasa
seven and eight times, respectively, even as two other persons
named Rey and Budda held Longasa’s arms. Longasa fell to the
ground. Edgar saw all these because the scene of the incident was
illuminated by a big fluorescent lamp located about three arms’
length away. Edgar rushed to Longasa’s house and reported the
incident to the latter’s parents.

The RTC ruled against the presence of treachery, since LONGASA


was engaged in a fight with the accused before the fatal attack and
was, therefore, sufficiently warned of the assault against him.
However, it appreciated against the accused the qualifying
aggravating circumstance of taking advantage of superior strength
because of the superior number of the accused, most of whom were
armed with weapons; while the victim was alone, with his arms held
behind him by two of the assailants. Hence, this petition.
ISSUE/S Whether or not treachery and superior of strength as aggravating
circumstances should be appreciated
RULING/S Yes. The SC do not share the assessment of the RTC that there
was no treachery in this case because the victim had engaged in a
fight previous to the killing and was thus forewarned of an attack
against him. Treachery may still be appreciated even when the
victim was forewarned of danger to his person. What is decisive is
that the execution of the attack made it impossible for the victim to
defend himself or to retaliate. The overwhelming number of the
accused, their use of weapons against the unarmed victim, and the

420
fact that the victim’s hands were held behind him preclude the
possibility of any defense by the victim. However, on the other hand,
the other qualifying circumstance of abuse of superior strength,
which the trial court appreciated, will no longer be taken against
accused-appellants, for it is absorbed in treachery.

421
CONTRIBUTOR Valdez, Ariane Faye V.
MODULE Module 8- Art. 14, RPC- Aggravating Circumstances
TOPIC
CASE TITLE People v. Guzman G.R.NO. 169246
PONENTE CHICO-NAZARIO, J. DATE: 26 January 2007
DOCTRINE Treachery is a sudden and unexpected attack under the
circumstances that renders the victim unable and unprepared to
defend himself by reason of the suddenness and severity of the
attack.
FACTS On 25 November 1999, at about 9:00 in the evening, after attending
a worship service at the Iglesia ni Kristo church in his barangay,
Michael Balber (victim) proceeded home.

While Michael was casually walking along the corner of Sto. Nino
Street and Mactan Street, Nicolas Guzman (accused-appellant) and
his two companions, who were drinking nearby, suddenly
approached and surrounded Michael. Appellant positioned himself
at the back of Michael while his two companions stood in front of
Michael.

In an instant, they grabbed the shoulders of Michael and


overpowered the latter. One of the appellant’s companions, whom
the prosecution witnesses described as a male with long hair, drew
out a knife and repeatedly stabbed Michael on the
stomach. Unsatisfied, the appellant’s other companion, whom the
prosecution witnesses described as a male with flat top hair, took
the knife and stabbed Michael on the stomach. As the finale,
appellant went in front of Michael, took the knife and also stabbed
Michael on the stomach. When Michael fell on the ground, appellant
kicked him at the body. Upon noticing that the bloodied Michael was
no longer moving, appellant and his two companions fled the scene.

After the disturbing events, the prosecution’s witnesses together


with Danilo, Michael’s father, went to Batasan Hills Police Station 6
and gave statement about the incident.

On 12 November 2001, the RTC rendered its Decision convicting


appellant of murder. Appellant filed a Notice of Appeal on 26
November 2001.

On 28 February 2005, the Court Appeals promulgated its Decision


affirming with modification the RTC Decision. The modification
pertains only to the penalty imposed by the RTC. Hence, this
petition.
ISSUE/S 1. Whether or not the lower court erred in appreciating the
qualifying circumstance of treachery.

422
2. Whether or not the killing of Michael was attended by an
aggravating circumstance of evident premeditation.
RULING/S 1. No. Treachery is a sudden and unexpected attack under the
circumstances that renders the victim unable and unprepared
to defend himself by reason of the suddenness and severity
of the attack. It is an aggravating circumstance that qualifies
the killing of a person to murder. Article 14, paragraph (16) of
the Revised Penal Code states the concept and essential
elements of treachery as an aggravating circumstance, thus:
Art. 14 par.16. That the act be committed with
treachery (alevosia).
There is treachery when the offender commits any of
the crimes against the person, employing means,
methods, or forms in the execution thereof which tend
directly and specially to insure its execution, without
risk to himself arising from the defense which the
offended party might make.
As viewed from the foregoing facts, the suddenness and
unexpectedness of the attack of appellant and his two
companions rendered Michael defenseless, vulnerable and
without means of escape. It appears that Michael was
unarmed and alone at the time of the attack. Further, he was
merely seventeen years of age then. In such a helpless
situation, it was absolutely impossible for Michael to escape
or to defend himself against the assault of appellant and his
two companions. Being young and weak, Michael is certainly
no match against adult persons like appellant and his two
companions.
The two essential elements/conditions are required in
order that treachery may be appreciated:
a. The employment of means, methods or manner of
execution that would ensure the offender’s safety from any
retaliatory act on the part of the offended party, who has, thus
no opportunity for self-defense or retaliation;
b. Deliberate or conscious choice of means, methods or
manner of execution.
In applying the essential elements of treachery in the case at
bar:
a. Michael was also outnumbered since he had three
assailants, and, was unarmed when he was stabbed to death.
Appellant and his two companions took advantage of their size,
number, and weapon in killing Michael.

423
b. They also deliberately adopted means and methods in
exacting the cruel death of Michael by first surrounding him, then
grabbing his shoulders and overpowering him. Afterwards, each of
them repeatedly stabbed Michael with a knife at the stomach until
the latter fell lifeless to the ground. The stab wounds sustained by
Michael proved to be fatal as they severely damaged the latter’s
large intestine.
Therefore, the lower court did not err in appreciating the
qualifying circumstance of treachery.
2. No. The essence of evident premeditation as an
aggravating circumstance is that the execution of the criminal act
was preceded by cool thought and reflection upon the resolution to
carry out the criminal intent during a space of time sufficient to arrive
at a calm judgment. It implies a deliberate planning of the crime
before executing it. It must also be shown how and when the plan
to kill was hatched or what time elapsed before it was carried out.
Further, there must be proof that the accused meditated and
reflected on his intention between the time when the crime was
conceived by him and the time it was actually perpetrated.
In the case at bar, there is no evidence to show that appellant
and his two companions had previously planned and
reflected in killing Michael. When appellant and his two
companions saw Michael on that fateful night, they
immediately pounced on him. The thought of killing Michael
came into the minds of appellant and his two companions
only when they saw Michael walking on the road. Indeed, the
killing of Michael was sudden and unplanned.
Therefore, killing of Michael was not attended by an
aggravating circumstance of evident premeditation. The
aggravating circumstance that can only be appreciated in the
case at bar is treachery (alevosia).

424
CONTRIBUTOR VILLANUEVA, Sean Ruthie L.
MODULE AGGRAVATING CIRCUMSTANCES
TOPIC
CASE TITLE PEOPLE v. TORRIEFEL G.R.NO. 659-R
PONENTE TUASON, J.: DATE: NOV. 29, 1947
DOCTRINE Ignominy is a circumstance pertaining to the moral order, which
adds disgrace and obloquy to the material injury caused by the
crime. This aggravating circumstance is applicable when the crime
committed is against chastity.
FACTS On December 17, 1942 of the afternoon, Torriefel and Ormeo were
on their way to USSAFE headquarters in the mountains. Passing
Eady’s place, they asked for khakis but Eady had none except for
what he was wearing. Ceferina Cordero appeared and inquired
about their mission. She scolded Torriefel and Ormeo because their
belongings were looted by USSAFE soldiers. Torriefel threatened
and slapped her while holding his revolver. Eady and Cordero were
charged with being fifth columnists as they refused to give aid and
was subsequently, taken to the USSAFE headquarters.

Torriefel held Eady while Ormeo held Cordero. Both of their hands
were free but were blindfolded. Cordero kept calling out for Eady in
order to know if he was following, until later Eady did not respond
anymore so they stopped to wait for them. Torriefel had taken the
wrong way so he went back to a guardhouse and left Eady there.
He tried to find a way to overtake Ormeo and Cordero but was
unsuccessful. At the guardhouse, Eady was able to escape.
Torriefel followed a different path enabling him to find Ormeo and
Cordero. Ormeo ran to find Eady upon knowledge of his escape
which left Cordero with Torriefel. As Cordero was about to urinate,
Torriefel pushed, carried and laid her on a log. He began
unbuttoning his pants and wrapped cogon leaves around his genital
which Cordero witnessed when her blindfold fell down a little.
Pressing her neck in order to suppress her voice, Torriefel had
sexual intercourse with her. Ormeo also raped her. The soldiers
desisted from bringing Cordero to their headquarters and returned
her to her home. A servant informed Cordero that Eady left. Upon
his return, Cordero instantaneously told Eady that she was abused
by the soldier.
ISSUE/S Whether or not the lower court erred in appreciating nighttime as
aggravating circumstance instead of ignominy.
RULING/S Yes, the lower court erred in appreciating nighttime since the whole
incident started early in the afternoon while ignominy should be
present since the novelty of the act of wrapping cogon leaves on his
genital before raping Cordero increased the wrong done through the
heightened pain with added moral disgrace. The exertion of force or

425
violence is implied in the term “rape”. Pushing the victim down
proves that there is force.

426
CONTRIBUTOR AGUILAR, Jose Maria L.
MODULE AGGRAVATING CIRCUMSTANCES
TOPIC
CASE TITLE PEOPLE v. SULTAN G.R.NO. 132470
PONENTE BELLOSILLO, J.: DATE: APRIL 27, 2000
DOCTRINE Additional rapes committed should not be appreciated as an
aggravating circumstance despite a resultant "anomalous situation"
wherein robbery with rape would be on the same level as robbery
with multiple rapes in terms of gravity.
FACTS According to the victim herself, at sometime in the evening she was
on her way home she was accosted by the accused-appellant who
pointed a sharp instrument at her neck and announcing it was a
"hold-up." He grabbed her and brought her to a house along the
alley. He made her sit down and then he started divesting her of her
watch, ring, earrings, and necklace. After taking her valuables, he
started kissing her on the lips and cheeks. As if to discourage him
from making further sexual advances, she told him that she was
married with two children. While pointing an ice pick at her he
ordered her to undress. She was under constant threat. After she
had completely undressed, still she could not resist him because of
fear.

After satisfying his lust, he tied her hands and went out of the room
to smoke. After few minutes he came back and once again with
threat and intimidation sexually abused her. Thereafter, he told her
that he loved her and that he would answer for what he had done to
her. In her effort to release herself, she convinced that she was
going to run away with him, he allowed her to go home at noon to
get her things. When she arrived home, they Immediately called her
brother who is a police officer (SPO1). SPO1 then advised the victim
to go back to the house of accused-appellant for the "planned
elopement.”

When she arrived at accused-appellants place, he was already


waiting for her outside the store nearby. They boarded a passenger
bus while SPO1 and his companions trailed them. When the bus
slowed down because of the traffic, SPO1 and his companions
boarded the bus. They immediately approached accused-appellant
and boxed him before they could arrest him. Special complex crime
of robbery with rape was filed against accused-appellant.
ISSUE/S Whether or not the multiplicity of rapes committed could be
appreciated as an aggravating circumstance in this case.
RULING/S No. In the recent case of People v. Regala, the Court realized that
there was no law providing for the additional rape/s or homicide/s
for that matter to be considered as aggravating circumstance. It

427
further observed that the enumeration of aggravating circumstances
under Art. 14 of the Revised Penal Code is exclusive, unlike in Art.
13 of the same Code which enumerates the mitigating
circumstances where analogous circumstances may be considered,
hence, the remedy lies with the legislature. Consequently, unless
and until a law is passed providing that the additional rape/s or
homicide/s may be considered aggravating, the Court must
construe the penal law in favor of the offender as no person may be
brought within its terms if he is not clearly made so by the statute.
Under this view, the additional rape committed by accused-
appellant is not considered an aggravating circumstance. Hence,
applying Art. 63 of the Revised Penal Code, the lower penalty
of reclusion perpetua should be imposed on accused-appellant.

428
CONTRIBUTOR ALGURA, Nino N.
MODULE Aggravating Circumstances
TOPIC
CASE TITLE PEOPLE OF THE PHILIPPINES G.R.NO. 136149-51
v
WALPAN LADJAALAM y MIHAJIL
alias "WARPAN”
PONENTE PANGANIBAN, J.: DATE: Sept. 19,
2000
DOCTRINE Section 1 of RA 8294 substantially provides that any person who
shall unlawfully possess any
firearm or ammunition shall be penalized, “unless no other crime
was committed”.
FACTS Walpan Ladjaalam y Mihajil, also known as "Warpan," appeals
before us the September 17, 1998 Decision of the Regional Trial
1

Court (RTC) of Zamboanga City (Branch 16), which found him guilty
of three out of the four charges lodged against him.
In the afternoon of September 24, 1997, more than thirty
(30) policemen proceeded to the house of appellant and his wife to
serve the search warrant when they were met by a volley of gunfire
coming from the second floor of the said house.
They saw that it was the appellant who fired the M14 rifle
towards them. Policemen took cover in the concrete fence.
Upon seeing to policemen 1 old woman went up with a child.
This opted one of this policeman to shout “Don’t shoot the second
floor! There are children”

The two of the police officers proceeded to the second floor where
they earlier saw appellant firing the rifle.

As he noticed their presence, the appellant


went inside the bathroom, dismantled the window, and jumped from
the window to the roof of a neighboring house.

He was subsequently arrested at the back of his house after a brief


chase.

Several firearms and ammunitions were recovered from appellant’s


house.

Also found was a pencil case with fifty (50) folded aluminum foils
inside, each containing methamphetamine hydrochloride.

A paraffin test was conducted and the casts taken both hands
of the appellant yielded positive for gunpowder nitrates.

429
Records show that appellant had not filed any application
for license to possess firearm and ammunition, nor has he been
given authority to carry firearms.

ISSUE/S Whether or not such use of an unlicensed firearm shall be


considered as an aggravating circumstance.
RULING/S No.
The appealed Decision was affirmed with modifications. Appellant
is found guilty only of two offenses:
(1) Direct assault and multiple attempted homicide with the use of a
weapon and;
(2) Maintaining a drug den. Section 1 of RA 8294 substantially
provides that any person who shall unlawfully possess any
firearm or ammunition shall be penalized, “unless no other crime
was committed”.
Furthermore, if homicide or murder is committed with the use of an
unlicensed firearm, such use of an unlicensed firearm shall be
considered as an aggravating circumstance. Since the crime
committed was direct assault and not homicide or murder, illegal
possession of firearms cannot be deemed an aggravating
circumstance. The law is clear: the accused can be convicted of
simple illegal possession of firearms, provided that
“no other crime was committed by the person arrested.”
If the intention of the law in the second paragraph were to refer only
to homicide and murder, it should have expressly said so, as it did
in the third paragraph. Verily, where the law does not distinguish,
neither should [the courts].

430
CONTRIBUTOR AROZA, Maria Minette R.
MODULE Aggravating Circumstances – Illegal Possession of Firearms
TOPIC
CASE TITLE CELINO v. COURT OF G.R.NO. 170562
APPEALS
PONENTE CARPIO-MORALES, J DATE: JUN 29, 2007
DOCTRINE The accused can be convicted of illegal possession of firearms,
provided no other crime was committed by the person
arrested. The word “committed” taken in its ordinary sense, and in
light of the Constitutional presumption of innocence, necessarily
implies a prior determination of guilt by final conviction resulting
from successful prosecution or voluntary admission.
FACTS Two separate information were filed before the Regional Trial Court
of Roxas City charging petitioner with violation of COMELEC
Resolution No. 6446 prohibiting during the election period the
bearing, carrying and transporting of firearms or deadly weapons by
any person, and Republic Act No. (R.A.) 8294 for illegal possession
of firearm.
ISSUE/S Whether or not the mere filing of an information for gun ban
violation against him necessarily bars his prosecution for illegal
possession of firearm.
RULING/S The accused can be convicted of illegal possession of firearms,
provided no other crime was committed by the person arrested. The
word “committed” taken in its ordinary sense, and in light of the
Constitutional presumption of innocence, necessarily implies a prior
determination of guilt by final conviction resulting from successful
prosecution or voluntary admission. When the other offense
involved is one of those enumerated under R.A. 8294, any
information for illegal possession of firearm should be quashed
because the illegal possession of firearm would have to be tried
together with such other offense, either considered as an
aggravating circumstance in murder or homicide, or absorbed as an
element of rebellion, insurrection, sedition or attempted coup d’etat.
Conversely, when the other offense involved is not one of those
enumerated under R.A. 8294, then the separate case for illegal
possession of firearm should continue to be prosecuted.

431
CONTRIBUTOR PROVIDO, Gemy Hale A.
MODULE Aggravating Circumstance – Qualifying Circumstance of
TOPIC Treachery, Dwelling and the Use of Disguise
CASE TITLE PEOPLE v. SIBBU G.R.NO. 214757
PONENTE DEL CASTILLO, J: DATE: MAR. 29, 2017
DOCTRINE Article 14, Paragraph 16 of the Revised Penal Code.. The act be
committed with treachery. There is treachery when the offender
commits a crime against a person employing means, methods or
forms which insures its execution and makes the victim defenseless.
FACTS Tirso Sibbu is charged with four criminal cases for three counts of
murder and one attempted murder. On December 6, 2004, in Brgy.
Elizabeth, Marcos, Ilocos Norte, Tirso Sibbu, committed an
attempted murder with Bryan Julian having two shots against him
being missed. In addition to that, he is charged with the three counts
of murder for killing Trisha May Julian, Ofelia Julian and Warlito
Julian on the same day.
ISSUE/S Whether or not the qualifying circumstance of treachery is
appreciated
RULING/S Yes, treachery is to be appreciated as a qualifying circumstance.
Treachery is present as the victims were unaware of the impending
attack of the accused which made them defenseless from their
attack. There is also security in the execution of their act as the
victims were unarmed. The victims were also defenseless and there
was a swift attack from the offender.
Another aggravating circumstance to be appreciated is the
aggravating circumstance of dwelling. The victims were shot inside
their house, even when the accused shot from the outside of the
victim´s house. As long as the victims stand inside their house, the
aggravating circumstance of dwelling is to be appreciated.
The last aggravatng circumstance to be appreciated is the
aggravating circumstance of the use of disguise. The accused used
disguise when he had covered his face using a bonnet while
committing the crime.
Wherefore, the accused is charged with three counts of murder and
one count of attempted murder. Since there is a presence of two
aggravating circumstances, namely dwelling and the use of
disguise, the accused is hereby sentenced to death.

432
CONTRIBUTOR ROMERO, Ma. Camille Concepcion M.
MODULE Aggravating Circumstances : Abuse of Superior Strength
TOPIC
CASE TITLE People v. Nazareno G.R.NO. 196434
PONENTE ABAD, J. DATE: OCT 24, 2012
DOCTRINE There is abuse of superior strength when the aggressors purposely
use excessive force rendering the victim unable to defend himself.
The notorious inequality of forces creates an unfair advantage for
the aggressor.
FACTS Magallanes, and Francisco attended the wake of a friend, they
drank liquor with Nazareno and Saliendra. Magallanes and
Nazareno got into a heated argument but their companions stopped
them

On the next day David, Magallanes, and Francisco returned to the


wake. The accused and Saliendra also arrived and told the three to
not mind what happened the previous night. At around 9:30 in the
evening, while Francisco and David were walking on the
street, Nazareno and Saliendra blocked their path. Nazareno boxed
Francisco who fled but Saliendra went after him with a balisong.

Francisco, who succeeded in hiding saw that Nazareno hit David on


the body with a stick while Saliendra struck David's head with a
stone. David then ran towards a gasoline station but Nazareno and
Saliendra, aided by some barangay tanods, caught up with him. As
David fell, the barangay tanods took over the assault.

Dr. Rebosa performed surgery on David's head but he died

In his defense, Nazareno claimed that he left his house at around


9:30 in the evening to buy milk. While on a street near his house,
he noted a commotion taking place nearby. He then bumped into
Saliendra. Nazareno proceeded home and went to bed. His wife
supported his testimony.
ISSUE/S Whether or not a qualifying circumstance of abuse of superior
strength attended the killing of David.
RULING/S Yes, there is abuse of superior strength when the aggressors
purposely use excessive force rendering the victim unable to defend
himself. The notorious inequality of forces creates an unfair
advantage for the aggressor.
Here, Nazareno and Saliendra evidently armed themselves
beforehand with a stick and heavy stone. David was unarmed. The
two chased him even as he fled from them. And when they caught
up with him, aided by some unnamed barangay tanods, Nazareno
and Saliendra exploited their superior advantage and knocked the

433
defenseless David unconscious. He evidently died from head
fracture caused by one of the blows on his head.
On the matter of penalty, the Court affirms the imposition of
reclusion perpetua.

434
CONTRIBUTOR SAMONTE, Vanessa Antoinette
MODULE AGGRAVATING AND MITIGATING CIRCUMSTANCES
TOPIC
CASE TITLE PEOPLE v. ALFREDO DULIN A.M. NO. G.R. NO.: 171284
PONENTE BERSAMIN ,J.: DATE: JUNE 29, 2015
DOCTRINES 1.Unless the victim committed unlawful aggression against the
accused, self-defense, whether complete or incomplete, should not
be appreciated, for the two other essential elements of self-defense
would have no factual and legal bases without any unlawful
aggression to prevent or repel.
2.The essence of treachery is that the attack comes without
warning, or is done in a swift, deliberate and unexpected manner,
affording the hapless, unarmed and unsuspecting victim no chance
to resist or to escape, without the slightest provocation on the part
of the victim.

FACTS Appellant was called by Angel Bancud, and as he approached him,


the victim Batulan, who was the cousin of appellant’s mother,
stabbed him on the right side of his body and in the left hand.
Appellant tried to run away but the victim pursued appellant and
stabbed him several times.They grappled for the weapon until
appellant was able to wrest it from the victim. When appellant was
able to gain possession of the weapon, he inflicted 12 stab wounds
on the victim which caused the death of the latter.

ISSUE/S 1. Whether or not appellant’s acts constituted incomplete self-


defense, and must be appreciated as a privileged mitigating
circumstance.
2. Whether or not treachery should be appreciated in the killing as
a qualifying circumstance.

RULING/S 1. No. Pursuant to Article 69 of the Revised Penal Code, the


privileged mitigating circumstance of incomplete self-defense
reduces the penalty than that prescribed by law. For this purpose,
the accused must prove the existence of the majority of the
elements for self-defense, but unlawful aggression, being an
indispensable element, must be present.
In the case at bar, with Batulan’s aggression having already ceased
from the moment that Dulin divested Batulan of the weapon, there
would not be any incomplete self-defense.
Whatever Dulin did thereafter like stabbing Batulan with the
weapon, constituted retaliation against Batulan. In this regard,
retaliation was not the same as self-defense. As such, there was no
unlawful aggression on the part of Batulan to justify his fatal
stabbing by Dulin.

435
2. No. Under Article 14, paragraph 16, of the Revised Penal Code,
treachery is present when the offender commits any of the crimes
against a person, employing means, methods or forms in the
execution thereof which tend directly and specially to insure its
execution, without risk to himself arising from the defense which
offended party might make.
Under the circumstances, treachery should not be appreciated in
the killing of Batulan because the stabbing by Dulin did not take
Batulan by surprise due to his having been sufficiently forewarned
of Dulin’s impending assault,and being thus afforded the opportunity
to defend himself, or to escape, or even to recover control of the
weapon from Dulin.

436
CONTRIBUTOR SAMSON, Jessa Viena D.
MODULE Classes of aggravating circumstances
TOPIC
CASE TITLE FANTASTICO v. MALICSE SR. G.R.NO. 190912
PONENTE PERALTA, J.: DATE: JAN. 12, 2015
DOCTRINE Abuse of superior strength is present whenever there is a notorious
inequality of forces between the victim and the aggressor, assuming
a situation of superiority of strength notoriously advantageous for
the aggressor selected or taken advantage of by him in the
commission of the crime. Further, the evidence must establish that
the assailants purposely sought the advantage, or that they had the
deliberate intent to use this advantage.
FACTS This is a petition for review on certiorari of petitioners Gary
Fantastico and Rolando Villanueva assailing the decision of the
Court of Appeals, affirming the decision of the Regional Trial Court
finding petitioners guilty of attempted murder.
Respondent Elpidio Malicse was outside the house of his sister
Isabelita Iguiron when all of a sudden, he heard Isabelita's son,
Winston, throwing invectives at him. Thus, Elpidio confronted
Isabelita, but she also cursed him, which prompted the former to
slap the latter. On that occasion, [he] was under the influence of
alcohol. He was eventually persuaded to go home where he drank
some coffee. Afterwards, he went back to the house of Isabelita to
offer reconciliation.
Upon reaching Isabelita’s house, Elpidio saw the former's son, Titus
and her son-in-law Gary Fantastico and asked the two where he can
find their parents. The [two] responded, “putang ina mo, and kulit
mo, lumayas ka, punyeta ka.” Elpidio kicked the door
open. Consequently, Salvador, also Isabelita’s son, hit Elpidio on
the right side of his head that forced the latter to bow his head, but
the former delivered a second blow that hit the latter on the right
eyebrow.
Gary hit Elpidio on the right side of his head with a tomahawk axe
when the latter was about to go out of the house. Elpidio tried to
defend himself, but was unable to take the tomahawk axe from
Gary. The former walked away from Titus; however, Gary was still
armed with the tomahawk axe and Salvador, with his arnis, including
Titus, chased him.
Roland Villanueva, without any warning, hit Elpidio on the back of
his head with a lead pipe which caused the latter to fall on the
ground. Elpidio begged his assailants to stop, but to no avail.
Salvador hit him countless times on his thighs, legs and knees using
the rattan stick.
Thereafter, a certain Mang Gil tried to break them off but Titus and
Gary shouted at him, “Huwag makialam, away ng mag-anak ito” and
the two continued to maul Elpidio. Elpidio then pretended to be

437
dead. The concerned neighbors approached him and rushed him
to the emergency room of the Philippine General Hospital (PGH).
ISSUE/S Whether or not the accused is guilty of qualifying circumstance of
employment of superior strength
RULING/S The Court affirmed the trial court in appreciating the qualifying
circumstance of abuse of superior strength.
The injuries inflicted on the complainant after he went back to
Isabelita's house. When he kicked the door, the melee began; and
the sequence of the injuries is proven by victim's
testimony. However, it was a lopsided attack as the victim was
unarmed, while his attackers were all armed (rattan stick, tomahawk
and lead pipe). Moreover, the victim was also drunk. This
establishes the element of abuse of superior strength.
Abuse of superior strength is present whenever there is a notorious
inequality of forces between the victim and the aggressor, assuming
a situation of superiority of strength notoriously advantageous for
the aggressor selected or taken advantage of by him in the
commission of the crime. The fact that there were two persons who
attacked the victim does not per se establish that the crime was
committed with abuse of superior strength, there being no proof of
the relative strength of the aggressors and the victim. The evidence
must establish that the assailants purposely sought the advantage,
or that they had the deliberate intent to use this advantage. To take
advantage of superior strength means to purposely use excessive
force out of proportion to the means of defense available to the
person attacked. The appreciation of this aggravating circumstance
depends on the age, size, and strength of the parties.

438
CONTRIBUTOR TERTE, Karen A.
MODULE Conspiracy and Proposal to commit Felony
TOPIC
CASE TITLE PEOPLE v. BOKINGCO G.R.NO. 187536
PONENTE PEREZ, J. DATE: AUGUST 10, 2011
DOCTRINE Principal by direct participation is those who materially execute the
crime and also who carried out and directly participated in the
execution of the crime. Conspiracy exists when two or more
persons come to an agreement to commit an unlawful act. It may
be inferred from the conduct of the accused before, during, and
after the commission of the crime. Conspiracy may be deduced
from the mode and manner in which the offense was perpetrated
or inferred from the acts of the accused evincing a joint or common
purpose and design, concerted action, and community of interest.
Unity of purpose and unity in the execution of the unlawful
objective are essential to establish the existence of conspiracy.
In order that the admission of a conspirator may be received
against his or her co-conspirators, it is necessary that first, the
conspiracy be first proved by evidence other than the admission
itself; second, the admission relates to the common object; and
third, it has been made while the declarant was engaged in
carrying out the conspiracy
FACTS Accused Michael Bokingco and Reynante Col are both employed by
the deceased Noli Pasion for the construction of his apartment.
From the testimonies of Pasion’s wife and brother-in-law Dante
Vitalicio, on February 29, 2000, they hear a commotion at Apartment
no. 3. Vitalicio was spin-drying his clothes inside his apartment
when he heard a commotion from Apartment No. 3. He headed to
said unit to check. He peeped through a screen door and saw
Bokingco hitting something on the floor. Upon seeing Vitalicio,
Bokingco allegedly pushed open the screen door and attacked him
with a hammer in his hand. A struggle ensued and Vitalicio was hit
several times. Vitalicio managed to push him away and Bokingco
tried to chase Vitalicio but was eventually subdued by a co-worker.
Vitalicio proceeded to his house and was told by his wife that Pasion
was found dead in the kitchen of Apartment No. 3. Vitalicio went
back to Apartment No. 3 and saw Pasion’s body lying flat on the
kitchen floor. Pasion and Vitalicio were brought to the hospital.
Pasion expired a few hours later while Vitalicio was treated for his
injuries.

Elsa testified that she was in the master’s bedroom on the second
floor of the house when she heard banging sounds and her
husband’s moans. She immediately got off the bed and went down.
Before reaching the kitchen, Col blocked her way. Elsa asked him
why he was inside their house but Col suddenly ran towards her,

439
sprayed tear gas on her eyes and poked a sharp object under her
chin. Col then instructed her to open the vault of the pawnshop but
Elsa informed him that she does not know the combination lock.
Elsa tried offering him money but Col dragged her towards the back
door by holding her neck and pulling her backward. Before they
reached the door, Elsa saw Bokingco open the screen door and
heard him tell Col: "tara, patay na siya." Col immediately let her go
and ran away with Bokingco. Elsa proceeded to Apartment No. 3.
Thereat, she saw her husband lying on the floor, bathed in his own
blood.

On arraignment, Bokingco entered a guilty plea while Col pleaded


not guilty. During the pre-trial, Bokingco confessed to the crime
charged.
ISSUE/S Whether or not Col is guilty of murder as co-conspirator.
RULING/S No, Col is not a co-conspirator in the crime done against Noli
Pasion.
In order to convict Col as a principal by direct participation, it is
necessary that conspiracy between him and Bokingco be proved.
Conspiracy exists when two or more persons come to an
agreement to commit an unlawful act. It may be inferred from the
conduct of the accused before, during, and after the commission of
the crime. Conspiracy may be deduced from the mode and
manner in which the offense was perpetrated or inferred from the
acts of the accused evincing a joint or common purpose and
design, concerted action, and community of interest. Unity of
purpose and unity in the execution of the unlawful objective are
essential to establish the existence of conspiracy.
The finding of conspiracy was premised on Elsa’s testimony that
appellants fled together after killing her husband and the
extrajudicial confession of Bokingco. Nobody witnessed the
commencement of the attack. Col was not seen at the apartment
where Pasion was being attacked by Bokingco. Based on these
acts alone, it cannot be logically inferred that Col conspired with
Bokingco in killing Pasion. At the most, Col’s actuations can be
equated to attempted robbery.
Elsa testified that she heard Bokingco call out to Col that Pasion
had been killed and that they had to leave the place. This does not
prove that they acted in concert towards the consummation of the
crime. It only proves, at best, that there were two crimes
committed simultaneously and they were united in their efforts to
escape from the crimes they separately committed. Their acts did
not reveal a unity of purpose that to kill Pasion. Bokingco had
already killed Pasion even before he sought Col. Their moves
were not coordinated because while Bokingco was killing Pasion

440
because of his pent-up anger, Col was attempting to rob the
pawnshop.

441
CONTRIBUTOR TIDALGO, Aimee Diane A.
MODULE Aggravating Circumstances; Treachery and Abuse of Superior
TOPIC Strength
CASE TITLE PEOPLE V. TABERNERO G.R.NO. 168169
PONENTE DAVIDE, JR. J. DATE: FEB. 24, 2010
DOCTRINE Despite the yelling which should have warned the victim of a
possible attack, the mere fact that the accused’s companions held
the hands of the victim while the accused stabbed him was
considered by this Court to constitute alevosia; The act of the
accused in stabbing the victim while two persons were holding
him clearly shows the deliberate use of excessive force out of
proportion to the defense available to the person attacked.
FACTS Gary, a 22-year-old construction worker who stayed in Ernesto’s
house from 1997 to 1999, as he and Mary Jane, daughter of
Teresita Acibar, the wife of Ernesto were living together. However,
Gary left the house shortly when Ernesto allegedly stopped the
planned marriage of Gary and Mary Jane, who was pregnant at that
time.

One day, to overcome with emotion over being separated from Mary
Jane, Gary went to Ernesto’s house, but was not able to enter. He
instead shouted his pleas from the outside, asking what he did
wrong and how much he loves Mary Jane. When Gary was about to
leave, Ernesto purportedly struck him with a lead pipe aiming at
Gary’s head, but the latter blocked the blow with his hands. Gary
embraced Ernesto, but the latter strangled him. At that point, Gary
felt that there was a bladed weapon tucked at Ernesto’s back.
Losing control of himself, Gary took the bladed weapon and stabbed
Ernesto. Gary was stunned, and did not notice his father, Alberto,
coming. Alberto asked what happened without noticing him coming,
Gary responded: “Nasaksak ko po ata si Ka Erning. Then, both fled
out of fear and parted ways. Alberto went to Norzagaray while Gary
was nowhere to be found.

On August 5, 2001, Alberto was apprehended. On August 20, 2001,


he pleaded NOT GUILTY to the charge. However, while Alberto’s
defense is denial and not self-defense like Gary’s, the court decided
to proceed with the reverse trial, as it had already started that way.

Alarma testified that while he was in a meeting at around 4:00 p.m.


on April 22, 2001, Gary arrived and told him of his intention to
surrender to him. Together with his fellow barangay tanod Zaldy
Garcia, Alarma brought Gary to the Malolos Police Station, where
the surrender was entered in the blotter report.
The first to testify for the prosecution was its eyewitness, Emerito
Acibar. Emerito, the brother of Mary Jane, was inside their house in

442
Daang Bakal, Longos, Malolos, Bulacan with his brother and his
stepfather, Ernesto, at around eleven o’clock on the night of the
incident on October 23, 1999. He heard somebody calling for
Ernesto, but ignored it. He then heard a “kalabog,” followed by
Ernesto’s plea for help. Emerito was about to go outside, but, while
he was already at the door of their one-room house, he saw Ernesto
being held by a certain Toning “Kulit” and another person, while
Gary and Alberto were stabbing Ernesto with fan knives. Emerito
lost count of the number of thrusts made by Gary and Alberto, but
each inflicted more than one, and the last stab was made by Alberto.
Emerito shouted for help. The four assailants left when somebody
arrived, allowing Emerito to approach Ernesto and bring him to the
Bulacan Provincial Hospital.
ISSUE/S Whether or not treachery could be qualified for the killing of Ernesto
RULING/S YES. Treachery is defined under Article 14(16) of the Revised Penal
Code, which provides: “There is treachery when the offender
commits any of the crimes against the person, employing means,
methods, or forms in the execution thereof which tend directly and
specially to insure its execution, without risk to himself arising from
the defense which the offended party might make.”

In the cases cited by the appellants, the eyewitnesses were not able
to observe any means, method or form in the execution of the killing
which rendered the victim defenseless. In Amamangpang, the first
thing the witness saw was the victim already prostrate on the
bamboo floor, blood oozing from his neck and about to be struck by
the accused. In Icalla, the witnesses merely saw the accused fleeing
from the scene of the crime with a knife in his hand. In Sambulan,
the witness saw the two accused hacking the victim with a bolo.
Since, in these cases, there was no restraint upon the victims or any
other circumstance which would have rendered them defense less,
the Court ruled that it should look into the commencement of the
attack in order to determine whether the same was done swiftly and
unexpectedly. However, the swiftness and unexpectedness of an
attack are not the only means by which the defenselessness of the
victim can be ensured.

SC therefore, rule that the killing of Ernesto was attended by


treachery. However, even assuming for the sake of argument that
treachery should not be appreciated, the qualifying circumstance of
abuse of superior strength would nevertheless qualify the killing to
murder. Despite being alleged in the Information, this circumstance
was not considered in the trial court as the same is already absorbed
in treachery. The act of the accused in stabbing Ernesto while two
persons were holding him clearly shows the deliberate use of

443
excessive force out of proportion to the defense available to the
person attacked.

444
MODULE 9
ALTERNATIVE
CIRCUMSTANCES AND
PERSONS CRIMINALLY LIABLE

445
CONTRIBUTOR Belano, Renato Jr. P.
MODULE Alternative Circumstances and Persons Criminally Liable
TOPIC
CASE TITLE People v. Atop G.R. NO. 124303-05
PONENTE Panganiban, J. DATE: Feb 10, 1998
DOCTRINE Outside these enumerations and consistent with the
doctrine that criminal laws must be liberally construed in favor of the
accused, no other relationship, kinship or association between the
offender and the victim may aggravate the imposable penalty for the
crime committed. The fact, then, that the offended party is the
granddaughter or descendant of appellant's live-in partner cannot
justify the imposition of death upon the rapist.
FACTS Four separate informations against accused-appellant charging him
with rape on three separate occasions - on October 9, 1992,
sometime in 1993 and on December 26, 1994 – as well as with
attempted rape on December 31, 1994.

Version of the Prosecution

Private complainant Regina Guafin told the court that she is a


granddaughter of Trinidad Mejos and the accused Atop is the
common law husband of said Trinidad Atop. Her mother is a
daughter of said Trinidad Atop. She is an illegitimate child and
doesn’t even know her father. Sometime in 1991 when she was
already 10 years of age the accused started having lustful desire on
her. The accused then inserted his finger into her vagina. She told
her grandmother about this, but her grandmother did not believe
her. She was then told by her grandmother, Trinidad Mejos, that
what her grandfather did to her was just a manifestation of fatherly
concern.

On October 9, 1992, she was called by the accused Alejandro Atop


to do something for him. When she approached him the accused
rushed towards her, removed her panty and inserted his male organ
into her vagina. She was not able to do anything to resist him
because the accused gagged her mouth and was carrying a knife
with him.

She also remember [sic] of another incident wherein she was raped
again by the accused Alejandro Atop. It was in the year 1993 but
she could not recall the month when it was committed.

On December 26, 1994, the accused again raped her. She could
not ask for help because her mouth was gagged by the accused.
Aside from gagging her, the accused also carried a knife which he
placed at his side.

446
On December 31, 1994, while she together with her Aunt Gloria
Montealto and her two (2) nieces Rubilen and Jubilen Atop were
about to go to sleep, she noticed that the accused was looking for
her. Upon seeing her the accused rushed towards her and was
about to lay on top of her. She kicked him. After that, the accused
caressed and touched his nieces but his nieces also kicked him.
Thereafter, the accused stopped molesting her and his nieces and
went to sleep instead.

She reported the incidents of rape that happened in 1992, 1993 and
1994 only in January 1995. It took her so long to report the said
incidents because she was afraid. The accused threatened to kill
her should she tell anybody about the incidents.

Version of the Defense

Appellant denied the accusations of Guafin and imputed ill motive


upon her aunts, who were the daughters of his live-in partner.

He denied committing rape against Regina Guafin on October 9,


1992, in the year 1993 and on December 26, 1994. He also denied
committing an offense against Regina Guafin on December 31,
1994. He testified also that he did not evade arrest by going out of
Matag-ob, Leyte because during that time he was working in Hideco
as a laborer. The reason why Regina Guafin filed a case against
him because the said private complainant was coached by her aunt
who wanted him and his wife Trinidad to be separated.

Ruling of the Trial Court

This court then gives the testimony of the private offended party full
faith and credit." The trial court also ruled that the circumstances of
nighttime and relationship aggravated all the three incidents of rape,
but that there was no sufficient evidence proving attempted rape
on December 31, 1994. Considering that the last rape occurred
after the effectivity of RA 7659, the death penalty law, the court
meted out the capital punishment to accused-appellant.
ISSUE/S Whether or not the aggravating circumstances of nighttime and
relationship should be appreciated against the accused for the rape
allegedly committed on October 9, 1992, in 1993 and on December
26, 1994
RULING/S Nighttime (no)
The time-settled rule is that nocturnity, as an aggravating
circumstance, must have been deliberately sought by the offender
to facilitate the crime or prevent its discovery or evade his capture

447
or facilitate his escape. The culprit must have purposely taken
advantage of the cover of night as an indispensable factor to attain
his criminal purpose.We find merit in Appellant Atop's contention, to
which the solicitor general agrees, that the prosecution failed to
prove that nighttime was deliberately sought by appellant to facilitate
his dastardly acts. In fact, the prosecution failed to show that
appellant consummated his carnal designs at night, except only for
the December 26, 1994 incident which the victim said occurred at
11:00 p.m. 16 Much less is here any evidence substantiating the
trial court's conclusion that appellant intentionally sought the
darkness to advance his criminal exploits.

Relationship (no)

The scope of relationship as defined by law encompasses:


1. the spouse;
2. an ascendant;
3. a descendant;
4. a legitimate, natural or adopted brother or sister; or
5. a relative by affinity in the same degree.

Relationship by affinity refers to a relation by virtue of a legal bond


such as marriage. Relatives by affinity therefore are those
commonly referred to as "in-laws," or stepfather, stepmother,
stepchild and the like; in contrast to relatives by consanguinity or
blood relatives encompassed under the second, third and fourth
enumeration above. The law cannot be stretched to include persons
attached by common-law relations. Here, there is no blood
relationship or legal bond that links the appellant to his victim.
Thus, the modifying circumstance of relationship cannot be
considered against him.

Neither is the following provision of Sec. 11, RA 7659 applicable:


"Sec. 11. Article 335 of the [Revised Penal] Code is hereby
amended to read as follows:
xxx
The death penalty shall also be imposed if the crime of rape is
committed with any of the following attendant circumstances:

1. when the victim is under eighteen (18) years of age and the
offender is a parent, ascendant, step-parent, guardian, relative by
consanguinity or affinity within the third civil degree, or the common
law spouse of the parent of the victim.
xxx
Undisputed is the fact that appellant is not the common-law spouse
of the parent of the victim. He is the common-law husband of the

448
girl's grandmother. Needless to state, neither is appellant the
victim's "parent, ascendant, step-parent, guardian, relative by
consanguinity or affinity within the third civil degree." Hence, he is
not encompassed in any of the relationships expressly enumerated
in the aforecited provision.

449
CONTRIBUTOR BILTZ, Aralind Louise A.
MODULE Alternative Circumstances
TOPIC
CASE TITLE US vs. McMann G.R.NO. L-2229
PONENTE WILLARD, J. DATE: JULY 1, 1905
DOCTRINE The exact degree of intemperance which constitutes a drunkard it
may not be easy to define, but speaking in general terms, and with
the accuracy of which the matter is susceptible, he is a drunkard
whose habit is to get drunk, 'whose ebriety has become habitual.'
To convict a man of the offense of being a common drunkard it is,
at the least, necessary to show that he is a habitual drunkard.
FACTS Defendant McMann and on Mckay were packert at Camp Vicars in
Mindano. McKay went to the house of a Moro, Amlay Pindolonan,
for the purpose of getting matches with which to light his cigar. With
his revolver in his hand he attempted to enter the house, but the
owner would not allow him to do so. A few moments later, defendant
McMann arrived and tried to enter the house but the owner refused.
Soon after, McKay and the Moro Pindolonan were seated side by
side at a distance of 3 to 6 feet form the defendant, the latter raised
his pistol and fired at McKay. The bullet struck the back of his head
and killed him instantly. The Moro jumpred up and lookd around to
see where the shot came. He started to run and the defendant shot
him. The defendant and McKay were both drunk at this time.
ISSUE/S Whether or not the drunkness of the defendant may be taken into
consideration to lessen his sentence
RULING/S NO.
The defendant was drunk at the time the act was committed, but
such drunkenness was habitual with him and therefore his
condition could not be taken into consideration for the purpose of
lessening the sentence. Such habitual drunkness of the defendant
was proven by the testimony of a witness.
In the case of Ludwick vs. Commonwealth, the court said:
"A man may be an habitual drunkard, and yet be sober for days
and weeks together. The only rule is, Has he a fixed habit of
drunkenness? Was he habituated to intemperance whenever the
opportunity offered?"
Therefore, the judgment of the court below is affirmed.

450
CONTRIBUTOR CACHERO, Luis III L.
MODULE Aggravating Circumstances
TOPIC
CASE TITLE PEOPLE V LUISITO SAN G.R.NO. L-44274
PEDRO, ET AL.
PONENTE PER CURIAM DATE: January 22,
1980
DOCTRINE When the aggravating circumstances of craft and treachery are
present, craft is not to be absorbed with treachery in the special
complex crime of Robbery with Homicide as craft was employed not
with a view to making treachery more effective.
When the mitigating circumstance of lack of instruction is alleged, it
must be proven that there is lack of sufficient intelligence to commit
an offense. Illiteracy alone is not grounds for this mitigating
circumstance to be appreciated.
FACTS In the afternoon of June 2, 1970, a lifeless body of a person was
found between the barrios of Masay and Paciano Rizal in the
municipality of Bay, Laguna. The deceased was identified to be
Filemon Rivera, a driver of a passenger jeep belonging to Pablito
Delos Reyes.
It was not until June 11, 1971 that the police authorities found a
concrete lead to the solution of the case. One Rodrigo Esguerra was
apprehended and interviewed by police, admitted his participation
and named his companions. Another suspect named Artemio
Banasihan was then apprehended sometime in 1972. He has given
a sworn statement confessing his participation in the robbery and
killing of Felimon Rivera. Carrying out their plan, he and Luisito San
Pedro approached Rivera in the afternoon of June 2, 1970 on the
pretext of hiring Rivera’s jeep to haul coconuts. They then
proceeded to Bo. Puypuy in Bay, Laguna where they were joined by
Salvador Litan and Rodrigo Esguerra. Upon reaching a river, San
Pedro ordered Rivera to stop. This was then followed by Litan hitting
Rivera with a water pipe on his nape. Rivera jumped out of the jeep
but was chased by San Pedro and Litan who stabbed him at the
back several times with a dagger. Esguerra then drove the jeep and
the group proceeded to Makati, Rizal. The jeep was then brought to
Cavite City where it was sold for Php 2,000. Four days later, San
Pedro and Litan, and Banasihan were paid 50 pesos each, with
promise that the balance would be given later. However, the
promised balance was not given to them.
Alleged in the Information were the aggravating circumstances of
craft, treachery and the mitigating circumstance of lack of
instruction.

451
ISSUE/S Whether or not the aggravating circumstance of craft is absorbed by
treachery.

Whether or not the single aggravating circumstance of treachery


should be offset by the mitigating circumstance of lack of
instruction.
RULING/S The Court does not subscribe to the theory of craft being absorbed
by treachery. Craft was not employed with a view to making
treachery more effective. It was directed actually towards facilitating
the taking of the jeep in the robbery scheme as planned by the
culprits. Treachery was then correctly appreciated by the lower
court.

The mitigating circumstance of lack of instruction is not applicable


to crimes of theft and robbery, much less to the crime of homicide.
The criteria in determining lack of instruction is not illiteracy alone,
but rather lack of sufficient intelligence.

The Court affirmed the lower court’s decision imposing the death
penalty.

452
CONTRIBUTOR CALZADO, Anne Valerie L.
MODULE Persons Criminally Liable
TOPIC
CASE TITLE PEOPLE v. YANSON- G.R.NO. 133527-28
DUMANCAS
PONENTE MELO, J. DATE: DECEMBER 13,
1999
DOCTRINE Person may be convicted as principal by inducement, the following
must be present: (1) the inducement be made with the intention of
procuring the commission of the crime, and (2) such
inducement be the determining cause of the commission by the
material executor.
FACTS Jeanette Yanson Dumancas was swindled in a fake gold bat
transaction losing P352,000 to Danilo Lumangyao and Rufino
Gargar, jr, both deceased. On August 5, 1992, Dominador Geroche,
Jaime Gargallano, Edwin Divinagracia, Teody Delgado, Mario
Lamis and Moises Grandez were at the house of Rolando
Fernandez when they planned to abduct the deceased because of
the swindling incident. They went to the office of Col. Nicolas Torres
at PNP Headquarters where they met the colonel who told to bring
and hide them at Dragon Lodge Motel. Grandeza, state witness,
invited the deceased to Tinolahan Eatery where the deceased were
abducted. The deceased were brought to Ceres Compound where
they were manhandled and asked on the the whereabouts of the
money that they swindled from Jeanette. It was then that Jeanette
ordered Geroche to take care of the two. The deceased were
brought to different motels and their last destination was in the Hd.
Pedrosa where the they were shot dead. Thereafter, the two dead
bodies were loaded on board the land cruiser and brought to Hda.
Siason where Pecha and Hilado buried them in the shallow grave
they dug.
ISSUE/S Whether or not accused Jeanette Yanson-Dumancas could be held
liable of the crime as a principal by inducement
RULING/S No.
Article 17, Revised Penal Code, provides: The following are
considered principals: (i) Those who take a direct part in the
execution of the act; (ii) Those who directly force or induce others to
commit it; and (iii) Those who cooperate in the commission of the
offense by another act without which it would not have been
accomplished.
What the Court now has to examine is whether or not sufficient
evidence was adduced by the prosecution to prove beyond
reasonable doubt that Jeanette indeed performed any of the

453
following acts: (a) directly forcing the killers to commit the crime, or
(b) directly inducing them to commit the crime.
There are 2 ways of directly forcing another to commit a crime,
namely: (i) by using irresistible force, or (ii) by
causing uncontrollable fear. Upon review of the testimony of all the
witnesses of the prosecution, we find nothing to conclude that
Jeanette used irresistible force or caused uncontrollable fear upon
the other accused-appellants.
The record is entirely bereft of any evidence to show that Jeanette
directly forced the participants of the said meeting to come up with
such plan, by either using irresistible force or causing uncontrollable
fear. The only basis relied upon by the trial court in arriving at its
conclusion that Jeanette is guilty of the crime as principal by
inducement, is the supposed “commands” or order given by her to
accused-appellant Dominador Geroche.
Likewise, there are 2 ways of directly inducing another to commit a
crime, namely: (i) by giving a price, or offering reward or promise,
and (ii) by using words of command. The Court finds no evidence,
as did the trial court, to show that Jeanette offered any price, reward,
or promise to the rest of accused-appellants should they abduct and
later kill the victims in this case.
By the foregoing standards, the remark of Jeanette to “take care of
the two” does not constitute the command required by law to justify
a finding that she is guilty as a principal by inducement.
Furthermore, the utterance which was supposedly the act of
inducement, should precede the commission of the crime itself. In
the case at bar, the abduction, which is an essential element of the
crime charged (kidnapping for ransom with murder) has already
taken place when Jeanette allegedly told accused-appellant
Geroche to “take care of the two.” Said utterance could, therefore,
not have been the inducement to commit the crime charged in this
case.

It was also duly proven by the witness that the intention of Jeanette
was to allow the law to run its course when she said that “Doming,
bring these two to the PC or police and will call Atty. Geocadin so
that proper cases could be filed against them.”

454
CONTRIBUTOR CARPIO, Anna Clarissa C.
MODULE Module 9: Alternative Circumstances and persons criminally liable
TOPIC
CASE TITLE PEOPLE v. MALUENDA G.R.NO. 115351
PONENTE Panganiban, J. DATE: March 27, 1998
DOCTRINE Conspiracy and/or direct participation in a crime may be proven by
circumstantial evidence. However, the comprising circumstances
must be duly proven, consistent with each other and lead with moral
certainty to only one conclusion: that the accused is guilty. If the
totality of such circumstances eliminates beyond reasonable doubt
the possibility of innocence, the conviction is proper; otherwise, the
accused must be acquitted. If said accused, however, took
advantage of the effects of the crime and profited thereby, he can
be held criminally liable as an accessory.
FACTS Accused-appellants Raul Mondaga, Rodrigo Legarto and Daniel
Maluenda were convicted of kidnapping Engr. Miguel Resus. The
Information provides that at about 9:00 p.m. in August 1992 at
Surigao Del Sur, the appellants, together with one Gil Bueno in
consiparcy kidnapped Engr. Resus and detained him for 4 days for
the purpose of extorting money from the spouses Engr. and Mrs.
Resus in the amount of Php 200,0000. While Bueno eluded arrest
and remained at large, the appellants pleaded “not guilty” with the
assistance of counsel. After trial, all were found guilty as charged,
as co-principals of the crime of kidnapping for ransom and
sentenced with reclusion perpetua with all the accessory penalties
provided by law. As Mondaga withdrew his appeal, only the criminal
liabilities of Legarto and Maluenda are thereby reviewed by the
Supreme Court.

According to the prosecution, that evening, Engr. Resus and wife


Dr. Bernardita Resus arrived at their residence/clinic at Lianga,
Surigao del Sur from the novena and three men who identified
themselves as Commander Bobong (Mondaga), Commander
Bongkoy (Maluenda) and one alias “Alex” were waiting for them.
Appellant-Mondaga then declared that they came upon the orders
of an alleged NPA Commander, Father Simon to solicit money and
medicines needed for the victims of the recent military-NPA
encounter. The trio demanded spouses medicines and Php20,000
in cash but the couple declared that they did not have such amount.
The amount was lowered to Php10,000, then to Php5,000 but the
spouses still cannot produce the same but instead gave the amount
of Php500 plus assorted medicines worth Php800. Afterwhich, the
appellants demanded that Engr. Resus drive them using the latter’s
car to Barobo, Surigao del Sur but the couple appealed and said the
car was not in good working condition to drive at night and that it did
not have enough gasoline. Mondaga then demanded that it be

455
prepared very early the following morning so Engr. Resus may drive
them, and left thereafter with a threat that should the spouses tell
anyone, they shall kill the members of their family and blow-up the
clinic. The next day, at 5:00 a.m., Mondaga arrived and Engr. Resus
drove Mondaga to Andanan to meet his other companions. Instead
of going to Borobo, despite the desitate of Engr. Resus, they went
to Alegria, undergone a 2-hour mountain hike at the boundary of
Cardon and Algeria whereupon reaching the hut, Engr. Resus was
told to stay there for the night together with Maluenda, Alex, and
Bueno while Mondanga will return to retrieve something he said he
had forgotten.

Meanwhile, at the Resus’ residence, a midwife has informed Dr.


Resus that Mondaga came at about 4:00 p.m. while the doctor was
out. Mondanga returned at about 7:00 p.m. that night and
demanded the amount Php300,000 in exchange for the release of
Engr. Resus. Dr. Resus said she can only produce Php10,000, so
she was told to reserve the money and Mondaga would get it the
following morning. Mondaga also asked the wife to look for her
husband’s firearm. The following morning, Mondanga arrived and
shortly after, so is Legarto. Dr. Resus was demanded by the 2 to
come with them but due to her health, the former asked that her
helper Maria Abne should instead join. Carrying the Php10,000, and
Engr. Resus’ Magnum 22, the appellants together with Abne went
to the mountain where Engr. Resus was the latter was then asked
to sign a note stating that it will be up to Dr. Resus to produce the
amount of Php200,000 (initially 300,000) as ransom. Abne left with
Engr. Resus while Legarto and Mondaga went to deliver the note to
Dr. Resus. The amount was reduced to Php 100,000 plus Php
50,000 as payment for Legarto’s motocycle. In the afternoon, Dr.
Resus, together with Nora Gubantes went to Lianga to secure the
ransom money from the former’s relatives but since the relatives
were not home, she then proceeded to seek the assistance of San
Francisco mayor Manatad to which the mayor gave Php 150,000
cash in lieu of the PNB check that Dr. Resus will issue. Dr. Resus
instructed Gubantes to give the cash to Legarto and the latter has
acknowledged receipt thereof the following day. Thereafter,
Mondaga informed Engr. Resus that he would be released on the
condition that appellant would come back to get the balance of the
Php300,000 (Php150,000) in 3 months. Engr. Resus and Abne were
released and the 2 were driven by Legarto in Engr. Resus’ car. Soon
after, Mondaga, Maluenda, and Legarto were arrested.

The defense however denies any involvement in the kidnapping.


Legarto avers that he only participated in the delivery of the ransom
money to Mondaga at the insistence of Dr. Resus herself. Maluenda

456
likewise denies knowledge of Mondaga’s plan to commit the crime.
He avers that he accompanied Mondaga to Mahilom only to mine
gold and not to plan, much less, to commit any crime and further
alleges that he guarded the victim at the hut only because Mondaga
threatened to kill him and his family. Appellants aver that the trial
court erred in declaring them co-principals in the crime when their
participation in the crime was not proven beyond reasonable doubt,
for failure to give credence to the testimonies of their witnesses and
their lack of motive in kidnapping Engr. Resus.
ISSUE/S Whether or not appellants Maluenda and Legarto are properly
convicted as co-principals in the crime as charged?
RULING/S NO for Legarto, YES for Maluenda.

Maluenda asserts that only Mondaga executed the acts constituting


the kidnapping with ransom and that his mere presence at the
hideout was involuntary due to Mondaga’s threats of killing him and
his family. Such contention is unmeritorious and his conviction
affirmed. The prosection was able to establish beyond cavil was
Maluenda’s direct participation in the kidnapping of Engr. Resus and
both Spouses testified and corroborated their statements stating
that Maluenda was one of the men who had, extorted money and
medicine from them that evening. Engr. Resus also testified that he
drove Maluenda and his companions to Alegria in his car the
following day and that the same guarded him at the farm hut
(hideout). The statement of Engr. Resus was corroborated by Abne
who too, was left in the hideout his employer. Although only
Mondaga verbally extorted money and demanded ransom from the
spouses, it is evident that the kidnapping was committed with
Maluenda’s participation as his actions exhibited a community of
interest and a concurrence of sentiment with Mondanga.

However, in Legarto's case, conspiracy was not at all established


by the prosecution. The familiarity between Legarto and Mondaga
is insufficient proof, as conspiracy transcends companionship.
Moreover, Mondaga's act of meeting Legarto on the road to
Andanan does not show conspiracy, because a merely casual or
unintended meeting, like passive presence, is not proof of
conspiracy. Similarly insufficient as circumstantial evidence to
prove conspiracy were Mondaga's demand for the use of Legarto's
motorcycle, Legarto's collecting the ransom money and delivering
part of it, and Legarto's failure to testify against Mondaga due to
either refusal or neglect. Conspiracy must be founded on facts,
not mere inferences and conjectures. Without an allegation of
any overt act showing community with the kidnappers, inferences
do not adequately establish participation in a criminal conspiracy.

457
Legarto was found by the trial court guilty as principal by
indispensable cooperation on the basis of several pieces of
evidence but appellant asserts that such evidence is frail and
inconclusive. Legarto’s contention is meritorious. A principal by
indispensable cooperation is one who cooperate in the commission
of the offense by another act without which it would have not been
accomplished (Article 17, RPC), and Legarto cannot be convicted
under this definition for failure of the prosecution to allege, much
less prove, any overt act on his part showing direct participation in
the kidnapping itself as his participation is limited to the acts
committed after the abduction was already consummated. He was
not with the kidnappers when they forcibly solicited money and
medicine from the Spouses; when they brought the victim to Alegria;
and when Mondaga demanded ransom for the victim’s release.
Together with Resus’ housemaid, he accompanied Mondaga to the
hideout in Alegria only upon the request of Dr. Resus’. In short, the
prosecution failed to piece together a clear story as to show how
Legarto figured in the kidnapping.

While circumstantial evidence may be sufficient to convict an


accused when the following requisites concur: (a) there is more than
one circumstance; (b) the facts from which the inferences are
derived are proven; and (c) the combination of all the circumstances
produces a conviction to withstand judicial scrutiny, the prosecution
must further show that all the circumstances are inconsistent with
the hypothesis except that of his guilt. In the case, the totality of the
pieces of circumstantial evidence being imputed to Legarto does not
foreclose the possibility that he took no part in the criminal enterprise
and does not, therefore, overcome his constitutional right to be
presumed innocent.

However, Legarto may not have had a direct hand in the kidnapping,
but he received part of the ransom and used it to pay off his arrears
in his motorcycle loan. Thus, having knowledge of the kidnapping
for ransom and without having directly participated therein, he took
part in the crime subsequent to its commission by profiting
from its effects. Therefore, Legarto is criminally liable as an
accessory to the crime of kidnapping for ransom. Article 19 of the
RPC defines accessories are defined as those who (1) have
knowledge of the commission of the crime, (2) did not take part in
its commission as principal or accomplice, but (3) took part in its
subsequent to its commission by any of the three modes
enumerated in this article, one of which is by profiting or by assisting
the offender to profit from the effects of the crime. These elements
are all present and proven in Legarto's case. Further, as an
accessory to the consummated crime, the penalty imposable upon

458
Legarto is two degrees lower than that prescribed by law (under
Article 267, RPC). Since no modifying circumstance is appreciated
for or against him, the imposable penalty should be in the medium
period of the indeterminate sentence applicable under RA 4103, as
amended.

459
CONTRIBUTOR CHUA, Kristy Anne
MODULE ALTERNATIVE CIRCUMSTANCES AND PERSONS CRIMINALLY
TOPIC LIABLE (Article 50 to 57 of the Revised Penal Code)
CASE TITLE PEOPLE V. MONTEALEGRE G.R.NO. L-67948
PONENTE CRUZ, J.: DATE: MAY 31, 1988
DOCTRINE Principal by indispensable cooperation which have the following
requisites: participating in the criminal resolution, that is, there is
either anterior conspiracy or unity of criminal purpose and intention
immediately before the commission of the crime charged; and
cooperation in the commission of the offense by performing another
act without which it would not have been accomplished.

Conspiracy need not be established by direct proof as it can be


inferred from the acts of the appellants. It is enough that, at the time
the offense was committed, participants had the same purpose and
were united in its execution; as may be inferred from the attendant
circumstances.
FACTS On March 11, 1983 around 11:30 P.M., Abadilia was at Meding’s
Restaurant in Cavite City eating where he caught a whiff of
marijuana smoke from the nearby table. He quietly went outside and
saw Pfc. Camantigue in his car to whom he reported the matter to.
After he parked his vehicle, both of them went inside the restaurant
and immediately caught the smell of marijuana smoke from the table
of Capalad which made Camantigue confront their table by asking
if they are using marijuana and forcing them up. While Camantigue
was holding the two, Montealegre’s right hand and Capalad’s left
hand. Capalad pulled out a knife from his waist and stabbed
Camantigue on the back. This caused Camantigue to let go of
Montealegre and draw his gun but was restrained by Montealegre
with his both hands while Capalad continued stabbing Camantigue.
When Capalad was able to free himself, he rose and fled to the door
while Camantigue fired shots at him while continuing the pursuit
outside which eventually Camantigue lost sight of Capalad, not
knowing that he fled into a dark alley. Camantigue abandoned the
chase and requested to be brought to a hospital. Capalad later was
found slumped in the alley with a gunshot wound in his chest. Both
of them died the following day.

Montealegre escaped during the altercation. Capt. Gilera of the


Cavite police immediately organized a team within that night after
being informed of the incident. Montealegre was not found in his
house but was arrested in the morning of the following day, on board
a vehicle bound for Baclaran, the name he gave was Alegre but later
confessed that he was the fugitive the police officers were after. Dr.
Sosa reported on the autopsy of Camantigue’s body and testified
that death was caused by the numerous stab wounds which

460
corroborated with the stabbing incident narrated by Abadilla and
Generoso San Juan.
ISSUE/S Whether or not Napoleon Montealegre was properly considered as
a co-principal for cooperating with Capalad in the killing of
Camantigue.
RULING/S Yes, according to the Supreme Court, Montealegre was correctly
considered a co-principal for having collaborated with Capalad in
killing the police officer when he was doing his duty. Both of the
accused acted in unison, with Capalad stabbing Camantigue seven
times and Montealegre holding both of the police officers' hands in
order to prevent him from drawing his gun to defend himself. He
might not be the one who stabbed Camantigue but he was equally
guilty for preventing the police officer from resisting the attack. Thus,
considered as a principal by indispensable cooperation under Article
17, par. 3, of the RPC. As correctly stated by the prosecution, even
if there was no evidence corresponding to a prior agreement
between Capalad and Montealegre but their subsequent acts
should be sufficient to show that there was presence of conspiracy.
It is enough that during the commission of the offense, the accused
had the same purpose and they acted in unison during the
execution. Montealegre cooperated in the commission of the murder
by holding the police officer’s hands which if he did not perform such
act then the crime would not have been accomplished. Therefore,
as per final verdict of the Supreme Court, the accused will suffer the
penalty of reclusion perpetua without aggravating and mitigating
circumstances, plus the civil indemnity, which is increased to
P30,000, and the actual, medical and funeral expenses in the sum
of P37,380 as stated in the trial.

461
CONTRIBUTOR CRUZ, Jakielyn Anne O.
MODULE MODULE 9: Persons Criminally Liable
TOPIC
CASE TITLE PEOPLE v. MADALI G.R.NO. L-67803
PONENTE FERNAN, C.J DATE: JUL. 30, 1990
DOCTRINE Direct proof is not essential to prove conspiracy as it may be shown
by acts and circumstances from which may logically be inferred the
existence of a common design among the accused to commit the
offense(s) charged, the evidence to prove the same must be
positive and convincing considering that conspiracy is a facile
devise by which an accused may be ensnared and kept within the
penal fold. With this and the principle that in criminal prosecution,
doubts must be resolved in favor of the accused.
As an accomplice, Annie Mortel Madali should be imposed the
penalty next lower in degree than that prescribed by law for the
consummated felonies.
FACTS The victim Felix Gasang figured in a fistfight with someone who was
a friend of Ramon Madali. Ramon interceded and mauled Felix with
a "chako". The next day, the police summoned Felix to the municipal
building and was accompanied by his mother Desamparada
Gasang. At the police station, Ricarte Madali, a police officer,
angrily scolded Felix and his cousin, Arnaldo Fadriquilan, and told
them that because they were "very brave", he would put them in jail
for twelve hours. Felix was eventually sent home together with his
mother.
At around 9:00 pm of October 31,1979, Felix and his cousin,
Agustin Reloj, went home together from the town plaza and parted
ways at the Marawi bridge. Around fifteen meters from the house of
Ricarte Madali, Agustin was confronted and was held by Ricarte
and said, "So you are here, you devil, now you are finished. I have
been waiting for you. I have been watching for you for three nights
already." Then Ricarte dragged Agustin towards the gate of his
house. Agustin asked Ricarte why he was dragging him, Ricarte
said that the reason was because Agustin helped in fighting his
son.
As one of Agustin's feet stepped over the knee-high fence at the
gate of the Madali residence, he was clubbed by Annie Madali with
a piece of wood. Annie struck him first on the left shoulder and
would have given him another blow had not Agustin freed himself
from Ricarte's hold. Annie landed that blow on Ricarte instead.
Agustin ran away but was shot by Ricarte in his right hip. Agustin
did not get up as he feared that Ricarte might shoot him again.

462
Annie beamed her flashlight as Felix approaches and was shot by
Ricarte twice despite Felix told them that he would not fight with him.
The father of Felix, Cipriano Gasang arrived and was also shot by
Ricarte. Merlinda Gasang was accompanying Cipriano was also
shot, Desamparada Gasang arrived and shouted for help.
Merlinda was brought to the hospital but the bodies of Felix and
Cipriano was not removed from the road until midnight.
Felix and Cipriano died due to the gunshot wounds according to the
postmortem examinations conducted by the rural health physician.
Merlinda and Agustin gunshot wound that would incapacitate them
for couple of days.
Both Ricarte and Annie denied the allegations and claimed self-
defense since there wa s someone who hurled stone in their house
and intruded them when they are about to sleep. Ricarte went to
investigate and claimed that someone hit his left shoulder. Ricate
and the intruder grappled the club and the intruder fell on Ricarte’s
stomach. That is the instance that Ricarte was able to get hold of
his gun and saw that there were two more persons approaching him
who were armed with a club and a knife. Ricarte shouted and
warned the intruders that he is a policeman and do not come near
but the intruder did not heed, thus, Ricarte fired his gun.
ISSUE/S Whether or not Annie Mortel Madali is guilty as principal by direct
participation?
RULING/S NO. The Court rules that the liability of Annie Mortel Madali with
respect to the crimes committed herein, is only that of an
accomplice.
Annie's participation in the shooting of the victims consisted of
beaming her flashlight at them and warning her husband of the
presence of other persons in the vicinity. By beaming her flashlight
at a victim, Annie assisted her husband in taking a good aim. Ho
ever, such assistance merely facilitated the commission of the
felonious acts of shooting.
Considering that, according to both of the Madali spouses, "it was
not so dark nor too bright" that night or that "brightness and
darkness were equally of the same intensity." Ricarte Madali could
have nevertheless accomplished his criminal acts without Annie's
cooperation and assistance.
Neither may Annie's shouts of "here comes, here comes another,
shoot" be considered as having incited Ricarte to fire at the victims
to make Annie a principal by inducement. There is no proof that
those inciting words had great dominance and influence over Madali
as to become the determining cause of the crimes. The rapidity with
which Madali admittedly fired the shots eliminated the necessity of
encouraging words such as those uttered by Annie.

463
The fact that Annie dealt a blow on Agustin while he was being
dragged by Madali to their yard does not make her a principal by
direct participation. Annie's act, being previous to Madali's act of
shooting Agustin, was actually not indispensable to the crime
committed against Agustin.

464
CONTRIBUTOR DELA CRUZ, Ma. Luzelle P.
MODULE Persons Criminally Liable
TOPIC
CASE TITLE ABEJUELA v. PEOPLE THE G.R.NO. 80130
PHILIPPINES
PONENTE FERNAN, C.J.: DATE: AUG. 19,
1991
DOCTRINE A conviction of a criminal offense must be based on clear and
positive evidence and not on mere assumptions.
Extinction of the penal action does not carry with it extinction of the
civil, unless the extinction proceeds from a declaration in a final
judgment that the fact from which the civil might arise did not exist.
FACTS Petitioner Benjamin Abejuela, a businessman engaged in the
manufacture and fabrication of hand tractors and other agricultural
equipment, had a savings deposit with Banco Filipino, Tacloban
Branch. Sometime in April or May 1978, petitioner was befriended
by Glicerio Balo, Jr., an employee of Banco Filipino in the same
Tacloban Branch. They became close friends. Balo even became
the godfather of Abejuela's daughter. Moreover, Balo offered
Abejuela financial assistance in the latter's welding business,
claiming that he was expecting a large sum of money out of the
insurance policy of his late father.
Balo went to Abejuela's welding shop to borrow the latter's
passbook. Abejuela was surprised and thought that it was not
possible for Balo to use his passbook. Balo showed Abejuela some
checks purporting to be the proceeds of his father's insurance
policy. He wanted to deposit the checks in Abejuela's account with
Banco Filipino. Abejuela then suggested that Balo open his own
account. However, Balo explained that he was prohibited from
opening an account with Banco Filipino since he was employed with
that bank as a savings bookkeeper. Abejuela advised Balo to open
an account instead with another bank but Balo insisted that he
wanted the checks deposited with Banco Filipino so that he could
facilitate their immediate encashment as well as avail himself of
some privileges. Balo assured Abejuela that there was nothing
wrong in allowing him to use his passbook and even reassured
Abejuela that he would accompany him to the bank to make the
deposit.
Accepting Balo's explanations and assurances Abejuela entrusted
his passbook to Balo.
Balo returned Abejuela's passbook where a deposit in the amount
of P20,000.00 was already reflected. Once again, Balo assured
Abejuela that there was nothing wrong with the deposit, and stated
that he just deposited one of his checks. On the same, day Balo
requested Abejuela himself to withdraw, in the former's behalf,
money from his account with Banco Filipino. Again with assurances

465
from Balo, Abejuela reluctantly agreed. He went to Banco Filipino
and withdrew the amount of P15,000.00 which he gave to Balo at a
restaurant called Felisa's Cafe.
Balo's practice of depositing and withdrawing money using
Abejuela's passbook continued for quite some time. During the
month of August 1978, the account of Abejuela with Banco Filipino
reflected a total deposits of P176,145.00 and a total withdrawal of
P175,607.96.
In the meantime, Abejuela borrowed P20,000.00 from Balo, payable
within 90 days from August 9, 1978. But feeling apprehensive over
Balo's constant use of his passbook, Abejuela decided to pay his
loan on August 31, 1978 by borrowing P10,000.00 from his father
and taking the other P10,000.00 from his business profits. Abejuela
also closed his account with Banco Filipino by surrendering his
passbook and withdrawing the balance of his deposit.
Thereafter, the bank's accountant and interest bookkeeper
discovered a discrepancy between the interest reconciliation
balance and the subsidiary ledger balance. The interest bookkeeper
could not locate the posting reconciliation and the proof
reconciliation. He also notice that Account No. 6701-0160 in the
name of Benjamin Abejuela reflected four (4) large deposits on
various dates from August 3, 1978 to August 23, 1978, totaling
P176,145.25, but the deposits slips thereof could not be located.
After further examination of the bank records, the manager,
accountant and interest bookkeeper were convinced that the
irregularities were caused by Balo who was the savings bookkeeper
at that time and who had access to Abejuela savings account
ledger. They concluded that Balo was able to manipulate the ledger,
by posting the fictitious deposits after banking hours when the
posting machine was already closed and cleared by the bank
accountant.
The bank officials confronted Balo, who feigned ignorance and
initially denied the accusations, but later admitted having posted the
false deposits. Petitioner Abejuela was also implicated because he
was the owner of the passbook used by Balo in accomplishing his
fraudulent scheme.
On December 5, 1978, an information was filed against Glicerio
Balo, Jr. and Benjamin Abejuela for the crime of estafa thru
falsification of commercial documents. Separately arraigned, both
pleaded "not guilt to the crime charged.
The court finds the accused Benjamin Abejuela guilty beyond
reasonable doubt as accomplice of the complex crime of estafa thru
falsification of a commercial document.
ISSUE/S WON Abejuela, having no knowledge of the criminal intent of his co-
accused, Glicerio Balo, Jr., should be convicted for estafa.

466
RULING/S From Balo's own admissions, it was he who deceived Abejuela
through sweet talk, assurances, drinking sprees and parties and
cajoled him into giving in to his requests. Furthermore, during that
time, nobody would have questioned Balo's source of money and
since he had a perfect alibi, i.e. the insurance proceeds of his later
father. When Balo showed Abejuela some checks purporting to be
his father's insurance proceeds, Abejuela was hoodwinked into
believing that Balo indeed had money. Balo's request to borrow
Abejuela's passbook in order to facilitate the encashment of the
checks seemed reasonable enough, considering that they were
close friends and "compadres", Abejuela's acquiescence to Balo's
overtures is understandable.
Furthermore, the court takes judicial notice of the practice of banks
in allowing anybody to deposit in an account even without the
owner's passbook, as long as the account number is known. Thus,
even without Abejuela's passbook, the false deposits could still have
been posted by Balo in the savings account ledger of Abejuela. After
all, the ledger is the record of the bank reflecting the transactions of
the depositor, while the passbook is the record of the depositor.
More often than not, it is the ledger which is more accurate and up-
to-date. This is the reason why depositors have their passbooks
updated for unrecorded transactions like interests, checks
deposited beyond clearance cut-off time and bank charges.
Knowledge of the criminal intent of the principal in this case,
(Glicerio Balo, Jr.) is essential in order that petitioner Abejuela
can be convicted as an accomplice in the crime of estafa thru
falsification of commercial document. To be convicted as an
accomplice, there must be cooperation in the execution of the
offense by previous or simultaneous acts. However, the cooperation
which the law punishes is the assistance rendered knowingly or
intentionally, which assistance cannot be said to exist without the
prior cognizance of the offense intended to be committed.
It has been satisfactorily established that Banco Filipino suffered
damage in the amount of P176,145.25 representing the fictitious
deposits posted by Glicerio Balo, Jr. and systematically withdrawn
through the passbook of petitioner Abejuela. Although Abejuela,
was unaware of the criminal workings in the mind of Balo, he
nevertheless unwittingly contributed to their eventual consummation
by recklessly entrusting his passbook to Balo and by signing the
withdrawal slips. Abejuela failed to exercise prudence and care.
Therefore, he must be held civilly accountable.
ACQUITTED.

467
CONTRIBUTOR DIMAKUTA, Jasmine A.
MODULE PLURALITY OF CRIMES
TOPIC
CASE TITLE PEOPLE VS. DOBLE G.R. NO. L-30028
PONENTE DE CASTRO, J. DATE: May 31, 1982
DOCTRINE Mere fact that appellant was present when the other accused met
in his house to plan a bank robbery and that he told them he cannot
join the latter because of a foot injury will not make said appellant a
co-conspirator.

FACTS Late in the night of June 13, 1966, ten (10) men, almost all of them
heavily armed with pistols, carbines and thompsons, left the shores
of Manila in a motor banca and proceeded to Navotas, Rizal. Their
mission: to rob the Navotas Branch of the Prudential Bank and Trust
Company. Once in Navotas and taking advantage of the darkness
of the night, eight (8) men disembarked from the banca and
proceeded to the beach in the direction of the branch bank. Within
a few minutes, shots were heard throwing the people around in
panic. As confusion reigned, the people ran in different directions
scampering for safety. As time went on, the shots grew in intensity.
As the commotion died down, the eight men returned to their banca,
still fully armed and some of them carrying what looked like
“bayongs”. They boarded the waiting motor banca and sped away.
As a result of the shooting, many people got killed and some injured.
Among those who were killed were agents of the law, like Sgt.
Alejandro Alcala of the Philippine Constabulary, Sgt. Eugenio
Aguilos and Cpl Teofilo Evangelista of the Navotas Police
Department. Dominador Estrella, a market collector,was also killed.
Those who were injured were Pat Armando Ocampo, Exequiel
Manalus, Jose Fabian, Rosalina Fuerten and Pedro de la Cruz.
The Prudential Bank and Trust Company branch office located at
the North Bay Boulevard, Navotas, Rizal, the object of the bloody
mission, has an unusual banking hours. It opens at midnight and
closes at 8:00 in the morning. The bank has ten employees, more
or less, including a security guard. It has two cages or compartments
for tellers. One cage was under the care of Melvin Domingo and the
other one under the care of Alejandro San Juan. At around 12:30
a.m. of June 14, 1966, Cesar Reyes, assistant cashier of the bank,
was near the cage of Domingo when two men entered the bank
asking that their money be changed. Domingo refused, saying that
they had no small denominations. Suddenly, three men armed with
long guns barged in and fired at the ceiling and the walls of the bank.
They ordered the employees to lie down, face downward and then
demanded the key to the vault. When Reyes answered that they do
not have the key, the armed men aimed their guns at the vault and
fired upon it until its doors were opened. They entered the vault and

468
found that they could not get anything as the compartments inside
the said vault were locked. Not being able to get anything from the
vault, the armed men went to the two teller cages and took whatever
they could lay their hands on. Not long afterwards, the men left,
carrying with them the sum of P10,439.95.
Just beside the bank was a police outpost. On the night in question,
Pat. Nicolas Antonio was in the outpost, together with Sgt. Aguilos,
Pats. Pangan, Burgos, Rosal, Ocampo and Cpl Evangelista. They
were on duty watching the fish landing. Suddenly, Antonio said, at
around 1:30 a.m., he heard a burst which he believed came from a
thompson. Hesaid he saw a man pointing a thompson upwards
while he was in front of the bank. Afterwards, Antonio said, he heard
another burst coming from the same direction. Antonio and his
companions then went to the middle of the road and again they
heard shots, and this time they were successive, coming from their
left. Antonio could not see who was firing the shots. Suddenly, he
said, he saw one of this companions Cpl. Evangelista topple down.
He saw also Dominador Estrella sitting down folding his stomach.
They were both felled by the shots coming from the left side of the
bank. Antonio told Ocampo to go beside the outpost and held Sgt.
Aguilos by the arm. Sgt. Aguilos, however, collapsed and fell down.
He was hit. Later on, Antonio said, he went to the outpost and told
Pat. Ocampo to go too. He said that from the outpost he heard some
more shots. Then he saw Ocampo hit in the thigh. After the firing
ceased, Antonio saw his wounded companions placed in a vehicle,
together with Evangelista and Aguilos who were already dead. Later
on, he said he saw Sgt. Alcala, a member of the PC, lying prostrate
in the ground already dead.”

ISSUE/S Whether or not the appellants are co-principals in the in the crime
charged
RULING/S Like the Solicitor General, We, therefore, find no culpable
participation of Simeon Doble in the commission of the crime, for,
indeed, by his physical condition alone, he could not in any way be
of help to the malefactors in the pursuit of their criminal design, nor
could he have been desired by the latter to be one of them.

The circumstances pointed out would not make [the other 2]


appellants liable as co-principals in the crime charged. At the most
their liability would be that of mere accomplices. They joined in the
criminal design when Cresencio consented to look for a banca and
Romaquin provided it when asked by the gang leader Joe Intsik,
and then brought the malefactors to the scene of the robbery,
despite knowledge of the evil purpose for which the banca was to
be used. It was the banca that brought the malefactors to the bank
to be robbed and carried them away from the scene after the

469
robbery to prevent their apprehension. Appellants thus cooperated
but not in an indispensable manner. Even without appellants
providing the banca, the robbery could have been committed,
specially with the boldness and determination shown by the robbers
in committing the crime.

If it is true that he never voluntarily made the trip with knowledge of


the planned robbery, and with Cresencio saying that he returned the
gun given him with which to prevent Romaquin from speeding away,
Romaquin could have tried a get-away, as should have been his
natural impulse had he not joined in the criminal design. His act of
hiding the money he received from the malefactors, and repainting
his boat, all attest to his guilty conscience arising from the act of
cooperation he knowingly extended to the principal culprit to achieve
their criminal purpose.

What [the other 2] appellants may be said to have joined is the


criminal design to rob, which makes them accomplices. Their
complicity must, accordingly, be limited to the robbery, not with the
killing. Having been left in the banca, they could not have tried to
prevent the killing, as is required of one seeking relief from liability
for assaults committed during the robbery (Art. 296. Revised Penal
Code).

The finding that appellants are liable as mere accomplices may


appear too lenient considering the gravity and viciousness of the
offense with which they were charged. The evidence, however, fails
to establish their complicity by a previous conspiracy with the real
malefactors who actually robbed the bank and killed and injured
several persons, including peace officers.

Accordingly, We find appellants Cresencio Doble and Antonio


Romaquin guilty beyond reasonable doubt, but only as accomplices
for the crime of robbery in band. As discussed earlier, appellant
Simeon Doble is entitled to acquittal as so recommended by the
Solicitor General who finds no sufficient evidence, to which We
agree, to establish his guilt beyond reasonable doubt.

470
CONTRIBUTOR FLORES, Precious Eureka D.
MODULE Article 50 to 57 of the Revised Penal Code (RPC)
TOPIC
CASE TITLE PEOPLE v. DOCTOLERO G.R.NO. 34386
PONENTE REGALADO, J. DATE: FEBRUARY 7, 1991
DOCTRINE Where the accomplices consented to help in the commission of
forcible abduction, they were responsible for the resulting homicide
even if the purpose of the principal to commit homicide was
unknown to the accomplices.

Where one goes with the principals, and in staying outside of the
house while the others went inside to rob and kill the victim, the
former effectively supplied the criminals with material and moral aid,
making him guilty as an accomplice.
FACTS On the 8th day of November, 1970, in barrio Binday, municipality of
San Fabian, province of Pangasinan, Philippines, the accused,
armed with bolos, went up the house of Marcial Sagun and once
thereat, conspiring together and mutually aiding one another, with
intent to kill and with evident premeditation and treachery, with
abuse of superior strength and with extreme cruelty, did, then and
there, willfully, unlawfully and feloniously attack, assault, hack, stab
and strike Lolita de Guzman Oviedo, Epifania Escosio and Jonathan
Oviedo and immediately thereafter, the same accused while already
on the road, conspiring together and mutually aiding one another,
with intent to kill and with evident premeditation and treachery,
attack, assault, hack and stab Marcelo Doctolero, thereby inflicting
upon him a total of nine (9) mortal wounds which caused his death.
ISSUE/S Whether or not the penalty imposed is correct and proper.
RULING/S The penalty imposed on Ludovico Doctolero as principal by direct
participation is correct and since he had withdrawn his appeal, the
judgment rendered by the trial court became final and executory and
any modification made by the Supreme Court in the penalties will
not be applicable to him.

The penalty imposed on Conrado and Virgilio Doctolero as


accomplices to the crime is correct. In the case of Conrado
Doctolero, even though he denied his participation in the
commission of the crime, his pretension is not corroborated by any
evidence other than the testimony of his co-accused. While such
testimony is admissible, it still came from a polluted source and must
be scrutinized with great caution as it is subject to suspicion.
Whatever doubt the court a quo entertained on the criminal
responsibility of appellants Conrado and Virgilio Doctolero did not
refer to whether or not they were liable but only with regard to
the extent of their participation. There being ample evidence of their

471
criminal participation, but a doubt exists on the nature of their
liability, the courts should favor the milder form of liability or
responsibility which is that of being mere accomplices.

However, since Virgilio Doctolero died during the pendency of the


appeal, his criminal liability is hereby terminated but not his civil
liability.

472
CASE TITLE People v. De Vera G.R NO. G.R. No. 128966
PONENTE PANGANIBAN, J. DATE: 18 August 1999
CONTRIBUTO GARCIA, LEXANNE O.
R
DOCTRINE
Mere presence does not amount to conspiracy.
Where an accused knew that the other
malefactors had intended to kill somebody and he cooperated
with
them but did not himself participate in the decision to kill, joini
ng
them only after the decision to kill had already been agreed u
pon, he could only be considered as an accomplice.
FACT/S
Witness saw a car passing by, driven by victim Frederick Capulong
together with four (4) other passengers. He recognized and identified
two of the passengers as Kenneth Florendo and Roderick Garcia,
both familiar in the subdivision. He heard unintelligible voices coming
from the car as it was cruising around Denver Loop Street, a circular
road. He saw the car already parked. Moments later, he saw the
victim dragged out of the car by Florendo and brought to a grassy
place. Florendo was holding a gun. Upon reaching the grassy spot,
Florendo aimed and fired the gun at the victim, hitting him between
the eyes.

Appellant claims that he had no part in the killing, and that it was
Kenneth Florendo who had shot the victim. He avers that he merely
accompanied to Filinvest the other accused and Florendo, who was
his friend, upon the latter’s request.
MAIN ISSUE/S
What is the proper degree of participation of the petitioner?
RULING ON
MAIN ISSUE/S Appellant should be convicted only as an accomplice, not as a
principal.

The eyewitness account is not enough to establish conspiracy. Mere


presence does not amount to conspiracy. Appellant knew of Kenneth
Florendo’s malevolent intention. Appellant’s companions were armed
that day, which revealed the unmistakable plan of the group. Knowing
that Florendo intended to kill the victim and that the three co-accused
were carrying weapons, he had acted as a lookout to watch for
passersby. He was not an innocent spectator; he was at the locus
criminis in order to aid and abet the commission of the crime. He
joined them that afternoon after the decision to kill had already been
agreed upon; he was there because “nagkahiyaan na.”

473
Where an accused knew that the other
malefactors had intended to kill somebody and he cooperated
with
them but did not himself participate in the decision to kill, joini
ng
them only after the decision to kill had already been agreed u
pon, he could only be considered as an accomplice. In the case
of an accomplice, the liability is one degree lower than that of a
principal.

474
CONTRIBUTOR CARPIO, Anna Clarissa C.
MODULE Module 9: Alternative Circumstances and persons criminally liable
TOPIC
CASE TITLE PEOPLE v. GARCES G.R.NO. 173858
PONENTE Ynares-Santiago, J. DATE: July 17, 2007
DOCTRINE Accomplices are those who, not being included in Article 17,
cooperate in the execution of the offense by previous or
simultaneous acts. The two elements necessary to hold petitioner
liable as an accomplice are present: (1) community of criminal
design, that is, knowing the criminal design of the principal by direct
participation, he concurs with the latter in his purpose; and (2)
performance of previous or simultaneous acts that are not
indispensable to the commission of the crime.
FACTS In August 1992, between 8:00 and 9:00 in the evening, AAA was on
of her way to the chapel when the five accused, Rosendo Pacursa,
Senando Garces, Antonio Pira, Jr., Aurelio Pira and petitioner
Ernesto Garces appeared and approached her. Pacura covered
AAA’s mouth with his hands and told her not to should else she’ll be
killed. Pacursa then brough AAA inside a nearby tobacco barn while
the 4 others stood guard outside. Inside the barn, despite AAA’s
desistance, Parcusa succeeded in having carnal knowledge of her.
After a while, having heard people shouting and calling the victim’s
name, petitioner Ernesto entered the barn, covered AAA’s mouth
and dragged her outside. Ernesto also threatened the latter that she
will be kill if she will report the incident. Thereafter, upon reaching
the house of Florentino Garces, Ernesto released AAA. The
complainant was found by her relatives crying, wearing only one
slipper with her hair disheveled. AAA, though in a state of shock was
able to recount the incident after a while.

At trial, all denied the charges. Petitioner Ernesto, Antonio and


Aurelio denied seeing Pacursa that night and averred that they were
watching a televised basketball game at the house Antonio when
the alleged rape happened. On the other hand, Pacursa testified
that he and AAA were sweethearts for almost a year prior to the
incident. He avers that he was on his way to the house of co-
accused Pira, Jr. to watch a televised basketball game when he saw
AAA and that the latter wanted have a talk so he led her to the
tobacco barn about 15 meters away so no one might see them.
Pacursa denied having sexual intercourse with AAA and alleged
that they were only talking, embracing and kissing by the barn door.
After the incident, he received a letter from AAA asking him to
elope.

The Abra Regional Trial Court (RTC Abra) found Pacursa and
Ernesto guilty of Forcible Abduction with Rape as principal and

475
accessory respectively. Ernesto’s participation was after or
subsequent to the rape and that his acts were employed as a means
of concealing the commission of the crime and assisting Pacursa to
escape. The aggravating circumstances of uninhabited place and
nighttime were also appreciated. Pacursa, being 16 years old at the
time of the commission of the offense was sentenced to one degree
lower than that prescribed for the offense pursuant to Article 68 of
the RPC. Petitioner Garces, as accessory is penalized two degrees
lower than that prescribed by law for the offense. The two others,
Antonio and Aurelio were acquitted for insufficiency of evidence.
The Court of Appeals affirmed the RTC’s decision with modification,
eliminating the aggravating circumstances. Petitioner now assails
his conviction claiming that no rape was committed and that there
was no evidence to show that he covered the mouth of AAA and
brought her out of the barn.
ISSUE/S Whether or not Ernesto Garces is liable as an accessory to the crime
of Forcible Abduction with Rape of AAA?
RULING/S NO, petitioner Ernesto is an accomplice as facts show that
petitioner is guilty in the commission of the crime even before
the complainant was raped.

Accomplices are those who, not being included in Article 17,


cooperate in the execution of the offense by previous or
simultaneous acts. The two elements necessary to hold petitioner
liable as an accomplice are present: (1) community of criminal
design, that is, knowing the criminal design of the principal by
direct participation, he concurs with the latter in his purpose;
and (2) performance of previous or simultaneous acts that are
not indispensable to the commission of the crime. Ernesto was
present when Pacursa abducted complainant and when he brought
her to the barn. Petitioner positioned himself outside the barn
together with the other accused as lookout. When he heard the
shouts of people looking for AAA, Ernesto entered the barn and took
complainant away from Pacursa. Having known of the criminal
design and thereafter acting as a lookout, petitioner is liable as an
accomplice, there being insufficient evidence to prove conspiracy,
and not merely as an accessory.

The penalty imposed upon accomplices in consummated crime is


penalty next lower in degree than that prescribed for the felony. The
crime committed in the case is simple rape, punishable with
reclusion perpetua. The penalty of reclusion temporal should be
imposed on petitioner in its medium period in the absence of any
aggravating or mitigating circumstances. Applying the
Indeterminate Sentence Law, the imposable penalty should range

476
from prision mayor as minimum, to reclusion temporal in its medium
as maximum.

Complainant’s testimony was credible, consistent and unwavering


even during the cross-examination. Ernesto’s defense of alibi
cannot prevail over complainant’s positive identification of her
assailants. Denial and alibi are inherently weak defenses and
constitute self-serving negative evidence which cannot be accorded
greater evidentiary weight than the positive declaration of credible
witness. For alibi to prosper, the accused must establish by clear
and convincing evidence (a) his presence at another place at the
time of the perpetration of the offense and (b) the physical
impossibility of his presence at the scene of the crime. The house
Aurelio where Ernesto averred he was about 20 meters away from
the barn. It was not physically impossible for him to be at the scene
of the crime at the time of the incident.

477
CONTRIBUTOR MARCELINO, Ferilynn T.
MODULE Persons Criminally Liable
TOPIC
CASE TITLE PEOPLE V. TALINGDAN G.R.NO. 32126
PONENTE PER CURIAM: DATE: 06 July 1978
DOCTRINE Accessories are those who, having knowledge of the commission of
the crime, and without having participated therein, either as
principals or accomplices, take part subsequent to its commission
FACTS Bernardo Bagabag was murdered in his house in Abra on 24 June
1967 by Nemesio Talingdan, Magellan Tobias, Augusto Berras,
Pedro Bides and his wife Teresa Domogma. There was only one
witness of the murder, Corazon, the eldest daughter of Bernardo
Bagabag.

Teresa and Bernardo’s married life was beset with troubles. Teresa
had deserted their family home for a couple of times and the longest
was three weeks and each time Bernardo would look for her. She
was suspected to have illicit relationship with the appellant,
Nemesio Talingdan, a policeman who happens to live nearby. Two
nights before Bernardo was killed, he and Teresa had a violent
quarrel, slapping the latter several times. Teresa then went out to
seek help from the police. Thereafter Talingdan came threatening
Bernardo that he would kill him someday. The day before Bernardo
was killed, Corazon saw her mother having a meeting with
Talingdan and their co-appellants and heard one of them saying
“could he elude a bullet.” Teresa noticed the presence of Corazon,
shoved her away saying “You tell your father that we will kill him.”
On the night of the murder, while Corazon was preparing for supper,
she saw Teresa talking with her co-appellants in their “batalan”
armed with long guns. After eating supper alone, Corazon told her
father about the persons outside their house but the latter ignored
her. Bernardo went to the kitchen and sat on the floor near the door
and there he was gunned down. Bides threatened to kill Corazon if
she would ask for help. However she confessed to her father’s
relatives the identities of the murderers.

Contention of the Accused (Teresa):


Teresa had only known Talingdan because they were neighbors.
There was no illicit relationship between the two. She loved her
husband so much and contrary to the testimony of Corazon, they
never quarreled nor did she left their family home. On the night of
the murder, it was Teresa who was preparing supper and not
Corazon. Teresa contends that her in-laws are using Corazon to
testify against her. Furthermore, the defendants had their own alibi.
Talingdan was a bodyguard of the Mayor, hence was not in
Sallapadan when Bernardo was killed. Bides and Berras also

478
alleged that they were in the same house of Mrs. Bayongan on that
date.
ISSUE/S 1. Whether or not Corazon’s testimony is truthful?
2. Whether or not Teresa is an accessory to the murder of
Bernardo?
RULING/S 1. YES. The sworn statement of Corazon was true. Corazon knew
the accused because they live nearby their house. The court is
also convinced from the reading of her whole testimony that it
could not have been a fabrication. It is too consistent for a child
of thirteen years to be able to substantially maintain throughout
her stay on the witness stand without any fatal flaw, in the face
of severe and long cross-interrogations, if she had not actually
witnessed the event she had described. There is also no cogent
explanation why she would attribute the assault on her father to
three other men aside from Talingdan whom she knew had
relations with her mother, nor is she merely making-up her
account of how he was shot.

2. YES. Teresa was a mere accessory to the murder. Teresa’s


direct participation in the conspiracy is not beyond reasonable
doubt. She cannot have the same liability for she had no hand in the
actual shooting. Before the killing, Teresa was passive but after the
killing she became active in her cooperation with them. These
subsequent acts shows that she was an accessory by “concealing
or assisting in the escape of the principal of the crime.”

Meanwhile, male appellants were found guilty beyond


reasonable doubt of the crime of murder with two aggravating
circumstances without any mitigating circumstances and hereby
sentence to DEATH.

479
CONTRIBUTOR MAYUGA, Eunice Allaine G.
MODULE Anti-Fencing Law (Presidential Decree No. 1612)
TOPIC
CASE TITLE DIZON-PAMINTUAN v. G.R.NO. 111426
PEOPLE
PONENTE DAVIDE JR., J: DATE: 11 JULY 1994
DOCTRINE Fencing is the act of any person who, with intent to gain for himself
or for another, shall buy, receive, possess, keep, acquire, conceal,
sell or dispose of, or shall buy and sell, or in any manner deal in any
article, item, object or anything of value which he knows, or should
be known to him, to have been derived from the proceeds of the
crime of robbery or theft.
FACTS Teodoro Encarnacion arrived at his residence at around 9:45PM of
12 February 1988 coming from the airport. When he came to his
house, he went inside leaving behind his driver and two housemaids
to pick up his personal belongings. Five unidentified masked armed
persons appeared, poked their guns to them and dragged them
inside the house. They were also made to lie on the floor together
with Teodoro's wife. Thereafter, the robbers ransacked the house
and took away jewelries and other personal property including cash.
After the intruders left the house, Teodoro reported the matter to the
police immediately. He was asked to prepare a list of items of
jewelry and other valuables that were lost. Then, he was told that
some of the lost items were in Chinatown hence, entrapment would
be made with their participation to confirm if it was the lost items.
Teodoro and his spouse, posed as a buyer, went to Chinatown and
they were able to recognize the jewelry stolen being tended by
Norma Dizon Pamintuan. Norma testified that he has no prior
knowledge of the stolen jewelries of Teodoro.

The Regional Trial Court and Court of Appeals found Norma guilty
of violating the Anti-Fencing Law hence, this petition.
ISSUE/S Whether or not Norma Dizon-Pamintuan shall be held liable for
violating PD 1612.
RULING/S The court held that the constitutional presumption will not apply as
long as there is some logical connection between the fact proved
and the ultimate fact presumed, and the interference of one fact to
another will not hinder it.
Section 5 of P.D. No. 1612 expressly provides that “mere
possession of any good, article, item, object, or anything of value
which has been the subject of robbery or thievery shall be prima
facie evidence of fencing."
In this case, the petitioner was unable to rebut the presumption. She
relied solely on the testimony of her brother which was insufficient
to overcome the presumption of innocence, and, on the contrary,

480
even disclosed that the petitioner was engaged in the purchase and
sale of jewelry and that she used to buy from a certain Fredo who
has the jewelry.

481
CONTRIBUTOR Valdez, Ariane Faye V.
MODULE Module 8- Art. 14, RPC- Aggravating Circumstances
TOPIC
CASE TITLE People v. Vilbar G.R.NO. 186541
PONENTE LEONARDO-DE CASTRO, J. DATE: 01 Feb. 2012
DOCTRINE Where the meeting between the accused and the victim was casual
and the attack was done impulsively, there is no treachery even if
the attack was sudden and unexpected.
FACTS On May 5, 2000 at around 7:00pm, Guilbert Patricio (victim) arrived
on the store of his wife located in the Ormoc City public market from
work. He was met by their child whom he then carried in his arms.
Moments later, Guilbert noticed a man urinating at one of the tables
in front of their store. The man urinating was among those engaged
in a drinking spree in a nearby store. Guilbert immediately
admonished the man urinating but the latter paid no attention and
continued relieving himself. Guilbert then put down his child when
Vicente Vilbar (accused) rose from his seat, approached Guilbert,
drew out a knife and stabbed him below his breast. Thereafter, the
accused, as well as his companions, scampered away. At the time,
the wife of the victim was getting her child from Guilbert and easily
recognized the accused because he would sometimes drink at their
store. Guilbert was immediately brought to the hospital and later
expired 11:35 of the same evening.

On the point of view of the accused: Denial was his main plea in
exculpating himself of the charged against him. He claimed that in
the evening of May 5, 2000, he and his wife went to the public
market to collect receivables out of the sale of meat. Afterwards,
they chanced upon his wife’s acquaintances who were engaged in
a drinking spree. Among them were Dodong Danieles (Dodong for
brevity) and his younger brother. They invited him (the accused) and
his wife to join them. While they were drinking, Dodong had an
altercation with Guilbert that stemmed from Dodong’s younger
brother who had earlier urinated at the Patricio’s store premises.
Suddenly, Dodong assaulted Guilbert and stabbed him. Fearing that
he might be implicated in the incident, the accused fled. When he
learned that the victim was brought to the Hospital, he went there to
verify the victim’s condition. He was able to talk with the mother and
the wife of Guilbert as well as the police. He was thereafter invited
to the precinct so that the police can get his statement. Half a month
later, he was arrested and charged for the death of Guilbert Patricio.

On August 6, 2001, the RTC promulgated its Decision finding


accused-appellant guilty of murder. However, the Court of Appeals
on its decision dated February 14, 2008, subsequently held

482
accused-appellant liable only for the lesser crime of homicide.
Hence, this petition.
ISSUE/S 1. Whether or not the accused-appellant should be held liable
for murder as held by the RTC or homicide as adjudged by
the Court of Appeals.

2. Whether or not credibility should be given on the


prosecution’s witnesses since there were alleged inconsistencies in
their testimony.
RULING/S 1. The accused-appellant should be held liable for homicide as
adjudged by the Court of Appeals in the absence of the
qualifying circumstance of treachery. The jurisprudence
provides in a number of cases that, treachery cannot be
appreciated simply because the attack was sudden and
unexpected.
In People v. Escoto, it was held that: “We can not
presume that treachery was present merely from the
fact that the attack was sudden. The suddenness of
an attack, does not of itself, suffice to support a
finding of alevosia, even if the purpose was to kill,
so long as the decision was made all of a sudden
and the victim’s helpless position was accidental.
. . .”
In People v. Bautista, it was held: “. . . The
circumstance that an attack was sudden and
unexpected to the person assaulted did not
constitute the element of alevosia necessary to
raise homicide to murder, where it did not appear
that the aggressor consciously adopted such
mode of attack to facilitate the perpetration of the
killing without risk to himself. Treachery cannot be
appreciated if the accused did not make any
preparation to kill the deceased in such manner as to
insure the commission of the killing or to make it
impossible or difficult for the person attacked to
retaliate or defend himself. . ..”
Applying these principles to the case at bar, the prosecution
has not proven that the killing was committed with treachery.
It was not shown that he consciously adopted the mode of
attacking the victim from behind to facilitate the killing without
risk to himself. Moreover, the mere fact that the accused-
appellant stabbed the victim in a public market at 7:00pm
wherein the place was well-lighted and teeming with people.
He showed no concern that the people in the immediate

483
vicinity might retaliate in behalf of the victim. There are no
indications that accused-appellant deliberately planned to
stab the victim at said time and place.
Therefore, accused-appellant should be held liable for
homicide in the absence of the qualifying circumstance of
treachery.
2. Yes. Credibility should be given on the prosecution’s
witnesses.
Case laws mandate that “when the credibility of a
witness is in issue, the findings of fact of the trial court,
its calibration of the testimonies of the witnesses and
its assessment of the probative weight thereof, as well
as its conclusions anchored on said findings are
accorded high respect if not conclusive effect. This is
more true if such findings were affirmed by the
appellate court, since it is settled that when the trial
court’s findings have been affirmed by the appellate
court, said findings are generally binding upon this
Court.”
The inconsistencies and inaccuracies in the testimonies of
the prosecution witnesses are relatively trivial, minor, and do
not impeach their credibility. The positive identification and
categorical statements of the prosecution witnesses that it
was accused-appellant who stabbed Guilbert prevail over
accused-appellant’s self-serving denial.
Furthermore, these minor contradictions were expected from
said witnesses as they differ in their impressions of the
incident and vantage point in relation to the victim and the
accused-appellant. Therefore, credibility should be given on
the prosecution’s witnesses.

484
CONTRIBUTOR VILLANUEVA, Sean Ruthie
MODULE Classes of Aggravating Circumstances
TOPIC • Treachery
• Use of Unlicensed Firearm (RA 8294)
CASE TITLE PEOPLE V. MATIBAG G.R.NO. 206381
PONENTE PERLAS-BERNABE, J.: DATE: March 25, 2015
DOCTRINE • A frontal attack does not necessarily rule out treachery. The
qualifying circumstance may still be appreciated if the attack
was so sudden and so unexpected that the deceased had
no time to prepare for his or her defense.

• Presidential Decree No. (PD) 1866, as amended by


Republic Act No. (RA) 8294, treats the unauthorized use of
a licensed firearm in the commission of the crimes of
homicide or murder as a special aggravating circumstance
FACTS Around 8:40 in the evening, Enrico Clar de Jesus Duhan (Duhan),
was walking along the street when Matibag confronted Duhan, and
asked, “ano bang pinagsasasabi mo?” Duhan replied “wala,” and
without warning, Matibag delivered a fist blow hitting Duhan on the
left cheek and causing him to teeter backwards. Matibag then pulled
out his gun and shot Duhan, who fell face-first on the pavement.
While Duhan remained in that position, Matibag shot him several
more times.

Matibag contends that his action was self-defense. He alleges that


on said date, he was at the despedida party of his neighbor when
he came upon Duhan. Wanting to settle a previous
misunderstanding, Matibag approached Duhan and extended his
hand as a gesture of reconciliation. However, Duhan pushed it away
and said, “putang ina mo, ang yabang mo,” thereby provoking
Matibag to punch him in the face. Matibag saw Duhan pull
something from his waist and fearing that it was a gun and Duhan
was about to retaliate, Matibag immediately drew his own gun, shot
Duhan, and hurriedly left the place.

RTC refused to give credence to Matibag’s claim of self-defense as


he failed to prove the presence of the elements for self-defense.

RTC appreciated the existence of the qualifying circumstance of


treachery since the attack was sudden, unprovoked, and without
any warning on the victim who was unarmed and in a defenseless
position. Likewise, the special aggravating circumstance of use of
unlicensed firearm was appreciated since a firearm was used in the
commission of a crime and, hence, considered unlicensed.

485
ISSUE/S Whether or not the aggravating circumstances of treachery and
use of unlicensed firearm were correctly appreciated
RULING/S Yes, the aggravating circumstances of treachery and use of
unlicensed firearm were correctly appreciated.
There is treachery when the offender commits any of the crimes
against the person, employing means, methods, or forms in the
execution thereof which tend directly and specially to ensure its
execution, without risk to himself arising from the defense which
the offended party might make.
In this case, the prosecution was able to prove that Matibag, who
was armed with a gun, confronted Duhan, and without any
provocation, punched and shot him on the chest.
Although the attack was frontal, the sudden and unexpected
manner by which it was made rendered it impossible for Duhan to
defend himself, adding too that he was unarmed. Matibag was
ready and destined to effect such dastardly act, considering that
he had an axe to grind when he confronted Duhan, coupled with
the fact that he did so, armed with a loaded handgun. Based on
these findings, the treachery was correctly appreciated.
Likewise, the special aggravating circumstance of use of
unlicensed firearm is correctly appreciated in the imposition of
penalty. Presidential Decree No. (PD) 1866, as amended by
Republic Act No. (RA) 8294, treats the unauthorized use of a
licensed firearm in the commission of the crimes of homicide or
murder as a special aggravating circumstance.
Therefore, when Matibag killed Duhan with his firearm, the use
thereof was unauthorized under the purview of RA 8294 and is
equally appreciated as a special aggravating circumstance. As a
result, the imposition of the maximum penalty of death, which is
reduced to reclusion perpetua in light of RA 9346, stands proper.
To this, the Court adds that Matibag is not eligible for parole.

486
MODULE 10
PLURALITY OF CRIMES

487
CONTRIBUTOR PALALA, Amer B.
MODULE Plurality of Crimes
TOPIC
CASE TITLE People v. Tabaco G.R.NO. 100382
PONENTE HERMOSISIMA, JR., J. DATE: March 19, 1997
DOCTRINE The firing of several bullets by the accused although resulting from
one continuous burst of gunfire constitutes several acts and cannot
be considered a complex crime.
FACTS In four related information, Mario Tabaco was charged with four
counts of Murder for shooting to death on March 22, 1987 Capt.
Oscar Tabulog (Criminal Case No. 10-259), Ex-Mayor Jorge Arreola
(Criminal Case No. 10-270), Felicito Rigunan (Criminal Case No.
10-284) and Pat. Romeo Regunton (Criminal Case No. 10-317).

In the evening of March 22, 1987, the 117th PC stationed at Aparri,


Cagayan, under then Lt. James Andres Melad, sponsored a cock
derby, under the name of Jose Ting, at the Octagon Cockpit Arena
located at Aparri, Cagayan.

This being so, peace officers in uniform with long firearms were
assigned as guards to maintain peace and order at the cockpit arena
namely: (1) Sgt. Benito Raquepo; (2) CIS Roque P. Datugan, both
from the 117th PC and (3) Pat. Andres Semana, INP, Aparri,
Cagayan. Accused Mario Tabaco who was in civilian clothes claims
to have been also assigned by his Commanding Officer of 117th
PC, to verify the presence of NPAs and assist in the protection of
VIPs in the cockpit arena, bringing with him his M-14 issued firearm.

Other peace officers who came to participate were: (1) Policeman


Mariano Retreta of INP, Buguey, Cagayan, who arrived with the
deceased Jorge Siriban and Licerio Antiporda, Jr., Licerio Antiporda
II; (2) Sgt. Rogelio Ferrer of 117th PC Company; (3) Policeman
Romeo Regunton (deceased) who was also armed, arrived in
company with the deceased Ex-Mayor Arreola; (4) Fireman Rogelio
Guimmayen, INP Buguey; (5) Pat. Barba; and (6) CIC PC Paragas.

At about nine o'clock in the evening of same date, the group of the
late Mayor Jorge Arreola of Buguey, Cagayan, arrived at the cockpit
arena. His companions were (1) Antonio Villasin; (2) Rosario
Peneyra; (3) victim Loreto Pita, Jr. and/or five (5) of them including
the Mayor. They occupied and were (4th row) north western part
cockpit-gate. Others seated with the Mayor were: (1) the late Capt.
Oscar Tabulog; (2) the late Pat. Romeo Regunton, who was at the
back of the mayor; (3) the late Felicito Rigunan. The accused CIC
Tabaco was seated on the arm of the bench situated at the lower
portion of the arena about more than three (3) meters away, (infront

488
and a little bit in the west), from the place where the late Mayor and
his group were seated (at the 4th row of seats upper portion). During
the ocular inspection conducted, the Court noticed the distance to
be more than three (3) meters, and/or probably 4-5 meters.

At about ten (10) o'clock 1987, while the accused Mario Tabaco was
seated as described above, he suddenly without warning or
provocation, shot the late mayor Jorge Arreola, with his M-14 rifle,
followed by several successive burst of gunfire, resulting in the
shooting to death of the late Mayor Arreola, Capt. Oscar Tabulog,
Felicito Rigunan and Pat. Romeo Regunton, although the latter
managed to run passing through the western gate near the gaffers
cage but was chased by accused Tabaco. Regunton was later found
dead inside the canteen of Mrs. Amparo Go inside the Octagon
cockpit arena.
ISSUE/S Whether or not the criminal cases Nos. 259, 270, 284 and 317,
involving the killings of Oscar Tabulog, Jorge Arreola, Felicito
Rigunan and Romeo Regunton, respectively, are complex crimes
and should have been prosecuted under only one information.
RULING/S No, the criminal cases Nos. 259, 270, 284 and 317, involving the
killings of Oscar Tabulog, Jorge Arreola, Felicito Rigunan and
Romeo Regunton, respectively, are not complex crimes and should
not have been prosecuted under only one information. The trial
court was in error in imposing only a single penalty of reclusion
perpetua for all four murder cases. The trial court holding that a
complex crime was committed since "the evidence shows that the
four (4) victims were felled by one single shot/burst of fire and/or
successive automatic gun fires, meaning continuous (emphasis
ours) does not hold water.

In the case at bar, Article 48 of the Revised Penal Code is not


applicable because the death of each of the five persons who were
killed by appellant and the physical injuries inflicted upon each of
the two other persons injured were not caused by the performance
by the accused of one simple act as provided for by said article.
Although it is true that several successive shots were fired by the
accused in a short space of time, yet the factor which must be taken
into consideration is that, to each death caused or physical injuries
inflicted upon the victims, corresponds a distinct and separate shot
fired by the accused, who thus made himself criminally liable for as
many offenses as those resulting from every single act that
produced the same. Although he perpetrated a series of offenses
successively in a matter of seconds, yet each person killed and each
person injured by him became the victim, respectively, of a separate
crime of homicide or frustrated homicide. Except for the fact that five
crimes of homicide and two cases of frustrated homicide were

489
committed successively during the tragic incident, legally speaking
there is nothing that would connect one of them with its companion
offenses.

Consequently, the four murders which resulted from a burst of


gunfire cannot be considered a complex crime. They are separate
crimes. The accused-appellant must therefore be held liable for
each and every death he has caused, and sentenced accordingly to
four sentences of reclusion perpetua.

490
CONTRIBUTOR PANGAN, Gabrielle L.
MODULE Compound crime (delito compuesto)
TOPIC
CASE TITLE PEOPLE V. VALDEZ G.R.NO. 127663
PONENTE MELO, J: DATE: MARCH 11, 1999
DOCTRINE Each act by each gunman pulling the trigger of their respective
firearms, aiming each particular moment at different persons
constitute distinct and individual acts which cannot give rise to the
complex crime of multiple murder.
FACTS Accused-appellant Rolando Valdez is charged with the complex
crime of Multiple Murder with Double Frustrated Murder, and of
Illegal Possession of Firearms and Ammunitions (Presidential
Decree No. 1866).

William Montano (16 years old), Randy Tibule (17 years old), Jean
Marie Garcia, Willie Acosta, Sandra Montano and Ramon Garcia,
Jr. were in a tricycle when they met appellant Rolando Valdez and
his companions who were each armed with unlicensed caliber .30
carbines. The tricycle's headlight flashed on their faces. Without
warning, they pointed their guns and fired at Montano's group.
Thereafter, after uttering the words, "nataydan, mapan tayon" (They
are already dead. Let us go), Valdez and companions.

The shooting incident left Ramon Garcia, Jean Marie Garcia,


Sandra Montano and Willie Acosta dead. On the other hand, William
Montano and Randy Tibule survived the attack. They suffered
serious gunshot injuries that could have caused their death were it
not for the timely medical attention given them.
ISSUE/S Whether or not the offense is correctly classified as complex crime
of Multiple Murder with Double Frustrated Murder
RULING/S The offense is incorrectly classified as complex crime of Multiple
Murder with Double Frustrated Murder because the definition of the
complex crime is not met.
A complex crime is committed when a single act constitutes two or
more grave or less grave felonies or when an offense is a necessary
means for committing the other. The penalty for the most serious
crime shall be imposed, the same to be applied in its maximum
period.
The four crimes of murder resulted not from a single act but from
several individual and distinct acts. The evidence indicates that
there was more than one gunman involved, and the act of each
gunman is distinct from that of the other. It cannot be said therefore,
that there is but a single act of firing a single firearm. There were
also several empty bullet shells recovered from the scene of the
crime. This confirms the fact that several shots were fired.

491
Furthermore, considering the relative positions of the gunmen and
their victims, some of whom were riding the motorized tricycle itself
while the others were seated inside the sidecar thereof, it was
absolutely impossible for the four victims to have been hit and killed
by a single bullet. Each act by each gunman pulling the trigger of
their respective firearms, aiming each particular moment at different
persons constitute distinct and individual acts which cannot give rise
to the complex crime of multiple murder.
Accused-appellant is guilty, not of a complex crime of multiple
murder, but of four counts of murder for the death of the four victims
in this case. In the same manner, accused-appellant is likewise held
guilty for two counts of frustrated murder.
On the matter of accused-appellant's conviction for illegal
possession of unlicensed firearm under Presidential Decree No.
1866, there can be no separate conviction of the crime of illegal
possession of firearms under Presidential Decree No. 1866 in view
of the amendments introduced by Republic Act No. 8294. Instead,
illegal possession of firearms is merely to be taken as an
aggravating circumstance.
Accused-appellant is found guilty beyond reasonable doubt of four
counts of murder and sentenced to suffer the penalty of four
sentences of reclusion perpetua. He is also found guilty beyond
reasonable doubt of two counts of frustrated murder and meted two
indeterminate sentences, each, ranging from prision mayor as
minimum, to reclusion temporal as maximum.

492
CONTRIBUTOR PROVIDO, Gemy Hale A.
MODULE Complex Crime proper
TOPIC
CASE TITLE PEOPLE v. SANCHEZ G.R.NO. 131116
PONENTE PARDO, J: DATE: AUG. 27, 1999
DOCTRINE Article 48 of the Revised Penal Code provides that when a single
act constitutes two or more grave or less grave felonies, or when an
offense is a necessary means of committing the other, the penalty
for the more serious crime in its maximum period shall be imposed.
FACTS On April 13, 1991, Mayor Sanchez of Calauan, Laguna, Artemio
Averion, Peradillas and Luis Corcolon were charged for killing
Nelson Penalosa and his son, Rickson. The accused fired at the
victim´s jeep using M-16 and baby armalite rifles in automatic firing
mode. The three shots killed Nelson Penalosa and his son, Rickson.
His son also fell from the jeep after being shot and the jeep
continued its speed in a zigzag position until it overturned in Irais
Farm. After the completion of shooting the victims, the three men
went to the office of Mayor Sanchez to inform him of the succesful
killing of Penalosa. The accused denied the charges made against
them and provided alibi and denial. Luis Corcolon stated that he was
in the poultry farm on the day the incident happened. Artemio
Averion stated that he was in Lucena City to attend to his ailing
father on that day. Mayor Sanchez claimed that he went to Anilao,
Batangas and Tagaytay City on the day of the incident. However,
the trial court ruled that accused conspired in committing the crime.
Mayor Antonio L. Sanchez and Artemio Averion is to be charged
with the crime.
ISSUE/S Whether or not the accused committed a complex crime of double
murder
RULING/S No, the court held that the accused are liable for two counts of
murder and not with a complex crime of double murder. The
accused are criminally liable for the offenses to be committed in the
number of bullets that is produced and not by pressing the trigger
itself. The important part here is the number of bullets produced by
pressing the trigger of the armalites, the same as what have been
ruled in People v Vargas Jr. Eventhough the armalite is in automatic
firing mode and would result in several bullets in one firing, it still
does not constitute a complex crime.
Wherefore, the accused are guilty beyond reasonable doubt of two
(2) counts of murder, where the qualifying circumstance of treachery
is present and the aggravating circumstances of evident
premeditation and the use motor vehicle are appreciated.

493
CONTRIBUTOR ROMERO, Ma. Camille Concepcion M.
MODULE Plurality of Crimes: Compound Crime
TOPIC
CASE TITLE People vs. Nelmida G.R.NO. 184500
PONENTE PEREZ, J. DATE: SEP 11, 2012
DOCTRINE When conspiracy was not proven, joint criminal responsibility could
not be attributed to the accused. Each accused could not be held
liable for separate crimes because of lack of clear evidence showing
the number of persons actually killed by each of them.
FACTS On the afternoon of June 5, 2001, Mayor Tawan-tawan together with
his security escorts composed of members of Philippine Army and
several police, to wit: (1) T/Sgt. Dacoco; (2) PFC Angni; (3) PFC
Tomanto; (4) PO3 Dela Cruz; (5) Juanito; (6) Mosanip; (7)
Macasuba; and (8) a certain Jun were at Tubod, Lanao del Norte.
By the afternoon they went home to Salvador, Lanao del Norte.
Juanito was driving the yellow pick-up vehicle on the passenger side
is the Mayor. Sitted at the back seat were Mosanip, Jun, and
Macasuba, who was sitting immediately behind Juanito. Sitted on
the installed wooden seat near the rear of the vehicle were PFC
Tomanto and PFC Angni were sitting beside each other facing the
right side of the road while PO3 Dela Cruz and T/Sgt. Dacoco were
both seated behind PFC Tomanto and PFC Angni facing the left
side of the road.

At 3:00pm of same day appellants and co-accused instructed


Samuel to stay on the waiting shed of San Manuel, Lano del Norte
while they assembled themselves in a diamond position on both
sides of the road and waited for the vehicle of the Mayor to arrive.
Samuel saw the yellow pick-up vehicle arrive and the appellants and
co-accused open-fired using high powered firearms.

Macasuba sitting behind the driver and PFC Tomanto sitting on the
rear of the vehicle saw appellant Wenceslao on a squat firing from
the right side of the road using M-16 armalite rifle. Macasuba was
also able to identify appellants Ricardo, Pedro, Eduardo, Sr.,
Eduardo, Jr., Brigido and Alfredo as among the ambushers. The
Mayor instructed Juanito to keep driving until they reach the army
and Civilian Armed Forces Geographical Unit (CAFGU) detachment
in Curva.

After the ambush the appellants went to Samuel mother’s house


which they stayed prior the incident to get their bags and left.

PO3 De la Cruz and T/Sgt. Dacoco died while the others has serious
wounding that without the medical assistance would have caused

494
their deaths, except for Mayor Tawantawan and Jun Palanas who
were not hit.

Mayor Tawan-tawan and members of CAFGU went back to the site


of ambush. SPO4 Medrano arrived. Mayor Tawan-tawan informed
him that appellant Wenceslao was one of those responsible. They
conducted investigation and saw Samuel at the scene of the crime,
he denied involvement in the ambush. Medrano turned him over to
SPO4 Micabalo and was then brought to Lala Municipal Jail in
Lanao del Norte.

They then found footprints which Medrano believed to be from the


culprits. Samuel told the police that there were electrical supplies
and radio antenna in San Manuel, Lala, Lanao del Norte, left by the
malefactors. He then executed his sworn statement identifying
appellants and their co-accused, Samuel was, thereafter,
incarcerated at the Bureau of Jail Management and Penology
(BJMP) in Tubod, Lanao del Norte.

Two (2) months after the ambush, appellant Wenceslao was


arrested. Appellant Ricardo, on the other hand, was arrested on 20
December 2001 while working in Puting Bato. Appellant Ricardo, on
the other hand, was arrested on 20 December 2001 while working
in Puting Bato. They denied having any involvement.
ISSUE/S Whether or not the conviction of appellants must be for the separate
crimes of two (2) counts of murder and seven (7) counts of
attempted murder or of the complex crime of double murder with
multiple frustrated murder and double attempted murder.
RULING/S Yes, the court holds appellants liable for two separate crimes of two
(2) counts of murder and seven (7) counts of attempted murder.
The Information filed against appellants and their co-accused
alleged conspiracy. Although the trial court did not directly state that
conspiracy existed, such may be inferred from the concerted actions
of the appellants and their co-accused, to wit: (1) appellants and
their co-accused brought Samuel to a waiting shed located on the
left side of the road where the yellow pick-up service vehicle
boarded by Mayor Tawan-tawan and his group would pass; (2)
appellants and their co-accused, thereafter, assembled themselves
on both sides of the road and surreptitiously waited for the aforesaid
yellow pick-up service vehicle; (3) the moment the yellow pick-up
service vehicle passed by the waiting shed, appellants and their co-
accused opened fire and rained bullets thereon resulting in the
killing and wounding of the victims; (4) immediately, appellants and
their co-accused ran towards the house of Samuel’s aunt to get their
bags and other stuff; (5) Samuel followed appellants and their co-
accused; and (6) appellants and their co-accused fled. Therefore

495
their acts were coordinated. They were motivated by a single
criminal impulse, to kill the victims. They also had a common
purpose to ambush the victims and spontaneous agreement or
active cooperation during the commission of the crime which
constitutes joint criminal responsibility.
As to penalty, under Article 248 of RPC penalty imposed for murder
is reclusion perpetua to death. There being neither aggravating nor
mitigating circumstance, the penalty to be imposed upon appellants
is reclusion perpetua for each count, pursuant to paragraph 2,
Article 63 OF RPC. Appellants are also guilty of seven (7) counts of
attempted murder, the Court, therefore, imposed upon the
appellants the indeterminate penalty of 4 years and 2 months of
prision correccional, as minimum, to 10 years of prision mayor, as
maximum, for each count of attempted murder.

496
CONTRIBUTOR SAMONTE, Vanessa Antoinette
MODULE PLURALITY OF CRIMES- COMPLEX CRIME PROPER
TOPIC
CASE TITLE PEOPLE VS. HERNANDEZ A.M. NO. G.R. No. L-6025
PONENTE LABRADOR, J.: DATE: MAY 30, 1964

DOCTRINE Doctrine of absorption of common crimes or Hernandez


doctrine.The crime of rebellion under the Revised Penal Code of the
Philippines is charged as a single offense, and that it cannot be
made into a complex crime.

FACTS The Philippine government was in the height of action against


communists and guerillas. Appellant Amado Hernandez was a
bonafide member of Congress of Labor Organizations. CLO was an
affiliate of Hukbong Magpalayang Bayan, a known group performing
rebellious activities. He gave and rendered speeches favoring
Communism.
Subsequently, the said "Hukbong Mapagpalaya Ng Bayan" or
Hukbalahap took arms against the Government, by making armed
raids, sorties and ambushes, attacks against police, constabulary
and army detachment, and as a necessary means to commit the
crime of rebellion, in connection therewith and in furtherance
thereof, they committed wanton acts of murder, spoilage, looting,
arson, kidnappings, planned destruction of private and public
buildings, to create and spread terrorism in order to facilitate the
accomplishment of their purpose.
Hernandez was charged with, and was convicted of, rebellion
complexed with murders, arsons and robberies and was sentenced
to life imprisonment.
The defense contends, among other things, that rebellion cannot be
complexed with murder, arson, or robbery.

ISSUE/S 1.Whether or not rebellion can be complexed with murder, arson, or


robbery.
2.Whether or not Article 48 of the Revised Penal Code is applicable.

RULING/S Yes. Prosecutions under Article 365 should proceed from a single
charge regardless of the number or severity of the consequences.
In imposing penalties, the judge will do no more than apply the
penalties under Article 365 for each consequence alleged and
proven. In short, there shall be no splitting of charges under Article
365, and only one information shall be filed in the same first level
court. Article 48 is a procedural device allowing single prosecution
of multiple felonies falling under either of two categories: (1) when a
single act constitutes two or more grave or less grave felonies (thus
excluding from its operation light felonies); and (2) when an offense
is a necessary means for committing the other. The legislature

497
crafted this procedural tool to benefit the accused who, in lieu of
serving multiple penalties, will only serve the maximum of the
penalty for the most serious crime. In contrast, Article 365 is a
substantive rule penalizing not an act defined as a felony but the
mental attitude behind the act, the dangerous recklessness, lack of
care or foresight, a single mental attitude regardless of the resulting
consequences. Thus, Article 365 was crafted as one quasi-crime
resulting in one or more consequences. Article 48 is incongruent to
the notion of quasi-crime resulting in one or more consequences.

498
CONTRIBUTOR SAMSON, Jessa Viena D.
MODULE Complex crime proper (delito complejo)
TOPIC
CASE TITLE ENRILE v. SALAZAR G.R.NO. 92163
PONENTE NARVASA, J.: DATE: JUN. 5, 1990
DOCTRINE The Hernandez Doctrine states that murder and arson are crimes
inherent when rebellion is taking place. In the Revised Penal Code
(RPC), rebellion is just a single crime and there is no reason to
complex it with other crimes inherent in its commission. Thus,
Article 48 applies only when there are two crimes committed and
not when there is only one such as in this case.
FACTS Senate Minority Floor Leader Juan Ponce Enrile was arrested by
law enforcement officers led by Director Alfredo Lim of the National
Bureau of Investigation (NBI) on the strength of a warrant issued on
an information charging him and the spouses Rebecco and Erlinda
Panlilio, and Gregorio Honasan with the crime of rebellion with
murder and multiple frustrated murder allegedly committed during
the period of the failed coup attempt.

Senator Enrile was taken to and held overnight at the NBI


headquarters without bail, none having been recommended in the
information and none fixed in the arrest warrant. On the next day,
he was brought to Camp Tomas Karingal where he was given over
to the custody of the Superintendent of the Northern Police District,
Brig. Gen. Edgardo Dula Torres.

On the same day, Senator Enrile, through counsel, filed the petition
for habeas corpus herein, alleging that he was deprived of his
constitutional rights.
ISSUE/S Whether or not the petitioner has committed complex crimes (delito
complejo) arising from an offense being a necessary means for
committing another (with reference to Article 48)
RULING/S In the absence of aggravating circumstances, the extreme penalty
could not be imposed upon him. However, under Article 48 said
penalty would have to be meted out to him, even in the absence of
a single aggravating circumstance. Thus, said provision, if
construed in conformity with the theory of the prosecution, would be
unfavorable to the movant.
The plaint of petitioner’s counsel that he is charged with a crime that
does not exist in the statute books, while technically correct so far
as the Court has ruled that rebellion may not be complexed with
other offenses committed on the occasion thereof, must therefore
be dismissed as a mere flight of rhetoric. In the context of
Hernandez, the information does indeed charge the petitioner with
a crime defined and punished by the RPC as simple rebellion.

499
As regards the claim that the petitioner was denied the right to bail,
the Court's reaffirmation of Hernandez as applicable to this case,
and of the logical and necessary corollary that the information
against him should be considered as charging only the crime of
simple rebellion, which is bailable before conviction, that must now
be accepted as a correct proposition. However, the question
remains on whether or not the petition for habeas corpus in the
Court was the appropriate vehicle for asserting a right to bail or
vindicating its denial. The criminal case before the respondent
Judge was the normal venue for invoking the petitioner's right to
have provisional liberty pending trial and judgment. The original
jurisdiction to grant or deny bail rested with said respondent. The
correct course was for petitioner to invoke that jurisdiction by filing
a petition to be admitted to bail, claiming a right to bail per se by
reason of the weakness of the evidence against him. Only after that
remedy was denied by the trial court should the review jurisdiction
of this Court have been invoked, and even then, not without first
applying to the Court of Appeals if appropriate relief was also
available there.
The Court reiterates that based on the doctrine enunciated in the
Hernandez case, the questioned information filed against
petitioners Enrile and the spouses Rebecco and Erlinda must be
read as charging simple rebellion only, hence said petitioners are
entitled to bail, before final conviction, as a matter of right. The
Court’s earlier grant of bail to petitioners being merely provisional in
character, the proceedings in both cases are ordered remanded to
the respondent Judge to fix the amount of bail to be posted by the
petitioners. Once bail is fixed by said respondent for any of the
petitioners, the corresponding bail bond filed with this Court shall
become functus oficio. No pronouncement as to costs.

500
CONTRIBUTOR TERTE, Karen A.
MODULE Plurality of Crimes; Complex Crimes
TOPIC
CASE TITLE PEOPLE v. GARCIA G.R.NO. 141125
PONENTE Per Curiam DATE: FEBRUARY 28, 2002
DOCTRINE Under Article 48 of the Revised Penal Code, a complex crime is
when a single act constitutes two or more grave or less grave
felonies, or when an offense is a necessary means for committing
the other. In the eyes of the law, the two crimes stem from one
criminal intent – this is less perverse in the crimes of the law
compared to punishing him for two crimes. The penalty for complex
crimes is the penalty for the most serious crime which shall be
imposed in its maximum period. The reason for the single penalty is
that the basis of the felony is the singularity of the act.
FACTS The victim Cleopatra Changlapon was abducted by a white van on
her way home. Inside the van, one of the perpetrators sprayed some
mist on her face causing her to lose consciousness. She woke up
naked inside a room and looking around, she saw the four
perpetrators. The four had forcible carnal knowledge with her. They
even had to hold her arms to proceed with the act. After the ordeal,
one of the men sprayed a mist on Cleopatra’s face which made her
vision blur. After that, she blacked out. When she regained
consciousness, she was lying by the roadside somewhere between
Tam-awan and Longlong. She already had her clothes on. She felt
pain all over her body and was unable to move. A taxi passed by
and picked her up. Although she was afraid to ride the taxi, she
boarded it just to get home. The taxi brought her to her house.

Her aunt, Rufina Angog, saw Cleopatra alight the taxi crying. She
also noticed that Cleopatra's clothes were inverted and she smelled
bad. After some time, when she was able to regain her composure,
she told them that she had been raped by four men. The following
day, Cleopatra was brought to the Baguio City Police Station. After
giving her statement to the police, she was brought to the Crime
Laboratory of the Baguio City Police, where she was examined.
Cleopatra was also able to give a description of the four rapists to
the cartographer and the sketches were published in the
newspaper. Meanwhile, accused-appellant was arrested at 4:30
p.m. of July 17, 1998 in connection with another rape charge against
him filed by a certain Gilda Mangyo.

Police Officers Gilbert Bulalit and Archibald Diaz saw the sketches
and noticed that one of the suspects depicted in the cartographic
sketch bore a striking resemblance to accused-appellant, who was
in their custody. Cleopatra positively identified the accused-
appellant as one of her abductors.

501
ISSUE/S Whether or not the Trial Court erred in convicting accused-appellant
of the complex crime of forcible abduction with rape
RULING/S No, the trial court did not err in convicting accused-appellant of the
complex crime of forcible abduction with rape.
The two elements of forcible abduction, as defined in Article 342 of
the Revised Penal Code, are: (1) the taking of a woman against her
will and (2) with lewd designs. The crime of forcible abduction with
rape is a complex crime that occurs when there is carnal knowledge
with the abducted woman under the following circumstances: (1) by
using force or intimidation; (2) when the woman is deprived of
reason or otherwise unconscious; and (3) when the woman is under
twelve years of age or is demented. Clear in this case that the the
information sufficiently alleged the elements of forcible abduction,
i.e., the taking of complainant against her against her will and with
lewd design. It was likewise alleged that accused-appellant and his
three co-accused conspired, confederated and mutually aided one
another in having carnal knowledge of complainant by means of
force and intimidation and against her will.
Hence, accused-appellant is guilty of the complex crime of forcible
abduction with rape. He should also be held liable for the other three
counts of rape committed by his three co-accused, considering the
clear conspiracy among them shown by their obvious concerted
efforts to perpetrate, one after the other, the crime. As borne by the
records, all the four accused helped one another in consummating
the rape of complainant. While one of them mounted her, the other
three held her arms and legs. They also burned her face and
extremities with lighted cigarettes to stop her from warding off her
aggressor. Each of them, therefore, is responsible not only for the
rape committed personally by him but for the rape committed by the
others as well.
There can only be one complex crime of forcible abduction with
rape. The crime of forcible abduction was only necessary for the first
rape. Thus, the subsequent acts of rape can no longer be
considered as separate complex crimes of forcible abduction with
rape. They should be detached from and considered independently
of the forcible abduction. Therefore, accused-appellant should be
convicted of one complex crime of forcible abduction with rape and
three separate acts of rape.

502
CONTRIBUTOR TIDALGO, Aimee Diane A.
MODULE Complex crime proper (delito complejo)
TOPIC
CASE TITLE BUTULANON V. PEOPLE G.R.NO. 139857
PONENTE YNARES-SANTIAGO, J. DATE: SEPT. 15, 2006
DOCTRINE When a single act constitutes two or more grave or less felonies, or
when an offense is a necessary means for committing the other, the
penalty for the most serious crime shall be imposed, the same to be
applied in its maximum period.
FACTS In June 19821, Leonila Batulanon, manager-cashier of Polomolok
Credit Cooperative, Inc., (PCCI), entrusted with the duty of
managing the affairs of the cooperative, receiving payments to, and
collections of, the same, and paying out loans to members have
released four cash vouchers representing varying amounts to four
different individuals. Thereafter, four pieces of information for estafa
thru falsification of documents were filed against Butulanon.

The prosecution presented Maria Theresa Medallo, Benedicto


Gopio, Jr., and Bonifacio Jayoma as witnesses. Medallo testified
that Omadlao, Oracion, and Dennis Batulanon were not eligible to
apply for loan because they were not bona fide members of the
cooperative. Ferlyn Arroyo, on the other hand, was a member of the
cooperative but there was no proof that she applied for a loan with
PCCI in 1982. She subsequently withdrew her membership in 1983.
Medallo stated that pursuant to the cooperative’s by-laws, only bona
fide members who must have a fixed deposit are eligible for loans.

Gopio, Jr. stated that Oracion is Batulanon’s sister-in-law while


Dennis Batulanon is her son who was only 3 years old in 1982. He
averred that membership in the cooperative is not open to minors.
Then, Jayoma testified that the loans made to Oracion, Omadlao,
Arroyo and Dennis Batulanon did not pass through the cooperative’s
Credit Committee and PCCI’s Board of Directors for screening
purposes.

Batulanon denied all the charges against her. She claimed that she
did not sign the vouchers in the names of Omadlao, Oracion and
Arroyo.

Batulanon admitted that she took out a loan in her son’s name
because she is no longer qualified for another loan as she still has
to pay off an existing loan; that she had started paying off her son’s
loan but the cooperative refused to accept her payments after the
cases were filed in court. She also declared that one automatically
becomes a member when he deposits money with the cooperative.
When she was Cashier/Manager of PCCI from 1980 to 1982, the

503
cooperative did not have by-laws yet. On rebuttal, Jayoma belied
that PCCI had no by-laws from 1980-1982, because the cooperative
had been registered since 1967.

The motion for reconsideration was denied, hence this petition.


ISSUE/S Whether or not Butulanon should be charged of Estafa through
falsification of Private documents
RULING/S No. Although the offense charged in the information is estafa
through falsification of commercial document, appellant could be
convicted of falsification of private document under the well-settled
rule that it is the allegations in the information that determines the
nature of the offense and not the technical name given in the
preamble of the information.

The elements of falsification of private document under Article 172,


paragraph 236 of the Revised Penal Code are: (1) thatthe offender
committed any of the acts of falsification, except those in paragraph
7, Article 171; (2) that the falsification was committed in any private
document; and (3) that the falsification caused damage to a third
party or at least the falsification was committed with intent to cause
such damage.

The CA correctly ruled that the subject vouchers are private


documents and not commercial documents because they are not
documents used by merchants or businessmen to promote or
facilitate trade or credit transactions nor are they defined and
regulated by the Code of Commerce or other commercial law.
Rather, they are private documents, which have been defined as
deeds or instruments executed by a private person without the
intervention of a public notary or of other person legally authorized,
by which some disposition or agreement is proved, evidenced or set
forth.

Since Batulanon’s conviction was for 3 counts of falsification of


private documents, she shall suffer the aforementioned penalties for
each count of the offense charged.

However, the other case was charged differently for the trial court
convicted petitioner Batulanon for falsifying Dennis Batulanon’s
signature in the cash voucher based on the Information charging her
of signing the name of her 3 year old son, Dennis. What she did was
to sign: “by: lbatulanon” to indicate that she received the proceeds
of the loan in behalf of Dennis. Said act does not fall under any of
the modes of falsification under Article 171 because there in nothing
untruthful about the fact that she used the name of Dennis and that
as representative of the latter, obtained the proceeds of the loan

504
from PCCI. The essence of falsification is the act of making
untruthful or false statements, which is not attendant in this case. As
to whether, such representation involves fraud which caused
damage to PCCI is a different matter which will make her liable
for estafa, but not for falsification.

505
CONTRIBUTOR Valdez, Ariane Faye V.
MODULE Module 10: Continuing crime (delito continuado)
TOPIC
CASE TITLE People v. Madrigal-Gonzales G.R.NO. L-16688
PONENTE PAREDES, J. DATE: 30 April 1963
DOCTRINE Motive is not an element of a felony; it is merely a prospectorant
circumstantial evidence. What renders an act a felony is the criminal
intent.
FACTS On August 23, 1956, the accused-appellee Pacita Madrigal-
Gonzales was charged with malversation of public funds, in the
amount of P104,000.00 before the Court of First Instance of Manila
under an information alleging that said accused, while administrator
of the Social Welfare Administration (SWA), appropriated, took and
misappropriated the said amount on five different occasions
comprised within the period from February, 1954 to
September,1955, in the City of Manila.

That simultaneously on the same date, the same accused-appellee


Pacita Madrigal-Gonzales was charged together with her seven co-
accused with the crime of falsification of public documents under 27
separate information filed before the same Court of First Instance of
Manila. They were alleged to have conspired in the commission of
said offense in or about and during the period comprised between
December, 1954 and September, 1955, by having allegedly caused
it to appear, that:
1. Cash aids were given when no such aids were indeed
distributed to the persons named and at the time and place
and in such amounts specified;
2. By making and/or causing it to appear that certain relief
supplies or merchandise were purchased by the accused
Pacita Madrigal-Gonzales when in truth and in fact no such
relief supplies were purchased, thereby making untruthful
statements in a narration of fact in said public and official
documents.

After protracted quibbling on various procedural and technical


aspects of the distribution of said 27 falsification cases the herein
accused-appellees filed a motion to quash the criminal cases. After
which, hearing the arguments pro and con on the said motion to
quash, the prosecution and the defense filing memorandum and
rebuttal memorandum on the said issues, the Court of First Instance
of Manila, all for falsification, in an order promulgated on January
19, 1960, granted the aforesaid motion to quash on the ground of
double jeopardy, and held:

506
(a) That the 27 separate cases of falsification should indeed
be tried and/or prosecuted only under one information for the
crime of falsification as a continuing offense resulting from
only one criminal intent or impulse, and

(b) That according to the spirit rather than the letter of the
constitutional safeguard against double jeopardy and the
complementary provision on the matter in our Rules of Court,
the accused Pacita Madrigal Gonzales and her seven co-
accused were placed in jeopardy of trial for the same offense
immediately after entering their pleas with Branch X of the
said court, and therefore, said plea could be pleaded as a bar
to their further prosecution for the other offenses of
falsification pending before said Branch XVIII.

That dissatisfied with the foregoing dismissal, the prosecution has


interposed the instant appeal on purely questions of law. Hence, this
petition.
ISSUE/S Whether or not the twenty-seven (27) falsifications were the product
of only one criminal intent.
RULING/S No. Although, it is argued that since all the falsifications were
supposedly committed within a specific period (from December,
1954 to September, 1955), and that the allegations in the different
informations for falsification are the same, said acts were but the
product or result of a singular criminal intent and that the
falsifications were committed to conceal the malversation. The
appellees seem to confuse motive with criminal intent.
Motive is not an element of a felony; it is merely a
prospectant circumstantial evidence. Criminal intent
renders an act a felony. Motive is a state of the mind of the
accused, and it is he who can state his real motive in
committing a crime.
The existence of the motive to conceal malversation, in the cases at
bar, is a question off act which should be ventilated in a formal trial,
in connection with the defense of double jeopardy. The Court cannot
assume that the purpose of committing the twenty-seven (27)
falsifications was to conceal the malversation. This is so because
there is no showing that for every particular amount they had
malversed on a certain period, they had purposedly perpetrated the
corresponding falsification to cover up such amount, until the whole
amount proposed to be malversed, shall have been completely
misappropriated.
In the absence of such showing, it is to be presumed that in the
falsification of each document, the criminal intent was separated

507
and distinct. Moreover, the jurisprudence provides in a number of
cases that:
In U.S. v. Infante & Barreto, 36 Phil. 146. “The falsification
of each of these six money orders committed separately by
means of different acts constitutes independent crimes of
falsification”.
In People v. Cid, 66 Phil.354. “It may therefore be said that
the malversations as well as the falsifications imputed to the
accused in the four cases under consideration were not the
result of only one purpose or of only one resolution to
embezzle and falsify, but of four or as many abstractions or
misappropriations had of the funds entrusted his care, and of
as many falsifications also committed to conceal each of said
acts.”
In Regis v. People, 67 Phil. 43, “The conclusion of the Court
of Appeals that the falsifications committed on April 30, 1931
and on May 2 of the same year were not necessary means
for the commission of the malversations on the same dates,
is correct. Each falsification and each malversation
constitute independent offenses which must be
punished separately x x x. The acts being independent
from each other and executed by different voluntary
actions, each constitutes an independent offense.”
In the case at bar and upon the basis of the above jurisprudence, it
can be said that the 27 falsifications perpetrated on separate
vouchers, at different dates and in various amounts, constitute 27
separate and independent crimes, as such, they were not
continuous crimes. Hence, were not a product of a one criminal
intent.

508
CONTRIBUTOR VILLANUEVA, Sean Ruthie
MODULE CONTINUING CRIME (delito continuado)
TOPIC
CASE TITLE JOSE L. GAMBOA and UNITS OPTICAL G.R L-41054
SUPPLY COMPANY, petitioners, vs. NO.
COURT OF APPEALS and BENJAMIN
LU HAYCO, respondents
PONENTE MARTIN, J DATE: November
28, 1975
DOCTRINE The term "continuing" must be understood in the sense similar to
that of "transitory" and is only intended as a factor in determining the
proper venue or jurisdiction for that matter of the criminal action
pursuant to Section 14, Rule 110 of the Rules of Court because "a
person charged with a transitory offense may be tried in any
jurisdiction where the offense is part committed

FACTS Respondent Benjamin Lu Hayco was an employee of petitioner


Units Optical Supply Company. Petitioner company filed 124
complaints of estafa were filed against him, 75 of which were
formally charged after the preliminary investigation. The common
charges only differ in dates and amounts complained of.
A civil action for accounting was also filed against him, this time by
the owner. Lu CHiong Son alleged that respondent initiated taking
over the business while the former was confined in a hospital.
Through fraud, deceit and machinations, Hayco duped him into
affixing signature and thumbmark in a general power of attorney that
caused the closing of Son’s accounts in a bank. Respondent
transferred accounts into his own name with the same bank whew.
Respondent commenced petition for prohibition with preliminary
injunction pendent lite, claiming that the filing, prosecutuion and trial
of the 75 estafa cases is oppressive, whimsical and capricious and
without or in excess of jurisdiction of City Fiscal and the City Court
Judges of Manila. He asserts that all the indictments with regard to
the 75 estafa informations were mere components of only one
crime, since the same were only impelled by a single criminal
resolution or intent.

ISSUE/S WON the accusations contained in the 75 informations against


respondent constitute a single crime of estafa

RULING/S No. Apart from the crimes defined and punished under Art. 48 of the
Revised Penal Code is the "delito continuado" or "continuous
crime". This is a single crime consisting of a series of acts arising
from a single criminal resolution or intent not susceptible of division.
In order that it may exist, there should be "plurality of acts performed
separately during a period of time; unity of penal provision infringed

509
upon or violated and unity of criminal intent and purpose, which
means that two or more violations of the same penal provision are
united in one and the same intent leading to the perpetration of the
same criminal purpose or aim." So long as the act or acts
complained of resulted from a single criminal impulse it is usually
held to constitute a single offense to be punished with the penalty
corresponding to the most serious crime, imposed in its maximum
period. The test is not whether one of the two offenses is an
essential element of the other. "to apply the first half of Article 48, ...
there must be singularity of criminal act; singularity of criminal
impulse is not written into the law."

The characterization or description of estafa as a continuing offense


cannot be validly seized upon by private respondent as basis for its
inference that the acts of abstraction in question constitute but a
single continuing crime of estafa. The sole import of this
characterization is that the necessary elements of estafa may
separately take place in different territorial jurisdictions until the
crime itself is consummated. The moment, however, that the
elements of the crime have completely concurred or transpired, then
an individual crime of estafa has occurred or has been
consummated.

510
CONTRIBUTOR PROVIDO, Gemy Hale A.
MODULE Continued Crime
TOPIC
CASE TITLE PEOPLE v. MALLARI G.R.NO. L-58886
PONENTE FERNAN, C.J.: DATE: DEC. 13, 1988
DOCTRINE For a continued crime to be committed, a series of acts must be
done but it must only arise to a single crime. It can be continuous,
unlawful act or series of acts committed even with long intervals of
time. There is only one crime committed, hence, only one penalty
should be given.
FACTS Petitioner Consuelo E. Mallari, with three others, was accused of the
crime of Estafa thru Falsfication of Public Document, however, the
case of Mallari was the only one which proceeded. She was found
guilty and was charged and sentenced to imprisonment of one (1)
year and to pay costs and P1500,00 to Remegio Tapawan, the
offended party. Petitioner appealed as she was already punished
for the same crime in another case, however, the court denied the
motion for reconsideration. Therefore, this case is for the instant
petition for review.
ISSUE/S Whether or not the crime of estafa thru falsification of public
document committed by Consuelo Mallari committed through a
series of acts constitute a continued crime
RULING/S Yes, from the information gathered, there is singularity in the offense
committed by the petitioner. The crime was committed on the same
date, in the same place, at the same time and on the same occasion,
hence, it can be concluded as a continued crime and the petitioner
should only be punished with one penalty. Eventhough there was a
series of acts committed by the offender, it only arises to one crime
committed, hence should be treated as a continued crime with only
one penalty.
Since the petitioner has already been convicted of the complex
crime of estafa thru falsification of public document in the case
docketed as G.R. No. 20817-CR, she can no longer be held liable
for the crime in the present case using the defense of double
jeopardy. The instant petition to review the case to use the defense
of double jeopardy, wherein three requisites must be present
namely:
1. A first jeopardy must have attached prior to the second;
2. The first jeopardy must have been validly terminated, and;
3. The second jeopardy must be for the same offense as that in the
first.
Hence, the instant petition is granted and the petitioner will no longer
be held liable in the present case.

511
CONTRIBUTOR ROMERO, Ma. Camille Concepcion M.
MODULE Plurality of Crimes
TOPIC
CASE TITLE People v. Garchitorena G.R.NO. 109266
PONENTE QUIASON, J. DATE: DEC. 2, 1993
DOCTRINE For delito continuado to exist there should be a plurality of acts
performed during a period of time; unity of penal provision violated;
and unity of criminal intent or purpose, which means that two or
more violations of the same penal provisions are united in one and
same instant or resolution leading to the perpetration of the same
criminal purpose or aim
FACTS This is a petition for certiorari under Rule 65 of the Revised Rules of
Court to set aside: (a) the Resolution dated March 3, 1993 in
Criminal Case No. 16698 of the Sandiganbayan (First Division) and
to declare Presiding Justice Francis Garchitorena of the
Sandiganbayan, disqualified from acting in said criminal case; and
(b) the Resolution of said court promulgated on March 14, 1993,
which deemed as "filed" the 32 Amended Informations against
petitioner.

On May 1, 1991, petitioner Santiago was charged by the


Sandiganbayan with violation of Section 3(e) of R.A. No. 3019, as
amended, otherwise known as the Anti-Graft and Corrupt Practices
Act, allegedly committed by her favoring "unqualified" aliens with the
benefits of the Alien Legalization Program.

On May 24, 1991, petitioner filed a petition for certiorari and


prohibition to enjoin the Sandiganbayan from proceeding with
criminal case on the ground that said case was intended solely to
harass her as she was then a presidential candidate. She alleged
that this was in violation of Section 10, Article IX-C of the
Constitution which provides that "(b)ona fide candidates for any
public office shall be free from any form of harassment and
discrimination." The petition was dismissed on January 13, 1992.

On October 16, 1992, petitioner filed a motion for inhibition of


Presiding Justice Garchitorena, which motion was set for hearing on
November 13, 1992. ten days after, the Sandiganbayan (First
Division), of which Presiding Justice Garchitorena is a member, set
the criminal case for arraignment on November 13, 1992. On
November 6, 1992, petitioner moved to defer the arraignment on the
grounds that there was a pending motion for inhibition, and that
petitioner intended to file a motion for a bill of particulars. However,
on November 9, 1992, the Sandiganbayan (First Division) denied
the motion to defer the arraignment.

512
More so, the petitioner cannot accept the legal morality of
Sandiganbayan Justice Francis Garchitorena who would her from
going abroad for a Harvard scholarship because of graft charges
against her. It appears that petitioner tried to leave the country
without first securing the permission of the Sandiganbayan,
prompting it to issue the hold-departure order which. The letter of
Presiding Justice Garchitorena, written in defense of the dignity and
integrity of the Sandiganbayan, merely stated that all persons facing
criminal charges in court, with no exception, have to secure
permission to leave the country.

The court issued the Resolution dated March 25, 1993, ordering
Presiding Justice Garchitorena "to CEASE and DESIST from sitting
in the case until the question of his disqualification is finally resolved
by this Court and from enforcing the resolution dated March 11,
1993, ordering petitioner to post bail bonds for the 32 Amended
Informations and from proceeding with the arraignment on April 12,
1993.
ISSUE/S Whether or not the petitioner is charged with continued crime (delito
continuado) under Article 48 of the Revised Penal Code
RULING/S The 32 Amended Informations charged to the petitioner is known as
delito continuado or "continued crime" and sometimes referred to as
"continuous crime." In fairness to the Ombudsman's Office of the
Special Prosecutor, it should be borne in mind that the concept of
delito continuado has been a vexing problem in Criminal Law —
difficult as it is to define and more difficult to apply.
According to Cuello Calon, for delito continuado to exist there
should be a plurality of acts performed during a period of time; unity
of penal provision violated; and unity of criminal intent or purpose,
which means that two or more violations of the same penal
provisions are united in one and same instant or resolution leading
to the perpetration of the same criminal purpose or aim.
The concept of delito continuado, although an outcry of the Spanish
Penal Code, has been applied to crimes penalized under special
laws, e.g. violation of R.A. No. 145 penalizing the charging of fees
for services rendered following up claims for war veteran's benefits
(People v. Sabbun, 10 SCRA 156 [1964] ). Under Article 10 of the
Revised Penal Code, the Code shall be supplementary to special
laws, unless the latter provide the contrary. Hence, legal principles
developed from the Penal Code may be applied in a supplementary
capacity to crimes punished under special laws.
In the case at bench, the original information charged petitioner with
performing a single criminal act that of her approving the application
for legalization of aliens not qualified under the law to enjoy such
privilege. The original information also averred that the criminal act
: (i) committed by petitioner was in violation of a law - Executive

513
Order No. 324 dated April 13, 1988, (ii) caused an undue injury to
one offended party, the Government, and (iii) was done on a single
day, i.e., on or about October 17, 1988.
The Resolution dated March 3, 1993 in Criminal Case No. 16698 of
the Sandiganbayan (First Division) is affirmed and its Resolution
dated March 11, 1993 in Criminal Case No. 16698 is modified in the
sense that the Office of the Special Prosecutor of the Office of the
Ombudsman is directed to consolidate the 32 Amended
Informations (Criminal Cases Nos. 18371 to 18402) into one
information charging only one offense under the original case
number, i.e., No. 16698. The temporary restraining order issued by
this Court on March 25, 1993 is lifted insofar as to the
disqualification of Presiding Justice Francis Garchitorena is
concerned.

514
CONTRIBUTOR SAMONTE, Vanessa Antoinette
MODULE PLURALITY OF CRIMES- COMPLEX CRIMES IN RELATION TO
TOPIC QUASI-CRIMES
CASE TITLE IVLER VS. SAN PEDRO A.M. NO. G.R. NO. 172716
PONENTE CARPIO, J.: DATE: NOVEMBER 17,2010

DOCTRINE Article 48 is incongruent to the notion of quasi-crimes under Article


365. It is conceptually impossible for a quasi-offense to stand for (1)
a single act constituting two or more grave or less grave felonies; or
(2) an offense which is a necessary means for committing another.

FACTS Ivler was charged before the Metropolitan Trial Court of Pasig City
(MeTC), with two separate offenses: (1) reckless imprudence
resulting in slight physical injuries for injuries sustained by
respondent Evangeline L. Ponce; and (2) reckless imprudence
resulting in homicide and damage to property for the death of
respondent Ponce’s husband Nestor C. Ponce and damage to the
spouses Ponce’s vehicle.
Ivler pleaded guilty to the charge in reckless imprudence resulting
in slight physical injuries and was meted out the penalty of public
censure. Invoking this conviction, Ivler moved to quash the
Information of reckless imprudence resulting in homicide and
damage to property for placing him in jeopardy of second
punishment for the same offense of reckless imprudence.

ISSUE/S Whether or not prior conviction or acquittal of reckless imprudence


bars subsequent prosecution for the same quasi-offense.

RULING/S Yes. Prosecutions under Article 365 should proceed from a single
charge regardless of the number or severity of the consequences.
In imposing penalties, the judge will do no more than apply the
penalties under Article 365 for each consequence alleged and
proven. In short, there shall be no splitting of charges under Article
365, and only one information shall be filed in the same first level
court. Article 48 is a procedural device allowing single prosecution
of multiple felonies falling under either of two categories: (1) when a
single act constitutes two or more grave or less grave felonies (thus
excluding from its operation light felonies); and (2) when an offense
is a necessary means for committing the other. The legislature
crafted this procedural tool to benefit the accused who, in lieu of
serving multiple penalties, will only serve the maximum of the
penalty for the most serious crime. In contrast, Article 365 is a
substantive rule penalizing not an act defined as a felony but the
mental attitude behind the act, the dangerous recklessness, lack of
care or foresight, a single mental attitude regardless of the resulting
consequences. Thus, Article 365 was crafted as one quasi-crime

515
resulting in one or more consequences. Article 48 is incongruent to
the notion of quasi-crime resulting in one or more consequences.

516
MODULE 11
PENALTIES

517
CONTRIBUTOR AGUILAR, Jose Maria L.
MODULE IMPOSABLE PENALTIES
TOPIC
CASE TITLE PEOPLE v. BON G.R.NO. 166401
PONENTE TINGA, J.: DATE: OCT. 30, 2006
DOCTRINE It should be understood that the debarring of the death penalty
through Rep. Act No. 9346 did not correspondingly declassify those
crimes previously catalogued as "heinous". The amendatory effects
of Rep. Act No. 9346 extend only to the application of the death
penalty but not to the definition or classification of crimes. True, the
penalties for heinous crimes have been downgraded under the
aegis of the new law. Still, what remains extant is the recognition by
law that such crimes, by their abhorrent nature, constitute a special
category by themselves. Accordingly, Rep. Act No. 9346 does not
serve as basis for the reduction of civil indemnity and other damages
that adhere to heinous crimes.
FACTS Eight Informations were filed within by Assistant Provincial
Prosecutor against appellant, charging him with the rape of
AAA and BBB, the daughters of his older brother. The rapes were
alleged to have been committed in several instances over a span of
six years.
Both AAA and BBB testified against appellant, their uncle, and both
identified him as the man who had raped them. AAA testified that
she was only six years old when she was first molested in the house
appellant had shared with her grandmother. She recounted that the
incident took place when she and appellant were alone in the house.
Appellant threatened that she and her parents would be killed
should she disclose the incident to anyone. She thereafter stopped
sleeping in the house of her grandmother. It was only three years
after, that she slept in the said house, yet again she was sexually
abused by appellant. She was then nine years old. BBB, on the
other hand, testified that she was first raped by appellant when she
was ten years old, also at the house appellant shared with her
grandmother. While alone in the house, appellant poked a knife at
her, removed her clothes and inserted his penis in her vagina.
The accused offered a general denial of the other charges against
him by BBB and AAA. He claimed that he seldom saw the two
minors. He further asserted that prior to the institution of the criminal
case against him he had a smooth relationship with his nieces and
the only reason the case was filed against him was that CCC, his
sister-in-law and the mother of his nieces, harbored ill-feelings
towards his deceased father, who would call CCC "lazy" within
earshot of other family members.
The RTC convicted appellant on all eight counts of rape. The RTC
pronounced appellant's defense of denial and alibi as unconvincing.
It further considered the qualifying circumstances of minority of the

518
victims and the relationship of the victims and appellant, the latter
being the former's relative by consanguinity within the third degree.
As the penalty imposed consisted of eight death sentences, the
records of the case were automatically elevated to this Court for
review.
ISSUE/S Whether or not the penalty imposed for the crimes committed was
proper.
RULING/S No. We shall not dwell at length on the proper penalty imposable on
appellant for the six counts of rape. The sentence of death imposed
by the RTC and affirmed by the Court of Appeals can no longer be
affirmed in view of Rep. Act No. 9346, titled "An Act Prohibiting the
Imposition of Death Penalty in the Philippines." Section 2 of the law
mandates that in lieu of the death penalty, the penalty of reclusion
perpetua shall be imposed. Correspondingly, the Court can no
longer uphold the death sentences imposed by lower courts, but
must, if the guilt of the accused is affirmed, impose instead the
penalty of reclusion perpetua, or life imprisonment when
appropriate. Since the passage of Rep. Act No. 9346
Article 51 of the Revised Penal Code establishes the penalty to be
imposed upon the principals of an attempted felony. As provided,
penalty lower by two degrees than that prescribed by law for the
consummated felony shall be imposed upon the principals in an
attempt to commit a felony. Following the scale prescribed in Article
71, the penalty two degrees lower than death is reclusion temporal,
which was the maximum penalty imposed by the Court of Appeals
on appellant for attempted rape. Reclusion temporal is a penalty
comprised of three divisible periods, a minimum, a medium and a
maximum. Thus, convicts sentenced to suffer death penalty or life-
imprisonment are ineligible under that law, as are persons
sentenced to reclusion perpetua, an indivisible penalty without
minimum or maximum periods.
Hence, the Court of Appeals sentenced appellant to suffer the
penalty for attempted rape, with a maximum penalty within the range
of reclusion temporal, and a minimum penalty within the range of
the penalty next lower, or prision mayor. If Rep. Act No. 9346 had
not been enacted, the Court would have affirmed such sentence
without complication. However, the enactment of the law has given
rise to the problem concerning the imposable penalty. Appellant was
sentenced to a maximum term within reclusion temporal since that
is the penalty two degrees lower than death. With the elimination of
death as a penalty, does it follow that appellant should now be
sentenced to a penalty two degrees lower than reclusion perpetua,
the highest remaining penalty with the enactment of Rep. Act No.
9346? If it so followed, appellant would be sentenced to prision
mayor in lieu of reclusion temporal.

519
Admittedly, the impact of Rep. Act No. 9346 is less dramatic in
relation to frustrated and attempted felonies which were punishable
by death if consummated. The consummated felony previously
punishable by death would now be punishable by reclusion
perpetua. At the same time, the same felony in its frustrated stage
would, under the foregoing premise in this section, be penalized one
degree lower from death, or also reclusion perpetua. It does not
seem right, of course, that the same penalty of reclusion
perpetua would be imposed on both the consummated and
frustrated felony. However, the anomaly would be mainly in theory,
as we recognize that those felonies previously punishable by death
are improbable of commission in their frustrated stage, unlike
several felonies punishable by "reclusion perpetua to death," such
as murder, which may be frustrated.
Still, it cannot be denied that these felonies previously punishable
by death are capable of commission in their attempted stages and
that the Revised Penal Code provides that the penalty for attempted
felonies is "a penalty lower by two degrees than that prescribed by
law for the consummated felony." The Court has thus consistently
imposed reclusion temporal, the penalty two degrees lower than
death, as the maximum term for attempted felonies which, if
consummated, would have warranted the death penalty. If it were to
be insisted that Rep. Act No. 9346 did not affect at all the penalties
for attempted felonies, then those found guilty of the subject
attempted felonies would still be sentenced to reclusion temporal,
even though the "penalty lower by two degrees than that prescribed
by law for the consummated felony" would now be prision mayor.
Hence, the maximum term of his penalty shall no longer be reclusion
temporal, but instead, prison mayor.

520
CONTRIBUTOR ALGURA, Nino N.
MODULE Penalties
TOPIC
CASE TITLE PEOPLE OF THE PHILIPPINES G.R.NO. 108172
v.
CONRADO LUCAS Y BRIONES
PONENTE DAVIDE, JR., J.: DATE: May 25, 1994
DOCTRINE Prior to R.A. No. 7659, the presence of modifying circumstances
would not affect the penalty of reclusion perpetua prescribed for the
crime of rape because such a penalty was then indivisible and under
Article 36, when the law prescribes a single indivisible penalty, it
shall be applied by the courts regardless of any mitigating or
aggravating circumstances that may have attended the commission
of the deed

FACTS In a sworn statement taken on 16 February 1991, Chanda Lucas y


1

Austria, then seventeen years old, charged her natural father,


accused Jose Conrado Lucas, of attempted rape committed against
her on 12 February 1991. She revealed therein that she was first
raped by him when she was only nine years old, or, as disclosed in
a handwritten note at the left-hand margin of her sworn statement,
"noong Nov. 26, 1982 . . . at naulit ng maraming beses."

On 19 February 1991, Chanda, assisted by her mother, Ofelia


Austria-Lucas, filed two separate sworn criminal complaints for
rape and for attempted rape against her father with the Regional
2 3

Trial Court of Quezon City. The complaints, docketed as Criminal


Cases Nos. Q-91-18465 and Q-91-18466, were subsequently
assigned to Branch 104 of the said court.

That on or about the 26th day of November 1982 and sometime


thereafter in Quezon City, Philippines and within the jurisdiction of
this Honorable Court, the above-named accused, with lewd designs
and by means of violence and intimidation did then and there,
wilfully, unlawfully and feloniously have sexual intercourse with the
undersigned CHANDA LUCAS Y AUSTRIA, who was then nine (9)
years old, now 17 yrs. of age, against her will, to her damage and
prejudice in such amount as may be awarded to her under the
provisions of the New Civil Code.

He also contends that the testimony of Cynthia is not convincing; it


was contrary to human experience and conduct for her to simply
close her eyes and cover her face with a blanket upon witnessing
the rape of her younger sister by their own father instead of helping
Chanda. If she was afraid of her father at that time, she could have
convinced Chanda to temporarily leave their house and seek shelter

521
with her relatives. It was also unnatural for her to abandon Chanda
when, as she claims, she fully knew the bestial tendencies of her
father. As to his wife, Ofelia, he attributes to her an ulterior motive
when she consented to the filing of the charges against him. Except
for the souring of their relationship which ended in their separation,
he finds no possible explanation why Ofelia believed Chanda's
report on the 12 February 1991 incident when she, Ofelia, refused
to heed Chanda and Cynthia's report concerning the 26 November
1983 incident.

ISSUE/S Whether or not the appellant is guilty of the crime rape.


RULING/S No.
Taking into account the presence of the aggravating circumstance
of relationship in Criminal Case No. Q-91-18465, the accused may
finally be sentenced to thirty-four (34) years, four (4) months and
one (1) day of reclusion perpetua.
Considering again such aggravating circumstance, the accused
may be sentenced in Criminal Case No. Q-91-18466 to an
indeterminate penalty ranging from four (4) years, two (2) months
and one (1) day of prision correccional maximum as minimum to ten
(10) years and one (1) day of prision mayor maximum as maximum.
There should also be awards for damages in each of the two cases.
WHEREFORE, the challenged Decision of 28 October 1992 of
Branch 104 of the Regional Trial Court of Quezon City in Criminal
Case
No. Q-91-18465 and Criminal Case No. Q-91-18466 is hereby
AFFIRMED, subject to the modifications indicated above. As
modified:
(1) In Criminal Case No. Q-91-18465, accused JOSE CONRADO
LUCAS y BRIONES is hereby sentenced to suffer the penalty
of Thirty-four (34) years, Four (4) months and One (1) day
of reclusion perpetua and to pay the offended party the sum
of P50,000.00 as civil indemnity; and
(2) In Criminal Case No. Q-91-18466, said accused is hereby found
GUILTY beyond reasonable doubt of the crime of ATTEMPTED
RAPE only and is hereby sentenced to suffer an
indeterminate penalty ranging from Four (4) years, TWO (2) months
and One (1) day of prision correccional maximum as minimum to
Ten (10) years and one (1) day of prision mayor maximum
as maximum and to pay the offended party the sum of P30,000.00
as civil indemnity.

522
CONTRIBUTOR AROZA, Maria Minette R.
MODULE Penalties – Reclusion Perpetua, R.A. 7659
TOPIC
CASE TITLE PEOPLE v. LUCAS G.R.NO. 1108172
PONENTE DAVIDE, JR., J DATE: JAN. 9, 1995
DOCTRINE Penalty for reclusion perpetua continues to be classified as
indivisible despite fixing the duration.
FACTS In the decision in this case, promulgated on 25 May 1994, the First
Division touched on the nature of the penalty of reclusion perpetua
in the light of Section 21 of R.A. No. 7659 which amended Article 27
of the Revised Penal Code by specifically fixing the duration of
reclusion perpetua at twenty (20) years and one (1) day to forty (40)
years. It opined that since no corresponding amendment to Article
76 of the Revised Penal Code was made, the said law has not made
explicit an intention to convert reclusion perpetua into a divisible
penalty. Nevertheless, it applied Article 65 of the Revised Penal
Code. the accused in the Criminal Case No. Q-91-18465 was
sentenced to thirty-four (34) years, from (4) months and one (1) day
of reclusion perpetua.
In a motion for clarification, the appellee asks the Court to correct
the duration of the maximum period of reclusion perpetua from
thirty-four (34) years, four (4) months and one (1) day to forty (40)
years, as stated in the decision, to thirty-three (33) years, four (4)
months and one (1) day to forty (40) years.
ISSUE/S Whether or not the amendment of Article 27 of the Revised Penal
Code by Section 21 of R.A. No. 7659 has made reclusion perpetua
a divisible penalty.
RULING/S The Supreme Court held that R.A. No. 7659 did not amend the
Revised Penal Code making the penalty of reclusion perpetua still
an indivisible penalty as there was no clear legislative intent to alter
its original classification. Senator Tolentino described reclusion
perpetua as a " flexible or divisible" penalty, yet in the portion of his
sponsorship speech immediately succeeding the foregoing
description, he explicitly stated that the said penalty is one of the
two indivisible penalties in the Revised Penal Code. Senator
Tolentino had this three-grade penalty in mind when he spoke of
flexibility and divisibility and that he stood by his subsequent
statement that reclusion perpetua is one of two indivisible penalties
is further borne out by his explanations in relation to the rule in
Article 63 of the Revised Penal Code on the application of mitigating
and aggravating circumstances. If reclusion perpetua was
reclassifed as a divisible penalty, then Article 63 of the Revised
Penal Code would lose its reason and basis for existence. If Article
63 of the Code were no longer applicable because reclusion
perpetua is supposed to be a divisible penalty, then there would be

523
no statutory rules for determining when either reclusion perpetua or
death should be the imposable penalty.

524
CONTRIBUTOR Belano, Renato Jr. P.
MODULE Penalties
TOPIC
CASE TITLE People v. Latupan G.R. NO. 112453
PONENTE Pardo, J. DATE: June 28, 2001
DOCTRINE Penalty for complex crimes - Article 48 of the Revised Penal Code
provides: "When a single act constitutes two or more grave or less
grave felonies or when an offense is a necessary means for
committing the other, the penalty for the most serious crime shall
be imposed, the same to be applied in its maximum period."
FACTS On April 29, 1991, at around 4:00 in the afternoon while Ceferino
was chopping wood outside his house, he suddenly heard a woman
and child’s shout coming from the north.

Moments later, Ceferino saw the accused Latupan walking toward


him, carrying a thin bladed knife. The Accused started chasing after
Ceferino’s wife who was able to escape to another house nearby.
Unable to catch Ceferino’s wife, Latupan turned to Ceferino and
said, “I will kill you al”. Latupan thrusted his knife to Ceferion but
Ceferion was able to parry it. Later on, accused Latupan told
Ceferino to bring him to the authorities and tried to give the knife to
Ceferino. Ceferion refused and told Latupan to go to the authorities
himself. Hearing this advice, the accused ran away.

At around 4:00 in the afternoon of the same day, Emy Asuncion was
returning to his house from a store. Upon reaching his house he
found his wife lying on the ground with several stab wounds and his
one-year-old son, Leo, lying on top of her. Emy picked up Leo and
saw that the left side of his face was lacerated. He saw Jaimee, his
three-year-old son and asked where Jose, his eldest son, was. At
that moment Emy heard the voice of Jose upstairs asking for
medicine. He ran upstairs and saw that Jose was wounded. He
asked Jose who stabbed him. Jose replied, "Uncle Jerry, Tatang."
Emy brought Jose to Ceferino’s house and then returned to his
house to get his two other children, Leo and Jaime.

After being unsuccessful in asking for help with the barangay.


Ceferino went to a military camp to borrow a vehicle to bring the
children to the hospital. The military men provided them with a jeep.
Riding in the jeep were five soldiers, the accused Latupan, Emy
Asuncion, Ceferino Dagulo, Ceferino's wife, and the three children,
Leo, Jaime and Jose Asuncion.

During the trip to the hospital, Emy's son, Jose, saw accused
Latupan inside the jeep. Jose pointed to accused Latupan as the
one who stabbed him.

525
At the hospital, the doctors were able to treat the injuries of Leo and
Jaime. However they were unable to treat Jose and advised Emy to
bring him to another hospital. So, they proceeded to Cagayan Valley
Regional Hospital. Sadly, Jose was dead on arrival.[8] He was only
nine years old.

Jaime, five-year-old son of Emy Asuncion, testified that he was


three years old when the incident occurred. He stated that accused
Latupan stabbed his mother, stepped on him, threw his brother, Leo,
outside the window and stabbed his other brother, Jose.

On August 25, 1993, the trial court rendered a decision, finding the
accused Latupan, guilty beyond reasonable doubt of the complex
offense of Double Murder sentencing him to suffer life imprisonment
and to suffer ten days imprisonment for the physical injuries suffered
by Jamie and likewise for Leo.
ISSUE/S Whether or not the trial court was convicting the accused-appellant
Latupan of the “complex crime of double murder” and separate
offenses of serious physical injuries and if in the affirmative what will
be the proper penalty.
RULING/S Yes, the trial court erred in convicting accused-appellant of the
"complex crime of double murder" and separate offenses of serious
physical injuries. Article 48 of the Revised Penal Code provides:
"When a single act constitutes two or more grave or less grave
felonies or when an offense is a necessary means for committing
the other, the penalty for the most serious crime shall be imposed,
the same to be applied in its maximum period." The instant case
does not fall under any of the two mentioned instances when a
complex crime is committed. Thus, accused-appellant is liable, not
for a complex crime of double murder, but for two separate counts
of murder, and separate counts of physical injuries.

Under Article 248 of the Revised Penal Code, the penalty for murder
at the time of the commission of the crime in April 1991 was
reclusion temporal maximum to death. The trial court convicted
accused-appellant of murder and sentenced him to "life
imprisonment." The proper imposable penalty is reclusion perpetua,
not life imprisonment. Obviously, the trial court intended to impose
reclusion perpetua.

We likewise note that the trial court sentenced accused to "ten days
of imprisonment" for each count of slight physical injuries. We
reiterate the rule that it is necessary for the courts to employ the
proper legal terminology in the imposition of penalties because of
the substantial difference in their corresponding legal effects and

526
accessory penalties. Thus, the courts must employ the proper
nomenclature specified in the Revised Penal Code, such as
"reclusion perpetua," not "life imprisonment" or "ten days of arresto
menor," not "ten days of imprisonment."

Hence, the proper penalty for each murder committed in April 1991,
considering the absence of aggravating and mitigating
circumstances, is reclusion perpetua, with its accessory penalties.
Further, accused-appellant is liable for two counts of slight physical
injuries and must be sentenced to twenty (20) days of arresto
menor, each, likewise with its accessory penalties under the
Revised Penal Code.

527
CONTRIBUTOR BILTZ, Aralind Louise A.
MODULE Penalties
TOPIC
CASE TITLE PEOPLE vs. LANUZA G.R.NO.
PONENTE LEONARDO-DE CASTRO, J DATE: AUGUST 17, 2011
DOCTRINE Under the indeterminate sentence law, the maximum of the
sentence shall be that which could be properly imposed in view of
the attending circumstances, and the minimum shall be within the
range of the penalty next lower to that prescribed by the Revised
Penal Code.
FACTS The incident subject of this case took place at the basement of the
BIR office in Laoag City while the private complainant as outgoing
security guard was handing his shotgun to the accused, the
incoming security guard. Because the accused did not report for
duty on the scheduled time, the private complainant reprimanded
him. After the accused had affixed his signature on the pertinent
portion of the logbook enumerating the items turned-over to him by
the outgoing security guard, the private complainant handed to him
their service firearm, a shotgun. Allegedly, the private complainant
held it with both hands, with the muzzle pointed at him and the butt
towards the accused. At that moment, the accused gripped the
firearm with one hand, with his pointer finger inside the trigger guard
and on top of the trigger itself.

the accused stated that "I immediately held opposite the muzzle of
the gun where the trigger is, I almost slip with it while in the act of
gripping and then immediately the gun went off; the incident
happened so fast that I was stunned then realized that I accidentally
shot my fellow guard." The accused, thereafter, proceeded to the
Laoag City police station and surrendered.
ISSUE/S Whether or not the RTC correctly imposed the penalty and
damages against accused-appellant
RULING/S YES. The penalty prescribed by law for the crime of frustrated
homicide is one degree lower than that prescribed by law for the
crime of homicide. Under the indeterminate sentence law, the
maximum of the sentence shall be that which could be properly
imposed in view of the attending circumstances, and the minimum
shall be within the range of the penalty next lower to that
prescribed by the Revised Penal Code.
Considering that the penalty prescribed by law for the crime of
homicide is reclusion temporal, the penalty for the crime of
frustrated homicide would be prision mayor. Applying the
indeterminate sentence law, there being the mitigating
circumstance of voluntary surrender and no aggravating
circumstance, the maximum of the sentence should be within the

528
range of prision mayor in its minimum term which has a duration of
six (6) years and one (1) day to eight (8) years, and that, on the
other hand, the minimum should be within the range of prision
correccional which has a duration of six (6) months and one (1)
day to six (6) years. Thus, the imposition of imprisonment from four
(4) years of prision correccional, as minimum, to seven (7) years of
prision mayor, as maximum, is in order.

529
CONTRIBUTOR CACHERO, Luis III L.
MODULE Penalties
TOPIC
CASE TITLE VIRGILIO TALAMPAS V G.R.NO. 180219
PEOPLE
PONENTE BERSAMIN, J. DATE: November 23, 2011
DOCTRINE Under Section 1 of the Indeterminate Sentence Law, the court, in
imposing a prison sentence for an offense punished by the Revised
Penal Code, or its amendments, is mandated to prescribe an
indeterminate sentence the maximum term of which shall be that
which, in view of the attending circumstances, could be properly
imposed under the rules of the Revised Penal Code, and the
minimum term shall be within the range of the penalty next lower to
that prescribed by the Revised Penal Code for the offense.
FACTS
Accused-appellant Virgilio Talampas was charged with the crime of
Homicide for the death of Ernesto Matic. Prosecution witness Jose
Sevillo testified that on July 5, 1995 at around 7 o’clock in the
evening, he together with Eduardo Matic and Ernesto Matic were in
front of his house repairing his tricycle when he noticed the appellant
who was riding on a bicycle passed by and stopped. The latter
alighted, walked a few steps and brought out a short gun and poked
the same to Eduardo and fired it hitting Ernesto, which caused his
untimely death. Another shot hit Eduardo on his nape and fell down
on his back.

Talampas interposed self-defense and accident. He has insisted


that his enemy had been Eduardo Matic, not Ernesto Matic. He also
contends that during the incident, Eduardo had hit him with a
monkey wrench, but he had parried the blow; that he and Eduardo
had then grappled for the monkey wrench; that while they had
grappled, he had notice that Eduardo had held a revolver; that he
had thus struggled with Eduardo for control of the revolver, which
had accidentally fired and hit Ernesto during their struggling.

The RTC found Talampas guilty beyond reasonable doubt of


homicide and sentenced him to suffer an indeterminate penalty of
imprisonment ranging from ten (10) years and one (1) day of prision
mayor, as minimum to fourteen (14) years and eight (8) months of
reclusion temporal, as maximum.

The Court of Appeals affirmed the decision of the lower court as far
as the penalty is concerned.

Hence, this petition.

530
ISSUE/S Whether or not the indeterminate sentence of ten years and one day
of prision mayor, as minimum, to fourteen years and eight months
is erroneous.
RULING/S The Court held that the indeterminate sentence imposed by the
lower court is legally erroneous. It may be true that the increment
did not constitute an error, because the minimum term thus fixed
was entirely within the parameters of the Indeterminate Sentence
Law. Yet, the addition of one day to the 10 years as the minimum
term of the indeterminate sentence of Talampas may occasion a
degree of inconvenience when it will be time for the penal
administrators concerned to consider and determine whether
Talampas is already qualified to enjoy the benefits of the
Indeterminate Sentence Law. Hence, in order to simplify the
computation of the minimum penalty of the indeterminate sentence,
the Court deletes the one-day increment from the minimum term of
the indeterminate sentence.

The Court finds the accused guilty beyond reasonable doubt of the
crime of homicide and imposes the indeterminate sentence of 10
years of prision mayor, as minimum, to 14 years, eight months, and
one day of reclusion temporal, as maximum.

531
CONTRIBUTOR CALZADO, Anne Valerie L.
MODULE Indeterminate Sentence Law
TOPIC
CASE TITLE PEOPLE v. TEMPORADA G.R.NO. 173473
PONENTE YNARES-SANTIAGO, J. DATE: DECEMBER 17, 2008
DOCTRINE The plain terms of the Indeterminate Sentence Law show that the
legislature did not intend to limit “attending circumstances” as
referring to Articles 13 and 14 of the Revised Penal Code—the
wording of the law clearly permits other modifying circumstances
outside of Articles 13 and 14 of the Revised Penal Code to be
treated as “attending circumstances” for purposes of the application
of the Indeterminate Sentence Law, such as quasi-recidivism under
Article 160 of the Revised Penal Code.

There are attending circumstances” that operate differently than


these generic mitigating and ordinary aggravating circumstances,
and that, expectedly, leads to a different result from the one-degree
difference. Thus, for as long as the incremental penalty rule is
consistent with the letter and spirit of “attending circumstances”
under the ISL, there is no obstacle to its treatment as such.
FACTS Temporada was accused and found guilty by both the RTC and the
CA of willfully, unlawfully and feloniously defrauding 5 individuals by
means of false manifestation and fraudulent representations to the
effect that they have the power and capacity to recruit and employ
the complainants for jobs in other countries. These induced the 5
individuals to give and deliver the placement fees, which was noted
to be in excess than that specified in the scheduled of allowable fees
prescribed by the POEA. Without fault of the said complainants,
Temporada failed to deploy them to the promised employment and
failed to also reimburse the expenses incurred by complainants in
connection with the documentation and processing of their papers
for the purposes of their deployment.

RTC Judgment convicted the said accused, as principal of the


offenses charged and she is sentenced to suffer the penalty of LIFE
IMPRISONMENT and a fine of Five Hundred Thousand Pesos
(P500,000.00) for illegal recruitment; and the indeterminate penalty
of four (4) years and two (2) months of prision correctional as
minimum, to nine (9) years and one (1) day of prision mayor, as
maximum for the estafa committed against complainant Rogelio A.
Legaspi, Jr.; the indeterminate penalty of four (4) years and two (2)
months of prision correctional as minimum to ten (10) years and one
day of prision mayor as maximum each for the estafas committed
against complainants, Dennis Dimaano, Soledad B. Atte and Luz T.
Minkay; and the indeterminate penalty of four (4) years and two (2)
months of prision correctional as minimum, to eleven (11) years and

532
one (1) day of prision mayor as maximum for the estafa committed
against Evelyn Estacio.

CA Judgment affirmed with modification. Appellant is sentenced to


suffer the indeterminate penalty of six (6) years of prision
correccional maximum, as minimum, to ten (10) years and one (1)
day of prision mayor maximum, as maximum; and in Criminal Case
No. 02-208374, she is sentenced to suffer the indeterminate penalty
of eight (8) years and one (1) day of prision mayor medium, as
minimum, to twelve (12) years and one (1) day of reclusion temporal
minimum, as maximum.
ISSUE/S Whether or not the indeterminate penalties imposed for the five (5)
counts of estafa were proper.
RULING/S SC affirms the Decision of the CA, except as to the indeterminate
penalties imposed for the five (5) counts of estafa.
The Indeterminate Sentence Law is intended to favor the accused,
particularly to shorten his term of imprisonment. The reduction of his
period of incarceration reasonably helps "uplift and redeem valuable
human material, and prevent unnecessary and excessive
deprivation of personal liberty and economic usefulness." The law,
being penal in character, must receive an interpretation that benefits
the accused. This Court already ruled that "in cases where the
application of the law on indeterminate sentence would be
unfavorable to the accused, resulting in the lengthening of his prison
sentence, said law on indeterminate sentence should not be
applied." In the same vein, if an interpretation of the Indeterminate
Sentence Law is unfavorable to the accused and will work to
increase the term of his imprisonment, that interpretation should not
be adopted. It is also for this reason that the claim that the power of
this Court to lighten the penalty of lesser crimes carries with it the
responsibility to impose a greater penalty for grave penalties is not
only wrong but also dangerous.
Nowhere does the Indeterminate Sentence Law prescribe that the
minimum term of the penalty be no farther than one degree away
from the maximum term. Thus, while it may be true that the minimum
term of the penalty in an indeterminate sentence is generally one
degree away from the maximum term, the law does not mandate
that its application be rigorously and narrowly limited to that
situation.
Modifications with respect to the indeterminate penalties imposed
on appellant for the five (5) counts of estafa, to wit:
(1) In Criminal Case No. 02-208372, the accused is sentenced to an
indeterminate penalty of 4 years and 2 months of prisión
correccional as minimum, to 9 years, 8 months and 21 days of
prisión mayor as maximum.

533
(2) In Criminal Case Nos. 02-208373, 02-208375, and 02-208376,
the accused is sentenced to an indeterminate penalty of 4 years and
2 months of prisión correccional as minimum, to 10 years, 8 months
and 21 days of prisión mayor as maximum for each of the aforesaid
three estafa cases.
(3) In Criminal Case No. 02-208374, the accused is sentenced to an
indeterminate penalty of 4 years and 2 months of prisión
correccional as minimum, to 12 years, 8 months and 21 days of
reclusión temporal as maximum.
PUNO, C.J., Dissenting Opinion
For purposes of determining the minimum of the indeterminate
sentence, the “penalty prescribed” for estafa should include the
incremental penalty, since the penalty for estafa, as that in theft,
hinges on the value or amount involved.

CORONA, J., Separate Opinion


The fundamental principle in applying and interpreting criminal laws,
including the Indeterminate Sentence Law (ISL), is to resolve all
doubts in favor of the accused; Intimately intertwined with the in
dubio pro reo principle is the rule of lenity—“a court, in construing
an ambiguous criminal statute that sets out multiple or inconsistent
punishments, should resolve the ambiguity in favor of the more
lenient punishment.”

534
CONTRIBUTOR CARPIO, Anna Clarissa C.
MODULE Module 11: Penalties
TOPIC
CASE TITLE In the matter of the PETITION FOR G.R.NO. 146270
HABEAS CORPUS OF PETE
LAGRAN
PONENTE Puno J. DATE: August 15,
2001
DOCTRINE Article 70 of the RPC allows simultaneous service of two or more
penalties only if the nature of penalties so permit. The penalties
consisting in deprivation of liberty cannot be served simultaneously
by reason of the nature of such penalties. Where the accused is
sentenced to two or more terms of imprisonment, the terms should
be served, successively.
FACTS On April 18, 1994, petitioner Pete Lagran was convicted by the
Quezon City RTC for 3 counts of violation of B.P. 22 (Bouncing
Checks Law) and was sentenced to suffer imprisonment of 1 year
for each count and to pay a fine of Php125,000 with subsidiary
imprisonment in case of insolvency. Lagran appealed the decision
but the CA dismissed the same on July 11, 1997 for failure to file
appellant’s brief, hence the decision became final and executory on
August 6, 1997 and entry of judgment was made on March 5, 1998.

On February 24, 1999, petitioner Lagran was committed to the


Quezon City Jail (QCJ) and was transferred to the New Bilibid
Prision (NBP) on April 3, 1999 where he has been serving his
sentence until present (the penning of this decision).

On March 19, 2001, Lagran, citing Article 70 of the RPC, filed an


instant petition for habeas corpus, praying for his immediate release
for allegedly having completed the service of his sentence. Lagran
argues that if the penalties or sentences imposed on the accused
are identical, and such penalties or sentences emanated from 1
count and 1 complaint, the accused shall serve them
simultaneously. Lagran avers that he has been incarcerated for 2
years and 4 days counted from February 28, 2001, thus, his
detention the NBP is now without legal basis.
ISSUE/S Whether or not the contention of Lagran is meritorious?
RULING/S NO, Lagran’s prayer for the issuance of a writ habeas corpus
is untenable.

Article 70 of the RPC allows simultaneous service of two or more


penalties only if the nature of penalties so permit. The penalties
that can be simultaneously served are:
1. perpetual absolute disqualification;

535
2. perpetual special disqualification;
3. temporary absolute disqualification;
4. temporary special disqualification;
5. suspension;
6. destierro*;
7. public censure;
8. fine and bond to keep the peace;
9. civil interdiction; and
10. confiscation and payment of costs.

These penalties, *except destierro, can be served simultaneously


with imprisonment. The penalties consisting in deprivation of liberty
cannot be served simultaneously by reason of the nature of such
penalties. Where the accused is sentenced to two or more terms of
imprisonment, the terms should be served, successively.

In the case, Lagran was sentenced to suffer 1 year imprisonment


for every count of the offense committed. The nature of the sentence
does not allow petitioner to serve all the prison terms
simultaneously. Applying the rule on successive service of
sentence, petitioner has not yet completed the service of his
sentence as he commenced serving his sentence only on February
24, 1999. Thus, his prayer is without basis.

536
MODULE 12
EXTINCTION OF CRIMINAL
LIABILITY

537
CONTRIBUTOR CHUA, Kristy Anne
MODULE EXTINCTION OF CRIMINAL LIABILITY
TOPIC
CASE TITLE SERMONIA V. CA G.R.NO. 109454
PONENTE BELLOSILLO, J.: DATE: JUNE 14, 1994
DOCTRINE The non-application to the crime of bigamy of the principle of
constructive notice is not contrary to the well entrenched policy that
penal laws should be construed liberally in favor of the accused. To
compute the prescriptive period for the offense of bigamy from
registration thereof would amount to almost absolving the offenders
thereof for liability therefor.
FACTS On May 26, 1992, Sermonia was charged with the crime of bigamy
which prescribes in fifteen years with Unson before the Regional
Trial Court of Pasig, Branch 151, they contracted such marriage on
February 15, 1975 while his prior marriage with Nievera was still
existing and valid. Sermonia moved to quash the information on the
ground that such criminal liability has been considered extinguished
by prescription. On October 1, 1992, the judge denied the motion to
quash and likewise on October 27, 1992, the judge again denied the
motion for reconsideration of his order of denial. On January 21,
1993, the Court of Appeals dismissed the petition for lack of merit.

Sermonia contends that the criminal liability from the crime of


bigamy has been extinguished by prescription, he avers that since
the subsequent marriage contract was duly registered with the
Office of the Civil Register in 1975 which shall be considered as a
notice to the whole world. Nievera is considered to have had
constructive notice of the second marriage as of 1975, thus,
explains where the prescription commenced to run on the day the
marriage contract was registered in the Civil Registry. The
corresponding information for bigamy should have been filed on or
before 1990 and not in 1992. As countered by the prosecution, it
explains that the prescriptive period begins from the time of
discovery by the complainant not from the time the crime was
committed.
ISSUE/S Whether or not the prosecution of Jose C. Sermonia for the crime
of bigamy has prescribed already.
RULING/S No, the criminal liability regarding the crime of bigamy has not
prescribed since the Supreme Court agrees that it might be more
favorable to the accused if the time commences when the crime was
committed. The principle of constructive notice should not be
applied in regard to the crime of bigamy as judicial notice may be
taken of the fact that a bigamous marriage is entered by the guilty
party secretly from the prior marriage. Also, usually bigamous
marriage takes place in places that the offender is not known to be

538
a married individual to cover the legal impediment to contract
another marriage. The celebration of the bigamous marriage may
be registered in the Office of the Civil Registry but Sermonia was
not truthful and that he concealed the information on his first
subsisting marriage and also information of his bigamous marriage.
Thus, the rule on constructive notice will make de rigueur the
routinary verification of marriages as listed in the National Census
Office and in other multiple local civil registries in the Philippines to
ascertain that there are no subsequent marriages that was
contracted by any married individual without the knowledge of their
respective legitimate spouses which is too much of a task. Sermonia
also explained that there was no concealment at all because his
marriage with Uson was recorded in the Civil Registry but as per
inspection, it was proven that he wrote “single” as a civil status to
which implies that he was hiding the truth and gambled on the
impossibility of any individual that is related to Nievera or to him
would even bother investigating in relation to his subsequent
bigamous marriage.

539
CONTRIBUTOR CRUZ, Jakielyn Anne O.
MODULE MODULE 12: Extinction of Criminal Liability
TOPIC
CASE TITLE REPUBLIC v. DESIERTO G.R.NO. 136506
PONENTE DE LEON, JR., J DATE: AUG. 23, 2001
DOCTRINE The applicable provisions of law on prescription of offenses are
found in Article 90 and Article 91 of the Revised Penal Code for
offenses punishable thereunder and Act No. 3326 for those
penalized by special laws. R.A. No. 3019 being a special law, the
commencement of the period for the prescription for any act
violating it is governed by Section 2 of Act No. 3326, which provides:
"Sec. 2. Prescription shall begin to run from the day of the
commission of the violation of the law, and if the same be not known
at the time, from the discovery thereof and the institution of judicial
proceedings for its investigation and punishment. The prescription
shall be interrupted when proceedings are instituted against the
guilty person and shall begin to run again if the proceedings are
dismissed for reasons not constituting jeopardy." As a rule, if the
commission of the crime is known, the prescriptive period shall
commence to run on the day it was committed. However, in cases
where the time of commission is unknown, prescription shall only
run from its discovery and institution of judicial proceedings for its
investigation and punishment. Ordinarily, there is no problem in
determining the date when the crime consists of a series of acts,
especially when some or all of these acts are innocent in
themselves.
FACTS The Office of Solicitor General initiated a complaint before the
Presidential Commission on Good Government (PCGG) against the
private respondents for violation of R.A. No. 3019 (Anti-Graft and
Corrupt Practices Act). The said complaint was referred to the
Office of the Ombudsman. Private respondent Eduardo Cojuangco,
Jr. was alleged that he was taking advantage of his close
relationship with then President Ferdinand Marcos, had caused
President Marcos to issue favorable decrees to advance his
personal and business interests.
In 1972, when President Marcos declared Martial Law, Eduardo
“Danding” Cojuanco, Jr. started growing a coconut seed garden in
Bagsuk Island, Palawan through Agricultural Investors, Inc. (AII).
Two years later, the President Marcos issued Presidential Decree
No. 582 creating the Coconut Industry Development Fund (CIDF).
The CIDF was envisioned to finance a nationwide coconut-
replanting program using "precocious high-yielding hybrid
seednuts" to be distributed for free to coconut farmers.

540
Six (6) days after the issuance of P.D. No. 582 AII and NIDC entered
into a Memorandum of Agreement, stipulating that AII shall
develop the Bugsuk property for the growing of hybrid seednuts and
sell the entire production to NIDC, which shall in turn pay AII part of
the costs in the development and operation of the seed garden and
the support facilities.
In 1982, President Marcos lifted the coconut levy. UCPB terminate
the agreement with the AII because the financial source of the CIDF
was depleted. Thus, AII demanded an arbitration. A Board of
Arbitrators was created and rendered a decision awarding to AII
liquidated damages for Php 958,650,000.00 from the CIDF. From
this award was deducted the Php 426,261,640.00 advanced by the
NIDC for the development of the seed garden, leaving a balance
due to AII amounting to Php 532,388,354.00. Costs of arbitration
and the arbitrator's fee of Php 150,000.00 were also taken from the
CIDF.
In 1983, the UCPB Board of Directors adopted Resolution No. 111-
83, resolving to "note" the decision of the Board of Arbitrators,
allowing the arbitral award to lapse with finality.
Thus, in 1978, P.D. No. 1468 (Revised Coconut Industry Code) was
issued to substitute the United Coconut Planters Bank (UCPB) for
the NIDC as administrator-trustee of the CIDF. UCPB is a
commercial bank acquired by the government through the CCSF for
the benefit of the coconut farmers.
ISSUE/S Whether or not the ombudsman acted with grave abuse of
Discretion in declaring that the offense charged in the Complaint for
violation of ra no. 3019 had already Prescribed when the complaint
was filed?
RULING/S YES. The task to determine and find whether probable cause to
charge the private respondents exists properly belongs to the
Ombudsman. We only rule that the Office of the Ombudsman
should not have dismissed the complaint on the basis of
prescription which is erroneous as hereinabove discussed. The
Ombudsman should have given the Solicitor General the
opportunity to present his evidence and then resolve the case for
purposes of preliminary investigation. Failing to do so, the
Ombudsman acted with grave abuse of discretion.
R.A. No. 3019, as applied to the instant case, covers not only the
alleged one-sidedness of the MOA, but also as to whether the
contracts or transactions entered pursuant thereto by private
respondents were manifestly and grossly disadvantageous to the
government, whether they caused undue injury to the government,
and whether the private respondents were interested for personal
gain or had material interest in the transactions.

541
The applicable provisions of law on prescription of offenses are
found in Article 90 and Article 91 of the Revised Penal Code for
offenses punishable thereunder and Act No. 3326 for those
penalized by special laws. R.A. No. 3019 being a special law, the
commencement of the period for the prescription for any act
violating it is governed by Section 2 of Act No. 3326, 31 which
provides:
SECTION 2. Prescription shall begin to run from the day of
the commission of the violation of the law, and if the same be
not known at the time, from the discovery thereof and the
institution of judicial proceedings for its investigation and
punishment.
The prescription shall be interrupted when proceedings are
instituted against the guilty person, and shall begin to run
again if the proceedings are dismissed for reasons not
constituting jeopardy.
As a rule, if the commission of the crime is known, the prescriptive
period shall commence to run on the day it was committed.
However, in cases where the time of commission is unknown,
prescription shall only run from its discovery and institution of
judicial proceedings for its investigation and punishment. Ordinarily,
there is no problem in determining the date when the crime consists
of a series of acts, especially when some or all of these acts are
innocent in themselves.

542
CONTRIBUTOR DELA CRUZ, Ma. Luzelle P.
MODULE Extinction Criminal Liability
TOPIC
CASE TITLE ROMUALDEZ v. HON. G.R.NO. 165510-33
MARCELO
PONENTE YNARES-SANTIAGO, J.: DATE: JULY 28, 2006
DOCTRINE The statute is not statute of process, to be scantily and grudgingly
applied, but an amnesty, declaring that after a certain time oblivion
shall be cast over the offense; that the offender shall be at liberty to
return to his country, and resume his immunities as a citizen; and
that from henceforth he may cease to preserve the proofs of his
innocence, for the proofs of his guilt are blotted out.
FACTS Petitioner claims that the Office of the Ombudsman gravely abused
its discretion in recommending the filing of 24 informations against
him for violation of Section 7 of Republic Act (RA) No. 3019 or the
Anti-Graft and Corrupt Practices Act; that the Ombudsman cannot
revive the aforementioned cases which were previously dismissed
by the Sandiganbayan in its Resolution of February 10, 2004; that
the defense of prescription may be raised even for the first time on
appeal and thus there is no necessity for the presentation of
evidence thereon before the court a quo. Thus, this Court may
accordingly dismiss Criminal Case Nos. 28031-28049 pending
before the Sandiganbayan and Criminal Case Nos. 04-231857–04-
231860 pending before the Regional Trial Court of Manila, all on the
ground of prescription.
In its Comment, the Ombudsman argues that the dismissal of the
informations in Criminal Case Nos. 13406-13429 does not mean
that petitioner was thereafter exempt from criminal prosecution; that
new informations may be filed by the Ombudsman should it find
probable cause in the conduct of its preliminary investigation; that
the filing of the complaint with the Presidential Commission on Good
Government (PCGG) in 1987 and the filing of the information with
the Sandiganbayan in 1989 interrupted the prescriptive period; that
the absence of the petitioner from the Philippines from 1986 until
2000 also interrupted the aforesaid period based on Article 91 of the
Revised Penal Code.
For its part, the PCGG avers in its Comment that, in accordance
with the 1987 Constitution and RA No. 6770 or the Ombudsman Act
of 1989, the Omdudsman need not wait for a new complaint with a
new docket number for it to conduct a preliminary investigation on
the alleged offenses of the petitioner; that considering that both RA
No. 3019 and Act No. 3326 or the Act To Establish Periods of
Prescription For Violations Penalized By Special Acts and Municipal
Ordinances and to Provide When Prescription Shall Begin To Run,
are silent as to whether prescription should begin to run when the

543
offender is absent from the Philippines, the Revised Penal Code,
which answers the same in the negative, should be applied.
ISSUE/S Whether the preliminary investigation conducted by the
Ombudsman in Criminal Case Nos. 13406-13429 was a nullity.
Whether the offenses for which petitioner are being charged have
already prescribed.
RULING/S Mr. Justice Carpio also remarks that the liberal interpretation of the
statute of limitations in favor of the accused only relates to the
following issues: (1) retroactive or prospective application of laws
providing or extending the prescriptive period; (2) the determination
of the nature of the felony committed vis-à-vis the applicable
prescriptive period; and (3) the reckoning of when the prescriptive
period runs.
Statutes of limitation are to be liberally construed in favor of the
defendant, not only because such liberality of construction belongs
to all acts of amnesty and grace, but because the very existence of
the statute, is a recognition and notification by the legislature of the
fact that time, while it gradually wears out proofs of innocence, has
assigned to it fixed and positive periods in which it destroys proofs
of guilt.
The applicable 10-and-15-year prescriptive periods in the instant
case, were not interrupted by any event from the time they began to
run on May 8, 1987. As a consequence, the alleged offenses
committed by the petitioner for the years 1963-1982 prescribed 10
years from May 8, 1987 or on May 8, 1997. On the other hand, the
alleged offenses committed by the petitioner for the years 1983-
1985 prescribed 15 years from May 8, 1987 or on May 8, 2002.
Therefore, when the Office of the Special Prosecutor initiated the
preliminary investigation of Criminal Case Nos. 13406-13429 on
March 3, 2004 by requiring the petitioner to submit his counter-
affidavit, the alleged offenses subject therein have already
prescribed. Indeed, the State has lost its right to prosecute
petitioner for the offenses subject of Criminal Case Nos. 28031-
28049 pending before the Sandiganbayan and Criminal Case Nos.
04-231857–04-231860 pending before the Regional Trial Court of
Manila.

544
CONTRIBUTOR DIMAKUTA, Jasmine A.
MODULE EXTINCTION OF CRIMINAL LIABILITY
TOPIC
CASE TITLE RICARDO RANIER G. CRUZ III, IN HIS G.R. 223446
CAPACITY AS DIRECTOR GENERAL OF NO.
THE BUREAU OF CORRECTIONS;
RICHARD W. SCHWARZKOPF, IN HIS
CAPACITY AS SUPERINTENDENT, NEW
BILIBID PRISON, BUREAU OF
CORRECTIONS; AND EMERENCIANA M.
DIVINA, IN HER CAPACITY AS THE
OFFICER-IN-CHARGE, INMATE
DOCUMENTS AND PROCESSING
DIVISION OF THE NEW BILIBID PRISON,
BUREAU OF CORRECTIONS, petitioners,
vs. ROLITO T. GO, DETAINED AT THE
MAXIMUM SECURITY COMPOUND, NEW
BILIBID PRISON, JOINED BY HIS WIFE
ELSA ANG GO
PONENTE DATE: November
28, 2016
DOCTRINE To implement the provisions of Article 97, the law has granted the
Director of Prisons the power to grant good conduct allowances. The
mandate of the Director of Prisons embodied in Article 99 of the
RPC is clear and unambiguous. In fact, once granted, such
allowances shall not be revoked.

FACTS By virtue of the 4 November 1993 Decision of the RTC, Branch 168,
Pasig City in Criminal Case No. 87411, respondent Rolito T. Go was
convicted of murder and sentenced to suffer the penalty of reclusion
perpetua. He began serving his sentence on 30 April 1996 at the
New Bilibid Prison.
On 30 July 2008, in carrying out the Resolution and Certificate of
Eligibility by then Bureau of Corrections (BuCor) Director Oscar C.
Calderon, the New Bilibid Prison Classification Board granted Go,
along with other 24 inmates, a colonist status. Accordingly, in view
of his commuted sentence, Go filed a petition for habeas corpus on
30 January 2014, pleading for his release. He posits that his original
prison sentence which shall expire on 31 January 2022 instead
should have expired on 21 August 2013 upon deduction of lawful
and proper allowances for good conduct, colonist status, and
preventive imprisonment based on the provisions of Act No. 2489,
otherwise known as "An Act Authorizing Special Compensation,
Credits, and Modification in the Sentence of Prisoners as a Reward
for Exceptional Conduct and Workmanship and for Other
Purposes."

545
In opposition to Go's release, petitioners maintained that Go's
sentence neither has expired nor was commuted. According to
petitioners, the grant of colonist status on Go did not carry with it the
automatic commutation of his sentence from the indivisible penalty
of reclusion perpetua to 30 years because only the President has
the power to commute a sentence. Sans the signature of the
President, any commutation is ineffectual.
On 28 April 2014, the RTC granted the petition and issued a Writ of
Habeas Corpus. The RTC found that Go's sentence was validly
commuted from reclusion perpetua to 30 years pursuant to Section
7, Chapter 3 of the BuCor Manual:
Section 7. Privileges of a colonist. — A colonist shall have the
following privileges:
a. Credit of additional GCTA of five(5) days for such calendar
month while retains said classification aside from the regular
GCTA authorized under Article 97 of the Revised Penal Code;
b. Automatic reduction of life sentence imposed on the colonist to
a sentence of thirty (30) years;

The BuCor Manual is very clear. No ambiguity attends that provision


that once an inmate is granted a colonist status, his life sentence is
commuted to 30 years. The RTC further held that, "[w]hile it is true
that the President may commute the service of sentence of a
prisoner, the law also recognizes partial reduction of sentences
under Art. 97 of the Revised Penal Code which provides for
allowances of good conduct." Contrary to petitioners' contention that
the penalty of reclusion perpetua cannot be commuted to 30 years,
the RTC cited Article 70 of the Revised Penal Code, which
specifically provides that for perpetual penalties like reclusion
perpetua, the duration shall be computed at 30 years. Clearly, it is
not correct that only the President can commute a sentence as
these provisions, i.e., Articles 70 and 97, warrant partial
extinguishment or commutation of sentence.
The pertinent portion of the Decision of the RTC granting the Writ of
Habeas Corpus reads:
The court adheres therefore, to the computation of GO's expiration
of sentence on August 21, 2013 which is based on the 30 year
reduction of his life sentence. His further detention beyond this
period to the mind of the court is illegal.
WHEREFORE, premises considered, the petition is hereby
GRANTED. The petitioner ROLITO GO y TAMBUNTING is
ordered released from custody having fully served his sentence
unless detained for some other legal cause.

On 27 August 2015, the CA in a Decision dated 27 August 2015,


dismissed the appeal as an improper remedy. The CA resolved that
because the appeal raised only pure question of law, as the sole

546
issue involved in the present case is whether the BuCor may validly
commute a sentence, the proper recourse should have been a
petition for review on certiorari under Rule 45 of the Rules of Court
before the Supreme Court (SC).

ISSUE/S Whether or not the BuCor has authority to commute a prison


sentence.

RULING/S Yes. Petitioners aver that Go's commutation of sentence as a result


of the grant of penal colonist status, deduction of lawful and proper
allowances for good conduct, and preventive imprisonment of Go is
ineffective without prior approval by the President because it
violates Section 19, Article VII of the Constitution, which mandates
that only the President has the power to exercise executive
clemency. We deny the petition. As correctly resolved by the trial
court, while only the President can commute a prison sentence,
Articles 70 and 97 of the Revised Penal Code (RPC) recognize
partial reduction or commutation of sentences by providing that "for
penal penalties, the duration shall be computed for 30 years and the
allowances of good conduct must be applied on top of the [good
conduct time allowance] accorded to an inmate with a colonist
status."

Accordingly, to implement the provisions of Article 97, the law has


granted the Director of Prisons the power to grant good conduct
allowances. The mandate of the Director of Prisons embodied in
Article 99 of the RPC is clear and unambiguous. In fact, once
granted, such allowances shall not be revoked. Article 99 of the RPC
explicitly states:

Art. 99. Who grants time allowances. — Whenever lawfully


justified, the Director of Prisons shall grant allowances for good
conduct. Such allowances once granted shall not be revoked.

Therefore, after crediting his preventive imprisonment of nine (9)


months and sixteen (16) days, and the regular Good Conduct Time
Allowance (GCTA) under Act No. 3815 and Special Credit Time
Allowance (SCTA) under Act No. 2409 granted upon him, Go has
completed serving his sentence of thirty (30) years on 21 August
2013, which he commenced to serve on 30 April 1996.

The intent and spirit of the law in affording persons the remedy of
writ of habeas corpus is to devise a speedy and effective means to
relieve persons from unlawful restraint. To rule otherwise would
render Article 99 of the RPC as a mere surplusage and would unduly
impose excessive imprisonment on inmates in violation of the basic
right to liberty.

547
WHEREFORE, the petition is DENIED. This Resolution is
IMMEDIATELY EXECUTORY. The Director of the Bureau of
Corrections is ordered to immediately RELEASE petitioner Rolito T.
Go from detention unless he is detained for any other lawful cause.

548
CONTRIBUTOR FLORES, Precious Eureka D.
MODULE Exempting Circumstance
TOPIC
CASE TITLE PEOPLE v. BAYOTAS G.R.NO. 102007
PONENTE ROMERO, J. DATE: SEPTEMBER 2, 1994
DOCTRINE Death of the accused pending appeal of his conviction extinguishes
his criminal liability as well as the civil liability
FACTS Rogelio Bayotas was charged with Rape and was convicted of the
same crime on June 19 ,1991. Pending appeal of his conviction,
Bayotas died on February 4, 1992 due to cardio respiratory arrest
secondary to hepatic encephalopathy secondary to hepato
carcinoma gastric malingering. Consequently, the Supreme Court
dismissed the criminal aspect of the appeal. However, it required
the Solicitor General to file its comment with regard to Bayotas’ civil
liability arising from his commission of the offense charged.

The Solicitor General expressed his view that the death of accused-
appellant did not extinguish his civil liability as a result of the
commission of the offense charged.
ISSUE/S Does death of the accused pending appeal of his conviction
extinguish his civil liability?
RULING/S Article 89 of the Revised Penal Code states that:

Art. 89. How criminal liability is totally extinguished – criminal liability


is totally extinguished:

1. By the death of the convict, as to personal penalties; and as


to the pecuniary penalties liability therefor is extinguished
only when the death of the offender occurs before final
judgment;

The law is plain, statutory construction is unnecessary, Criminal


liability is extinguished. However, the civil liability poses a problem
because such liability is extinguished only when the death of the
offender occurs before final judgment. Is it final judgment as
contradistinguished from an interlocutory order or is it a judgment
which is final and executory?

Based on the legal precepts and opinions, final judgment in the


Revised Penal Code means judgment beyond recall wherein as
long as a judgment has not become executory, it cannot be truthfully
said that the defendant is definitely guilty of the felony charged
against him.

549
Death of the accused pending appeal of his conviction extinguishes
his criminal liability as well as the civil liability since based on the
Castillo case, the court said that the civil liability is extinguished only
when death of the accused occurred before the final judgment.

550
CASE TITLE Securities and Exchange Commission G.R NO. G.R. No.
v. Interport Resources Corp 135808
PONENTE CHICO-NAZARIO, J. DATE: 6 October
2008
CONTRIBUTOR GARCIA, LEXANNE O.
DOCTRINE
Indubitably, the prescription period is interrupted by commencing
the proceedings for the prosecution of the accused. In criminal
cases, this is accomplished by initiating the preliminary
investigation.

A preliminary investigation is essentially a determination whether


an offense has been committed, and whether there is probable
cause for the accused to have committed an offense. A preliminary
investigation interrupts the prescription period.
FACT/S
The Board of Directors of IRC approved a Memorandum of
Agreement with Ganda Holdings Berhad (GHB), where IRC
acquired 100% or the entire capital stock of Ganda Energy
Holdings, Inc. (GEHI), which would own and operate a 102
megawatt (MW) gas turbine power-generating barge. However,
the fax machine of the SEC could not receive it. Upon the advice
of the SEC, the IRC sent the press release on the morning of 9
August 1994.

The SEC averred that it received reports that IRC failed to make
timely public disclosures of its negotiations with GHB and that some
of its directors, respondents herein, heavily traded IRC shares
utilizing this material insider information.

Thereafter, investigative proceedings by the SEC ensued as early


as 1994. Respondents were called to appear before the \ SEC and
explain their failure to disclose pertinent information on 14 August
1994.

But on appeal, the Court of Appeals later issued a writ preliminary


injunction where the SEC is prohibited from taking cognizance or
initiating any action, be they civil, criminal, or administrative against
the respondents.
MAIN ISSUE/S
Has the crime prescribed?
RULING ON
MAIN ISSUE/S
The instant case has not yet prescribed. Respondents point
out that the prescription period applicable to offenses

551
punished under special laws, such as violations of the Revised
Securities Act, is twelve years. Since the offense was committed in
1994, they reasoned that prescription set in 2006 and now moot.

A preliminary investigation is essentially a


determination whether an offense has been committed, and
whether there is probable cause for the accused to have committed
an offense. A preliminary investigation interrupts the prescription
period.

The Court declared that it is imperative that the


criminal prosecution be initiated before the SEC. It should be noted
that the SEC started investigative proceedings against the
respondents as early as 1994. This investigation
effectively interrupted the prescription period. However, said
proceedings were disrupted by a preliminary injunction issued\ by
the Court of Appeals.

The law on the prescription period was never intended to put the
prosecuting bodies in an impossible bind in \ which \ the \
prosecution of a case would be place way beyond their control; for
even if they avail themselves \ of the proper remedy, they would still
be barred from investigating and prosecuting the case.

Indubitably, the prescription period is interrupted by commencing


the proceedings for the prosecution of the accused. In criminal
cases, this is accomplished by initiating the preliminary
investigation.

Given the nature and purpose of the


investigation conducted by the SEC, which is equivalent to the
preliminary investigation conducted by the DOJ in criminal cases,
such investigation would surely interrupt the prescription period.

552
CONTRIBUTOR Manango, John De Divine B.
MODULE Module 12: Extinction of Criminal Liability
TOPIC
CASE TITLE People v. Consorte G.R.NO. G.R. No. 194068
PONENTE J. Perez DATE: 26 November 2014
DOCTRINE Extinguishment of Criminal liablity (by death of the convict)
FACTS For the resolution of the Court is the Motion for Reconsideration[1]
of our Decision dated 9 July 2014,[2] which affirmed the conviction
of accused-appellant Benjie Consorte y Franco for the murder of

Elizabeth Palmar, the dispositive portion of which reads:

WHEREFORE, the Decision of the Court of Appeals dated 27 May


2010 in CA-G.R. CR HC No. 01806 is AFFIRMED with the following
MODIFICATIONS (1) that the amount of civil indemnity is increased
from P50,000.00 to P75,000.00; and (2) that the amount... of
exemplary damages is increased from P25,000.00 to
P30,000.00. An interest, at the rate of six percent (6%) per annum
shall be imposed on all the damages awarded in this case from the
date of finality of this judgment until they are fully paid.

Meanwhile, in a Letter dated 21 September 2014,[7] the Officer-in-


Charge of the New Bilibid Prison (NBP) informed the Court that
accused-appellant died on 14 July 2014
ISSUE/S Court now addresses the effect of death pending accused-
appellant's appeal with regard to his criminal and civil liabilities.
RULING/S Article 89(1) of the Revised Penal Code is illuminating:
Art. 89. How criminal liability is totally extinguished. - Criminal
liability is totally extinguished:
(1) By the death of the convict, as to the personal penalties; and as
to pecuniary penalties, liability therefor is extinguished only when
the death of the offender occurs before final judgment;
In the case at bar, accused-appellant died before final judgment,
as in fact, his motion for reconsideration is still pending resolution
by the Court. As such, it therefore becomes necessary for us to
declare his criminal liability as well as his civil liability ex... delicto
to have been extinguished by his death prior to final judgment.
WHEREFORE, the criminal and civil liability ex delicto of accused-
appellant Benjie Consorte y Franco are declared EXTINGUISHED
by his death prior to final judgment. The judgment or conviction
against him is therefore SET ASIDE.

553
THAT IN ALL THINGS,
GOD MAY BE GLORIFIED!

554

You might also like